Sie sind auf Seite 1von 210

 

1. PACIENTE DE 69 AÑOS, JUBILADO, QUE EN SU TIEMPO LIBRE SE DEDICA A CUIDAR,


EN UNA GRANJA ESCUELA, UN GRUPO DE ASNOS. TIENE DIVERSOS NÓDULOS EN EL
BRAZO DERECHO, ALGUNOS DE LOS CUALES HAN EVOLUCIONADO A ÚLCERAS, DE
LAS CUALES SURGEN UNOS TRAYECTOS ERITEMATOSOS QUE USTED IDENTIFICA
COMO LINFANGITIS. EN LA EXPLORACIÓN ENCUENTRA ADENOPATÍAS AXILARES Y
CERVICALES. TIENE DIFICULTAD RESPIRATORIA; REFIERE ODINOFAGIA Y CAMBIO EN
LA TEXTURA DE SU VOZ DESDE HACE UNOS DÍAS, QUE ATRIBUYE USTED A LA
AFECTACIÓN DE LAS MUCOSAS OROFARÍNGEA Y NASAL. LA PIEL DEL DORSO DE LA
NARIZ TIENE ASPECTO ERISIPELOIDE. USTED YA SOSPECHA DE LA SIGUIENTE
ETIOLOGÍA:

1. 1. BURKHOLDERIA MALLEI.
2. 2. BURKHOLDERIA PSEUDOMALLEI.
3. 3. PSEUDOMONAS AERUGINOSA.
4. 4. ALCALIGENES XYLOSOXIDANS.
Gráfico de respuestas
Comentario
No te preocupes si has fallado esta pregunta: se trata de un caso de muermo. Recuerda las
características principales de esta enfermedad:

- Causada por el cocobacilo gramnegativo Burkholderia mallei

- Los portadores habituales son el ganado equino (caballos, burros y mulas)

- Existen 4 formas clínicas:

o Forma aguda cutáneo- mucosa localizada

o Forma pulmonar aguda

o Forma septicémica ó generalizada

o Forma crónica supurada


(R1)  

2. ¿Cuál de los siguientes GN puede derivar a una glomeruloesclerosis focal y


segmentaria?

1. 1. Nefropatía de mínimos cambios.


2. 2. GN proliferativa mesangial.
3. 3. GN mesangial IgA.
4. 4. Más del 90% de las asociadas a lupus.
Gráfico de respuestas
Comentario

La GN focal y segmentaria puede ser secundaria prácticamente a cualquier nefropatía. Cuando se


produce una pérdida de masa nefronal, las nefronas restantes se ven sometidas a una situación de
hiperfiltración. Esto produce una hiperpresión intraglomerular, que favorece su aparición. Por ello,

 
 
 
 
puede ser debida a cualquiera de las opciones que aparecen en esta pregunta, por lo que la
pregunta realmente, de aparecer en el ENARM sería anulable.(R1)

3. Una biopsia hepática de una hepatitis crónica persistente presentará:

Infiltrado mononuclear en espacio porta que lo expande pero respetando la membrana


1. 1.
limitante.
Infiltrado inflamatorio portal con respeto de la membrana limitante y necrosis e
2. 2.
inflamación lobular.
Infiltrado inflamatorio portal y lobular con necrosis parcelar periférica y grados variables
3. 3.
de fibrosis.
4. 4. Infiltrado inflamatorio portal y lobular con necrosis en puentes y fibrosis.
Gráfico de respuestas
Comentario

Es importante que tengas claros los siguientes conceptos sobre la histopatología del hígado:

- Hepatitis crónica activa: Requiere, como condición mínima, la rotura de la membrana limitante
que separa el espacio porta del lobulillo hepático (“necrosis parcelar periférica”).

- Hepatitis crónica persistente: Infiltrado portal, pero sin afectación de la membrana limitante.

- Hepatitis crónica lobulillar: Infiltrados aislados en lobulillo hepático. No se afecta membrana


limitante.

- Cirrosis: Necrosis, fibrosis en puentes y nódulos de regeneración.


(R1)  

4. Mujer portadora de un DIU que presenta un intenso dolor en hipogastrio, náuseas y


vómitos. La temperatura es de 38.9ºC y en los exámenes de laboratorio se detecta
elevación de la VSG y leucocitosis con desviación a la izquierda. La exploración
ginecológica es dolorosa y existe un anexo aumentado de tamaño. Cuando se comprime
el cuello uterino con las valvas, sale un flujo purulento. ¿Cuál es, actualmente, el agente
más frecuente de este cuadro?:

1. 1. Neisseria gonorrheae.
2. 2. Chlamydia trachomatis.
3. 3. Mycobacteria tuberculosis.
4. 4. Candida albicans,
Gráfico de respuestas
Comentario

Una pregunta bastante difícil.

La causa más frecuente de cervicitis es Chlamydia trachomatis. Sin embargo, las cervicitis por este
microorganismo suelen ser poco sintomáticas, el flujo no suele ser purulento (sino seroso o
hemorrágico) y normalmente no hay fiebre (como mucho, febrícula). Por lo tanto, aunque el

 
 
 
 
microorganismo más frecuente no es el gonococo, en este caso sí que sería mucho más esperable
que Chlamydia, porque las manifestaciones clínicas son muy floridas. Respuesta 1 correcta.(R1)

5. A 75-year-old patient from Detroit comes to the ER due to massive hematemesis. His
previous medical history is significant for alcoholic cirrhosis, with previous episodes of
ascites and upper gastrointestinal bleeding. His current medication includes furosemide,
spirinolactone and propranolol. On physical examination, he has spider angiomas,
jaundice, caput medusae and flank dullness. BP is 120/50 mmHg, pulse is 75/min and
respirations are 22/min. Laboratory results are: Hemoglobin 10 g./dl, MCH 27 pg, MCV:
90fl., Leukocytes 5220/mm3, Platelets 150000/mm3, total bilirubin: 6.0 mg/dl, INR: 1.8,
albumin 2.5 g./dl., GOT: 340 U/L, GPT: 300 U/L. He is started on diuretics and a salt and
protein restricted diet. Supportive treatment is started, however three days later he
develops low level of consciousness and flapping tremor. Which of the following is the
pathogenesis of this condition?

1. 1. Relative hypocalcemia
2. 2. Anemia
3. 3. Accumulation of ammonia and production of false neurotransmisors
4. 4. Spontaneous bacterial peritonitis
Gráfico de respuestas
Comentario
Accumulation of ammonia and production of false neurotransmisors. Hepatic encephalopathy (HE)
is the occurrence of confusion, altered level of consciousness and coma as a result of liver failure. It
is caused by the accumulation in the bloodstream of toxic substances that are normally removed by
the liver. It occurs either because the hepatocytes are incapable of metabolising the waste products
or because portal venous blood bypasses the liver through collateral circulation. Mild forms of HE
are difficult to detect clinically. They are experienced as forgetfulness, mild confusion, and irritability.
In the intermediate stages, a characteristic jerking movement of the limbs is observed (asterixis,
"liver flap" due to its flapping character); this disappears as the somnolence worsens. There is
disorientation and amnesia, and uninhibited behaviour may occur. In the advanced stages,
neurological examination may reveal clonus and positive Babinski sign. Coma and seizures
represent the most advanced stage; cerebral oedema (swelling of the brain tissue) leads to
death.(R3)

6. ¿Cuál de las siguientes características NO pertenece al síndrome de feminización


testicular completa?:

1. 1. No existe hirsutismo.
2. 2. La vagina es corta y termina en fondo de saco.
3. 3. Existen testículos de tamaño normal.
4. 4. Las mamas habitualmente no se desarrollan.
Gráfico de respuestas
Comentario

Es una pregunta bastante difícil del síndrome de Morris del que debe conocer algunas
características principales para poder resolver un posible caso clínico. La feminización testicular o
resistencia androgénica completa se debe a un trastorno del receptor androgénico. Se hereda de
forma recesiva ligada a X. Los conductos de Wolf están ausentes y no existe ningún órgano genital
interno. Los testículos son normales sin espermatogénesis, pueden estar situados en el abdomen o
en cualquier lugar del trayecto del conducto inguinal. Seno urogenital femenino y vagina corta que
acaba en un fondo de saco ciego. Los genitales externos son femeninos con ausencia de vello

 
 
 
 
axilar y pubiano. La mama también es normal. La testosterona se encuentra elevada igual que los
estrógenos y la LH (resistencia a la acción androgénica en el hipófisis).(R4)

7. A patient is found to have high grade intraepitelial dysplasia on follow-up cervical


cytology. Colposcopy reveals no atypical lesions in the transition zone. Which of the
following is the most appropriate next step in management?

1. 1. New cervical cytology and colposcopy in 3 months.


2. 2. Cryotherapy.
3. 3. Endocervical curettage.
4. 4. Total hysterectomy.
Gráfico de respuestas
Comentario

Tema muy importante para el nacional. Según las guías, el tratamiento de una displasia grave es la
crioterapia o la conización.(R2)

8. Gestante de 37 semanas que ha estado en contacto con un niño al que han


diagnosticado varicela y que no recuerda si ella la padeció en la infancia. ¿Qué estaría
indicado?:

1. 1. Tranquilizarla; no es necesario ningún tratamiento.


2. 2. Administrar la gammaglobulina específica.
Solicitar la cuantificación de IgG y, si fuera negativa, administrar la gammaglobulina
3. 3.
específica.
4. 4. Finalizar la gestación.
Gráfico de respuestas
Comentario
Si la varicela se produce después de las 20 semanas, el riesgo de defectos congénitos es
prácticamente inexistente. El problema es el riesgo en esta paciente de la varicela neonatal que se
da cuando el cuadro aparece en las 3 semanas previas al parto y tiene gran mortalidad, por lo que,
si no está inmunizada, es necesario administrar gammaglobulina específica.(R3)

9. En un niño de 15 meses diagnosticado de enfermedad celíaca, ¿qué cereal, de los


siguientes, recomendaría porque NO se asocia a la producción de este cuadro?

1. 1. Trigo.
2. 2. Cebada.
3. 3. Arroz.
4. 4. Avena.
Gráfico de respuestas
Comentario

Es muy importante que el día del ENARM no olvide los cereales que contienen gluten: trigo,
cebada, centeno y posiblemente en avena (por contaminación cruzada). Para recordar qué
alimentos se pueden dar, recuerde que terminan con la letra z: maíz y arroz. Respuesta 3
correcta.(R3)

10. En una de las siguientes pacientes está totalmente CONTRAINDICADA la


anticoncepción hormonal:

 
 
 
 
1. 1. Paciente con miomas uterinos.
2. 2. Paciente con colelitiasis.
3. 3. Antecedentes de ictericia colestásica del embarazo.
4. 4. Hemangioma hepático.
Gráfico de respuestas
Comentario

Es muy importante que conozca el mecanismo de acción, los efectos secundarios y las
contraindicaciones de los anticonceptivos orales de cara al ENARM pues han sido preguntados en
diversas ocasiones.

Las contraindicaciones absolutas son pacientes con riesgo cardiovascular (fumadoras mayores de
35 o no fumadoras mayores de 40), antecedentes de enfermedad tromboembólica, HTA mal
controlada, diabetes con afectación vascular, vasculopatía inflamatoria, cardiopatías graves,
pacientes con afectación hepática importante (adenoma hepático o hepatopatías activas), pofiria
aguda intermitente, antecedentes de ictericia durante el embarazo (colestasis intrahepática),
cáncer de mama u otros tumores hormonodependientes, discrasia sanguínea y sangrado genital
anormal no filiado.(R3)

11. En relación con la bilirrubina en RN, marque la correcta:

1. 1. No hay riesgo de neuro –toxicidad con adecuados valores séricos de albúmina


2. 2. Hay más riesgo de Kernicterus en prematuros
3. 3. Tiene efectos antioxidantes a niveles altos.
4. 4. Hay excelente correlación entre la clínica y los valores séricos.
Gráfico de respuestas
Comentario

Esta pregunta puede resolverse por sentido común. La respuesta 1 es dificil de aceptar. En primer
lugar, todo puede pasar en medicina, y en segundo lugar, si hay un exceso de bilirrubina, tal que la
albúmina no pueda manejar, entonces, puede existir neurotoxicidad.

La opción 4 también es dificil de aceptar, ya que entra en juego la variabilidad interindividual.

La opción 2 es la respuesta correcta. Los prematuros tienen mayor susceptibilidad al daño


encefálico por hiperbilirrubinemia.(R2)

12. De los virus que se mencionan, ¿cuál tiene un sólo tipo serológico?:

1. 1. Herpes simple.
2. 2. Varicela-zoster.
3. 3. Parainfluenzae.
4. 4. Echo.
Gráfico de respuestas
Comentario

A primera vista esta pregunta puede asustar y parecer imposible, pero aunque no te hayas
memorizado todos los serotipos de los virus, con un poco de lógica y razonamiento de lo estudiado
en infecciosas se puede sacar.

 
 
 
 
Si piensas en las infecciones producidas por el VHS (Herpes simple) te sonará que existe el VHS-
1, el VHS-2... con lo que la opción 1 queda descartada.

Del virus varicela zoster VVZ... sabes que causa la varicela y el herpes zoster pero te suenan
distintos serotipos... pues será una posible opción.

Del parainfluenza, el causante de la gripe, también hay varios serotipos y de los virus Echo, quizá
no te suene pero suena a complicado. Por lo tanto lo más probable sin saber nada de serología
vírica sería la opción 2, que es la correcta.(R2)

13. Embarazada de 27 semanas de gestación, que acude a consulta por vez primera,
porque no se ha controlado el embarazo hasta la fecha. Tiene como antecedentes tres
embarazos y partos “normales”, aunque tampoco controló la gestación en ninguno de
ellos, con pesos comprendidos entre 4200-4700 g. No presenta otros antecedentes de
interés, salvo un IMC (índice de masa corporal) de 31. Se solicita exámenes de laboratorio
ecografía fetal. En la ecografía se observa una gestación única, longitudinal, cefálica, con
latido cardiaco positivo. Biometría acorde con amenorrea, salvo un perímetro abdominal
compatible con 31-32 semanas de gestación y presencia de una hipertrofia del tabique
interventricular. La placenta es posterior y presenta polihidramnios leve. Según su
sospecha diagnóstica, señale la afirmación CORRECTA:

Sería útil una determinación de HbA1c, aunque con bastante probabilidad no se


1. 1.
encontraría alterada.
Sería conveniente realizar el test de O´Sullivan, donde encontraríamos con mucha
2. 2.
probabilidad un valor a la hora postingesta < 140 mg/dl.
No estaría indicado aún el tratamiento con insulina, aunque posiblemente fuera necesario a
3. 3.
medida que avanza la gestación.
Esta patología puede dar lugar a diferentes malformaciones congénitas, entre las que se
4. 4. encuentran: hipoplasia de colon izquierdo, síndrome de regresión caudal, malformaciones
cardiovasculares o agenesia renal.
Gráfico de respuestas
Comentario

La respuesta correcta es la 4, porque esas son algunas de las malformaciones congénitas que
pueden aparecer en la diabetes gestacional que es nuestra sospecha diagnóstica.

La determinación de la HbA1c es útil para ver el control del metabolismo en las últimas semanas y
tiene valor pronóstico para las malformaciones. En este caso, dadas las repercusiones fetales, lo
más probable es que exista un mal control metabólico y que el valor de HbA1c se vea alterado. En
cuanto al O´Sullivan, se considera patológico un valor postingesta de una hora superior a 140
mg/dl y no inferior. En esta paciente, con mal control metabólico y repercusiones fetales
(macrosomía, polihidramnios), además de dieta y ejercicio, sería indicación de tratamiento con
insulina.(R4)

14. El exantema de la varicela se caracteriza por:

1. 1. Lesiones pleomórficas, de predominio periférico.


2. 2. Distribución a predominio periférico.
3. 3. No pruriginoso.
4. 4. Eritematopapular.
Gráfico de respuestas

 
 
 
 
Comentario

El exantema de la varicela son vesículas en diferentes estadios de evolución que se distribuyen en


cielo estrellado, es decir, por todos lados, incluso cuero cabelludo y mucosas. Si bién es cierto que
hay cierto predominio periférico. Y cualquiera que la haya pasado sabrá que produce mucho
prurito.(R1)

15. Paciente ictérico que presenta en los exámenes de laboratorio elevación de las
transaminasas 8 veces por encima de lo normal, de quince días de evolución. Se conocía
HBsAg +, HBeAg + desde hacía 3 años. Señale el marcador vírico que solicitaría
inicialmente para catalogar el cuadro:

1. 1. Determinación de anti HBc tipo IgM.


2. 2. Idem de Ac anti-HBe.
3. 3. Idem de anti-VHC.
4. 4. Idem ADN del virus de la hepatitis B.
Gráfico de respuestas
Comentario
El único marcador que es capaz de diferenciarnos una hepatitis aguda y una hepatitis crónica en
fase replicativa (AgHBs +, Ag Hbe +) sería el anticuerpo anti- HBc: Si fuera de tipo IgM nos
encontramos ante una hepatitis aguda y si es de tipo IgG estamos ante una hepatitis crónica.(R1)

16. Ante los cambios que se producen en el embarazo, ¿cuál de los siguientes
enunciados es el VERDADERO?:

1. 1. La dilatación de los uréteres y pelvis renales es patológica.


Ante cifras de 13,000 leucocitos y VSG de 35 mm, debemos buscar siempre un foco
2. 2.
infeccioso en la gestante.
3. 3. Se produce una disminución plasmática de las cifras de creatinina, urea y ácido úrico.
La actividad tiroidea está disminuida y se produce paso al feto de T3 y T4 de forma muy
4. 4.
apreciable.
Gráfico de respuestas
Comentario

A menudo se pregunta sobre los cambios fisiológicos en el embarazo. Pregunta sobre las
modificaciones gravídicas maternas que como puede ver no entraña ninguna dificultad que hemos
repasado previamente.

Durante la gestación se produce un aumento del flujo plasmático renal y la filtración glomerular en
un 40%, junto con un incremento de la reabsorción tubular; todo ello producido por el aumento del
volumen plasmático y gasto cardiaco. Es por todo lo anteriormente expuesto el que nos
encontremos un descenso de la creatinina, urea y ácido úrico (opción 3 correcta) queda diluido por
así decirlo.

El resto de opciones son falsas ya que se produce una dilatación de uréteres y pelvis renales de
forma fisiológica, aumentan también de forma fisiológica la cifra de leucocitos y VSG y aumenta la
síntesis de hormonas tiroideas.(R3)

17. Which of the following options is NOT a side effect of vasopressin?

 
 
 
 
1. 1. Ascites.
2. 2. Myocardial ischemia.
3. 3. Portal hypertension.
4. 4. Hyponatremia.
Gráfico de respuestas
Comentario

La vasopresina produce constricción arteriolar esplácnica, disminuyendo la presión portal


(respuesta 3 falsa). Sin embargo, tiene muchos efectos secundarios, como isquemia arterial a nivel
coronario, mesentérico, cerebral o de miembros. Por otra parte, disminuye la excreción de agua
libre, favoreciendo la sobrecarga de volumen, la hiponatremia y la ascitis. Todo esto hace que la
vasopresina, asociada o no a nitroglicerina, sea un fármaco actualmente en desuso.

En este momento, se utiliza mucho más la terlipresina o glipresina. Es un derivado de la


vasopresina, con menos efectos secundarios y una vida media más prolongada. También es eficaz
con este fin la somatostatina.(R3)

18. Adolescente de 16 años con rinitis crónica por alergia a los ácaros del polvo. El
tratamiento de elección a largo plazo más eficaz es:

1. 1. Inmunización con vacuna sublingual.


2. 2. Antihistaminicos orales.
3. 3. Medidas en domicilio particular para evitar y/o reducir exposición al polvo.
4. 4. Vivir en zonas costeras.
Gráfico de respuestas
Comentario

En las alergias al polvo lo más eficaz son las medidas de evitación, por este motivo la respuesta
correcta es la 3.(R3)

19. Mujer de 32 años, embarazada de gemelos y diagnosticada de polihidramnios,


comienza después del parto con una importante hemorragia. A la palpación se detecta la
ausencia de retracción uterina. Debemos pensar en:

1. 1. Placenta acreta.
2. 2. Desgarro de partes blandas.
3. 3. Atonía uterina.
4. 4. Rotura uterina.
Gráfico de respuestas
Comentario

La causa más frecuente de las hemorragias del alumbramiento es la atonía uterina. Además, esta
paciente que nos presentan tiene un riesgo aumentado, por la sobredistensión uterina mantenida
por el polihidramnios y el embarazo gemelar. La clínica de un útero que no se contrae nos apoya
en nuestra sospecha, y lo único que tenemos que descartar es la presencia de una rotura uterina
como causa de atonía uterina. Para ello, se revisaría la cavidad uterina y, si no existe solución de
continuidad, comenzaría el tratamiento de la atonía con masaje uterino, fármacos uterotónicos,
taponamiento uterino… Si las medidas conservadoras fracasan, la histerectomía sería el
tratamiento definitivo.(R3)

20. ¿A qué edad coge el lactante una bolita con ayuda del movimiento de pinza?

 
 
 
 
1. 1. A los 10 meses.
2. 2. A los 6 meses.
3. 3. A los 5 meses.
4. 4. A los 15 meses.
Gráfico de respuestas
Comentario

Pregunta memorística sin demasiada importancia sobre un aspecto muy concreto del desarrollo
infantil. Recordemos que en este tipo de preguntas, la respuesta correcta suele ser una de las
respuestas intermedias: en este caso, 10 meses.(R1)

21. Paciente de 30 años que acude "muerto de miedo" porque su padre ha fallecido
recientemente de un adenocarcinoma de colon y le han comentado que es hereditario.
Entre sus familiares cercanos, también un abuelo ha padecido esta enfermedad y a su tía
le acaban de diagnosticar un adenocarcinoma de endometrio. Usted le recomienda:

Que se tranquilice; aunque existe un condicionante genético, todavía no está en la edad de


1. 1.
riesgo.
2. 2. Colectomía profiláctica debido a su alto riesgo.
3. 3. Colonoscopia completa.
4. 4. Colonoscopia anual a partir de los 50 años.
Gráfico de respuestas
Comentario

El cáncer de colon tiene un componente hereditario incluso cuando aparece de forma esporádica…
Pero en esta ocasión puede que sea algo más. Observe que tiene varios familiares con el mismo
problema y una tía padece un cáncer de endometrio, por lo que posiblemente se trate de una
persona con síndrome de Lynch tipo II. En estos casos, se recomienda realizar colonoscopias
periódicas, a pesar de la juventud del paciente: a partir de los 25 años, o tres años antes del debut
del familiar más joven afecto.(R3)

22. Señale la falsa en relación al diagnóstico de hemofilia:

1. 1. Las hemofilias A y B son clínicamente indistinguibles.


2. 2. El tiempo de tromboplastina parcial activada se encuentra alargado.
3. 3. Es habitual encontrar un tiempo de hemorragia alargado.
Puede hacerse un diagnóstico prenatal por análisis de los fragmentos de restricción de
4. 4.
longitud polimórfica.
Gráfico de respuestas
Comentario
El tema de las hemofilias como tal no es muy preguntado en el MIR pero es importante conocerlas
porque entran en el diagnóstico diferencial de los trastornos de la coagulación que se basa
fundamentalmente en las pruebas de laboratorio. De tal manera que ante una clínica de
hemorragia lo primero que hacer es una historia clínica y exploración física. Si en éstas
encontramos datos que orientan a alteraciones de la hemostasia hacemos un tiempo de
hemorragia; si éste está prologado hay que determinar el número de plaquetas, porque la causa
más frecuente es la trombopenia y si es normal hay que sospechar una trombocitopatía
(Enfermedad de von Willebrand). Si el tiempo de hemorragia es normal, determinamos el TP y el
TTPA. Si TP alargado y TTPA normal, valorar ingesta de dicumarínicos; si TP alargado y TTPA
alargado valorar CID o hepatopatía grave; si TP normal y TTPA alargado, descartar utilización de
heparina y si no es por ésta, cuantificar factores de coagulación por posible hemofilia. Por ello la

 
 
 
 
opción 4 es la incorrecta, ya que si resumimos en la hemofilia encontramos tiempo de hemorragia
normal, TP normal y TTPA alargado (opción 3 correcta).(R3)

23. La "fiebre de Haverhill" y el "sodoku" se conocen también bajo un mismo nombre:


Fiebre por mordedura de rata, y sus respectivos agentes etiológicos son:

1. 1. Streptobacilus moniliformis y Spirilum minor.


2. 2. Togavirus y reovirus.
3. 3. Legionella pneumophila y Legionella micdadei.
4. 4. Rickettsiae prowazeki y Rickettsiae typhi.
Gráfico de respuestas
Comentario

Esta pregunta es muy complicada y poco provechosa, ya que no es útil estudiar las infecciones de
partes blandas y sus agentes infecciosos. Así que si quiere aprender algo, aquí tiene estas tres
cosas que son más que suficientes: las infecciones por mordedura de rata están producidas por
Streptobacillus moniliformis y Spirilum minor (opción 1 correcta), con sus dos nombres propios (que
aparecen en el enunciado), tratándose ambas con penicilina. Las infecciones por mordedura de
rata entran al diagnóstico diferencial del exantema palmoplantar, junto con la sífilis secundaria y las
infecciones por rickettsias.(R1)

24.
Masculino de 63 años que presenta como FRCV el ser fumador severo y DM tipo 2 de
reciente diagnóstico. Historia cardiológica compatible con ángor de moderados
esfuerzos de un año de evolución. Consulta por intensa opresión en hemitórax derecho
de una hora de evolución que asocia disnea con crepitantes a la auscultación pulmonar.
En urgencias se realiza un ECG. ¿Qué le sugiere el ECG?

1. 1. Pericarditis aguda.
2. 2. Infarto anteroseptal y lateral agudo.
3. 3. Infarto anteroseptal evolucionado.
4. 4. Síndrome de Brugada.
Gráfico de respuestas
Comentario

 
 
 
 
Se observa en el ECG una elevación del segmento ST en todas las derivaciones precordiales, así
como en I y aVL, por lo que la respuesta correcta es la número 2.(R2)

25. ¿Presenta algún trastorno de la conducción intraventricular?

1. 1. Bloqueo de rama derecha.


2. 2. Bloqueo de rama derecha y hemibloqueo anterior izquierdo.
3. 3. Bloqueo completo de rama izquierda.
4. 4. Bloqueo trifascicular.
Gráfico de respuestas
Comentario
La presencia de BCRD (QRS≥120 ms con morfología rSR´ en V1 o en definitiva QRS
predominantemente positivo en V1) y HBAI (desviación del eje hacia la izquierda, rS en II, III, aVF y
qR en I y aVL) supone la aparición de un bloqueo bifascicular, que implica necrosis de estos dos
fascículos y por tanto importante afectación del tabique interventricular; indicativo todo ello de un
infarto extenso de mal pronóstico.(R2)

26. Una de las siguientes afirmaciones sobre la patogenia de la preeclampsia es FALSA;


¿cuál?:

El hematócrito aumenta con la gravedad y la duración de la hipertensión inducida por el


1. 1.
embarazo.
2. 2. El péptido natriurético auricular aumenta significativamente en la preeclampsia.
3. 3. Existe una disminución de la síntesis de tromboxano A2.
4. 4. La perfusión renal y la filtración glomerular están disminuidas.
Gráfico de respuestas
Comentario

Lo que ocurre en la preeclampsia es un aumento de la producción de endotelina y tromboxano, un


aumento de la sensibilidad vascular a la angiotesnina II y una disminución en la formación ed
agentes vasodilatadores (oxido nitrico y prostaciclinas).

Todas estas alteraciones provocan un aumento de las resistencias vasculares, mayor


agregabilidad plaquetaria, activación del sistema de la coagulaión y disfunción endotelial que se
traducen en los síntomas y signos de la enfermedad.(R3)

27. Hombre de 33 años que llega a urgencias posterior a sufrir un accidente. En la


evaluación inicial se detecta: rotura esplénica, hematoma perirrenal izquierdo,
neumotórax a tensión, fractura de 3ª vértebra dorsal con lesión medular incompleta y
fractura de fémur izquierdo. La primera lesión a tratar es:

1. 1. La rotura esplénica.
2. 2. El neumotórax a tensión.
3. 3. El traumatismo renal.
4. 4. La fractura de fémur.
Gráfico de respuestas
Comentario

Una pregunta muy sencilla, recuerde el protocolo ABCDE. A continuación, resumimos su


significado:

 
 
 
 
A: Airway (vía aérea).

B: Breathing (respiración).

C: Circulation (control del shock, de los puntos sangrantes activos…).

D: Disability (lesiones neurológicas).

E: Exposure (exposición). Consiste en la exposición completa del paciente, desvistiéndole y


dándole la vuelta, así como la prevención de la hipotermia.

En este caso, siguiendo este protocolo de actuación, la respuesta correcta sería la 2.(R2)

28. Which of the following is most likely related to exophthalmos?

1. 1. Intracranial hypertension.
2. 2. Leukemia.
3. 3. Liver cirrhosis.
4. 4. Subarachnoid hemorrhage.
Gráfico de respuestas
Comentario

Esta es una pregunta difícil cuya respuesta puede obtenerse por exclusión. La hemorragia
subaracnoidea puede producir un síndrome de hipertensión intracraneal que producirá un
papiledema. Finalmente, la leucemia puede ser responsable de un exoftalmos por infiltración
directa del compartimento ocular.(R2)

29. Señale la FALSA:

1. 1. El síndrome CREST es un tipo de esclerodermia cutánea difusa.


La esclerodermia cutánea limitada tiene mejor pronóstico que la esclerodermia cutánea
2. 2.
difusa.
La cirrosis biliar puede aparecer como complicación tras varios años de esclerodermia
3. 3.
cutánea limitada.
4. 4. La morfea es una forma de esclerodermia sin afectación visceral.
Gráfico de respuestas
Comentario
Debes tener una idea básica de las diferencias entre esclerodermia difusa y localizada, pues ha
caído en el MIR. 1) El síndrome CREST corresponde a la forma LOCALIZADA, y sus iniciales
esbozan su extensión: Calcinosis, Raynaud, alteración Esofágica, eSclerodactilia y
Telangiectasias. Dicha forma va "despacio", el Raynaud precede en años al cuadro completo, es
de extensión cutánea limitada, y no afecta apenas a vísceras, siendo la hipertensión pulmonar y la
cirrosis biliar sus principales causas de morbi- mortalidad. Su anticuerpo es el anticentrómero. 2)
La forma difusa va "rápido", el Raynaud precede en pocos meses al cuadro, que compromete
extensamente a las vísceras, con mal pronóstico de la fibrosis pulmonar y con crisis renales. Su
anticuerpo es el antitopoisomerasa o anti- Scl- 70. Existen además formas viscerales sin afectación
cutánea, o lo contrario y más frecuente, formas cutáneas sin afectación visceral, como es la morfea
en placas, la esclerodermia lineal "en coup de sabre", en gotas,?(R1)

 
 
 
 
30. Un infante de 10 meses que gozaba de buena salud, repentinamente llora por “cólico
abdominal”, con severos episodios que ocurren en un período de 3 horas, algunos
acompañados de vómitos. El niño parece muy enfermo. Al examen hay signos de íleo,
distensión, vómitos, taquicardia. El abdomen parece no ser doloroso. El diagnóstico más
probable es:

1. 1. Vólvulo.
2. 2. Apendicitis aguda.
3. 3. Diverticulitis.
4. 4. Ruptura esplénica.
Gráfico de respuestas
Comentario

Cuadro compatible con apendicitis. Recuerde que en niños, el cuadro típico no es parecido al de
los adultos, pero se puede obtener datos clave como cólicos abdominales severos, vómitos, íleo y
el paciente se obsera muy enfermo y le solicitan el diagnóstico más probable, respuesta 2
correcta.(R2)

31. Señale la FALSA respecto al síndrome de Gorlin:

1. 1. Herencia autosómica dominante.


2. 2. Carcinomas espinocelulares múltiples de inicio infantil.
3. 3. Depresiones puntiformes palmoplantares.
4. 4. Quistes odontógenos y anomalías esqueléticas.
Gráfico de respuestas
Comentario
Las lesiones cutáneas precancerosas las preguntan en el MIR con poca frecuencia.
Concretamente, el síndrome de Gorlin no ha aparecido aún, por lo que no has de preocuparte por
fallar esta pregunta. Este síndrome es un trastorno polimalformativo de herencia autosómica
dominante. Consiste en la aparición de múltiples carcinomas basocelulares a edades precoces y
sin relación con la exposición solar. Junto a esto, se observan anomalías craneofaciales, como
quistes odontogénicos, hipertelorismo y protrusión frontal, que dan al paciente una facies
característica, así como anomalías óseas y neurológicas y tendencia a desarrollar neoplasias
viscerales. Recuerda que el tumor típico al que predisponen es el basocelular, no el epidermoide,
porque probablemente el aspecto más preguntable es éste (respuesta 2 correcta).(R2)

32. A 45-year-old woman comes to her physician complaining of repeated episodes of


axiety, cold sweats, pallor, palpitations that happens early in the morning and disappear
after breakfast. Her past medical history is significant for hypertension, recurrent renal
colic and cholecystectomy for cholelithiasis 10 years ago. Furthermore, her mother died
being a young woman from an intestinal tumor. The laboratory studies are as follows:
Glu: 63 mg / dl, creatinine 0.79 mg / dl; urea 42 mg / dl, Na 138 mEq, K 3.7 mEq / l, calcium:
11 mg / dl, phosphorus 3.2 mg / dl, elevated PTH. Which of the following statements is
FALSE about the suspected syndrome?

1. 1. It's frequently associated to pituitary tumors.


2. 2. It's related to menin gene (Chromosome 11).
3. 3. 95% of patients have ganglioneuromatosis.
4. 4. The presentation of this patient is not the most frequently seen.
Gráfico de respuestas

 
 
 
 
Comentario

Los MEN no es un tema muy preguntado en el ENARM. Sospechamos una Neoplasia Endocrina
Múltiple tipo 1. La paciente presenta síntomas compatibles con hipoglucemias de ayuno, así como
una glucemia basal baja, compatibles con un posible insulinoma. Por otra parte, presenta una
calcemia elevada con PTH elevada, habiendo presentado clínica asociada a la hipercalcemia:
cólicos nefríticos de repetición, colelitiasis, HTA, sospechando Hiperparatiroidismo Primario. Por
otra parte su madre pudiera haber presentado un tumor enteropancreático.

La neoplasia endocrina múltiple tipo 1 (MEN 1) se hereda con carácter autosómico dominante. Es
producida por una mutación en el gen de la menina, localizado en una región del cromosoma 11.
Asocia hiperparatiroidismo (90%), tumores enteropancreáticos (70%), tumores hipofisarios (40%),
así como adenomas no funcionantes de suprarrenal. Por otra parte en el 85% de los casos asocian
angiofibromas faciales, en el 70% colagenomas y en el 30% lipomas.

La ganglioneuromatosis cutánea es típica del MEN 2B(respuesta 3 falsa).(R3)

33. En relación con el trastorno por déficit de atención, señalar la afirmación FALSA:

1. 1. Nunca persiste después de los 7 años.


2. 2. Los psicoestimuladores son terapéuticamente eficaces.
3. 3. Los fármacos antihistamínicos pueden disminuir la eficacia de los estimulantes.
4. 4. A menudo existe fracaso escolar.
Gráfico de respuestas
Comentario

Es una pregunta de dificultad media, directa, sobre el trastorno por déficit de atención. Quizás en
psiquiatría infantil lo que merece la pena es mirarse las enfermedades más prevalentes y
asociadas a la infancia. Es el trastorno psiquiátrico más prevalente y sobre todo aparece en niños
varones, y los síntomas continúan en la adolescencia en un 25% e incluso se pueden persistir en la
edad adulta. Ya sabemos que el tratamiento son los psicoestimulantes. Esto es para compensar el
déficit dopaminérgico y la alteración noradrenérgica que se ha visto que pueden tener. El uso de
antihistamínicos pueden disminuir la eficacia del tratamiento al interactuar con estos
neurotrasmisores. Imagine a un niño con un niño con temperamento difícil, que no acaba de
terminar las tareas, que sufre muchos accidentes, probablemente está constantemente
moviéndose, corre o trepa y se cae, que se comporta mal en el colegio, probablemente sea
respondón, y que tiene cierto deterioro en su medio (la escuela). Estos son síntomas diagnósticos
del TDHA que según la DSMIV deben de estar presentes durante al menos 6 meses.(R1)

34. Una de las siguientes afirmaciones NO es correcta respecto a la trombastenia de


Glanzmann:

1. 1. Se hereda de forma autosómica recesiva.


2. 2. Las plaquetas no se agregan cuando se estimulan con ADP.
3. 3. Las plaquetas son de tamaño grande.
4. 4. Está alterada la unión de la plaqueta al fibrinógeno.
Gráfico de respuestas
Comentario
Dentro de las alteraciones plaquetarias, uno de los temas preguntados es el de las
trombocitopatías: alteración de la función plaquetarias cuando el tiempo de hemorragia se
encuentra prolongado y el número de plaquetas es normal. Entre ellas podemos encontrar: la

 
 
 
 
trombastenia de Glanzman(AR- opción 1 correcta), que consiste en un déficit de GpIIb/Ia (receptor
del fibrinógeno- opción 5 correcta) necesario para la agregación plaquetaria y que cursa con
plaquetas normales (opción 3 incorrecta); la enfermedad de Bernard- Soulier (AR), que consiste en
un déficit de GpIb (receptor para el FVW sintetizado por el endotelio) necesaria para la adhesión
plaquetaria al endotelio y que presenta plaquetas gigantes; la enfermedad de von Willebrand, que
se debe a una alteración cuantitativa y/o cualitativa del FVW y por tanto cursa también con
alteración de la adhesión plaquetaria. Estos dos tipos de alteraciones las podemos diferenciar
mediante el test de ristocetina que valora la adhesión y está alterada en Bernard- Soulier y en Von-
Willebrand, la diferencia radica en que en la primera no se corrige tras administrar plasma,
mientras que en la segunda sí. Este test es normal en la trombastenia de Glanzman (opción 4
correcta) donde encontramos alterado el test de agregación con ADP (opción 2 correcta),
adrenalina o tromboxano.(R3)

35. Recién nacido de 36 horas de vida que presenta llanto débil, hipotonía y convulsiones.
Tiene una ligera macrocefalia, con microoftalmía y coriorretinitis bilateral. Ya desde el
nacimiento presentaba hepatoesplenomegalia, y tras 12 horas comenzó con ictericia y
lesiones petequiales de predominio en flexuras. ¿Qué hallazgo esperaría encontrar en el
estudio de imagen cerebral?

1. 1. Calcificaciones periventriculares.
2. 2. Hipoplasia cerebelosa.
3. 3. Edema cerebral difuso.
4. 4. Calcificaciones dispersas.
Gráfico de respuestas
Comentario

Cuando nos hablan de macrocefalia, debe considerar una posible toxoplasmosis cerebral,
diagnóstico que además es compatible con la coriorretinitis y la hepatosplenomegalia que nos
describen.

Desde el punto de vista de imagen cerebral, nos encontraríamos con calcificaciones dispersas (RC
4). No se confunda con la opción 1... La infección que se asocia con calcificaciones
periventriculares es el CMV. Nemotecnia: Coriorretinitis, Microcefalia, calcificaciones
periVentriculares.(R4)

36. Paciente de 20 años de edad, con diagnostico clínico de plastrón apendicular, con
leucocitosis y desviación a la izquierda. ¿Cuál es la conducta a seguir?

1. 1. Antibioticoterapia.
2. 2. Drenaje percutáneo.
3. 3. Cirugía de emergencia.
4. 4. Hidratación y analgesia.
Gráfico de respuestas
Comentario

En casos de plastrón apendicular, lo primero que se realizará es la administración de


antibioticoterpia y posteriormente con el paso de los días cirugía. Respuesta 1 correcta.(R1)

37. Varón de 24 años que acude a urgencias por disfagia súbita mientras comía carne y
sialorrea. En la endoscopia practicada en urgencias tras extracción del bolo cárnico

 
 
 
 
impactado en esófago medio se observa una mucosa normal. Se toma una biopsia que
únicamente demuestra la existencia de 45 eosinófilos por campo en la mucosa. La
analítica era la siguiente: Hb 13.6 g/dl, 11.300 leucocitos/mm3 con neutrófilos 54%,
linfocitos 25%, monocitos 13%, eosinófilos 2 %, plaquetas 165.000/mm3, creatinina 1.0,
urea 66, GOT 25, GPT 38, GGT 13, bilirrubina 1.1, sodio 147, potasio 4.4. ¿Cuál es el
diagnóstico más probable?

1. 1. Estenosis péptica
2. 2. Achalasia.
3. 3. Esofagitis eosinofílica.
4. 4. Esofagitis candidiásica.
Gráfico de respuestas
Comentario
La esofagitis eosinofílica es un trastorno caracterizado por la existencia de disfagia e incluso
impactaciones de repetición con una mucosa normal o un aspecto ligeramente traquealizado del
esófago y por tanto, el diagnóstico es histológico (marcada infiltración de la mucosa por
eosinófilos). La diferencia con el síndrome hipereosinofílico radica en que no tiene eosinofilia
plasmática marcada como sucede así en este último.(R3)

38. La causa más común de muerte debida a enfermedad de Kawasaki en la fase inicial
de la enfermedad es:

1. 1. Rotura de un aneurisma.
2. 2. Ictus.
3. 3. Infarto de miocardio.
4. 4. Miocarditis.
Gráfico de respuestas
Comentario

Las tres primeras respuestas son causas de muerte tardías, como consecuencia de los aneurismas
que quedan como secuelas en los casos, sobretodo no tratados, de Kawasaki. La miocarditis sería
la causa precoz.(R4)

39. Una niña de 12 años tiene una talla en el P3 y ha crecido 4 cm en el último año. En la
exploración física se observa ausencia de signos puberales y la radiografía de la muñeca
muestra una edad ósea de 11 años. Al interrogar a su madre, ésta nos cuenta que tuvo la
menarquia a los 15 años. La niña presentará con mayor probabilidad:

1. 1. Talla baja familiar.


2. 2. Hipotiroidismo subclínico.
3. 3. Retraso constitucional del crecimiento y pubertad.
4. 4. Hipogonadismo hipogonadotropo.
Gráfico de respuestas
Comentario
La pubertad retrasada no ha sido preguntada aún en el MIR. Se considera retardo puberal en las
niñas si a los 13 años no existe desarrollo mamario y en los varones si a los 14 años no presentan
crecimiento testicular. También es anormal si transcurren más de 4 a 4.5 años entre el primer signo
puberal y la menarquia, o si los varones tardan más de 5 años en alcanzar un completo desarrollo
sexual. Dentro de las causas de retraso puberal se encuentra el cuadro de retraso constitucional
del crecimiento y desarrollo, que se considera una variante de la normalidad. Es más frecuente en

 
 
 
 
varones y siempre existe historia de familiares con el mismo problema. Estos niños nacen con un
peso y talla normales y a los 6- 12 meses se enlentece el crecimiento, situándose en los
percentiles inferiores, en los que se estabilizan. La edad ósea se retrasa hasta 2,5 años. Los
signos puberales también se demoran hasta los 14 a 18 años en los niños y los 13,4 a 16 en las
niñas. El estirón puberal no acontecerá hasta después de que se inicie la pubertad, pero la mayoría
de estos chicos alcanzan, finalmente, una talla definitiva normal.(R3)

40. Niño de 2 años de vida que en el examen físico presenta aniridia y masa abdominal,
un diagnostico podría ser:

1. 1. Hemangioma hepático.
2. 2. Neuroblastoma.
3. 3. Teratoma.
4. 4. Tumor de Wilms.
Gráfico de respuestas
Comentario

El tumor de Willms es el tumor renal más frecuente. Es el segundo tumor abdominal maligno con
mayor prevalencia. Es importante su asociación con malformaciones congénitas: anomalias
genitourinarias, síndrome con gigantismo-hipertrofia, aniridia, síndrome de WARG y síndrome de
dennys Drash(R4)

41. ¿Cuál de los fármacos siguientes NO es útil en el tratamiento de la enfermedad de


Parkinson?:

1. 1. Anticolinérgicos.
2. 2. Agonistas dopaminérgicos.
3. 3. Neurolépticos.
4. 4. IMAOs.
Gráfico de respuestas
Comentario
Pregunta de sencilla sobre un tema importante en el MIR, como son los trastornos
extrapiramidales. La pregunta hace referencia al tratamiento de la enfermedad de Parkinson, tema
que debes saber bien. En general la enfermedad de parkinson idiopática se trata con L- Dopa
asociada a un inhibidor de la dopa decarboxilasa periférica (carbidopa, benserazida, entacapona)
siendo especialmente útil en la bradicinesia y la rigidez. La falta de respuesta habla a favor de un
parkinsonismo no idiopático( fármacos, PSP, OPCA...).A pesar de la utilización de inhibidores de la
dopadecarboxilasa que permiten bajar la dosis, casi todos los pacientes que inicialmente mejoran
pierden su respuesta a L- dopa a los 3- 8 años; apareciendo fenómenos on- off, wearing- off...Por
lo que en pacientes jóvenes con muchos síntomas se prefiere empezar con agonistas dopa (
pergolide, lisuride...) usando también estos cuando aparecen los efectos secundarios del
tratamiento con L- dopa, a fin de disminuir la dosis de esta.(opción 3 cierta). Es muy importante
recordar que el fármaco más efectivo para el tratamiento del temblor de la enfermedad de
Parkinson son los anticolinergicos( trihexifenidil, biperiden...) y son el tratamiento de elección en
pacientes jóvenes en los que predomina el temblor (opción 1 cierta), estando contraindicados en
pacientes ancianos por los efectos confusionales. Los IMAOs, se pueden utilizar en el tratamiento
de la demencia asociada a la enfermedad de Parkinson, además el deprenilo o selegilina (inhibidor
selectivo de la MAO- B) se usa en estadios precoces,por su efecto neuroprotector.(opción 5 cierta).
La amantadina, aunque débilmente, también es efectiva para controlar los síntomas de la
enfermedad de Parkinson. (opción 2 cierta). Los neurolépticos son antagonistas dopaminérgicos, y

 
 
 
 
no solo no valen para el tratamiento de la enfermedad de Parkinson, sino que además son causa
de parkinsonismo secundario. (opción 4 incorrecta).(R3)

42. Paciente de 32 años con test de embarazo positivo y amenorrea de 9 semanas.


Consulta por hiperemesis gravídica de 6 días de evolución y metrorragia escasa de 3
horas de evolución. En la exploración se comprueba sangrado escaso de cavidad uterina
y útero aumentado como gestación de 14 semanas. En el ultrasonido presenta imagen
intrauterina en “copos de nieve” y ausencia de saco gestacional intraútero. La
determinación de β-HCG es de 110,000 mUI/ml. Tras evacuación por succión y curetaje
se diagnostica de mola hidatiforme completa. En el seguimiento posterior, cuál de las
siguientes es CIERTA:

1. 1. Tratamiento con metotrexate y ácido folínico semanales en cualquier caso.


Determinación semanal de β-HCG durante un mes tras la normalidad de la misma.
2. 2.
Después determinación mensual durante 1 año.
3. 3. Realización de radiografía simple de tórax cada 3 meses durante 1 año.
4. 4. Determinación trimestral de β-HCG, tras la normalidad de la misma, durante 1 año.
Gráfico de respuestas
Comentario

En este caso, se trata de una pregunta de dificultad media sobre un tema prioritario de Obstetricia:
las hemorragias del primer trimestre. Lo más importante es saber hacer el diagnóstico diferencial
entre las tres causas siguientes: aborto, embarazo ectópico y mola hidatiforme.

No obstante, en esta pregunta también es necesario conocer el algoritmo diagnóstico-terapéutico.

Ante una mola hidatiforme, lo primero es realizar un legrado por succión-aspiración, o una
histerectomía en mayores de 40 años sin deseo de tener hijos. Después, hay que realizar un
seguimiento mediante determinación semanal de b-hCG.

Tras varias determinaciones normales (un mes), se pasa a realizarlas mensualmente durante un
año para poder establecer la curación (respuesta 2 correcta).

Durante este tiempo, la paciente debe tomar medidas anticonceptivas para evitar que un posible
embarazo falsee los resultados del análisis de b-hCG.

Sobre el resto de opciones, el estudio de extensión y el tratamiento con metrotexato (o


poliquimioterapia) se realizan si la b-hCG está aumentada durante más de 8 semanas. Además:

•   Respuesta 1: "en cualquier caso" ya debería hacernos pensar que es falsa. El metotrexato
se usa cuando es una mola localizada persistente, o bien una mola diseminada sin factores
de mal pronóstico (poliQT si mal pronóstico).

•   Respuesta 3: TC o Rx cada tres meses durante un año debería hacernos pensar que es
excesivo, con un exceso de irradiación para la paciente, cuando disponemos de la b-hCG.

(R2)

Tratamiento de la enfermedad trofoblástica

 
 
 
 

43. Indique cuál de las siguientes situaciones NO produce falso positivo en el test del
sudor:

1. 1. Hipoparatiroidismo.
2. 2. Edema hipoproteinémico.
3. 3. Insuficiencia suprarrenal no tratada.
4. 4. Displasia ectodérmica.

 
 
 
 
Gráfico de respuestas
Comentario

Esta pregunta del diagnostico de la fibrosis quística es difícil y sin mucha relevancia. Lo que si es
importante es tener claros los criterios diagnósticos de fibrosis quística, las indicaciones del test del
sudor y en qué consiste.

En ciertas situaciones bien definidas se producen falsos negativos como en caso de edema, y
falsos positivos: insuficiencia suprarrenal, hipotiroidismo, hipopituitarismo, diabetes insípida
nefrogénica, displasia ectodérmica, malnutrición y enfermedades por deposito como
mucopolisacariosis, glucogenosis tipo I.

Una mala técnica puede ser causa de falsos negativos o positivos.

Algunos pacientes pueden presentar síntomas típicos pero leves con niveles de Cl normales
fibrosis quística atípica.

Recuerda que los epitelios derivan de la hoja ectodérmica y los principales órganos afectados en la
FQ son los epitelios.(R2)

44. Respecto a la menopausia, la afirmación CORRECTA es:

1. 1. Lo más eficaz es el estradiol oral.


Los suplementos de calcio son fundamentales, ya que se ha demostrado que reducen
2. 2.
considerablemente el índice de fracturas de cadera.
3. 3. Si hay contraindicación para los estrógenos, debemos al menos administrar progestágenos.
4. 4. Si aparece tensión mamaria, debemos bajar la dosis de estrógenos.
Gráfico de respuestas
Comentario

La tensión y el dolor mamario son síntomas derivados de un hiperestrogenismo, como también


puede ocurrir en la mastopatía fibroquística. Por tanto, en las pacientes con terapia hormonal
sustitutiva (THS), habría que disminuir la dosis de estas hormonas.

Respecto a la opción 1, conviene decir que el estradiol es igualmente eficaz independientemente


de la vía. Lo que puede variar en función de ésta son los efectos secundarios.

En pacientes histerectomizadas, no es necesario añadir progestágenos, ya que no existiría riesgo


de cáncer de endometrio, como sucede en otros casos. Por otra parte, carece de sentido
administrar progestágenos solos. Los síntomas climatéricos dependen del déficit estrogénico. La
razón para administrar progestágenos es para evitar el aumento de riesgo de cáncer de
endometrio.

En cuanto al calcio, los suplementos no son necesarios si en la dieta hay cantidad suficiente. En
caso de administrarse calcio, se asocia a vitamina D. Los suplementos de calcio sí que han
demostrado una menor pérdida en la densidad mineral ósea… Pero no alteran la incidencia de la
fractura de cadera.(R4)

45. El tipo de esquizofrenia caracterizado por la presencia de delirios de persecución o


referencia bastante sistematizados es:

 
 
 
 
1. 1. Esquizofrenia paranoide.
2. 2. Esquizofrenia catatónica.
3. 3. Esquizofrenia residual.
4. 4. Trastorno esquizotípico.
Gráfico de respuestas
Comentario
Repasemos los síntomas básicos de cada una de estas formas. En la esquizofrenia paranoide
predominan los síntomas psicóticos (alucinaciones y delirios), con un nivel de sistematización más
alto que en otras formas de esquizofrenia. En la esquizofrenia hebefrénica predominan los
síntomas desorganizados (en el pensamiento: disgregación; en la afectividad: reacciones
emocionales paradójicas; en el comportamiento: absurdo, pueril, impredecible). En la esquizofrenia
catatónica predominan los síntomas motores (estupor cataléptico, agitación, estereotipias,
manierismos). En la esquizofrenia residual predominan los síntomas negativos y en el trastorno
esquizotípico (o esquizofrenia latente) los síntomas no alcanzan la severidad de los anteriores pero
sí se les aproximan (lenguaje y pensamiento idiosincrásicos, frecuentes ilusiones, intereses y
comportamientos peculiares).(R1)

46. El crecimiento promedio de la cabeza durante el primer año de vida es:

1. 1. 5 cm.
2. 2. 12 cm.
3. 3. 15 cm.
4. 4. 20 cm.
Gráfico de respuestas
Comentario

En relación al perímetro cefálico simplemente debes recordar que es de 35 cm al nacimiento y es


mayor que el perímetro torácico. Al final del primer año se igualan y después el perímetro torácico
es mayor. El aumento del perímetro cefálico durante el primer año es de 1cm/mes (los 3 primeros
meses más rápido y luego más lento) y 10 cm durante el resto de la vida.(R2)

47. Acude a su consulta una joven de 20 años con una lesión hiperpigmentada en la
espalda, de 5 cms. de diámetro, más estrecha por un extremo que por otro y bordes
irregulares. Dice que toda la vida ha tenido un "lunar" ahí, pero que no era tan grande,
aunque tampoco lo había observado mucho dada su localización . ¿Cuál sería su actitud
ante esta paciente?:

Biopsia excisional de la lesión completa, para realizar un correcto estudio


1. 1.
anatomopatológico.
2. 2. Curetaje de la lesión estudiando anatomopatológicamente la muestra obtenida.
3. 3. Excisión quirúrgica hasta la fascia con márgenes de seguridad de 1 cm.
4. 4. Seguimiento regular de la lesión, que casi con seguridad es un caso de nevus displásico.
Gráfico de respuestas
Comentario
El melanoma es un tema de cierta importancia para el examen MIR. Debes conocer los signos de
alarma que deben hacerte sospechar que un nevus previo haya podido degenerar hacia
melanoma. Estos signos puedes recordarlos mediante las letras ABCD: Asimetría, Bordes
irregulares, Coloración heterogénea y Diámetro de más de 5 mm. En el caso que nos comentan, al
menos hay dos signos sospechosos: el tamaño y la asimetría. En este caso, se debe extirpar la
lesión completa (biopsia excisional) y proceder a un estudio histológico de toda ella, para
comprobar si se trata o no de un melanoma o simplemente era un nevus. En caso de que resulte

 
 
 
 
ser un melanoma, puede ser necesario ampliar márgenes de resección (1cm de margen o quizá
incluso 2 cm, dependiendo de si el índice de Breslow es mayor o menor de 1). Por tanto, la
respuesta correcta es la 1.(R1)

48. ¿Cuál de las siguientes enfermedades es un defecto primario de la capacidad


bactericida?

1. 1. Enfermedad granulomatosa crónica.


2. 2. Sarcoidosis.
3. 3. Sd. de Di George.
4. 4. Inmunodeficiencia combinada severa.
Gráfico de respuestas
Comentario

La enfermedad granulomatosa crónica se debe a mutaciones en los genes de la enzima NADPH


oxidasa, responsable de la formación del anión superóxido, fundamental para la eliminación de las
bacterias catalasa positivas.(R1)

49. El medicamento de primera elección como terapia de rescate en crisis asmática es:

1. 1. Cromoglicato.
2. 2. Corticoide inhalatorio.
3. 3. Teofilina.
4. 4. Beta 2 agonista de acción corta.
Gráfico de respuestas
Comentario

Respuesta muy sencilla, que no debe fallar. En casos de crisis asmática, el medicamento de
primera elección es el salbutamol que es un beta2 agonista de acción corta, respuesta 4
correcta.(R4)

50. Señale la FALSA, en relación a la apendicitis en mujeres embarazadas:

1. 1. Es más frecuente en los dos primeros trimestres.


2. 2. Ante la sospecha debe realizarse intervención quirúrgica.
3. 3. Su incidencia es mayor que en no embarazadas.
4. 4. Son frecuentes síntomas atípicos, como dolor menos agudo.
Gráfico de respuestas
Comentario

De cara al ENARM acerca de la apendicitis aguda, además del cuadro clínico, debe recordar sus
peculiaridades en ancianos, niños y embarazadas. Durante el embarazo no aumenta la frecuencia
de apendicitis (opción 3 falsa) pero es la urgencia quirúrgica más frecuente en este grupo. La
mayoría se presenta en el segundo trimestre del embarazo (opción 1) y su diagnóstico es difícil, ya
que síntomas como el dolor y las nauseas son típicas del embarazo y según progresa este el
apéndice se localiza en una posición más alta y lateral (opción 4). Recuerde que el tratamiento de
una apendicitis aguda siempre debe ser quirúrgico (opción 2)(R3)

51. De las siguientes familias de virus, tan sólo una tiene ADN como ácido nucleico.
Señálela:

 
 
 
 
1. 1. Retrovirus.
2. 2. Herpesvirus.
3. 3. Ortomyxovirus.
4. 4. Arenavirus.
Gráfico de respuestas
Comentario
El saber qué virus son de ADN y cuáles de ARN puede sernos útil a lo hora de responder
preguntas MIR. Existe una nemotecnia con la palabra anglosajona Happy (feliz) que resume las
familias virales que poseen ADN. HHAPPPy: Herpes, Hepadna (HVB), Adeno, Papova, Parvo, Pox.
El resto por tanto serán ARN (recuerda que los virus poseen un solo tipo de ácido nucléico). La
respuesta correcta por tanto es la 2.(R2)

52. Un paciente de cincuenta años con AR en tratamiento con AINE presenta dolores
moderados en muñecas y rodillas. En la radiografía de ambas manos hay osteoporosis
yuxtaarticular y erosiones en tercera, cuarta y quinta articulaciones metacarpofalángicas
de ambas manos. ¿Cuál es el tratamiento de elección?

1. 1. Aumentar la dosis de AINE.


2. 2. Mantener la misma dosis de AINE y revisar en un mes.
3. 3. Mantener AINE y añadir un fármaco modificador de la enfermedad
4. 4. Añadir infliximab.
Gráfico de respuestas
Comentario

La primera medida en el manejo de la artritis reumatoide es la administración de AINEs +/-


corticoides a dosis bajas para controlar los síntomas de la enfermedad. Cuando, como este caso,
no es suficiente para controlar totalmente los síntomas, se debe instaurar lo antes posible un
tratamiento modificador de la enfermedad (metotrexate fundamentalmente). En los casos en que
con estas medidas la enfermedad siga mostrando actividad, habría que plantearse otras medidas
como la administración de leflunomida o recientemente la instauración de tratamiento anti- TNF. En
el caso que nos presentan la única duda es si añadir (no sustituir) corticoides a los AINEs. Dado
que los síntomas son descritos como moderados, parece que se puede evitar. Además la opción
no se encuentra entre las respuestas.(R3)

53. An 18-month-old male is brought to the ED by his parents for presenting jelly-like
stoolfor the past 2 days. The child does not appear to be in acute distress at rest, although
his parents say that he seems to sleep more and looks paler than normal over the past 24
hours. He has not vomited and continues to feed regularly. Prior to the onset of the
current symptoms, he had good appetite and was a "healthy and active baby". Vital signs
and physical examination are normal. What is the most likely diagnosis?

1. 1. Cystic fibrosis
2. 2. Intussusception
3. 3. Pyloric stenosis
4. 4. Meckel's diverticulum
Gráfico de respuestas
Comentario
Meckel's diverticulum. The child in this case suffers from Meckel’s diverticulum. This diverticulum is
a remnant of tissue from the prenatal development of the digestive tract and is located in the distal
ileum. This structure is present in 2% of general population but is asymptomatic in the majority of
patients.The most common symptom is painless rectal bleeding, followed by intestinal obstruction,

 
 
 
 
volvulus and intussusception. Children with intussusception and bowel volvulus have a worse
general state than the child in this case, with abdominal distention and intense pain, and
vomiting.(R4)

54. Se exponen 5 patologías con sus localizaciones habituales. Señale la FALSA:

1. 1. Embarazo ectópico tubárico.


2. 2. Endometrioma tubárico.
3. 3. Gonococia endocervical.
4. 4. Chancro sifilítico exocervical.
Gráfico de respuestas
Comentario

Esta pregunta es muy sencilla sobre conocimientos básicos y no debe tener problemas al
contestarla. La localización más frecuente de embarazo ectópico es la porción ampular de la
trompa.

El endometrioma es un tumor ovárico de estirpe epitelial. Recuerde que la vulvovaginitis mas


sintomática es la producida por cándida y que no modifica el pH ácido de la vagina.

La causa mas frecuente de enfermedad inflamatoria pélvica es la Chlamydia y le sigue el


gononoco. Ambos producen infección ascendente del endocervix y tracto superior. No te olvides de
que el chancro sifilítico es duro indoloro y bilateral.(R2)

55. Señale la respuesta FALSA respecto al desarrollo del niño durante el primer año de
vida:

1. 1. El RN puede perder hasta un 10% de su peso en la primera semana de vida.


2. 2. El RN suele ganar 30 g/día durante el primer mes.
3. 3. Alrededor de los dos meses aparece la sonrisa social.
4. 4. El reflejo de Moro desaparece aproximadamente a los 6 meses.
Gráfico de respuestas
Comentario

Lo más importante de esta pregunta es que recuerde que el reflejo de Moro (la extensión súbita de
la cabeza provoca un movimiento de abrazo) desaparece en torno a los 3 meses.

El RN puede perder hasta un 10% de su peso en la primera semana de vida en relación a la


distinta proporción de agua corporal respecto al adulto y, así como por una ingesta escasa. Una
perdida >15 % se considera patológica.

A las 2 semanas deben tener o superar el peso al nacimiento.

El aumento de peso durante el primer año de vida suele ser entre 15 y 30 g/día, siendo el aumento
mayor en los primeros meses.(R4)

56. Paciente de 20 años que acude a la consulta por elevación leve de bilirrubina sin otros
hallazgos. Está asintomática y sólo refiere cierta coloración amarillenta de los ojos
durante episodios catarrales. ¿Qué prueba realizaría para su diagnóstico?:

 
 
 
 
1. 1. Medición de bilirrubina antes y después de 24 horas de ayuno.
2. 2. Biopsia hepática.
3. 3. Determinación de ceruloplasmina, cobre sérico y cobre en orina de 24 horas.
4. 4. Estudio del metabolismo férrico.
Gráfico de respuestas
Comentario
El paciente presenta un síndrome de Gilbert caracterizado por un aumento de la bilirrubina a
expensas de la fracción indirecta por defecto en la conjugación de la bilirrubina debido a una
alteración cualitativa en la UDP- glucuroniltransferasa. Aunque no debemos olvidar que puede
haber una alteración en la excreción de bilirrubina y un cierto grado de hemólisis oculta. Su patrón
de herencia es autosómico dominante siendo la causa más frecuente de ictericia metabólica
constitucional (hasta el 3% de la población). Se manifiesta con elevación de la bilirrubina cuando el
paciente está sometido a algún tipo de estrés físico o psíquico. De forma que en el test del ayuno
(someter al paciente a una dieta hipocalórica) provocará una elevación de la bilirrubina (menor de 5
mg/dl) y esa será por tanto, la prueba que deberá realizarse.(R1)

57. La asfixia perinatal se caracteriza por:

1. 1. Hiperoxia, isquemia y acidosis.


2. 2. Hipoxia, isquemia y alcalosis.
3. 3. Hipoxia, hipercapnea y acidosis.
4. 4. Hipoxia, hipocapnea y alcalosis.
Gráfico de respuestas
Comentario

Si es una asfixia es lógico que se produzca una hipoxia, el recien nacido deja de recibir oxígeno,
tambien retiene hidróxodo de carbono lo que produce una acidosis.(R3)

58. Recién nacido de 1,800 g, 45 cm de longitud y 32 cm de perímetro cefálico, sin


evidencias de anomalías físicas. ¿Cuál podría ser la complicación más probable en las
siguientes 12 horas?:

1. 1. Septicemia.
2. 2. Hiponatremia.
3. 3. Episodios apneicos.
4. 4. Hipoglicemia.
Gráfico de respuestas
Comentario

Es un caso en el que describen un neonato con retraso en el crecimiento intrauterino. La


morbimortalidad en estos pacientes suele estar dada por emaciación de partes blandas, circulación
fetal persistente, aspiración meconial, encefalopatía hipoxicoisquemica y alteraciones metabólicas
como: hipoglicemia, hipocalcemia, hiperviscocidad (policitemia) e hipotermia. Respueta 4
correcta.(R4)

59. A 5-year-old girl is brought to the emergency department presenting with generalized
edemas and decreasing urine output. Creatinine and BUN levels are at the upper limit of
normal. What should be done next?

1. 1. Observation at the emergency room

 
 
 
 
2. 2. Renal ultrasound
3. 3. Complete urinalysis
4. 4. Abdominal MRI
Gráfico de respuestas
Comentario
Complete urinalysis. The girl's symptoms are suggestive of primary nephropathy. The main
symptoms are edema and low urine output, both suggestive of nephrotic syndrome. The diagnosis
must be supported by urinalysis, to certify nephrotic-range proteinuria. The most common cause at
this age is minimal change disease. It has a good prognosis.(R3)

60. En el curso de un parto de nalgas, estando éstas están encajadas, la expulsión de


meconio se traduce en:

1. 1. Anoxia fetal.
2. 2. Compresión de cabeza.
3. 3. Compresión abdominal.
4. 4. Compresión y/o prolapso de cordón.
Gráfico de respuestas
Comentario

Pregunta difícil sobre el parto de nalgas. El meconio puede hacernos pensar que existe sufrimiento
fetal, pero su presencia no siempre tiene este significado. Por ejemplo, en las presentaciones
podálicas (como en este caso) es mucho más frecuente, sobre todo cuando las nalgas están
encajadas, ya que las piernas del feto comprimen el abdomen fetal en cada contracción, lo que
justifica este hallazgo (opción 3 correcta).(R3)

 
 
 
 

61. What is the most common complication


associated to the type of fracture shown in the picture?

1. 1. Musculocutaneous nerve palsy.


2. 2. Radial nerve palsy.
3. 3. Cubital nerve palsy.
4. 4. Circumflex nerve palsy.
Gráfico de respuestas
Comentario
En la imagen podemos observar una radiografía simple en proyección anteroposterior del codo. En
la región diafisaria y metafisaria distal del húmero se aprecia una fractura con un tercer fragmento.
El nervio que puede verse afectado con mayor frecuencia en este tipo de fracturas es el radial. La
lesión del nervio radial es la complicación aguda más frecuente en las fracturas de la diáfisis
humeral, y es más habitual en las fracturas oblicuas del tercio distal (fractura de Holstein-Lewis). La
lesión del nervio radial suele ser una neuroapraxia, que se recupera en unos tres a cuatro meses.
La lesión del radial, asociada a una fractura de húmero, no precisa cirugía salvo en los casos de
empeoramiento de sintomatología cuando se manipula la fractura (sugiere atrapamiento en el
foco), o en las fracturas abiertas.(R2)

62. Niño de 10 meses con cuadro febril de 3 días de duración, sin otra sintomatología
acompañante salvo irritabilidad con los periodos de hipertemia. El cuarto día presenta
aparición de exantema en tronco y desaparición de la fiebre. Respecto al cuadro clínico
citado, ¿cual de las siguientes afirmaciones es verdadera?

 
 
 
 
1. 1. El diagnóstico más probable es una infección por virus del sarampión.
2. 2. El signo físico diagnóstico es la presencia de una amigdalitis exudativa.
3. 3. Se asocia a una infección por virus herpético humano tipo 6.
4. 4. La duración del exantema suele ser de 4 semanas.
Gráfico de respuestas
Comentario

Las enfermedades exantemáticas son un tema relativamente importante. Lo esencial es saber


hacer diagnóstico diferencial entre ellas. Este caso clínico presenta datos muy típicos, con lo que el
diagnóstico no ofrece duda.

Fiebre alta + exantema cuando ésta desaparece = EXANTEMA SÚBITO

Es muy típico de esta enfermedad que el exantema respete la cara y extremidades inferiores. El
agente causal es el HHV-6. También es característico de esta enfermedad la presencia de
leucocitosis con neutrofilia en las primeras 24 horas y su asociación con las convulsiones
febriles.(R3)

63.
Un paciente presenta el ECG que se muestra en la imagen nº 20. ¿En cuál de los
siguientes casos no está indicada la profilaxis antitrombótica?

1. 1. Mujer de 40 años con antecedentes de enfermedad tromboembólica.


2. 2. Varón de 76 años hipertenso y con antecedentes de ingresos hospitalarios por ICC.
3. 3. Mujer de 60 años con antecedentes de infarto de miocardio sin elevación del ST.
4. 4. Varón de 80 años con disfunción ventricular severa (FEV1 20%).
Gráfico de respuestas
Comentario

Pregunta importante para repasar las indicaciones de la anticoagulación oral crónica (ACO) en la
fibrilación auricular, que es el ritmo que nos muestra el ECG de la pregunta. Puede reconocerlo
porque n o existen ondas “p” y porque el ritmo es absolutamente irregular.

Los factores de riesgo de enfermedad embólica se dividen en alto y bajo grado:

 
 
 
 
- Los factores de alto riesgo embolígeno son: antecedentes de embolia o AIT, estenosis mitral y
prótesis valvulares.

- Los factores de riesgo moderado son: la HTA, diabetes mellitus, edad > de 75 años y FEV1
menor o igual al 35%.

Considerando los factores de riesgo anteriores, estaría indicada la ACO si coexisten dos o más
factores de riesgo moderados o si existe un factor de alto riesgo. En el caso de un solo factor de
riesgo moderado se podría optar entre antiagregar o anticoagular. Teniendo en cuenta lo anterior,
en la opción 4 no estaría indicada la profilaxis antitrombótica porque no reúne ningún factor de
riesgo de los anteriormente mencionados.(R3)

64. De las siguientes infecciones de transmisión sexual cuáles producen úlcera genital:

1. 1. Candidiasis, herpes genital.


2. 2. Sífilis, herpes genital.
3. 3. Gonorrea, trichomoniasis.
4. 4. Clamidiasis, gonorrea.
Gráfico de respuestas
Comentario

La sífilis produce una úlcera genital, tambén llamada chancro, que es duro, no doloroso, y no
exudativo a diferencia del chancro blando causado por H. ducrey.

El virus herpes produce una serie de vesículas que debido al traumatismo se ulceran muy
fácilmente quedando en forma de úlceras pequeñas y agrupadas sobre una base eritematosa.

El resto de enfermedades de las otras opciones no se presentan en forma de úlcera genital.(R2)

65. ¿Cuáles son las tres exploraciones fundamentales que deben buscarse al valorar
inicialmente a un paciente de 50 años con un cuadro clínico sugerente de obstrucción
intestinal?:

1. 1. Búsqueda de hernias, existencia de laparotomías previas y tacto rectal.


2. 2. Percusión abdominal, BH y Rx abdomen.
3. 3. Inspección abdominal, electrolítos séricos y tacto rectal.
4. 4. Búsqueda de hernias, BH y Rx abdomen.
Gráfico de respuestas
Comentario

Pregunta importante sobre el abordaje ante la sospecha de una oclusión intestinal.

En un paciente con cuadro de obstrucción intestinal, tendrán que descartarse la presencia de


hernias, interrogar sobre laparotomías o cirugías previas y realizar un tacto rectal. Respuesta 1
correcta.(R1)

66. En el síndrome de ovario poliquístico, NO suele encontrarse:

1. 1. Hipertricosis.
2. 2. Hiperplasia endometrial.

 
 
 
 
3. 3. Quiste folicular.
4. 4. Hiperplasia medular ovárica.
Gráfico de respuestas
Comentario

Es importante que conozca el SOP pero esta pregunta es de una dificultad mayor que las que
suelen salir sobre este síndrome. Es una afección muy frecuente en que está aumentada la LH con
niveles de FSH bajos o inferiores a los normal, por lo que aumenta la relación LH/FSH. Hay un
aumento leve de andrógenos, aumento de la estrona (los andrógenos circulantes son convertidos a
estrona en la grasa periférica) y descenso del estradiol. La LH aumentada estimula la teca,
produciendo más andrógenos (también hay una sobreproducción suprarrenal) que va a provocar
obesidad, hirsutismo y anovulación. En la eco se ve que existen pequeños folículos subcorticales
(de 2-10 mm) y un aumento del estroma ovárico. La hiperplasia endometrial estará producida por el
efecto hormonal.(R3)

67. A 5-month-old infant is brought to the pediatrician's office for the study of delayed
psychomotor development. Physical examination shows spasticity and giant
hepatosplenomegaly. The macula appears cherry-like red upon funduscopic examination.
What is the most likely cause of this child's problem?

1. 1. Sphingolipidosis due to deficiency of sphingomyelinase


2. 2. Sphingolipidosis due to deficiency of galactosidase A
3. 3. Tay Sachs disease
4. 4. Friedrich's Ataxia
Gráfico de respuestas
Comentario
Sphingolipidosis due to deficiency of sphingomyelinase. Hepatosplenomegaly is suggestive of a
lysosomal storage disease, which is also related to the neurological deficit. In this case, the cherry
red macula helps us rule out deficit of glucocerebrosidase. Tay Sachs produces seizures but not
hepatosplenomegaly. Friedrich's ataxia is not a storage disease.(R1)

68. ¿Cuál de los siguientes es el signo más característico de la rubéola o sarampión


alemán?

1. 1. Enantema de Forcheimer.
2. 2. Manchas de Koplik.
3. 3. Lengua en fresa blanca.
4. 4. Adenopatías retroauriculares y occipitales.
Gráfico de respuestas
Comentario

El signo más característico de la rubeola es la presencia de adenopatías retroauriculares,


cervicales posteriores y posteroccipitales.
Las manchas de Forcheimer no son patognomónicas de la rubeola; las manchas de Koplik son
típicas del sarampión; la lengua en fresa blanca y las amídgalas cubiertas por un exudado
blancogrisáceoson típicas de la escarlatina.(R4)

69. Which of the following is FALSE regarding gastric lymphomas?

1. 1. Hodgkin lymphoma accounts for the majority of the cases.

 
 
 
 
In patients with non-Hodgkin lymphoma, stomach is the most commonly involved
2. 2.
extranodal site.
3. 3. Clinical presentation is similar to adenocarcinoma.
Typical endoscopic findings can be: multiple ulcers, thickened gastric folds or diffuse
4. 4.
infiltration.
Gráfico de respuestas
Comentario

Los linfomas gástricos son prácticamente siempre de tipo no Hodgkin. Su localización


extraganglionar más frecuente es la gástrica. Frecuentemente surgen sobre áreas de la mucosa
con metaplasia intestinal o gastritis crónica. La clínica puede ser similar a la del adenocarcinoma.
En la endoscopia, y el estudio radiológico, es característica la presencia de pliegues gástricos
engrosados (que también se pueden observar en el adenocarcinoma gástrico de tipo difuso). En
los linfomas de bajo grado puede haber respuesta, con curación, al tratamiento erradicador del
Helicobacter pylori, lo que les confiere un mejor pronóstico que al adenocarcinoma.(R1)

70. ¿Cuál es la única afirmación cierta para todas las bacterias de la familia
enterobacteriae?:

1. 1. Bacterias móviles.
2. 2. Metabolismo fermentativo.
3. 3. Producción de catalasa.
Todas las bacterias pertenecientes a este género son patógenos primarios del intestino
4. 4.
humano.
Gráfico de respuestas
Comentario
Puede sernos útil conocer de cara al MIR alguna característica de la microbiología de las
enterobacterias, como las que exponemos a continuación. Las enterobacterias son huéspedes
habituales del intestino humano. No obstante, dentro de esta familia existen bacterias patógenas
primarias ( Salmonella, Shigella, Edwarsiella, Citrobacter, Yersinia, algunas especies de E. Coli) y
patógenas oportunistas ( que aprovechan debilidad del huésped para producir enfermedad). Son
bacilos gramnegativos no esporulados, aerobios o anaerobios facultativos, oxidasa negativos,
siendo en su mayoría móviles por flagelos peritricos. Son poco exigentes nutricionalmente, y
poseen todas ellas una característica común: son fermentadoras de glucosa con producción de
ácido. Así, opción 2 correcta.(R2)

71. La presencia de la mutación genética de las células germinales BRCA-1 confiere a las
pacientes portadoras:

1. 1. Un mayor riesgo de presentar cáncer de mama exclusivamente.


2. 2. Un mayor riesgo de presentar cáncer de mama y de ovario.
Dado que es una mutación de transmisión autonómico recesiva el riesgo de presentar
3. 3.
cáncer de mama es menor del 10% a lo largo de la vida.
La mastectomía profiláctica de las pacientes portadoras de esta mutación sólo debe
4. 4. realizarse (si se indica) por encima de los 45 años (postmenopausia), ya que por encima de
esta edad existe la mayor incidencia de tumores de mama.
Gráfico de respuestas
Comentario

Una pregunta bastante sencilla, teniendo en cuenta que los genes BRCA-1 y BRCA-2 son muy
preguntados.

 
 
 
 
Estos dos genes se consideran responsables de más de la mitad de los casos de cáncer de mama
hereditario. Ambos han sido implicados en la génesis del cáncer de ovario, no solamente en el de
mama (respuesta correcta 2).

El riesgo de padecer cáncer de mama es de más del 50% en las mujeres portadoras, y no un 10%,
como se menciona en la respuesta 3. Lógicamente, cuando aparecen estos genes, el cáncer de
mama se comporta como más agresivo, con patrones histológicos más desfavorables (mayor
grado histológico de malignidad) y, por tanto, el pronóstico empeora.(R2)

72. A 28-year-old female patient, with a past medical history significant for IV drug abuse,
comes to the emergency department complaining of a 36-hour history of sudden pain and
swelling of her right knee, along with fever. Her temperature is 38,7ºC, blood pressure is
120/80 mm Hg, pulse is 90/min and respirations are 18/min. Physical examination reveals
redness, tenderness and swelling of her right knee. Chest auscultation reveals no
abnormal findings. Which of the following is the most important next step in
management?

1. 1. Plain X-ray of her right knee.


2. 2. Arthrocentesis.
3. 3. Blood studies including complete blood count.
4. 4. Nonsteroidal anti-inflammatory and empiric antibiotic therapy.
Gráfico de respuestas
Comentario
Arthrocentesis. An infected joint is an emergency that must be ruled out as soon as possible.
Furthermore, other entities may mimic that disease and the best way to ascertain a diagnosis is to
obtain a sample of the fluid inside the joint and perform some laboratory and microbiologic
studies.(R2)

73. En el tratamiento de la bronquiolitis aguda del lactante todo lo siguiente puede estar
indicado, EXCEPTO:

1. 1. Sulmetil-papaverina.
2. 2. Oxígeno frío humidificado.
3. 3. Ventilación mecánica.
4. 4. Colocar al niño en posición semiincorporada.
Gráfico de respuestas
Comentario

El tratamiento mas empleado es el sintomático.

En las formas leves y sin factores de riesgo puede realizarse tratamiento domiciliario con humedad,
posición semiincorporada, fisioterapia y aspiración de secreciones. Puede probarse el tratamiento
con broncodilatadores en aerosol como el salbutamol y la adrenalina aunque su utilidad es
discutida.

La ventilación mecánica puede ser necesaria en casos de insuficiencia respiratoria grave.

Los supositorios de sulmetil papaverina han sido usados de forma habitual en el tratamiento del
crup por su teórico, efecto beneficioso antitusígeno, sedante, antiemético y espasmolítico pero no
están indicados para la bronquiolitis.

 
 
 
 
La ribavirina, sólo puede considerarse en pacientes de alto riesgo como cardiópatas e
inmunodeprimidos.(R1)

74. La distinción entre fenómeno del alba y la hiperglucemia posthipoglucémica


(fenómeno de Somogyi) suele efectuarse:

1. 1. Utilizando un test de infusión rápida de glucosa.


2. 2. Determinando la glucosa en sangre a las 3 a.m.
3. 3. Observando glucemia tras pautar clorpropamida.
4. 4. Controlando glucemia antes y después de las comidas.
Gráfico de respuestas
Comentario

La distinción entre ambos fenómenos se realiza haciendo una determinación de glucemia a las 3
a.m. Si la glucosa a esta hora está BAJA será un fenómeno de Somogyi que consiste en la
elevación de la glucemia matutina por las hormonas contrarreguladoras en respuesta a una
hipoglucemia nocturna. En el fenómeno del alba lo que aparece se también una hiperglucemia
matutina por el efecto de la secreción nocturna de GH y el ritmo circadiano del cortisol. En este
caso la glucemia de las 3 a.m estará NORMAL O ELEVADA.(R2)

75. Don Tomás, de 85 años, es traído a urgencias por presentar hemiparesia y


hemihipoestesia derechas. Al intentar interrogarlo notamos que habla con dificultad,
pues tiene la boca desviada hacia la izquierda. ¿Cuál de las siguientes exploraciones NO
es básica para el estudio de este paciente?

1. 1. Medida de la tensión arterial.


2. 2. Electrocardiograma.
3. 3. TC cerebral.
4. 4. Angiografía cerebral.
Gráfico de respuestas
Comentario

Pregunta difícil de un tema muy importante para el MIR: las enfermedades cerebrovasculares
isquémicas.

El estudio básico de este paciente incluye:

- Medida de la tensión arterial.

- Auscultación de carótidas, en busca de lesiones arterioscleróticas.

- ECG, por la posible presencia de arritmias (fibrilación auricular) como causa de embolias.

- TAC cerebral, como prueba de elección en fase aguda, ya que nos permite distinguir entre
patología isquémica y hemorrágica.

La angiografía cerebral es muy útil ante una posible hemorragia subaracnoidea, o también para
localizar malformaciones vasculares… Pero el diagnóstico de sospecha, en este caso, es un ictus
isquémico, con lo que no estaría indicada.(R4)

 
 
 
 
76. La panarteritis nodosa afecta al riñón en un:

1. 1. 10%.
2. 2. 50%.
3. 3. 90%.
4. 4. 20%.
Gráfico de respuestas
Comentario
Esta pregunta es fácil de contestar, pese a ser un porcentaje, porque la PAN afecta a arterias de
mediano y pequeño calibre y es muy probable el daño renal por mecanismo isquémico. Como ves,
la afectación renal de la PAN ocurre en un 90% de los casos y la manifestación más frecuente es
la hipertensión vasculorrenal por hipoperfusión mantenida. También puede debutar con hematuria
y proteinuria. Otras formas menos frecuentes son: síndrome nefrítico, síndrome nefrótico y la
aparición de una GN extracapilar con aparición de oligoanuria.(R3)

77. ¿Cuál de las siguientes asociaciones del HLA-B27 con las espondiloartropatías es
FALSA?:

1. 1. Con las artritis reactivas del 70-80%.


2. 2. Con la Espondilitis Anquilosante del 50%.
3. 3. La población B-27 tiene riesgo de padecer una Esp. Anquilosante de un 2-5%.
4. 4. Con la Espondilitis de la A. Psoriásica del 50-60%.
Gráfico de respuestas
Comentario
Aunque es indudable la asociación del HLA B27 con las espondiloartropatias recuerda que la
mayoria de las personas que presentan HLA B27 (8% de la población general) no desarrollan
afortunadamente ninguna de estas enfermedades, aunque el riesgo de padecer EA se estima entre
el 2- 5% en los pacientes que presentan este hapllotipodel sistema HLA. Las demás
espondiloartropatías tambien se asocian al HLA especialmente cuando muestran afectación
sacroiliaca y del esqueleto axial. La espondilitis anquilosante se asocia en el 90 % de los casos y la
artritis reactiva en el 70- 80%. La artropatía psoriásica y la artritis que acompaña a la enfermedad
inflamatoria intestinal solo muestran una asociación con el B27 cuando producen afectación del
esqueleto axial y sacroileitis, especialmente si esta es bilateral.(R2)

78. ¿Cuál es la causa más frecuente de distrés respiratorio del adulto?:

1. 1. Politraumatismo.
2. 2. Politransfusiones.
3. 3. Broncoaspiración.
4. 4. Sepsis.
Gráfico de respuestas
Comentario

El síndrome de distrés respiratorio del adulto (SDRA) no es un tema muy preguntado en el


ENARM, pero esta pregunta es sencilla y no debería haber planteado dudas. El SDRA produce
una insuficiencia respiratoria aguda grave, refractaria a la oxigenoterapia, como consecuencia de
una lesión en la membrana alveolocapilar. Debido a ello, aumenta la permeabilidad de esta
membrana, de forma que los alveolos se llenan de líquido y se produce un edema agudo de
pulmón. La etiología de este síndrome es muy variada, pero la causa más frecuente es la sepsis
(opción 4 correcta). Puede aparecer también por traumatismos torácicos, infecciones pulmonares

 
 
 
 
diseminadas, broncoaspiraciones, semiahogamientos y enfermedades autoinmunes. Existen
fármacos que pueden producir un SDRA, como los opiáceos (recuerda el famoso "edema pulmonar
no cardiogénico" de los heroinómanos). Es importante que distinga el SDRA del edema pulmonar
de la insuficiencia cardiaca izquierda, desde el punto de vista conceptual y según la presión de
enclavamiento. En la insuficiencia cardiaca, se debe a un aumento de la presión hidrostática en los
capilares pulmonares, por el fracaso del corazón en su función de bomba (presión de
enclavamiento elevada). En cambio, en el SDRA es por una lesión en la membrana alveolocapilar
(presión de enclavamiento normal).(R4)

79. A 36-year-old Jewish woman complains of jaundice over the last four days. She does
not have any important event in her medical history, apart from being on
contraconceptives since one month ago. Laboratoty findings include: Total bilirubin 4.5
mg/dL, direct bilirubin 3.1 mg/dL, Alkaline phosphatase 71 U/L, AST 33 U/L, ALT 44 U/L.
Serologies are negative for every hepatotropic virus. Liver biopsy shows hepatocytes
containing dark pigment. What is the most likely diagnosis?

1. 1. Gilbert's syndrome
2. 2. Dubin-Johnson syndrome
3. 3. Criggler-Najjar syndrome
4. 4. Rotor syndrome
Gráfico de respuestas
Comentario
Dubin-Johnson syndrome. It is a benign inherited disorder of bilirubin metabolism. Clinically, it
presents with jaundice, usually non-pruritic, although it can also cause pain. It is charactetized by
normal liver enzymes and the deposition of a pigment that gives the liver a characteristic black
color. Bilirubin serum levels range between 2 and 5 mg/dL, but some cases elevate up to 25. Less
important is the fact that DJS affects individuals of all ethnic origins but is most common among
certain communities of Jews.(R2)

80. Un niño de 2 años presenta una reacción alérgica tras la picadura de un insecto, con
urticaria, angioedema, estridor laríngeo y broncoespasmo severo. Para administrarle
adrenalina, ¿qué vía NO debe utilizar?:

1. 1. Intratraqueal (si el paciente está intubado).


2. 2. Intraósea.
3. 3. Subcutánea.
4. 4. Intraarterial.
Gráfico de respuestas
Comentario

Esta pregunta sobre las vías de administración de la adrenalina no tiene ninguna relevancia .
Simplemente debes recordar que la droga de elección para el tratamiento de la anafilaxia es la
adrenalina. Produce un aumento de las resistencias sistémicas, relaja la musculatura bronquial,
mejora el angioedema y la urticaria.

La vía de administración más adecuada de la adrenalina es la vía intramuscular ya que, debido a


una mayor perfusión, se alcanza un pico de concentración plasmática más alto y con mayor
rapidez que por vía subcutánea. La localización de elección para la inyección es la cara anterior del
muslo donde se ha demostrado una mayor absorción en menos tiempo que en la región deltoidea.

 
 
 
 
Debido al riesgo de inducción de isquemia miocárdica, infarto o arritmia que supone la
administración de adrenalina, la vía endovenosa se reserva para situaciones límites, y se lleva a
cabo bajo monitorización cardíaca.

También se administra intracardiaca o endotraqueal en caso de extrema urgencia por parada


cardiorrespiratoria. La via intraósea es útil en caso de estado de paro cardiaco ya que la médula
ósea se comporta como un sistema venoso no colapsable.

Recuerde que la adrenalina en aerosol es el tratamiento más eficaz de la bronquiolitis. La


adrenalina vía intraarerial se utiliza para producir vasoconstricción y prolongar el efecto de los
anestésicos locales.(R4)

81. ¿A qué edad puede el niño realizar un juego en paralelo como logro madurativo?

1. 1. 12 meses
2. 2. 18 meses
3. 3. 24 meses
4. 4. 36 meses
Gráfico de respuestas
Comentario

Este tema es de vital importancia en el nacional, debes dominarlo, revise la siguiente tabla.(R4)

82. Ante un politraumatizado en aparente parada cardiorrespiratoria su primera medida


de entre las siguientes sería:

1. 1. Comprobar pulsos carotídeos.


2. 2. Comenzar masaje cardíaco.
3. 3. Comprobar la existencia de cuerpos extraños en la boca.
4. 4. Comprobar la existencia de hemorragias externas.
Gráfico de respuestas

 
 
 
 
Comentario

Esta pregunta es muy importante ya que el manejo del politraumatizado es un tema muy rentable
en el ENARM en los últimos años. Para valorar el orden de priorización ante un accidentado
utilizamos el acrónimo ABCDE. El primer paso ante un accidentado es mantener la vía aérea
permeable (airway), descartando la posible presencia de cuerpos extraños. El siguiente paso sería
asegurar la correcta ventilación, realizando en caso de escasos medios el boca a boca. El
siguiente paso sería controlar la circulación, hemorragias y TA.(R3)

83. Los consejos de los profesionales para estimular la lactancia materna deben ser
claros y bien documentados; señale un consejo ERRÓNEO:

Se debe conseguir un contacto materno-hijo inmediato después del parto con alimentación
1. 1.
por succión a demanda.
En la técnica de la toma es muy importante que se introduzca en la boca del niño, no sólo
2. 2.
el pezón, sino parte de la areola.
No debe utilizarse la doble pesada para saber lo que toma, ya que no crea más que
3. 3.
confusión en las madres.
Cuando pasen 3 ó 4 días convendría establecer un horario estricto de las tomas cada 3 ó 4
4. 4.
horas.
Gráfico de respuestas
Comentario

La respuesta errónea es la número 4, en establecer un horario estricto en las tomas, pasando 3 o 4


días, ya que deberá ser posteriormente.(R4)

84. La toxoplasmosis tiene como peculiaridad:

1. 1. Nunca es crónica.
2. 2. Pasa la placenta más facilmente cuanto mayor sea la edad gestacional.
3. 3. Es extracelular.
4. 4. La detección de IgM en el recién nacido no tiene importancia diagnóstica.
Gráfico de respuestas
Comentario
La toxoplasmosis pasa al feto con mas facilidad a medida que avanza la gestación, pero la
infección durante el primer trimestre es más grave. Cuando se contrae durante la gestación, la
enfermedad es transmitida al feto en casi la mitad de los casos. Es muy difícil su diagnóstico
anteparto salvo por cultivo de líquido amniótico o demostrando calcificaciones cerebrales,
hidrocefalia, etc. La detección de IgM en el recién nacido nos informa de infección intrauterina. Por
tanto la respuesta correcta es la nº 2.(R2)

85. Ingresa a urgencias paciente femenino de 28 años, por sufrir una herida
punzopenetrante a nivel del sexto espacio intercostal izquierdo, a nivel de la línea
medioclavicular. Presenta disnea severa, cianosis, ingurgitación yugular y ruidos
hidroaéreos en hemitórax izquierdo ¿Cuál es el diagnóstico más probable?

1. 1. Lesión diafragmática.
2. 2. Hemotórax mixto.
3. 3. Contusión pulmonar.
4. 4. Perforación.

 
 
 
 
Gráfico de respuestas
Comentario

Le hablan de una lesión en el sexto EII en la LMC. El paciente tiene disnea, IY y RUIDOS
HIDROAÉREOS EN EL HTX IZQ. El diagnóstico es la opción 1, lesión diafragmática.

Se debe hacer el diagnóstico diferencial con otras etructuras vecinas como el corazón y un
tamponade en el que es caraterístico el pulso paradójico.(R1)

86. NO parece aumentar el riesgo de padecer cáncer de cérvix:

1. 1. Obesidad.
2. 2. Tabaquismo.
3. 3. Promiscuidad del compañero sexual.
4. 4. Inicio precoz de las relaciones sexuales.
Gráfico de respuestas
Comentario

El cáncer de cérvix en un tema muy imortante para el ENARM y debe dominarlo. La obesidad es
un factor de riesgo para el cáncer de endometrio pero no para el cáncer de cérvix. Las demás
opciones sí son factores e riesgo para el cáncer de endometrio.(R1)

87. Hombre de 15 años de edad. Acude por aumento de volumen en región mamaria. ¿Qué
hallazgo clínico amerita mayor estudio?:

1. 1. Que afecte ambas mamas.


2. 2. Hiperplasia subareolar de las mamas.
3. 3. Hiperestesia mamaria.
4. 4. Pseudo ginecomastia.
Gráfico de respuestas
Comentario

En hombres con aumento de volumen mamario, tendrías que extender o el estudio de las mamas
si presentan hiperplasia subareolar Respuesta 2. Resto no sería necesario.(R2)

88. ¿Qué patrón es más sugestivo de un síndrome de ovario poliquístico?:

1. 1. FSH 150, LH 90, Estradiol 5.


2. 2. FSH 25, LH 105, Estradiol 435.
3. 3. FSH 10, LH 35, Estradiol 15.
4. 4. FSH 12, LH 10, Estradiol 35.
Gráfico de respuestas
Comentario

Esta pregunta es muy importante porque debe conocer perfectamente el síndrome de ovario
poliquístico y sus alteraciones hormonales. Es una afección muy frecuente en que está aumentada
la LH con niveles de FSH bajos o inferiores a los normal, por lo que aumenta la relación LH/FSH.
Hay un aumento leve de andrógenos, aumento de la estrona (los andrógenos circulantes son
convertidos a estrona en la grasa periférica) y descenso del estradiol. La LH aumentada estimula la
teca, produciendo más andrógenos (también hay una sobreproducción suprarrenal) que va a

 
 
 
 
provocar obesidad, hirsutismo y anovulación. La insulina estimula la actividad aromatasa en la
granulosa, convirtiendo los andrógenos de la teca en estrógenos, pero en las mujeres con SOP
hay insulinresitencia lo cual contribuye al aumento de andrógenos.(R3)

89. Una paciente con Ca vulvar, señale la sintomatología más común:

1. 1. Lesión ulcerativa exofítica en labio mayor


2. 2. Prurito vulvar
3. 3. Disuria
4. 4. Dispareunia
Gráfico de respuestas
Comentario

En esta pregunta, puede haber dudas entre las respuestas 1 y 2. Es cierto que el hallazgo más
común en la exploración es una lesión ulcerativa exofítica en el labio mayor; sin embargo, eso es
un signo (un hallazgo objetivo) , no un síntoma (una sensación subjetiva que el paciente refiere); y
nos preguntan el síntoma más común. Y el síntoma más común es el prurito vulvar (respuesta 2).

El típico caso de una viejita con mucho prurito vulvar.(R2)

90. Señale lo que cree que es correcto sobre el síndrome de Sjögren:

Sus características principales son un engrosamiento de las glándulas lacrimales y


1. 1.
parotídeas, simultáneo y generalmente unilateral.
Afecta con más frecuencia a la mujer en la edad media de la vida, sin aparecer en la
2. 2.
infancia.
3. 3. Lesiones cutáneas de vasculitis además de xerostomía y xeroftalmía.
4. 4. Ausencia de síntomas en un 40% de los pacientes con queratoconjuntivitis seca.
Gráfico de respuestas
Comentario
Pregunta de relativamente baja importancia de cara al MIR. Quizá lo más importante de esta
patología es recordar sus manifestaciones clínicas y la posible complicación con un linfoma. El
síndrome de Sjögren es una exocrinopatía autoinmune, en la que se produce una insuficiencia de
las glándulas exocrinas producida por la infiltración de éstas por lifocitos CD4. Las glándulas más
afectadas son las salivares (produciendo xerostomía) y las lacrimales (produciendo xeroftalmia) y
la afectación es bilateral. Además pueden aparecer manifestaciones extraglandulares articulares,
pulmonares, renales, neurológicas, vasculitis, etc. Es más frecuente en la mujer en la edad media
de la vida pero puede aparcer en hombres o en cualquier edad. Los anticuerpos anti- Ro y anti- La
son los característicos de esta enfermedad y aparecen en el 60% de los pacientes, siendo algo
menos frecuentes en la forma secundaria.(R3)

91. La complicación más frecuente de la rubéola congénita es:

1. 1. Sordera.
2. 2. Ductus persistente.
3. 3. Trombocitopenia.
4. 4. Microcefalia.
Gráfico de respuestas
Comentario

 
 
 
 
La rubéola congénita cursa clínicamente a través de una tríada (la de Gregg: hipoacusia
neurosensorial, alteraciones oculares muy diversas y cardiopatía) y lesiones óseas sin periostitis.
Es tanto más frecuente cuanto más precozmente se dé en el embarazo. La complicación más
frecuente es uno de los ítems de la tríada de Gregg: la hipoacusia neurosensorial.(R1)

92. RNPT de 13 días que comienza con distensión abdominal y evacuaciones con sangre
macroscópica. En la Rx de abdomen se objetiva neumatosis intestinal y patrón “en miga
de pan”. Señale lo CORRECTO:

1. 1. No es necesario suspender alimentación enteral.


2. 2. El hallazgo de gas en la vena porta en la Rx de abdomen es un signo de gravedad.
3. 3. Los antibióticos están contraindicados en el tratamiento de esta entidad.
El inicio precoz de alimentación enteral en recién nacidos con sufrimiento fetal agudo no
4. 4.
incrementa el riesgo de esta enfermedad.
Gráfico de respuestas
Comentario

La enterocolitis necrotizante afecta fundamentalmente a íleon distal y colon proximal. Suele


relacionarse con situaciones de hipoxia y bajo gasto. También se ha vinculado con un comienzo
precoz de la alimentación enteral (respuesta 4 falsa), y se ha visto que la lactancia materna tiene
un papel protector.

Clínicamente, debe sospecharla ante la presencia de distensión abdominal y evacuaciones


sanguinolentas. Existen varios signos radiológicos, pero el más sugestivo es laneumatosis
intestinal, que es diagnóstico de esta enfermedad. Por eso se lo ofrecen en el enunciado, para que
no quepa duda de qué diagnóstico debe sospechar. Por supuesto, signos como la presencia de
gas en la vena porta, o el neumoperitoneo, implicarían un pronóstico mucho peor.

El tratamiento consiste en dieta absoluta (respuesta 1 falsa), sonda nasogástrica y sueroterapia.


Debe administrase además antibioticoterapia (respuesta 3 falsa), cubriendo gérmenes anaerobios
y gramnegativos. En caso de perforación intestinal o refractariedad a antibióticos, se plantearía la
cirugía.(R2)

93. Sobre el pseudoquiste pancreático. Seleccione la CORRECTA.

1. 1. Su cavidad esta cubierto por epitelio columnar.


2. 2. La hemorragia intraquística es frecuente.
3. 3. Luego de 5 a 6 semanas generalmente requieren drenaje.
4. 4. El drenaje del quiste al árbol biliar es el tratamiento más adecuado.
Gráfico de respuestas
Comentario

Esta pregunta hace referencia a la "regla del 6" en el manejo del pseudoquiste pancreático. Según
esta regla se deben drenar los pseudoquistes con más de 6 cm o más de 6 semanas de
duración.Pero no se deje engañar, en las publicaciones más recientes, esta regla ha dejado de
tener validez. Recuerde que en el ENARM, las preguntas en ocasiones tienen respuestas con
información atrasada.(R3)

94. Una paciente de 57 años es valorada en su consulta. Refiere desde hace un año
molestias miccionales que no han respondido a tratamiento con antibióticos y una

 
 
 
 
intensa polaquiuria y nicturia que se asocian a cierta urgencia miccional. Presenta
citologías urinarias negativas, cistoscopia normal, biopsias de la mucosa vesical con
acúmulos de macrófagos y PCR de orina negativa. Su sospecha diagnóstica será:

1. 1. Tuberculosis genitourinaria.
2. 2. Cistitis intersticial.
3. 3. Carcinoma de tracto urinario superior.
4. 4. Litiasis vesical.
Gráfico de respuestas
Comentario
El diagnóstico de cistitis intersticial se realiza por exclusión, no existiendo pruebas específicas para
poder llegar al mismo.(R2)

95. Femenino de 50 años con hemorragia uterina anormal y examen pélvico normal. ¿Cuál
es la patología más importante a descartar?

1. 1. Aborto.
2. 2. Adenocarcinoma endometrial.
3. 3. Hemorragia anovulatoria.
4. 4. Mioma uterino.
Gráfico de respuestas
Comentario

En esta pregunta te hacen incapié en "la patología más importante a descartar"... por lo que de las
cuatro opciones ¿cuál es la que podía poner en riesgo la vida de la paciente? Obviamente la
número 2, el cáncer de endometrio, es un diagnóstico que no debes dejar pasar en mujeres
mayores de 50 años, generalmente en la menopausia, por lo que aunque no sepas la respuesta,
tienes otras herramientas que te pueden ayudar a resolver preguntas tipo ENARM.(R2)

96. Un niño RN de bajo peso, cuya placenta fue anormalmente grande, presenta edema
corporal generalizado y 8 millones de eritrocitos en sangre periférica. Su función renal
fue inicialmente normal, pero presenta proteinuria de 4 gr/día no selectiva. Señala la
afirmación FALSA en relación a este cuadro clínico:

1. 1. Es una forma de síndrome nefrótico con herencia recesiva.


2. 2. La sífilis congénita puede producir un síndrome similar.
3. 3. La alfa-fetoproteína está disminuida en líquido amniótico.
4. 4. Es muy frecuente en familias de origen finlandés.
Gráfico de respuestas
Comentario
Pregunta de dificultad muy elevada acerca del síndrome nefrótico congénito tipo finlandés, de
herencia AR, en que hay un defecto de la nefrina, proteína implicada en la estructura de la lámina
de los slits. No es típico de este cuadro la disminución de la AFP en líquido amniótico (al contrario
que en el síndrome de Down fetal, en que encontramos niveles disminuidos de AFP).(R3)

97. Mujer de 52 años de edad, acude por presentar desde hace 6 meses episodios de
metrorragia no asociados al coito. Su último período menstrual fue hace 4 años. El
diagnóstico probable es:

1. 1. Hiperplasia endometrial.

 
 
 
 
2. 2. Cáncer de cuello uterino.
3. 3. Hemorragia uterina disfuncional.
4. 4. Endometriosis.
Gráfico de respuestas
Comentario

Le preguntan el diagnóstico más probable en una paciente menopaúsica con hemorragia... En


ningún momento relatan consumo de hormonales, puede descartar la 1, obviamente descarta la 4,
ya que no relatan historia previa de dismenorrea...

La respuesta correcta es la 2, ya que en toda paciente postmenopaúsica con dicha sintomatología


debe descartarlo.(R2)

98. ¿Cuál de las siguientes respuestas acerca de la hernia umbilical en el niño es FALSA?:

El mayor número de hernias que aparecen en los primeros 6 meses de vida desaparecen
1. 1.
hacia el año.
Las hernias estranguladas, sintomáticas o que sigan creciendo tras los 2 años de vida
2. 2.
tienen indicación quirúrgica.
Una medida útil para acelerar la resolución de las hernias es el vendaje abdominal o
3. 3.
colocar alguna moneda dentro de las mismas.
4. 4. Se suelen deber al cierre incompleto y a la debilidad del anillo umbilical.
Gráfico de respuestas
Comentario

Las hernias umbilicales son extraordinariamente frecuentes en los recién nacidos, especialmente
en los de raza negra, cuy abdomen suele ser muy voluminoso. Suele evidenciarse cuando el niño
llora. En general, las hernias umbilicales son de anillo amplio, fácilmente reductibles; tan sólo si
progresan, se complican, o persisten más allá de los dos años, se propondrá tratamiento
quirúrgico.

Por lo tanto es falsa la respuesta número 3.(R3)

99. A 5-year-old child is brought to the pediatrician’s office complaining of an


erythematous rash that started on her face and neck and has now spread down all over
her body. She had a cough with conjunctivitis last week. Oral mucosa shows pale lesions
surrounded by red specks. Which of the following is the most likely causative agent of
this girl's disease?

1. 1. Paramyxovirus infection
2. 2. Herpesvirus infection
3. 3. Lice infestation
4. 4. Togavirus infection
Gráfico de respuestas
Comentario
Paramyxovirus infection. The paramyxovirus is the causative agent of measles. The disease starts
as a viral cold and a few days later the rash appears. Chickenpox and shingles produce a vesicular
rash. The rash of rubella spreads less.(R1)

 
 
 
 

100. Embarazada de 40 semanas de


gestación, que acude al hospital por dinámica uterina regular. A la exploración presenta
un cérvix de parto. En la vulva presenta las lesiones que se muestran en la imagen que
son dolorosas. Se palpan adenopatías inguinales bilaterales. Al intentar obtener más
información, nos indica que su pareja presenta lesiones parecidas desde hace unos días.
Dada su sospecha diagnóstica, señale la respuesta CORRECTA:

Como la paciente ha iniciado el trabajo de parto es precisa la administración inmediata de


1. 1.
aciclovir para evitar el contagio vertical durante el parto vaginal.
Se permite parto vaginal, siempre y cuando las lesiones no supongan un obstáculo
2. 2.
mecánico ni aumenten el riesgo de sangrado.
Es causada por el virus del papiloma humano y es la infección de transmisión sexual más
3. 3.
frecuente.
4. 4. Es indicación de cesárea.
Gráfico de respuestas
Comentario

Pregunta muy importante sobre indicaciones de cesárea. La paciente del caso clínico presenta un
episodio activo por herpes simple en el momento del parto, lo cual es indicación de cesárea y no
de parto vaginal. La infección genital por el virus del papiloma humano produce verrugas genitales
(condilomas) y no contraindica el parto vaginal, salvo que suponga un obstáculo mecánico en el
canal del parto.(R4)

101. Paciente de 22 años que acude a usted por diarrea de 3 semanas de evolución con
expulsión de sangre, fiebre de hasta 38º y dolor abdominal. No refiere antecedentes de
interés y en los últimos meses no ha viajado fuera de su localidad habitual. El
coprocultivo es negativo. ¿Cuál sería el primer proceso que deberiamos descartar?

1. 1. Enfermedad inflamatoria intestinal.


2. 2. Diarrea enteroinvasiva por Shigella.
3. 3. Isquemia intestinal.
4. 4. Neoplasia.
Gráfico de respuestas
Comentario

Una pregunta relativamente sencilla. Dada la negatividad del coprocultivo y la ausencia de viajes
recientes, se descartan las opciones 1 y 3. No parece probable una neoplasia, dada la edad del

 
 
 
 
paciente y la ausencia de síndrome constitucional (opción 5 falsa). La isquemia intestinal también
sería impropia de este grupo de edad (respuesta 4 falsa). La única opción razonable sería la 2,
cuya edad pico de presentación oscila entre los 20- 30 años, y cuyas manifestaciones clínicas son
compatibles con el cuadro descrito.(R1)

102. Un niño de 5 años de edad tiene fiebre (máximo 40ºC) desde hace 6 días. Tiene algo
de tos, pero no ha presentado respiratoria, ni otros síntomas catarrales, aunque refieren
que al inicio del cuadro tuvo una leve conjuntivitis. Desde hace 48 horas presenta un
exantema maculopapuloso en tronco y raíz de las extremidades. La exploración física
muestra además: regular estado general, buen color, buena hidratación, eritema
palmoplantar, auscultación cardiopulmonar normal, abdomen blando, no doloroso, no
palpándose masas ni megalias. La puñopercusión renal es negativa. Neurológicamento
consciente y orientado, sin signos meníngeos. ORL normal. Es FALSO que:

Se precisarían estudios complementarios (laboratorio, ECG y ultrasonido) para el


1. 1.
diagnóstico de confirmación de la entidad que padece el paciente.
Los corticoides se utilizan como tratamiento alternativo en los casos en que falla el
2. 2.
tratamiento de primera línea.
En caso de demostrarse lesión coronaria, la antiagregación se mantendría de forma
3. 3.
indefinida.
Si no hay lesión coronaria, se recomienda mantener aspirina en dosis bajas durante 6-8
4. 4.
semanas, hasta que descienda la VSG o la cifra de plaquetas.
Gráfico de respuestas
Comentario

Considere en el diagnóstico diferencial la enfermedad de Kawasaki siempre que en una pregunta


le hablen de un niño pequeño (<5 años) y con fiebre prolongada (>5 días). La enfermedad de
Kawasaki es una vasculitis sistémica febril y típica de la infancia. Su diagnóstico se hace por
criterios clínicos, reservándose las pruebas complementarias para el diagnóstico de las
complicaciones y de las formas atípicas o incompletas. Los criterios clínicos son:

Fiebre > 5días.

Inyección conjuntival bilateral aséptica.

Alteraciones bucales: lengua aframbuesada, fisuras labiales. Cuenta como 1 aunque estén todas.

Eritema y edema palmoplantar. Presente en casi todos los casos.

Exantema cutáneo. De cualquier tipo menos ampollas o vesículas. Típico que afecte al área del
pañal.

Adenopatía cervical. Bilateral o típicamente unilateral (>1.5 cm).

Deben cumplirse el primero y al menos 4 de los demás. No es necesario que estén todos
presentes al mismo tiempo. Cursa en tres fases:

- Fase aguda: 1- 2 semanas. Febril. Clínica típica (criterios clínicos), irritabilidad, anorexia,
meningitis aséptica, diarrea, disfunción hepática. En casi todos los casos destaca la afectación del
estado general.

 
 
 
 
- Fase subaguda: 2- 3 semanas. Afebril. Típico de esta fase es la descamación periungueal y las
artralgias/artritis. Aneurismas coronarios.

El tratamiento incluye:

- Gammaglobulina inespecífica intravenosa que disminuye la prevalencia de aneurismas coronarios


a largo plazo, la fiebre y los reactantes de fase aguda. Máxima efectividad si se da entre los días 5º
y 10º de la enfermedad.

- Corticoides intravenosos: se reservan para el 5% de casos refractarios a inmunoglobulinas.

(R1)

103. Uno de los siguientes datos NO es característico del síndrome cordonal posterior:

1. 1. Pérdida de la sensibilidad profunda (vibratoria, artrocinética).


2. 2. Paraplejía.
3. 3. Retención urinaria.
4. 4. Ataxia de las extremidades inferiores.
Gráfico de respuestas
Comentario
El síndrome cordonal posterior asocia una pérdida de la sensibilidad propioceptiva y se conserva la
sensibilidad dolorosa y térmica. Es típico de la tabes dorsal (neurolúes) que se desarrolla a los 10-
20 años después del inicio de la infección y resulta en atáxia sensitiva. Asocia dolores lancinantes
en las piernas, incontinencia urinaria y arreflexia rotuliana y aquílea. La disfunción de los cordones
posteriores en la región cervical da lugar a la sensación de descarga eléctrica (signo de Lhermitte).
La neurona motora no se afecta en el síndrome cordonal posterior así que no va a haber
paraplejía.(R2)

104. Paciente de 52 años de edad, con síndrome climatérico y cuadro de hemorragia


uterina anormal. ¿Cuál es la conducta inicial más importante?

1. 1. Histerosalpingografía.
2. 2. Ultrasonido transvaginal.
3. 3. Histeroscopía.
4. 4. Laparoscopía.
Gráfico de respuestas
Comentario

La primera prueba a realizar siempre debe ser una prueba no invasiva. En este caso una ecografia
transvaginal seria la prueba más indicada para inciar el estudio.(R2)

105. La toxicidad más importante de la Zidovudina es:

1. 1. Náuseas y vómitos.
2. 2. Mielosupresión.
3. 3. Azoospermia.
4. 4. Neuropatía periférica dolorosa.
Gráfico de respuestas

 
 
 
 
Comentario
La zidovudina es el primer antirretroviral utilizado en la infección por el VIH, y actualmente continua
utilizándose ampliamente, por lo que es preciso conocer sus efectos secundarios más habituales.
Es muy característico la producción de anemia y macrocitosis. Esta última característica se
aprovecha para comprobar el cumplimiento terapéutico de los pacientes. Otro efecto adverso
descrito con este fármaco es la miopatía. Se debe recordar que la neuropatía es mas característico
del d4T (estavudina) y del ddI.(R2)

106. Señale la afirmación INCORRECTA en relación con la poliposis adenomatosa


familiar:

1. 1. Se debe a alteraciones del gen APC, que está en el brazo largo del cromosoma 5.
2. 2. Puede haber manifestaciones extracolónicas, como pólipos en el fundus.
3. 3. El riesgo de malignización es prácticamente del 100%.
El manejo habitual de estos pacientes consiste en colonoscopías periódicas a partir de los
4. 4.
25 años, o cinco años antes del familiar más joven afecto.
Gráfico de respuestas
Comentario

Todos los miembros potencialmente afectos de poliposis colónica familiar deben realizarse
sigmoidoscopías a partir de los 12 años. Si se encuentran pólipos, debe repetirse cada 2 años, y
se debe realizar colectomía profiláctica cuando el paciente haya completado su desarrollo físico
(entre los 15-22 años, individualizando y tratando que sea lo más tarde posible, dentro de este
rango).(R4)

107. Mujer de 40 años consulta por diarrea crónica acuosa fluctuante, sin esteatorrea ni
sangre. La colonoscopía es macroscópicamente normal y en la biopsia se objetiva
infiltrado por linfocitos y células plasmáticas en la lámina propia e intraepitelial. Respecto
a la enfermedad que probablemente presenta esta paciente, señale la FALSA:

La etiología es desconocida, aunque se han descritos casos asociados a fármacos y a


1. 1.
enfermedades autoinmunes.
En la evolución de la enfermedad de esta paciente podría observarse engrosamiento de la
2. 2.
banda subepitelial de colágeno.
3. 3. Tiene un curso crónico recidivante y se han descrito casos de remisión espontánea.
4. 4. Debe instaurarse tratamiento con dieta sin gluten.
Gráfico de respuestas
Comentario

Esta pregunta nos presenta un caso de colitis colágena. Vamos a resumir sus características
principales:

- Principal síntoma: Diarrea acuosa crónico.

- Otros: Pérdida de peso fluctuante que luego se estabiliza.

- Es más frecuente en mujeres.

- Asociación con enfermedades autoinmunes.

 
 
 
 
- Colonoscopía normal.

- Histología característica: Aumento de fibras colágenas o infiltrado linfocitario (colitis linfocitaria, en


este caso, que por lo demás son iguales).

- Tratamiento: Sintomático (antidiarreico en principio). Sólo en ocasiones se utilizan AINEs y


corticoides.

- No predispone a cáncer.

Lógicamente, no precisa dieta sin gluten, porque no se trata de una enfermedad celíaca.(R4)

108. Indique la respuesta FALSA en relación con el tratamiento de los ACV:

1. 1. Los vasodilatadores son eficaces en el ictus establecido.


En un ictus masivo de causa cardioembolica no esta indicada la anticoagulación por riesgo
2. 2.
de transformación hemorrágica.
3. 3. El ácido acetilsalicílico (AAS) es útil en la prevención secundaria.
4. 4. Los fibrinolíticos han demostrado que es posible la recanalización del trombo.
Gráfico de respuestas
Comentario
Pregunta de dificultad baja de un tema muy importante para el Mir, como es el tratamiento de los
ACVs. El tratamiento en fase aguda de los ictus consiste en medidas generales, como ser evitar la
hipertermias, hiperglucemias, y elevación excesiva de la tensión arterial, así como descensos
bruscos. La respuesta falsa es la 1 dado que el tratamiento con vasodilatadores puede reducir la
tensión arterial, resultando en una disminución de la presión de perfusión cerebral, lo cual podría
aumentar la isquemia o extender la lesión a la zona de penumbra. La opción 2 es totalmente
correcta y no deberías dudarla. La opción 3 también es claramente correcta, y debes saber el
tratamiento de la ateromatosis carótidea a la perfección. Con respecto a los fibrinolíticos, es un
tratamiento eficaz dentro del período ventana, que son las tres - seis primeras horas desde la
instalación del cuadro.(R1)

109. Paciente femenino de 25 años consulta por un retraso menstrual de 3 semanas y


metrorragia escasa desde hace 24 horas. Su estado general es bueno, no tiene dolor, y
la exploración ginecológica no muestra alteraciones excepto el sangrado escaso
procedente de la cavidad uterina. El test de embarazo es positivo y la determinación de
beta-hCG en plasma es de 600 mUI/ml. Por ecografía transvaginal se observa un útero
normal con un endometrio homogéneo de aspecto secretor de 12 mm de espesor. En el
ovario derecho hay una formación que parece un cuerpo lúteo normal. No hay líquido
libre en la cavidad abdominal. ¿Cuál de las siguientes es la indicación más correcta?

1. 1. Repetir seriadamente cada 2-3 días la ecografía y la beta-hCG.


2. 2. Legrado uterino.
3. 3. Tratamiento con metotrexato por vía sistémica.
4. 4. Laparoscopia.
Gráfico de respuestas
Comentario

Pregunta de dificultad media acerca del embarazo ectópico.

 
 
 
 
Las hemorragias del primer trimestre son un tema bastante preguntado en el ENARM y que exige
su conocimiento. El diagnóstico diferencial se tiene que realizar entre el aborto, la gestación
ectópica y la enfermedad trofoblástica.

El aborto es la terminación de la gestación antes de la semana 20, denominándose precoz o tardío


según ocurra en las 12 primeras semanas o entre la 12 y la 20, respectivamente. La causa más
habitual son las anomalías ovulares. Otras son los factores maternos, inmunológicos, etc. Hay que
distinguir entre la amenaza de aborto en la que menos de la mitad aborta y no hay daño fetal tras
la amenaza, el aborto en curso, el aborto completo en el que se han expulsado la totalidad de los
restos y el incompleto. Hay que tener en cuenta que el aborto es la hemorragia del primer trimestre
que con más frecuencia sangra abundantemente.

En la gestación ectópica el signo cardinal es el dolor en anejo (dado que la localización más
frecuente es la ampular) acompañado de signos de gestación incipiente, siendo corriente un
discreto sangrado oscuro intermitente. A veces, puede debutar con signos de peritonismo como
consecuencia de la rotura tubárica, pudiendo entrar en shock y causar la muerte de la paciente.

En cuanto a la enfermedad trofoblástica, la metrorragia suele aparecer a partir del segundo mes de
amenorrea y se acompaña de un útero de mayor tamaño al que corresponde al tiempo de
amenorrea y de una elevación exagerada de la β-HCG, hecho que no se observa en el caso que
nos muestran.

Por tanto, en este caso se ha de hacer el diagnóstico diferencial entre una gestación incipiente,
una gestación ectópica o un aborto completo. Para ello, usaremos la determinación seriada cada
dos o tres días de los niveles sanguíneos de β-HCG, que nos permitirá diagnosticar una gestación
ectópica si no se duplica (durante las primeras semanas de un embarazo normal las cifras se
duplican cada dos días aproximadamente) o un aborto, en el que los niveles no aumentarían.

En un embarazo con niveles en sangre de β-HCG superiores a 1.000 mUI/ml se puede ver el saco
gestacional intrauterino, y es, además, la prueba de elección para el diagnóstico de la gestación
ectópica, del aborto y de la enfermedad trofoblástica. Por consiguiente, la ecografía también sería
imprescindible (respuesta 1 correcta).

La respuesta 2 (legrado uterino) es el tratamiento de la enfermedad trofoblástica y del aborto


incompleto, y la respuesta 4 es uno de los tratamientos de la gestación ectópica. No obstante, lo
primero sería confirmar el diagnóstico (respuestas 2, 3 y 4 incorrectas).(R1)

110. Cuál es la actitud terapéutica aconsejada de profilaxis en adultos postexposición a


la hepatitis virus B percutánea si la persona expuesta está vacunada:

1. 1. Si los marcadores de antiHBs son mayores o iguales a 10 mUI/ml tratar con vacuna HB.
2. 2. Si los marcadores de antiHBs son mayores o iguales a 10 mUI/ml no tratar.
Si los marcadores de antiHBs son menores a 10 mUI/ml esperar a elevación de las
3. 3.
transaminasas.
4. 4. Vacunación de forma independiente a nivel antiHBs.
Gráfico de respuestas
Comentario

Si en un vacunado se detectan cifras de antiHBs mayores o iguales a 10 mUI/mL, no es necesario


hacer nada pues consideramos que está protegido.(R2)

 
 
 
 
111. ¿Qué enfermedad pulmonar intersticial difusa se caracteriza por afectación de los
lóbulos inferiores?:

1. 1. Sarcoidosis.
2. 2. Fibrosis pulmonar idiopática.
3. 3. Neurofibromatosis.
4. 4. Síndrome de Goodpasture.
Gráfico de respuestas
Comentario
La fibrosis pulmonar idiopática se caracteriza radiológicamente por un patrón reticulonodular de
predominio en campos inferiores. La silicosis presenta pequenas opacidades redondeadas sobre
todo en los lóbulos superiores y pueden aparecer también calcificaciones en "cáscara de huevo ".
El síndrome de Goodpasture suele mostrar infiltrados pulmonares algodonosos difusos.(R2)

112. Como causas de la amenorrea de origen hipotálamo-hipofisario, incluiríamos todas


EXCEPTO:

1. 1. Ejercicio físico excesivo.


2. 2. Anorexia nerviosa.
3. 3. Síndrome de Stein-Leventhal.
4. 4. Tumores paraselares.
Gráfico de respuestas
Comentario

Puede contestar esta pregunta por descarte aunque no sepas que es síndrome de STEIN-
LEVENTHAL, lo que debe conocer es que las amenorreas de causa hipotálamo- hipofisaria se
deben a tumores, traumatismos, infartos, granulomas que lesionan el eje hipotálamo- hipofisario e
impiden el normal funcionamiento del ciclo.

La anorexia nerviosa cursa con un descenso de gonadotropinas, por lo que la única respuesta que
nos queda es la 3.

Quizá se aclare aún más si descubre que ese síndrome raro es el síndrome de ovario poliquístico
cuya causa de amenorrea es la anovulación por el exceso de andrógenos que también se
acompaña de obesidad e hirsutismo.(R3)

113. RNT de 38 semanas, peso 1,930 g, presenta a las 24 horas de vida fiebre, ictericia y
hepatoesplenomegalia. A la exploración destaca PC de 40 cm. Se le realiza ultrasonido
cerebral, donde se aprecian calcificaciones intracraneales dispersas. ¿Cuál de los
siguientes datos NO es característico de este proceso?:

Más de la mitad de los lactantes infectados congénitamente se consideran normales en el


1. 1.
período neonatal, aunque casi todos tendrán afectación ocular posteriormente.
Es más frecuente la afectación fetal en los casos de primoinfección materna que en los
2. 2.
casos de reactivación.
3. 3. La hidrocefalia puede ser la única manifestación clínica en estos pacientes.
Es más probable que se produzcan manifestaciones graves si el feto adquiere la infección
4. 4.
durante el 3º trimestre.
Gráfico de respuestas
Comentario

 
 
 
 
Cuadro característico de toxoplasmosis congénita.

El riesgo de toxoplasmosis en el feto es directamente proporcional a la edad gestacional (más


frecuente en el 3er trimestre) en la que se produce la infección; sin embargo, la gravedad lo es
inversamente a la madurez fetal (más grave en el 1er trimestre, lo contrario que afirma la respuesta
número 4).

Si la infección ocurre en el primer trimestre, el RN puede presentar la tétrada de Sabin


(coriorretinitis, que es la manifestación más frecuente, calcificaciones intracraneales difusas,
hidrocefalia y convulsiones).(R4)

114. Si en un paciente cirrótico encuentra una fosfatasa alcalina sérica


desproporcionadamente elevada en relación con otras pruebas de función hepática, es
un indicio de:

1. 1. Enfermedad de Wilson.
2. 2. Hemocromatosis.
3. 3. Cirrosis alcohólica.
4. 4. Carcinoma hepático infiltrante.
Gráfico de respuestas
Comentario

Un carcinoma hepático infiltrante podría generar un patrón conocido como colestasis disociada,
donde se eleva principalmente la fosfatasa alcalina, sin que le acompañe un incremento de la
bilirrubina ni de las transaminasas (o muy levemente, en el caso de estas últimas). Esta pregunta,
en caso de tratarse de un examen real, podría considerarse muy controvertida.(R4)

115. ¿Cuál es la indicación fundamental del tratamiento hormonal sustitutivo del


climaterio?:

1. 1. Aliviar los sofocos y síntomas climatéricos.


2. 2. Retrasar las lesiones del tejido conjuntivo (prolapso, incontinencia).
3. 3. Prevenir las lesiones vasculares.
4. 4. Prevenir el tumor endometrial y ovárico.
Gráfico de respuestas
Comentario

El motivo fundamental del tratamiento hormonal sustitutivo (THS) no es la prevención


cardiovascular ni la de la osteoporosis, sino aliviar los sofocos y el resto de los síntomas
relacionados con el climaterio. En el pasado, se creía que la THS podría tener algún papel
protector desde el punto de vista cardiovascular, pero algunos estudios demostraron que esto no
era así, e incluso un incremento del riesgo. Por otra parte, la THS tampoco ha demostrado
mayores beneficios en la prevención de la osteoporosis.(R1)

116. Mujer de 39 años. Como antecedentes personales destacan un embarazo y parto


normales hace 7 años. Gestante actualmente de 13 semanas, acude a su centro para
realización de ultrasonografía. En ella se observa gestación intrauterina, embrión con
CRL de 70 mm y movimientos cardiofetales positivos, líquido amniótico en cantidad
normal y placenta en cara posterior. En la exploración usted detecta un edema nucal de
5 mm. Señale lo FALSO:

 
 
 
 
La amniocentesis precoz para el diagnóstico de cromosomopatías tiene un riesgo de
1. 1.
pérdida fetal de un 1% aproximadamente.
2. 2. El edema nucal es una grave malformación fetal incompatible con la vida.
3. 3. El origen de este edema parece ser una alteración en el desarrollo del sistema linfático.
4. 4. El edema nucal puede aparecer en embriones cromosómicamente normales.
Gráfico de respuestas
Comentario

Pregunta complicada sobre el diagnóstico prenatal. Se presenta un caso clínico de una gestante de
39 años, a la 13ª semana. Dada la edad materna, habría que indicar una amniocentesis, cuyo
riesgo de pérdidas fetales es de 1%. Un CRL de 70 mm coincide más o menos con las 13 semanas
de amenorrea. Ante un edema nucal de más de 3 mm, se debe informar de la posible existencia de
cardiopatías fetales o cromosomopatías. Esto hace aún más necesaria la amniocentesis. El edema
nucal suele deberse a alteraciones del drenaje linfático, lo cual no implica que obligatoriamente el
feto tenga alguna malformación (pero sí tiene más probabilidades).

La opción falsa por tanto es la 2. Si bien es cierto que puede correlacionarse con malformaciones
fetales y alteraciones cromosómicas (síndrome de Down), también puede aparecer en embriones
cromosómicamente normales y sin malformaciones (respuesta 4). Cuando dos opciones son
excluyentes entre sí, con mucha probabilidad alguna de las dos será la opción a elegir.

Por otro lado, la correlación del edema nucal con posibles patologías no implica que estas tengan
que ser incompatibles con la vida, siendo otro argumento más para elegir la respuesta 2 como la
falsa.(R2)

117. El uso de ácido acetil-salicílico se ha relacionado con la aparición de síndrome de


Reye en niños. ¿A pesar de ello, en cuál de estas situaciones estaría indicado su uso?

Lactante de 6 meses con cuadro catarral y fiebre que no responde a ibuprofeno ni


1. 1.
paracetamol.
Niña de 18 meses con tumefacción y dolor articular en ambos carpos, rodilla izquierda y
2. 2.
tobillo derecho que no responde al tratamiento con naproxeno iniciado hace 3 semanas.
Niño de 10 años con más de 4 episodios mensuales de cefalea holocraneal pulsátil con
3. 3.
fotofobia y vómitos.
Niño de 5 años con fiebre elevada, adenopatías cervicales, faringe eritematosa sin exudado
4. 4.
y conjuntivitis sicca desde hace 5 días en el que aparece descamación ungueal.
Gráfico de respuestas
Comentario

Pregunta sobre un tema iportante en el ENARM. El síndrome de Reye es una encefalopatía aguda
con degeneración grasa del hígado que se ha relacionado con el uso de ácido- acetil salicílico en
niños, especialmente cuando durante procesos virales como la influenza o la varicela. Es por ello
que su uso está contraindicado en la edad pediátrica, salvo algunas indicaciones como la
enfermedad de Kawasaki, en la que se usa en un primer momento como antiinflamatorio y
posteriormente como antiagregante.(R4)

118. ¿En qué caso NO estaría indicado realizar un test del sudor para descartar fibrosis
quística en la infancia?:

1. 1. Tos crónica.
2. 2. Retraso pondoestatural.
3. 3. Ileo meconial.

 
 
 
 
4. 4. Hipotiroidismo.
Gráfico de respuestas
Comentario

Es importante el conocimiento de las indicaciones más importantes del test del sudor, método
diagnostico principal de FQ.

Sus indicaciones son entre otras: tos crónica, neumonía recurrente o crónica, retraso del
desarrollo, pólipos nasales o pansinusitis, íleo meconial o síndrome del tapón de meconio, ictericia
neonatal prolongada , atelectasias, hemoptisis, infección por Pseudomonas, esteatorrea, cirrosis
biliar, alcalosis hipoclorémica inexplicada.

La fibrosis quística no produce hipotiroidismo.(R4)

119. A 39-year-old woman presents with fatigue. On physical examination she is found to
have hepatomegaly and mild jaundice. Serum analysis show: HBsAg positive, HBeAg
positive, anti-HBcAg IgG antibodies positive, and high titers of HBV DNA. AST 80 U/L,
ALT 110 U/L, with minimal inflammation on the biopsy. What is the best recommendation?

1. 1. Hepatitis B immune globulin


2. 2. Hepatitis B vaccine
3. 3. Interferon or Lamivudine
4. 4. Do nothing at this time
Gráfico de respuestas
Comentario
Hepatitis B vaccine. Although there are constant changes regarding the clinical management of
infection by HBV, in this case we are considering a patient with clinically active infection by the
virus. Replication appears to be highly active. There is not much harm caused to the liver, yet, but
enzyme testing also shows active disease. Only one answer option includes effective therapeutic
agents against HBV infection.(R3)

120. Varón de 40 años de edad acude a urgencias acompañado por un familiar por un
cuadro de temblor y angustia. El paciente es bebedor habitual, pero hace ya tres días que
no prueba el alcohol. En la exploración física sus manos tiemblan y habla casi
constantemente de un modo descentrado. A veces reconoce al médico, pero otras le
confunde con su hermano. Hay períodos en que atrapa "bichos" que cree ver en las
sábanas. Muestra desorientación espacial y temporal. ¿Cuál es su diagnóstico?:

1. 1. Delirium por abstinencia de alcohol.


2. 2. Alucinosis alcóholica.
3. 3. Hematoma subdural.
4. 4. Beri-beri
Gráfico de respuestas
Comentario
Esta pregunta es de " verde y con asas" es sencillísima y además en forma de caso clínico por lo
que jamás se nos olvidará la "foto" de la enfermedad que nos preguntan. Nos presentan a un
varón, mediana edad, consumidor habitual de alcohol, que HA DEJADO DE TOMAR ALCOHOL!, y
presenta un cuadro de desorientación + atención fluctuante + hiperactividad autónoma, con
alucinaciones visuales muy vivas y que son típicamente bichos. Por lo que la opción correcta es la
1. Recuerda e la alucinosis ( opción 2) el nivel de conciencia está intacto y el paciente alucina

 
 
 
 
auditivamente!, en la intoxicación (opción 3) aparecería disártrico sin delirium En el hematoma
habría clínica más neurológica con posible focalidad. El beri- beri es un déficit de vitamina B que se
manifiesta con alteraciones neurológicas, polineuropatía, encefalopatía de Wernicke- Korsakoff (
trastorno de la memoria ) que no es lo que presenta el sujeto. Manual CTO 4ª Edición, Psiquiatría,
Tema 5, Págs. 34- 36(R1)

121. ¿Cuál de los siguientes cuadros NO consideraría de entrada entre las etiologías de
un dolor en fosa ilíaca derecha?:

1. 1. Apendicitis aguda.
2. 2. Gestación tubárica.
3. 3. Isquemia mesentérica aguda.
4. 4. Hematoma de la vaina de los rectos.
Gráfico de respuestas
Comentario

Una pregunta relativamente fácil. El dolor de la isquemia mesentérica aguda es preferentemente


periumbilical. Recuerde que es muy intenso y súbito, y habitualmente se trata de un paciente
mayor, con fibrilación auricular o algún otro factor que predisponga a embolias sistémicas.

Tenga cuidado con la respuesta 4. El hematoma de la vaina de los rectos podría lateralizarse, tanto
a la izquierda como a la derecha, con lo que puede doler a nivel de la fosa ilíaca.(R3)

122. Mujer obesa que ha sido sometida a una resección segmentaria de intestino delgado
por una hernia inguinal estrangulada y que previamente estaba en tratamiento con
corticoides. En el postoperatorio presenta un absceso en la herida quirúrgica. El germen
que nos encontramos con mayor probabilidad en el absceso será:

1. 1. Haemophilus.
2. 2. Staphylococcus epidermidis.
3. 3. Escherichia coli.
4. 4. Streptococcus pyogenes.
Gráfico de respuestas
Comentario

Los gérmenes implicados en la infección de una herida quirúrgica dependerán del tipo de cirugía
del que se trate.

Así en los casos de infección de una herida quirúrgica después de una cirugía limpia serán en
general los que existen en la flora saprófita cutánea (en general Staphylococcus) del paciente y
que por diferentes circunstancias (mala asepsia o antisepsia, inmunosupresión, etc.) contaminan y
proliferan en el seno de la herida quirúrgica.

Por el contrario en el caso de cirugías limpias-contaminadas, contaminadas o sucias los gérmenes


responsables de la infección de la herida serán los que corresponden a la flora saprófita más
habitual en un determinado tramo del tubo digestivo, y en el caso expuesto en el intestino existen
E. coli con mayor frecuencia que el resto de gérmenes enumerados.(R3)

 
 
 
 
123. An 84-year-old man with a past medical history remarkable for chronic obstructive
pulmonary disease, chronic kidney disease, type 2 diabetes and transient ischemic
attacks is diagnosed with severe symptomatic aortic stenosis. He has a good quality of
life and aortic surgical valve replacement is initially considered but finally his surgical
risk is deemed to be extremely high and the surgical option is dismissed. Which of the
following options may improve his symptoms?

1. 1. Aortic valve replacement surgery.


2. 2. Percutaneous aortic valve replacement.
3. 3. Percutaneous balloon aortic valvuloplasty.
4. 4. Angiotensin-Converting Enzyme Inhibitors.
Gráfico de respuestas
Comentario

Concepto que dudo sea preguntado en el nacional. Ante un paciente con estenosis aórtica severa
sintomática en el que nos plantearíamos una cirugía pero esta tiene un riesgo elevado y se
desestima, debemos valorar el implante de una prótesis valvular aórtica transcatéter. Esto mejora
la clínica y el pronóstico de los pacientes respecto a no hacer nada. Una valvuloplastía simple con
balón en pacientes adultos no se suele realizar ya que tiene malos resultados y la tasa de
reestenosis es muy alta a los pocos meses. El tratamiento médico no tiene ningún beneficio en la
estenosis aórtica.(R2)

124. Mujer joven que acude por aumento del vello corporal, alteraciones menstruales
(dice que tiene las menstruaciones muy separadas), nota que ha cambiado su voz y está
preocupada porque pierde pelo en la cabeza. Refiere que todo empezó bastante deprisa,
por lo que tiene bastante ansiedad. Como se sospecha un aumento de andrógenos,
iniciamos el estudio correspondiente, con el que apreciamos niveles muy elevados de
DHEA y aumento en orina de 17-cetoesteroides. Ambos metabolitos no se suprimen tras
la administración de 0,5 mg/6 h durante 2 días de DXM; la testosterona no se encuentra
aumentada. Se encuentra a su vez una leve elevación de cortisol y 17-hidroxiesteroides.
Ante este resultado, el origen más probable del cuadro será:

1. 1. Carcinoma suprarrenal.
2. 2. Arrenoblastoma ovárico.
3. 3. Hiperplasia suprarrenal de aparición tardía.
4. 4. Toma oculta de andrógenos.
Gráfico de respuestas
Comentario

Ante una paciente con un cuadro de rápida evolución de hirsutismo y virilización, se debe descartar
un tumor productor de andrógenos. Este hecho también se confirma con la no supresión de los
andrógenos tras la supresión con DXM. Una vez establecida la sospecha clínica, se debe buscar la
etiología del exceso androgénico. Para ello, es fundamental conocer el andrógeno que se
encuentra elevado. En este caso, es la DHEA y los cetoesteroides en orina, luego el origen es la
suprarrenal y la respuesta correcta la 1. Si hubiera sido la testosterona el andrógeno elevado, el
origen sería el ovario.(R1)

 
 
 
 

125. Niño de 11
años con dolor abdominal de varios meses de evolución que ha aumentado en el último
mes. Refiere sensación ocasional de atragantamiento con algunos alimentos, tanto
sólidos como líquidos. Ha recibido tratamiento con omeprazol a dosis plenas durante
este periodo con mejoría parcial de su sintomatología. Tras la realización de una
endoscopia digestiva alta (ver imagen) y una impresión diagnóstica inicial indique cuál
de las siguientes opciones es INCORRECTA:

El tratamiento será corticoides tópicos (budesonida, fluticasona) deglutidos administrados


1. 1.
en MDI.
2. 2. Se han descrito una predilección por el género masculino.
La impactación alimentaria es el síntoma principal en los pacientes de mayor edad si bien
3. 3.
el dolor abdominal aislado es un motivo frecuente de consulta en pacientes más jóvenes.
4. 4. La existencia de estenosis en el tercio medio esofágico es patognomónica.
Gráfico de respuestas
Comentario

No se preocupe si ha fallado esta pregunta, ya que se trata de un caso de dificultad muy alta.

La enfermedad que padece este niño es una esofagitis eosinofílica. Esta entidad se relaciona con
procesos alérgicos a otros niveles, su prevalencia está aumentando durante los últimos años y el
principal diagnóstico diferencial es el reflujo gastroesofágico. De ahí que nos insistan de una
mejoría solamente parcial con inhibidores de la bomba de protones. El tratamiento se realiza con
corticoides, y es típica la imagen endoscópica con “traquealización” del esófago (se vuelve
blanquecino y su imagen endoscópica recuerda la textura del interior de la tráquea).

126. Con respecto al paciente de la pregunta anterior indique la respuesta FALSA:

1. 1. Es típica la imagen endoscópica de un esófago blanquecino de aspecto traquealizado.


Su patogenia está basada en una inflamación crónica mediada por eosinófilos a nivel
2. 2. esofágico siendo diagnóstico un recuento mayor de 15 eosinófilos/campo de gran aumento
en muestras de tercios medio y distal esofágicos.
3. 3. La pHmetría 24 horas suele ser patológica.
4. 4. La retirada de los alimentos sospechosos puede ayudar a mejorar la sintomatología.
Gráfico de respuestas
Comentario

Tal como se explicaba en el comentario anterior, la traquealización esofágica es muy sugestiva de


este trastorno. En caso de asociarse alimentos a los que el paciente sea alérgico, su retirada

 
 
 
 
puede ayudarnos a controlar los síntomas. Es importante distinguir esta entidad de la enfermedad
por reflujo gastroesofágico, con la que se confunde en muchas ocasiones (respuesta 3 falsa).(R3)

127. A la vista del frotis de


sangre periférica de la imagen, todas las siguientes pruebas complementarias están
indicadas excepto:

1. 1. Prueba de Coombs.
2. 2. Anticuerpos antinucleares.
3. 3. LDH sérica.
4. 4. Test de Ham.
Gráfico de respuestas
Comentario

Aparte de algunos hematíes normales, lo que observamos en este frotis son otros de volumen
mucho más reducido, con una coloración más intensa (ya que albergan una cantidad semejante de
hemoglobina, pero en un volumen menor). Se trata, por tanto, de esferocitos. Como sabes, los
esferocitos pueden verse en la esferocitosis hereditaria, pero también en las anemias
inmunohemolíticas. Por ello, carece de sentido solicitar un test de Ham (test de la hemólisis ácida).
Esta prueba es útil para el diagnóstico de la hemoglobinuria paroxística nocturna, donde también
existe una hemólisis, pero que no es mediada por anticuerpos.(R4)

128. El hallazgo corresponde a una mujer de 35 años con clínica de debilidad de reciente
aparición con el siguiente hemograma: Hb 8 g/dl, VCM 102 fl, leucocitos 5.500/microl,
plaquetas 60.000/microl, reticulocitos 150.000/microl. ¿Cuál es el diagnóstico más
probable entre los siguientes?

1. 1. Síndrome de Evans.
2. 2. Anemia megaloblástica por déficit de folato.
3. 3. Anemia perniciosa.
4. 4. Esferocitosis hereditaria.
Gráfico de respuestas
Comentario

Dadas las cifras de reticulocitos, se trataría de una anemia regenerativa, lo que descarta las
respuestas 2 y 3. Tampoco tiene sentido la 4, ya que no explica la trombopenia.

 
 
 
 
La única solución posible es la 1. El síndrome de Evans consiste en una anemia hemolítica
autoinmune que, además, se acompaña de trombopenia (también autoinmune).(R1)

129. Y, por tanto, el tratamiento más adecuado para el proceso referido en la pregunta
anterior es:

1. 1. Esteroides.
2. 2. Ácido fólico.
3. 3. Ácido fólico y B12.
4. 4. Fludarabina.
Gráfico de respuestas
Comentario

El tratamiento de una anemia hemolítica autoinmune consiste, al menos de inicio, en corticoides


(respuesta 1 correcta). Lo mismo sucede cuando estamos ante una púrpura trombopénica
idiopática, que no deja de ser otro problema inmunológico basado en autoanticuerpos.(R1)

130. ¿En cuál de las siguientes patologías NO se incrementa el pH vaginal?

1. 1. Vaginitis por Trichomonas.


2. 2. Candidiasis vulvovaginal.
3. 3. Vaginitis inflamatoria.
4. 4. Vaginitis atrófica.
Gráfico de respuestas
Comentario

En el transcurso de la infección por cándida, el pH vaginal se muestra dentro de los valores


normales (en torno a 4.5); si bien, su desarrollo es favorecido a partir de una variación del pH
normal hacia la alcalinidad. Respuesta correcta, la número 2.(R2)

131. Paciente varón de 56 años de edad, que tras presentar cuadro clínico de hematuria
macroscópica, a la exploración física presentaba dolor a la palpación en el flanco
derecho. Tras los estudios de imagen pertinentes es diagnosticado de tumor renal de
células claras. Tras el estudio de extensión, que fue negativo, se procedió al tratamiento
de la lesión. Desde el momento del tratamiento hasta la actualidad el paciente ha
permanecido asintomático durante tres años. Sin embargo, en uno de los controles
rutinarios en el servicio de oncología, se detecta un nódulo en un pulmón. El tratamiento
más útil en el caso de este paciente es:

1. 1. Radioterapia.
2. 2. Tratamiento conservador.
3. 3. Nodulectomía.
4. 4. Quimioterapia.
Gráfico de respuestas
Comentario
No hay comentario(R3)

 
 
 
 
132. El síndrome de Pierre Robin se caracteriza por:

1. 1. Hepatomegalia, esplenomegalia y onfalocele.


2. 2. Hernia umbilical, ictericia prolongada y fontanela anterior amplia.
3. 3. Hipoplasia mandibular con desplazamiento posterior de la lengua.
4. 4. Debilidad de músculos abdominales, criptorquidia y anomalías genitourinarias.
Gráfico de respuestas
Comentario

El síndrome de Pierre Robin combina micrognatia, caída de la lengua hacia atrás y paladar
hendido. La opción 1 es el síndrome de Wiedemann-Beckwith; la opción 2 es el hipotiroidismo
congénito; la opción 4 es el síndrome de Eagle-Barret (o de prune belly o de vientre en ciruela
pasa).(R3)

133. Un paciente de 52 años con el diagnóstico de úlcera pilórica y vómitos de repetición


de una semana de duración acude al hospital con una tensión arterial de 100/58 mmHg y
los siguientes laboratorios: plasma Na + 140 mmol/l, K + 2.2, mmol/1, Cl- 86 mmol/l, HCO3
42 mmol/l, pH 7.53, pCO2 53 mmHg y creatinina 2.9 mg/dl; orina Na+ 2 mmol/l, K + 21
mmol/l, pH 5. ¿Cuál de los siguientes diagnósticos es cierto?

1. 1. Alcalosis mixta.
2. 2. Acidosis metabólica con brecha aniónica o anion gap normal.
3. 3. Alcalosis metabólica.
4. 4. Acidosis hiperclorémica.
Gráfico de respuestas
Comentario

Una pregunta bastante sencilla sobre la valoración del equilibrio ácido-base. A continuación, vamos
a proceder mediante el descarte de opciones:

•   Dado que el pH es 753, está claro que no es una acidosis, lo que descarta las opciones 2 y
4.
•   Para hablar de alcalosis mixta, debería existir un descenso de la PaCO2 y un aumento del
bicarbonato sérico. No es el caso (de hecho, la PaCO2 está aumentada), de manera que el
motivo de esta alcalosis es exclusivamente metabólico, no mixto (respuesta 3 correcta).
•   En general, siempre que se trate de vómitos de larga evolución, hay que pensar en una
pérdida ácida importante en forma de HCl. La descompensación hidroelectrolítica típica en
estos casos es la hipocloremia con alcalosis metabólica.

(R3)

134. Gestante secundigesta, con antecedente de un parto anterior en la semana 35,


consulta en la semana 32 por percibir contracciones. En la exploración se comprueba un
cérvix permeable al dedo, borrado en 30% y con 3 cm de longitud ecográfica. La
monitorización cardiotocográfica fetal revela una frecuencia cardiaca de 150 lpm. y una
contracción uterina cada 5 minutos. ¿Cuál de las siguientes afirmaciones es
INCORRECTA?:

Deberíamos realizar el test de la fibronectina exocervical, cuya negatividad habla a favor


1. 1.
de una amenaza de parto pretérmino establecida.
2. 2. La presencia de fibronectina exocervical es fisiológica hasta la semana 20 del embarazo.

 
 
 
 
Se aconseja la administración de corticoides para maduración pulmonar por el antecedente
3. 3.
de parto pretérmino.
La pérdida prematura del tapón mucoso (moco cervical) se relaciona con un aumento de
4. 4.
riesgo para parto pretérmino.
Gráfico de respuestas
Comentario

No debería pasar desapercibido que las respuestas 1 y 2 se contradicen (si la presencia de


fibronectina exocervical es fisiológica hasta la semana 20, más allá de dicha semana lo fisiológico
es su ausencia, por tanto la respuesta 1 no tendría sentido). Esto les tiene que hacer sospechar. Y
en efecto, un marcador de riesgo de parto pretérmino sería un test de fibronectina exocervical
POSITIVO en este caso de gestación de 32 semanas, por tanto la respuesta 1 es la falsa.(R1)

135. Mujer de 70 años con antecedentes personales de artritis reumatoide y enfermedad


de Crohn de larga evolución. Función renal en revisiones dentro de la normalidad. En un
control reumatológico rutinario se aprecia: Cr 2 mg/dl, CCr de 50 ml/min, proteinuria 5g
/día, colesterol 350 mg/dl y proteínas totales de 4,5 g/dl. En el espectro electroforético de
la orina no se aprecia ningún pico monoclonal, es decir, existe una proteinuria no
selectiva. ¿Cuál sería la lesión más frecuente que se podría encontrar en la biopsia
renal?:

1. 1. GN membranosa.
2. 2. Amiloidosis secundaria.
3. 3. GN mesangial.
4. 4. Nefropatía de cambios mínimos.
Gráfico de respuestas
Comentario
Esta pregunta ya ha caído en el MIR por lo que esto significa que es una pregunta que NO debes
fallar. Nos preguntan acerca de un síndrome nefrótico (proteinuria mayor de 3.5 gr/día) sin ningún
pico monoclonal en la electroforesis de orina en una paciente con antecedentes de enfermedad de
Crohn y artritis reumatoide. Debes recordar que esta última enfermedad es una causa importante
de amiloidosis secundaria, que además se manifiesta de esta forma en los pacientes. El tipo de
amiloide que nos encontraríamos en este caso sería AA visible en la biopsia renal a nivel
glomerular en el mesangio y en la pared capilar sin proliferación celular en forma de un material
amorfo e hialino.(R2)

136. Respecto a la etiopatogenia de la endometriosis, señale la respuesta FALSA:

Según la teoría del desarrollo “in situ” la endometriosis puede originarse en el sitio en el
1. 1. que se diagnostica y puede aparecer a partir de restos müllerianos o del conducto de
Wolff.
Según la teoría de la inducción, células mesenquimales se diferenciarían gracias a
2. 2.
sustancias liberadas por el endometrio que han llegado a la cavidad abdominal.
Según la teoría del implante, la endometriosis se produce por el transplante a través de
3. 3.
menstruación retrógrada de células endometriales.
Según la teoría del implante, la endometriosis se produce por implante directo vía
4. 4. hematógena de células endometriales procedentes de cavidad peritoneal a la que han
llegado a través de menstruación retrógrada.
Gráfico de respuestas
Comentario

 
 
 
 
Pregunta sencilla y directa sobre la endometriosis. La etiopatogenia es poco conocida pero en la
actualidad se proponen las siguientes teorías:

- Teoría del desarrollo “in situ”: se origina en el sitio en el que aparece a partir de restos
müllerianos, del conducto de Wolff o metaplasia de tejido peritoneal u ovárico.

- Teoría de la inducción: diferenciación de células mesenquimales activada por sustancias


generadas por el endometrio a su llegada a la cavidad endometrial.

- Teoría del trasplante o implante: trasplante e implante posterior durante la menstruación a través
de las trompas.

Otra vez se presentan dos opciones excluyentes entre si, opciones 3 y 4, por lo que alguna de las
dos muy probablemente sea la opción a elegir.(R4)

137. A mother comes to the emergency department with her 3-year-old child. Playing at a
local playground, the child tripped so her mother grabbed the child's left arm to prevent
a fall. The girl presents with pain and imobility of the left arm, holding it against her body
in a flexed, adducted position. The child is in no acute distress, her vital signs are stable
and you find no other signs of injury. The child has full passive range of motion of the
wrist and fingers. There is no swelling or ecchymosis. X-Rays of the left elbow are normal.
What is the most likely cause of these symptoms?

1. 1. Panner disease
2. 2. Nursemaid's elbow
3. 3. Lateral epicondilytis
4. 4. Elbow dislocation
Gráfico de respuestas
Comentario
Nursemaid's elbow. This is is the typical presentation of a case of nursemaid’s elbow. Nursemaid’s
elbow is a subluxation of the radial head caused by a sudden pull on the extended forearm. The
patients present with intense pain and functional impairment with normal passive mobility except for
supination.(R2)

138. El agente más frecuente de sepsis nosocomial en el recién nacido es:

1. 1. Estreptococo del grupo B.


2. 2. Candida albicans.
3. 3. Staphylococcus epidermidis.
4. 4. Haemophilus influenzae.
Gráfico de respuestas
Comentario

Los principales factores de riesgo son la prematuridad y los procedimientos invasivos.

El agente etiológico más frecuente son los estafilococos coagulasa negativos, como el
Staphylococcus epidermidis.

 
 
 
 
El diagnóstico de confirmación es a través de hemocultivos positivos y además podremos obtener
el antibiograma.(R3)

139. En el estudio histológico de un tumor ovárico se observan unas microcalcificaciones


denominadas "cuerpos de psamoma". ¿En qué tipo tumoral debemos pensar como más
probable?:

1. 1. Cistoadenoma seroso papilar.


2. 2. Cistoadenocarcinoma mucinoso.
3. 3. Tumor de Krukenberg.
4. 4. 4)Tumor de las células de Leydig.
Gráfico de respuestas
Comentario

Las microcaalcificaciones aparecen en la mayoría de los tumores seroso bien diferenciados.

Los tumores de Brenner se caracterizan por epitelio semejante al transicional de la vejiga.

El tumor de Krukenberg por las células en anillo de sello.

Recuerde que los cuerpos de Call - Exner (regiones acelulares PAS +) son típicos de los tumores
de la granulosa.(R1)

140. En un ciclo normal, de 34 días de duración, la ovulación ocurre:

1. 1. Dos semanas antes del flujo menstrual.


2. 2. A mitad del ciclo.
3. 3. En el momento de la producción máxima de progesterona.
4. 4. Antes de la producción máxima de estrógenos.
Gráfico de respuestas
Comentario

Una vez liberado el óvulo, el tiempo hasta la menstruación es generalmente de dos semanas.

141. A 4-year-old child is brought to the emergency department presenting with pain and
bulging of the right knee. He is afebrile. Articular examination shows a restriction of
movement with signs of intraarticular fluid. The mother says that the child has frequent
bruises and nosebleeds. What should be done next?

1. 1. Perform an MRI of the knee


2. 2. Initiate antibiotics
3. 3. Articular surgical drainage
4. 4. Request coagulation tests
Gráfico de respuestas
Comentario
Request coagulation tests. The disorder described is a knee hemarthrosis. Joint haemorrhages are
rare. They can occur in cases of high-energy trauma or coagulative disorders. In this case, the
history of nosebleeds favours a blood alteration, which should be studied.(R4)

 
 
 
 
142. ¿Cuál de los siguientes fármacos sería el MENOS útil para tratar a un enfermo con
una crisis palúdica aguda?:

1. 1. Quinina.
2. 2. Primaquina.
3. 3. Hidroxicloroquina.
4. 4. Mefloquina.
Gráfico de respuestas
Comentario
Sobre la malaria hay un aspecto que debemos recordar. Existe un primer ciclo reproductivo
(asexual) al interior de los hepatocitos, y posteriormente ciclos repetidos de reproducción al interior
de los eritrocitos (responsables de las crisis febriles a intervalos más o menos regulares). Dos
especies de Plasmodium, vivax y ovale, son capaces de quedar "durmientes" al interior de los
hepatocitos. Los antipalúdicos actúan sobre los trofozoitos del ciclo eritrocítico. Si no queremos
tener una recaída posterior por los "hipnozoitos" que quedaron en el hígado, debemos administrar
primaquina, que sí es capaz de erradicar los plasmodium del hígado. Por lo tanto, dado que es un
ataque AGUDO, y que no nos dicen si la etiología es por vivax u ovale, no está indicada la
primaquina, serán más útiles por tanto el resto de los antipalúdicos que nos presentan.(R2)

143. En el cáncer de mama, ¿cuándo se emplea la radioterapia?:

1. 1. Siempre que el tumor sea > 2 cm.


2. 2. Cuando la linfadenectomía es positiva.
3. 3. Como coadyuvante de la cirugía conservadora.
4. 4. Como coadyuvante a la mastectomía.
Gráfico de respuestas
Comentario

Pregunta bastante sencilla e importante si aplicamos el algoritmo del tratamiento del cáncer de
mama.

Las indicaciones de la radioterapia en el tratamiento del cáncer de mama son fáciles de conocer.
Así, serán subsidiarias de radioterapia siempre que se realice cirugía conservadora (tumorectomía)
para el tratamiento del cáncer de mama, ya que disminuye la tasa de recidivas y alcanza la misma
supervivencia que la mastectomía; por lo cual la respuesta correcta es la opción 3.

También se aplica la radioterapia sobre la pared torácica y áreas ganglionares regionales tras una
mastectomía en aquellas pacientes consideradas de alto riesgo (es la condición de ser de alto
riesgo la que hace que la opción 4 no sea válida, ya que no se aplica radioterapia a todas las
pacientes mastectomizadas) ya que disminuye la recaída locorregional y aumenta la supervivencia
global.(R3)

144. Señale la respuesta FALSA si hablamos del VIH en niños:

1. 1. La inversión del cociente CD4/CD8 es más precoz que en adultos.


2. 2. TB y hepatitis B son menos frecuentes en niños que en adultos.
3. 3. Un 10% cursan con hipogammaglobulinemia, que es factor de mala evolución.
4. 4. El aumento del TNF sería marcador precoz de alteración del SNC en niños con SIDA.
Gráfico de respuestas
Comentario

 
 
 
 
Debemos conocer unos datos fundamentales del SIDA pediátrico, sobre todo las diferencias con el
SIDA adulto y entre ellas tenemos:

Hay menos VHB, menos TB y menos tumores que en el adulto;

Clínicamente son típicas la NIL y la parotiditis crónica;

Laboratorio es típico una hipergammaglobulinemia más precoz, una linfopenia menos llamativa y
más tardía y una inversión del cociente CD4/CD8 también más tardía que en adultos.

Por lo tanto, respuesta 1 correcta.(R1)

145. A un paciente obeso y roncador intenso, sin somnolencia diurna, con antecedentes
de insuficiencia cardíaca, se le practica una polisomnografía después de haber detectado
su pareja pausas respiratorias durante el sueño. El registro muestra un índice de apnea-
hipopnea de 4/hora a expensas de apneas obstructivas, una SaO2 media durante el sueño
de 94% y una SaO2 mínima de 86%. El diagnóstico es:

1. 1. Se trata de un roncador simple que no precisa tratamiento.


2. 2. Se trata de un síndrome de apnea del sueño subsidiario de tratamiento con CPAP nasal.
3. 3. Se trata de un síndrome de apneas centrales secundarias a la insuficiencia cardíaca.
4. 4. Los resultados de polisomnografía no son compatibles con la historia clínica.
Gráfico de respuestas
Comentario

El método diagnóstico del síndrome de apnea obstructiva del sueño (SAOS) es la polisomnografía.
Esta técnica pone de manifiesto los episodios de cese del flujo aéreo con mantenimiento del
esfuerzo respiratorio. Se considera diagnóstico de SAOS un índice apnea-hipopnea mayor de diez.

En el paciente de la pregunta, las opciones 2, 3 y 4 se eliminan de forma inmediata, ya que no


cumple este criterio, de manera que no cabe hablar de SAOS. La única opción válida es la 1: se
trata de un roncador simple, dado que no cabe hablar de SAOS, y por el momento no precisa
intervención terapéutica.(R1)

146. En una paciente de 64 años que acude a consulta por aumento del perímetro
abdominal, dispepsia y astenia de cuatro meses de evolución y, que en la exploración
ultrasonográfica se observa una masa anexial derecha compleja bilateral y líquido libre
en pelvis, señale la respuesta correcta:

Dada la edad de la paciente, el primer diagnóstico de sospecha sería un tumor del seno
1. 1.
endodérmico, que es altamente maligno y puede producir alfa-fetoproteína.
Con probabilidad se trata de un tumor de Brenner, uno de los más frecuentes dentro de los
2. 2.
tumores epiteliales de ovario y, que en su mayoría son malignos.
3. 3. Es poco probable el carácter bilateral de los tumores serosos de ovario.
4. 4. Existe una alta posibilidad de que el Ca 125 se encuentre elevado.
Gráfico de respuestas
Comentario
Los tumores epiteliales son los más frecuentes dentro de los tumores de ovario, y dentro de los
epiteliales, el más frecuente es el seroso (60- 80 %), que con elevada frecuencia es bilateral. El Ca
125 suele aparecer elevado en tumores epiteliales de ovario, aunque también puede aparecer
elevado en otras circunstancias (menstruación, endometriosis,…). El tumor del seno endodérmico

 
 
 
 
es maligno y puede producir alfa- fetoproteína, pero es un tumor de estirpe germinal, que son más
frecuentes en mujeres jóvenes. El tumor de Brenner es el tumor epitelial menos frecuente y suele
ser benigno por lo general. Los tumores de ovario más frecuentes en mujeres con antecedentes de
endometriosis son el tumor de células claras y el carcinoma endometriode.(R4)

147. Niño de 5 años que presenta


fiebre de 39.5ºC en los últimos 7 días. Los padres describen que los primeros días además
de la fiebre le notaron enrojecimiento de las palmas y las plantas además de edema de
pies y manos. Por ello acudieron a su pediatra que le diagnosticó de viriasis y urticaria y
le pautó antitérmicos y antihistamínicos. Actualmente presenta un exantema de
predominio en tronco, una adenopatía cervical de 2 cm además de cambios bucales como
los que se aprecian en la imagen. En relación al cuadro clínico citado, señale la opción
CORRECTA:

1. 1. El diagnóstico más probable es el de reacción medicamentosa.


2. 2. El cuadro se ha relacionado claramente con el Streptococcus pyogenes.
La presencia de una adenopatía grande junto a fiebre obliga a descartar un cuadro
3. 3.
linfoproliferativo mediante una biopsia medular.
4. 4. La base patogénica de la enfermedad es una vasculitis.
Gráfico de respuestas
Comentario

Se trata de una pregunta de dificultad media sobre una enfermedad preguntada en varias
ocasiones en el ENARM como es la enfermedad de Kawasaki.

En este caso para contestar adecuadamente a la pregunta tenemos que en primer lugar darnos
cuenta de que todos los datos clínicos que la pregunta nos ofrece son muy sugerentes de
enfermedad de Kawasaki: fiebre alta de al menos 5 días de evolución, adenopatía cervical mayor
de 1.5 cm, exantema polimorfo, eritema palmo- plantar y edema de manos y pies además de los
labios rojos y fisurados que nos muestra la imagen.

Una vez hecho el diagnóstico nos piden la respuesta correcta que en este caso es la 4 ya que
efectivamente desde el punto de vista patogénico se trata de una vasculitis secundaria al daño
inmunitario del endotelio. La causa no es bien conocida pero se cree en la teoría de que la
respuesta inmune esté mediada por superantígenos.(R4)

 
 
 
 
148. El paciente descrito en la pregunta anterior fue ingresado para estudio y tratamiento
específico. Señale la respuesta FALSA:

1. 1. La trombocitosis es característica de la fase subaguda de la enfermedad.


2. 2. La aspirina a dosis antiagregantes es importante en la prevención de las trombosis.
3. 3. El pronóstico depende de la afectación renal y pulmonar.
4. 4. Los corticoides y el infliximab se recomiendan en casos refractarios a gammaglobulina.
Gráfico de respuestas
Comentario

La pregunta se centra en el manejo y las posibles complicaciones de la enfermedad de Kawasaki.


Recuerde que en la fase aguda de la enfermedad es frecuente la leucocitosis con desviación a la
izquierda así como la elevación de la VSG y PCR. Sin embargo la trombocitosis es un hallazgo
típico de la fase subaguda.

El tratamiento de elección es la gammaglobulina intravenosa que produce una desaparición rápida


de los síntomas y previene la formación de aneurismas siempre que se administre en los 10
primeros días de evolución, preferiblemente en los primeros 7.

La aspirina a dosis antiagregantes previene la aparición de trombos sobre los aneurismas y se ha


de mantener hasta normalización de las plaquetas en caso de no haber aneurismas o hasta la
desaparición de los mismos si es que se han producido.

En la mayoría de los casos la gammaglobulina permite controlar el cuadro pero si no es así y


persiste la fiebre pueden utilizarse corticoides de segunda línea e incluso un fármaco anti- TNF
como el infliximab.

En cuanto al pronóstico es excelente si no existe afectación cardíaca. La mortalidad oscila entre el


0.5% y el 2.8% siempre en relación con la afectación coronaria, no siendo el riñón ni el pulmón
órganos afectados por la enfermedad en la mayoría de los casos (respuesta 3 FALSA). Pese a
constituir una complicación potencialmente grave, el 50% de los aneurismas desaparecen en 1 o 2
años.(R3)

149. Un niño de 4 años acude al pediatra por aparición de lesiones papulosas rojas,
algunas vesículas blanquecinas no umbilicadas en tronco y mucosa oral, muy
pruriginosas. Dos días antes presentaba cuadro catarral con fiebre moderada. Respecto
al cuadro que presenta este niño, todo lo siguiente es cierto, EXCEPTO:

1. 1. Esta causado por un poxvirus, el virus de la varicela zoster.


2. 2. Las lesiones costrosas no contienen virus viables.
3. 3. La complicación más frecuente es la sobreinfección de las lesiones cutáneas.
La encefalitis postvaricela que se presenta con signos cerebelosos tiene mejor pronostico
4. 4.
que si lo hace con signos cerebrales.
Gráfico de respuestas
Comentario

Esta causada por un herpesvirus, no un poxvirus. Respuesta 1 correcta. El resto de respuestas son
correctas.

Insistimos en que debes dominar el tema que se pregunta año con año, esta hablando de VVZ que
es herpes virus tipo 3, la opción 2, 3 y 4 son correctas.(R1)

 
 
 
 
150. ¿Cuál de las siguientes afirmaciones es FALSA en referencia a los riesgos asociados
a una anestesia general y una intervención quirúrgica en una paciente gestante?

1. 1. Los riesgos teratógenos son mayores en el primer trimestre de gestación.


2. 2. Los riesgos de parto prematuro son máximos en el tercer trimestre de gestación.
3. 3. Los anestésicos generales pueden provocar hipoperfusión e hipoxemia fetal.
La hemorragia quirúrgica no influye sobre el bienestar fetal ya que la circulación
4. 4.
placentaria tiene auto-regulación.
Gráfico de respuestas
Comentario

Una cuestión complicada. Sin embargo, esto no debe impedir que luchemos por la pregunta (y, en
esta ocasión, podemos prácticamente asegurar el punto, si tenemos sentido común).

•   R1: como sucede con la mayoría de los fármacos en infecciones connatales, cuanto más
precozmente actúan sobre el embrión, los efectos teratógenos son mayores. Dado que los
anestésicos no dejan de ser fármacos, deberían ajustarse a este principio general, por lo
que la opción 1 es cierta.
•   R2: el parto prematuro es una complicación propia del tercer trimestre, por lo que esta
opción también debe ser cierta. ¡Antes de las 20 semanas, se llamaría aborto, por lo que
difícilmente encontraremos un parto pretérmino en el primer trimestre!
•   R3: la hipoxia es una de las causas más frecuentes de morbimortalidad en anestesia, por
lo que esta opción no debe sorprenderle. De todas formas, aunque no sepa esto, en un
examen tipo test, es de locos desafiar al poder del “puede” si no se está muy seguro (lee
atentamente la redacción de la pregunta).

Solamente nos queda la respuesta 4, por si nos cabía alguna duda, porque lo que dice es, en sí
mismo, bastante descabellado. Por supuesto, si la madre padece una hemorragia importante, esto
puede repercutir sobre el bienestar fetal (a pesar de los mecanismos autorreguladores de la
placenta, que por supuesto tendrán un límite).(R4)

151. Paciente femenino de 38 años consultó por prurito y astenia. En la exploración física
presentaba una hepatomegalia lisa a 2 cm. del reborde costal. Presentaba los siguientes
datos de laboratorio: bilirrubina 1.6 mg/dl; fosfatasa alcalina 1.040 (N<280); el resto de la
bioquímica hepática era normal. Una ecografía abdominal fue normal, así como una
colangiografía retrógrada endoscópica. ¿Cuál de los siguientes parámetros sería más útil
en el diagnóstico?:

1. 1. ANCA.
2. 2. AMA.
3. 3. Porfirina en orina.
4. 4. Ceruloplasmina sérica.
Gráfico de respuestas
Comentario

Se trata de una paciente con colestasis intrahepática, ya que presenta elevación marcada de la
fosfatasa alcalina y elevación leve de la bilirrubina, y la ecografía abdominal no muestra dilatcaión
de la vía biliar extrahepática. Estos hallazgos, en una mujer, nos deben hacer sospechar una
cirrosis biliar primaria. La CPRE descarta otras causas de colestasis intrahepática. El parámetro
más útil para su diagnóstico son los anticuerpos antimitocondriales (AMA), aunque se debe
confirmar con una biopsia hepática.(R2)

 
 
 
 
152. Un paciente de 81 años consulta por un episodio de síncope. Refiere disnea de
esfuerzo desde hace un año. En la auscultación cardíaca destaca un soplo sistólico 3/6
en borde esternal izquierdo que irradia a carótidas y punta, ¿qué patología le parece más
probable?

1. 1. Bloqueo auriculo-ventricular de tercer grado.


2. 2. Estenosis valvular aórtica degenerativa.
3. 3. Miocardiopatía hipertrófica.
4. 4. Miocardiopatía dilatada.
Gráfico de respuestas
Comentario

Se trata de un paciente añoso que sufre un síncope y presenta disnea de esfuerzo de un año de
evolución. Conviene recordar que los síntomas típicos de la estenosis valvular aórtica son la
presencia de angina de esfuerzo, síncope de esfuerzo y disnea. En los últimos años y dado que la
enfermedad cada vez con mayor frecuencia aparece como consecuencia de una valvulopatía
fibrocalcificada degenerativa-senil, los pacientes muy raramente describen angina de esfuerzo (que
era el síntoma mas frecuente en épocas previas en la que la etiología dominante era la reumática
que afecta a individuos de edad menos avanzada) sino “equivalentes anginosos” propios de la
edad avanzada, sobre todo, la disnea de esfuerzo progresiva que se ha hecho el síntoma mas
frecuente en la actualidad.

La auscultación corrobora el diagnóstico ante el hallazgos de un soplo sistólico en borde esternal


izquierdo (no nos refieren a qué altura se ausculta, pero probablemente es en la base cardíaca en
segundo espacio intercostal izquierdo paraesternal) que se irradia a las carótidas (lo que confirma
que se trata de un soplo aórtico) y a la punta (lo que se ha dado en llamar fenómeno de
Gallavardin). Esta es la valvulopatía más frecuente en la actualidad en Occidente.(R2)

153. Varón de 40 años que, en un control rutinario en su empresa, presenta una glucemia
de 130 mg/dl en una toma de plasma venoso cogida en ayunas. ¿Cuál es la correcta?

1. 1. Diagnóstico de diabetes mellitus (DM).


2. 2. Diagnóstico de intolerancia hidrocarbonada.
3. 3. Diagnóstico de glucemia en ayunas alterada.
Hay que descartar el diagnóstico de DM, por lo que necesito confirmarlo con una prueba
4. 4.
de sobrecarga oral de glucosa alterada o con otra analítica equivalente.
Gráfico de respuestas
Comentario

Como sabes, el diagnóstico de diabetes mellitus podría establecerse si el paciente tuviese una
glucemia de 126 mg/dL y, al menos, pudiésemos confirmarla en un día distinto mediante otra
determinación. Otra alternativa sería realizar una sobrecarga oral de glucosa (>200 mg/dL a las
dos horas tendría el mismo valor confirmatorio). Lo que tiene que quedar claro en esta pregunta es
que, con una medición aislada en un paciente asintomático (revisión rutinaria, como en este caso),
no podemos establecer el diagnóstico de diabetes mellitus.(R4)

154. Un paciente de 6 años es llevado a consulta para control en Salud. De sus


antecedentes se destacan: residencia en medio socioeconómico desfavorable hasta los
3 años, período durante el cual fué hospitalizado en 3 oportunidades por deshidratación
y desnutrición. Luego de esa edad fue incorporado a un ambiente urbano de nivel

 
 
 
 
socioeconómico medio con cobertura de todas sus necesidades básicas. ¿Cuál es el
indicador antropométrico actual MÁS probablemente afectado?:

1. 1. Talla/ Edad.
2. 2. Peso/ Talla.
3. 3. Velocidad de peso.
4. 4. Ninguna de las anteriores.
Gráfico de respuestas
Comentario

La evaluación del crecimiento es muy importante ya que puede ser o un síntoma de enfermedad o
un indicador de salud. Un retraso en la edad pondero-estatural nos debe hacer pensar en una
enfermedad subyacente.

- Una enfermedad crónica o sistémica (digestiva, hepática, renal, cardíaca...) afecta


fundamentalmente al PESO y luego a la talla.

- Una enfermedad endocrinológica o de crecimiento óseo afecta sobretodo a la TALLA y luego al


peso.

La velocidad de crecimiento es el parámetro más sensible para la identificación de un


hipocrecimiento. Consideramos hipocrecimiento cuando la velocidad de crecimiento es inferior al
percentil 3.

La respuesta correcta es la 1.(R1)

155. ¿Cuál de las siguientes bacterias es grampositiva?:

1. 1. Brucella melitensis.
2. 2. Corynebacterium diphtheriae.
3. 3. Acinetobacter calcoaceticus.
4. 4. Pasteurella multocida.
Gráfico de respuestas
Comentario
Corynebacterium diphtheriae es un bacilo grampositivo, aerobio, no esporulado. Algunas cepas de
C. diphtheriae producen toxina diftérica. El gen de la toxina diftérica se encuentra codificado en un
fago que se integra en el gen del Corynebacterium diphtheriae, que adquiere de esta forma la
capacidad de producir toxina diftérica. El medio selectivo para Corynebacterium diphtheriae es en
medio con telurito y cisteína, como el agar de Tinsdale.(R2)

156. A 5-year-old child with a history of severe persistent asthma that requires occasional
short cycles of steroids is brought to the emergency department for presenting
respiratory difficulty that do not respond to inhaled beta agonists. He is tachypneic and
the use of accesory muscles is evident upon physical examination. Arterial blood gases
and pulse oximetry results on room air are: pH 7.2, pO2 55 mmHg, pCO2 47 mmHg. What
should be done as the first step in the management of this patient?

1. 1. Cricothyroidotomy
2. 2. Intravenous epinephrine

 
 
 
 
3. 3. Start theophylline
4. 4. Endotracheal intubation and mechanical ventilation
Gráfico de respuestas
Comentario
Endotracheal intubation and mechanical ventilation. Respiratory distress in this child continues to
worsen despite treatment. In the control arterial blood gas there is hypercapnia, hypoxemia and
respiratory acidosis. This situation is extremely serious because there is imminent risk of respiratory
arrest. The first step to take is to secure the airway by intubation and mechanical ventilation
regardless of the cause of the respiratory distress.(R4)

157. Usted está de médico en un puesto de carretera de la Cruz Roja y le llaman porque
ha habido un accidente grave. Cuando llega al lugar del siniestro hay un hombre
inconsciente tendido en el asfalto junto a un charco de sangre. ¿Qué es lo primero que
haría?:

1. 1. Auscultarle el corazón.
2. 2. Buscar el origen de la hemorragia y cortarla.
3. 3. Realizar una exploración neurológica.
4. 4. Comprobar si la vía aérea está libre y si respira.
Gráfico de respuestas
Comentario

Esta pregunta es muy importante ya que el manejo del politraumatizado es un tema muy rentable
en el ENARM y lo repasaremos las veces que ea necesario. Para valorar el orden de priorización
ante un accidentado utilizamos el acrónimo ABCDE. El primer paso ante un accidentado es
mantener la vía aérea permeable (airway), es decir comprobar que la vía aérea esté libre y
comprobar si el paciente respira. Una vez comprobado este paso, el siguiente es comprobar si
ventila correctamente (breathing). Como tercer paso es controlar las hemorragias y la TA, es decir
las respuestas 1 y 2. El cuarto paso es control de las posibles lesiones NRL, respuesta 3, y valorar
posibles lesiones internas (disability). El último paso se reserva el valorar las fracturas
(extremities).(R4)

158. A 51-year-old patient undergoes a screening mammography, an 8-mm spiculated


nodule is detected in the upper outer quadrant of the left breast. Core biopsy reveals that
it is an infiltrating ductal carcinoma GII. The hormone receptor status is positive. Ki-67
10%, HER2-neu negative. Clinical examination: no palpable nodule, free armpit. Axillary
ultrasound reveals no suspicious nodes. What is the primary treatment of choice?

1. 1. Primary hormonal therapy.


2. 2. Neoadjuvant chemotherapy.
3. 3. Mastectomy and axillary lymphadenectomy.
4. 4. Tumorectomy and sentinel node biopsy.
Gráfico de respuestas
Comentario
Tumorectomy and sentinel node biopsy. In cases of small tumors, conservative resection
(lumpectomy) is prefered to complete mastectomy. Moreover, since it is a very small tumor found
incidentally, it is better to perform the technique of sentinel node (if negative, it would avoid a
complete lymphadenectomy).(R4)

 
 
 
 
159. Dentro de las funciones de la placenta, NO se incluye:

Transporte activo de sustancias como las vitaminas liposolubles o los aminoácidos hacia
1. 1.
el lado fetal.
2. 2. Síntesis hormonal, para mantener la gestación y favorecer el crecimiento y desarrollo fetal.
Transporte por pinocitosis de moléculas de gran tamaño, como fosfolípidos o
3. 3.
lipoproteínas.
4. 4. Síntesis de inmunoglobulinas, que serán útiles como defensa del neonato.
Gráfico de respuestas
Comentario

Una pregunta bastante sencilla que no debería haberte planteado problema alguno. La placenta
permite el paso de determinadas inmunoglobulinas a su través (IgG, únicamente), pero no participa
en modo alguno en su síntesis, tarea que está reservada a los linfocitos B y células plasmáticas
(respuesta 4 falsa).(R4)

160. La taquipnea transitoria del recién nacido o síndrome de dificultad respiratoria tipo
II se caracteriza por:

1. 1. Distress respiratorio severo de inicio tardío.


Taquipnea con distress leve a moderado y resolución espontanea dentro de las 72 horas de
2. 2.
vida.
3. 3. Taquipnea con broncograma aéreo.
4. 4. Requiere corticoides para su curación.
Gráfico de respuestas
Comentario

Debes dominar toda la patología respiratoria del recién nacido para el ENARM. Este síndrome se
suele producir por un parto rápido o por un parto por cesarea en la que el drenaje por presión de
los pulmones no se ha completado. Normalmente se trata de un distrés leve, que resuelve
espontaneamente y en menos de 3 días y en la radiografía se ve líquido en cisuras.(R2)

161. A 4-year-old girl is brought to consultation for presenting recurrent episodes of


nausea and vomiting during the last year. The episodes begin with pallor and intense
photophobia and then nausea with vomiting almost twelve times an hour. The hole
episode lasts over one week and is followed by an asymptomatic period. What is the most
common complication of this condition?

1. 1. Bulimia
2. 2. Dehydration and electrolyte abnormalities
3. 3. Intussusception
4. 4. Peptic ulcer
Gráfico de respuestas
Comentario
Dehydration and electrolyte abnormalities. Persistent vomiting and low water and food intake may
provoke dehydration, renal failure and electrolyte disorders.(R2)

162. What is the most frequent cause of death in patients admitted to hospital with acute
coronary syndrome?

 
 
 
 
1. 1. Thromboembolic complications.
2. 2. Acute heart failure.
3. 3. Malignant arrhythmias.
4. 4. AV block.
Gráfico de respuestas
Comentario

Pregunta básica. La causa más frecuente de muerte en el medio extrahospitalario son las arritmias
ventriculares malignas. En cambio, durante el ingreso, en medio hospitalario, sería la insuficiencia
cardíaca (shock cardiogénico).(R2)

163. ¿Cuál de los siguientes es el dato que más le ayudará para establecer el diagnóstico
del síndrome de Swyer?:

1. 1. Biopsia de ovario.
2. 2. Laparoscopia.
3. 3. Cariotipo.
4. 4. Perfil hormonal.
Gráfico de respuestas
Comentario

Es una pregunta bastante específica de este síndrome pero debemos conocer ciertos datos típicos
de la disgenesia gonadal que permitirán contestarla.

Consiste en la defectuosa formación de los ovarios, sustituidos por dos cintillas fibrosas con
ausencia de folículos ováricos. Los genitales externos son femeninos pero infantiles. Presentan
niveles elevados de gonadotropinas ya que no se producen las hormonas que llevan a cabo el
feedback negativo. Se presenta bajo estas formas: síndrome de Turner, síndrome de Swyer y
disgenesia gonadal mixta.

El síndrome de Swyer es una disgenesia gonadal pura, sin malformaciones ni enanismo. El


cariotipo es 46 XY. El cromosoma Y no se expresa, por lo que funciona como un 45 XO (que es el
cariotipo más frecuente del síndrome de Turner). Son frecuentes los cánceres de ovario (sobretodo
el gonadoblastoma).(R3)

164. El porcentaje de pacientes de 65 años o más que presentan un desprendimiento


vítreo posterior es de aproximadamente:

1. 1. 70%.
2. 2. 30%.
3. 3. 10%.
4. 4. 2%.
Gráfico de respuestas
Comentario
Pregunta poco importante para el MIR puesto que no es rentable aprenderse una gran cantidad de
porcentajes para responder preguntas como ésta que caen muy raramente. Además este tipo de
preguntas las suele fallar todo el mundo con lo cual no son discriminativas. Sólo debes recordar
que el desprendimiento de vítreo posterior es una patología muy frecuente en este grupo de
edad.(R1)

 
 
 
 
165. A CT scan picture with contrast (corresponds to a 87 years old patient without
cardiovascular risk factors diagnosed of Alzheimer Disease) shows a biconvex shape
hemorrhage . What is the diagnosis?

1. 1. Epidural hematoma.
2. 2. Subdural hematoma.
3. 3. Subcortical intracerebral hemorrhage.
4. 4. Primary central nervous system tumor.
Gráfico de respuestas
Comentario

En la imagen por tomografía de un hematoma epidural suele observarse una imagen biconvexa, a
diferencia del hematoma subdural que muestra una imagen concava en su cara interna (en
semiluna).(R1)

En la siguiente imagen la primera corresponde a un hematoma epidural la segunda a un hematoma


subdural.  

166. La radiografía de tórax de un antiguo recién nacido que ha requerido ventilación


asistida durante varios meses muestra una imagen poliquística donde alternan zonas
hiperaireadas con zonas condensadas en ambos campos pulmonares. A partir de estos
datos, ¿cuál es su diagnóstico de sospecha?:

1. 1. Déficit de alfa-l antitripsina.


2. 2. Displasia broncopulmonar.
3. 3. Enfermedad de la membrana hialina.
4. 4. Síndrome de aspiración-meconial.
Gráfico de respuestas
Comentario

La displasia broncopulmonar es la broncopatía crónica de los recién nacidos (una especie de


EPOC versión lactante). Entre sus agentes, destacan tres: inmadurez, toxicidad por oxígeno
(tiempo prolongado) y barotrauma. Su expresión radiológica típica es el llamado pulmón en

 
 
 
 
esponja. El pulmón en esponja combina zonas de atrapamiento de aire, condensaciones, patrón
intersticial y atelectasias.

El dato clínico más característico es la dependencia de oxigeno después del mes de vida más allá
después de las 36 seanas corregidas.(R2)

167. De las pruebas complementarias utilizadas en el diagnóstico de encefalitis viral,


¿cuál es la de MENOR utilidad?:

1. 1. Cultivo de LCR.
2. 2. TC y RM cerebral.
3. 3. Electroencefalograma.
4. 4. Biopsia cerebral.
Gráfico de respuestas
Comentario

Pregunta fácil de un tema muy preguntado sobre aspectos del diagnóstico que no deben fallarse.
Si ponemos como ejemplo de esta patología la encefalitis herpética (la más preguntada de las
encefalitis virales) debe saber que el diagnóstico es inicialmente clínico. El LCR presenta
incremento de linfocitos y proteínas con glucosa normal. En el TC y RM cerebral pueden
observarse imágenes sugerentes de hipodensidades en lóbulo temporal, lo mismo que en el EEG.
Previamente era necesaria la biopsia cerebral para la confirmación del diagnóstico pero
actualmente se realiza con la detección del ADN viral en el LCR. Lo que no es útil es el cultivo del
LCR, que es típicamente negativo ya que el virus no crece en él.(R1)

168. Sólo una de las afirmaciones siguientes en relación con la valvulopatía aórtica es
cierta:

1. 1. La causa más frecuente de estenosis aórtica es la fiebre reumática.


El hecho fisiopatológico más importante en la estenosis aórtica es un gradiente diastólico
2. 2.
entre el ventrículo izquierdo y la aorta.
Los vasodilatadores arteriales disminuyen el volumen regurgitante en la insuficiencia
3. 3.
aórtica.
4. 4. Los pacientes con insuficiencia aórtica toleran muy mal la taquicardia.
Gráfico de respuestas
Comentario
Esta pregunta es útil para repasar la valvulopatía aórtica, un tema bastante preguntado en el MIR.
La causa más frecuente de EAo es la congénita (la forma bicúspide degenerativa es la causa más
frecuente en adultos entre 30 y 70 años). En los mayores de 70 años, la causa más frecuente es la
senil degenerativa. En la EAo existe un gradiente de presión entre VI y aorta, pero es sistólico (no
diastólico). En la IAo los vasodilatadores arteriales disminuyen el volumen regurgitante, y de hecho,
el tratamiento con nifedipino ha demostrado aumentar el tiempo hasta que es necesaria la cirugía
(en pacientes asintomáticos con IAo). En esta enfermedad la taquicardia es muy bien tolerada (al
contrario que en la EAo), ya que disminue el tiempo de diástole, y así disminuye también el
volumen regurgitante. En estos pacientes el ejercicio físico es muy bien tolerado. El eje del QRS
menor de 0º es un signo de mal pronóstico, ya que indica dilatación del VI con sobrecarga de
volumen.(R3)

 
 
 
 

169. Respecto a las lesiones


cutáneas mostradas en la imagen, y al diagnóstico de sospecha que evocan, señale la
que considere FALSA:

Debe de comprobarse si las lesiones son palpables y si la coloración se mantiene con la


1. 1.
vitropresión. En caso afirmativo, se procederá a la biopsia cutánea.
La aparición de este tipo de lesiones en pacientes pediátricos con síndrome febril y dolor
2. 2. abdominal obliga a la práctica de un sedimento de orina y al estudio de la proteinuria
durante 24 h.
Dada la alta tasa de afectación renal severa en este tipo de dermatosis, se recomienda
3. 3.
practicar una biopsia renal de rutina mediante intervención ambulatoria.
En la histología de este tipo de dermatosis es frecuente observar necrosis vascular
4. 4.
fibrinoide y un infiltrado polimorfonuclear con leucocitoclasia.
Gráfico de respuestas
Comentario

Lo que nos muestra esta imagen son lesiones eritematosas, purpúricas, con necrosis en algunas
de ellas. En otras palabras, habría que sospechar una posible vasculitis. En este caso, estaría
indicada la realización de una biopsia cutánea, que posiblemente mostraría una infiltración
polimorfonuclear en los vasos afectos, como se explica en la respuesta 4. En la infancia, no es
infrecuente que este tipo de lesiones aparezcan en el contexto de la púrpura anafilactoide o
enfermedad de Schönlein- Henoch, que mostraría IgA si se practica una inmunofluorescencia.

La respuesta incorrecta sería la 3. La afectación renal es infrecuente (suele tratarse de lesiones


exclusivamente cutáneas). Además, antes de realizar una técnica cruenta como una biopsia renal,
habría que considerar muchas más cosas (orina, sedimento, función renal, etc), por lo que no se
justifica su práctica rutinaria (respuesta 3 falsa).(R3)

170. Hombre de 52 años, bebedor importante, que consulta por presentar lesiones
ampollosas y erosiones tras exposición solar. El laboratorio nos informa de la existencia

 
 
 
 
de un discreto aumento de transaminasas, siendo la excreción urinaria de uroporfirina
superior a 200 ug/dl. Ante este cuadro, señale la respuesta CORRECTA:

1. 1. Se trata de una enfermedad de herencia autosómica recesiva.


2. 2. El tratamiento incluye la realización de flebotomías periódicas.
3. 3. La absorción intestinal del hierro estará probablemente incrementada.
La fotosensibilidad no encaja en el diagnóstico de hemocromatosis primaria, lo que
4. 4.
descarta la sobrecarga férrica a nivel hepático.
Gráfico de respuestas
Comentario

Recuerda la siguiente descripción: alcohólico con pelos en la cara y ampollas en las manos, porfiria
cutanea tarda. Se trata de una enfermedad adquirida que produce una mala gestión hepática del
hierro y que requiere tratamiento con flebotomías periódicas.(R2)

171. Niño de 4 años de edad, con secreción nasal, conjuntivitis, alza térmica, erupción
maculopapular (no vesicular) generalizada. Hallazgos de laboratorio: IgM en suero. En su
escuela se presentaron casos parecidos hace 3 semanas ¿Cuál es la definición del caso?

1. 1. Confirmado.
2. 2. Sospechoso.
3. 3. Compatible.
4. 4. Descartado.
Gráfico de respuestas
Comentario

Según la epidemiología, un caso sospechoso se considera a quien cumple síntomas o signos


caracteristicos de la enfermedad y que ha estado previamente en contacto con casos confirmados
en un periodo plausible de contagiosidad.

Un caso probable es un caso sospechoso pero sin que se tengas las pruebas con las que se
diagnostica comunmentepara detectar la infección.

Un caso confirmado seria aquel con prueba de laboratorio que confirme infección.(R1)

172. ¿Para qué puede ser útil la medición de la presión vesical en el paciente intervenido
por traumatismo abdominal?

1. 1. Para detectar lesiones vesicales inadvertidas.


2. 2. Para detectar lesiones medulares que ocasionan hipotonía vesical.
Para detectar el síndrome de compartimiento abdominal, que exigirá descompresión
3. 3.
quirúrgica urgente del abdomen.
4. 4. No tiene utilidad reconocida.
Gráfico de respuestas
Comentario

La medición de la presión vesical es muy util en el caso de traumatismos para poder detectar un
síndrome compartimental, ya que la presión en la vejiga va a transmitir la presión intraabdominal.
Es importante detectar éstos pacientes para poder proceder a una descompresión quirúrgica
urgente del abdomen.(R3)

 
 
 
 
173. Los serotipos de Chlamydia trachomatis causantes de linfogranuloma venéreo son:

1. 1. L1, L2, Ba.


2. 2. L1
3. 3. B, D-K.
4. 4. L1, L2, L3.
Gráfico de respuestas
Comentario

Es fácil de recordar, L1, L2 y L3, como las vértebras lumbares. Son los serotipos causantes del
linfogranuloma venéreo.(R4)

174. El síndrome de Madelung consiste en:

1. 1. Afectación pulmonar por sarampión.


2. 2. Lipoma voluminoso alrededor del cuello.
3. 3. Torsión del extremo inferior del radio con dislocación del cúbito hacia atrás.
4. 4. Condrocalcinosis.
Gráfico de respuestas
Comentario

Es una pregunta difícil sobre un síndrome pediátrico poco frecuente. Es difícil encontrar preguntas
como ésta en el ENARM. Es síndrome de Madelung se llama también cuello de Madelung y
consiste en un lipoma voluminoso alrededor del cuello. No confundir con la deformidad de
Madelung que consiste en una torsión del extremo inferior del radio con dislocación del cúbito
hacia atrás.(R2)

175. Hombre de 39 años, que ingresó por persistencia de dos úlceras duodenales tras
ocho semanas de tratamiento con omeprazol, y habiendo completado previamente un
ciclo de diez días con omeprazol, claritromicina y amoxicilina. No tenía antecedentes de
alcoholismo, consumo de tabaco o fármacos gastroerosivos. En las últimas tres semanas
se había añadido diarrea de 3 a 4 deposiciones al día. En los exámenes de laboratorio
destacaba: Hb 10,2 mg/dl y vitamina B12 de 57 pg/ml (normal >200 pg/ml), grasas fecales
de 24 h: 10 g. La prueba de la D-Xilosa fue normal. Una sigmoidocolonoscopi¡ía fue
normal. Se realizó un TC abdominal, cuyo estudio fue informado como dentro de la
normalidad. Entre las siguientes medidas, ¿cuál le aportaría una mayor rentabilidad para
obtener el diagnóstico de este paciente?:

1. 1. Determinación de gastrina basal.


2. 2. Niveles de polipéptido pancreático.
3. 3. Ensayo durante una semana con enzimas pancreáticas.
4. 4. Arteriografía mesentérica.
Gráfico de respuestas
Comentario

Ante la presencia de úlceras duodenales múltiples, refractarias al tratamiento habitual, sería


posible un síndrome de Zöllinger- Ellison. Existen además otros datos orientativos:

- Existe diarrea con esteatorrea. En esta enfermedad, se debe a la hiperacidez duodenal, que
impide la correcta función de las enzimas pancreáticas (que precisan pH alcalino).

 
 
 
 
- Existe déficit asociado de B12: Como consecuencia de la excesiva acidez duodenal, se
desnaturaliza el factor intrínseco previamente secretado a nivel gástrico, lo que impide la correcta
absorción de la B12.

La TC abdominal normal no debe engañarte. En ocasiones, los gastrinomas son tan pequeños que
es difícil detectarlos incluso mediante una TC de abdomen. Por ello, no debes descartar esta
enfermedad y has de solicitar el nivel de gastrina (respuesta 1 correcta).(R1)

176. En reanimación neonatal, ¿a partir de que FC se inicia masaje cardiaco?

1. 1. < 100 lpm.


2. 2. < 80 lpm.
3. 3. < 60 lpm.
4. 4. < 40 lpm.
Gráfico de respuestas
Comentario

Se inicia masaje cardiaco en la renamiación enonatal a partir de <60 lpm(R3)

 
 
 
 
177. Un hombre de 30 años sufre caída de una altura de 2 metros de pie. A su llegada a
Urgencias en camilla está consciente y orientado, con sondaje vesical con orina clara.
Muestra tumefacción de talón izquierdo y hormigueos leves por cara anterior de muslo
derecho y escroto, así como disestesias en ambos pies. El paciente se despide del
camillero con un apretón de manos con una palmada simultánea en el hombro. La
rotación de los miembros inferiores en la camilla, es indolora y la compresión de la pelvis
es asintomática. A continuación:

1. 1. Lo sentaremos en la camilla para poder explorar la columna cervical.


Remitiremos a radiología para realizar estudio de extremidades inferiores (pies, fémures y
2. 2.
pelvis).
3. 3. Palparemos el abdomen y lo rodaremos en bloque para palpar la columna toracolumbar.
4. 4. Solicitaremos un ultrasonido abdominal.
Gráfico de respuestas
Comentario

En esta pregunta, por el tipo de caída de pie, debemos sospechar posibles fracturas de calcáneo,
tibia y vértebras. En este caso, el paciente ha mantenido el nivel de consciencia desde el inicio y
mantiene la fuerza y la movilidad en extremidades superiores. Además, nos indican que la rotación
de miembros inferiores y la compresión de la pelvis son indoloras, por lo que disminuye la
probabilidad de fractura a estos niveles, por lo que la respuesta 2 no sería prioritaria.

Lo que más llama la atención son los hormigueos en cara anterior de muslo derecho, escroto y
disestesias en ambos pies, que sugiere una compresión en niveles medulares lumbares por
posible fractura vertebral, por lo que la respuesta 3 es la que indica la mejor actitud clínica, ya que
la lesión dorsolumbar sería lo más prioritario a descartar. Por otra parte, es conveniente que ante la
sospecha de lesión vertebral y con lesión medular o de la cola de caballo, se haga la movilización
en bloque que sugiere la respuesta 3. Al menos de inicio, no sería tan importante el estudio de la
columna cervical, ya que el paciente ha estado consciente, no hay pérdida de fuerza en
extremidades superiores, y no hay más antecedentes de gravedad, por lo que las respuesta 1 no
sería prioritaria.

En el enunciado, nos aportan otra información útil: en el sondaje vesical, la orina es clara. Esto no
orienta a un traumatismo urológico, y tampoco se refieren a un dolor abdominal que hiciese
prioritario un ultrasonido abdominal, por lo que la respuesta 4 tampoco sería lo primero en
hacer.(R3)

178. Niña de 10 años, con fibrosis quística, acude por agravamiento de la dificultad
respiratoria basal, esputo con tapones marrones y febrícula. En la auscultación se
aprecian sibilancias (más que en exploraciones previas). No ha respondido a
antibioterapia. Usted sospecha aspergilosis broncopulmonar alérgica. Señale la
respuesta CORRECTA:

La presencia de Aspergillus fumigatus en esputo es un criterio menor de diagnóstico de


1. 1.
ABPA.
2. 2. Los corticoides están contraindicados en el tratamiento de esta entidad.
3. 3. Nunca se utiliza como tratamiento anfotericina B o 5-fluorocitosina.
4. 4. La ABPA se encuentra en más del 75% de los pacientes con fibrosis quística.
Gráfico de respuestas
Comentario

 
 
 
 
Aunque esta pregunta no es muy importante, debemos recordar algunas datos de la ABPA.

Se trata de una reacción inmunológica a Aspergillus fumigatus que complica el 1% de los casos de
asma y un 12% de fibrosos quística.

Nos describen la presentación típica que incluye asma de difícil control (FQ), tos productiva con
tapones marrones, febrícula disnea y sibilancias.

Los criterios menores son identificación del Aspegillus en esputo, expectoración de tapones
mucosos marronaceos y reacción cutánea retardada al hongo.

Los mayores son: asma, eosinofilia, IgE total elevada, IgG y E anti- Aspergillus elevada, infiltrados
pulmonares transitorios, bronquiectasias proximales y test cutáneo de hipersensibilidad inmediata
positivo para Aspergillus.

El tratamiento son corticoides sistémicos por lo menos durante 2 semanas. Se puede asociar algún
antimicótico.(R1)

179. RN de 32 semanas, con expulsión de meconio en las primeras 24 horas, que en el


10º día de vida presenta intolerancia alimentaria, vómitos biliosos, distensión abdominal
y una deposición sanguinolenta. En los exámenes de laboratorio: neutropenia y
trombopenia con hiponatremia grave; pH 7.19. En la Rx de tórax se observa
neumoperitoneo. ¿Cuál sería el tratamiento más adecuado?

1. 1. Piloromiotomía extramucosa.
2. 2. Enema de suero salino isotónico.
3. 3. Enema de contraste yodado.
4. 4. Resección del intestino necrosado y colostomía de descarga.
Gráfico de respuestas
Comentario

El cuadro clínico es sugestivo de una enterocolitis necrotizante que se presenta con datos de
inestabilidad y gravedad en los exámenes de laboratorio, por lo que el manejo más apropiado sería
la cirugía con resección del segmento necrosado.(R4)

180. Señale el enunciado INCORRECTO en relación con la incompatibilidad ABO:

1. 1. Puede existir afectación del primer hijo.


Sólo aparece enfermedad hemolítica en el 10% de los embarazos con incompatibilidad de
2. 2.
grupo.
3. 3. La ictericia no aparece hasta las 48-72 horas de vida.
Si se precisa exanguinotransfusión debe utilizarse sangre del grupo de la madre y Rh igual
4. 4.
al del niño.
Gráfico de respuestas
Comentario
La isoinmunización ABO se da por anticuerpos naturales de tipo IgG frente a los grupos opuestos.
No precisa sensibilización: de ahí que pueda afectar a primer hijo. Da lugar a cuadros más leves
que a isoinmunización Rh. En cualquier caso, la ictericia, si aparece, lo hace en las primeras 24
horas de vida del niño. Si precisa exanguinotransfusión, se hará con sangre isogrupo ABO con
respecto a la madre y del mismo Rh que el niño.(R3)

 
 
 
 

181. Niña de 1 año con fiebre de


hasta 39 ºC de 2 días de evolución. No presenta en la exploración foco claro para la fiebre,
salvo faringe algo hiperémica. Se toma muestra de orina mediante bolsa colectora,
detectándose nitritos y leucocitos mediante tira reactiva, por lo que se procede a toma
de orina estéril mediante punción suprapúbica y la tinción de Gram de dicha orina
muestra más de 1 bacteria por campo de gran aumento. En los exámes de laboratorio
sanguíneos destaca leucocitosis con neutrofilia, así como elevación importante de la
PCR. Tras el diagnóstico y el tratamiento adecuado para este caso, la niña es estudiada
en la consulta de nefrología, donde se le decide practicar una cistoureterografía
miccional seriada (CUMS) que se muestra en la imagen. Respecto al cuadro clínico citado,
señale la respuesta FALSA:

1. 1. La infección urinaria es la causa más frecuente de fiebre sin foco en menores de 1 año.
La presencia de nitritos y leucocitos en la orina no es diagnóstico de infección urinaria ya
2. 2.
que existen causas de falsos positivos.
3. 3. Para el diagnóstico de infección urinaria es necesario un cultivo positivo de orina.
El tratamiento de elección en estos casos de sospecha de infección urinaria con fiebre debe
4. 4.
ser siempre con antibióticos intravenosos de entrada.
Gráfico de respuestas
Comentario

La sola sospecha de infección urinaria no justifica la administración de antibióticos en todos los


casos. Cuando éstos se utilizan, tampoco tienen por qué ser necesariamente intravenosos
(depende de la gravedad y de la edad del paciente). Una respuesta como la 4, aunque sólo fuese
por lo radical de su expresión, debería hacerle desconfiar muy seriamente. La antibioticoterapia
intravenosa queda reservada para los casos más graves, aparte de que lo más adecuado es
confirmar la infección, como se explica en otra de las opciones.(R4)

 
 
 
 
182. La niña de la pregunta anterior fue diagnosticada finalmente de infección urinaria al
presentar un urocultivo con >1000 UFC/ml de E. coli que respondió bien al tratamiento
antibiótico. En relación al manejo adecuado de este tipo de pacientes tras la resolución
del proceso infeccioso, señale la opción FALSA:

Se recomienda realizar ultrasonido renal y vesical tras un episodio de IVU febril para
1. 1.
descartar hidronefrosis y abscesos renales.
La cistoureterografía miccional seriada no sólo permite el diagnóstico del reflujo vesico-
2. 2. ureteral sino que lo clasifica en grados que pueden determinar la actitud terapéutica más
adecuada.
El reflujo vesico-ureteral primario se debe a una anomalía congénita en la unión urétero-
3. 3. vesical, mientras que el secundario, mucho más frecuente que el primario, puede tener
múltiples etiologías.
El pilar básico del manejo de niños con reflujo vésico-ureteral es la profilaxis antibiótica
4. 4. con el objetivo de prevenir pielonefritis de repetición que puedan dejar cicatrices renales y
afectar a la función renal (nefropatía por reflujo).
Gráfico de respuestas
Comentario

Para el diagnóstico de IVU, se precisan más de 100 000 UFC, pero en este caso nos bastaría con
una sola, ya que el método de muestra ha sido la punción suprapúbica. Una vez resuelto el caso,
debemos plantearnos si existe una anomalía morfológica que lo justifique, por lo que realizaremos
un ultrasonido renal y vesical, para empezar el estudio. También se utilizará la cistoureterografía
miccional seriada (CUMS) para ver el grado de reflujo y elegir el tratamiento más adecuado. La
respuesta 3 es falsa por un pequeño detalle: el RVU es más frecuente en su forma primaria que el
secundario.(R3)

183. A 50-year-old male patient says that he has been having erythematous plaques in
different areas of his body for the last 5 years. These lesions tend to be itchy and covered
by scales. In the last few months bigger plaques have been appearing and some of them
have an ulcerated appearance. Which of the following is the most likely diagnosis?

1. 1. Histiocytosis X.
2. 2. Neurofibromatosis.
3. 3. Mycosis fungoides.
4. 4. Syphilis.
Gráfico de respuestas
Comentario

La micosis fungoides es un linfoma cutáneo de células T, seguramente no será preguntado, pero


es importante que sepas que es? bueno pues es de bajo grado de malignidad, cuya evolución dura
muchos años. Clínicamente, evoluciona en tres fases: eccematosa o macular, en placas o
infiltrativa y tumoral (respuesta 3 correcta).

Desde el punto de vista histológico, debes recordar los microabscesos de Pautrier, que son
cúmulos de linfocitos intraepidérmicos, de aspecto atípico y con núcleo cerebriforme. Cuando estas
células pasan al torrente sanguíneo, se habla del síndrome de Sézary, que es la fase leucémica de
la micosis fungoide. El síndrome de Sézary se define por la tríada: eritrodermia, linfadenopatías y
existencia de más del 10% de células de Sézary en sangre periférica. La célula de Sézary es un
linfocito T atípico con núcleo cerebriforme. En pacientes con este tipo de leucemia, el prurito es
muy intenso y debes considerarlo un síntoma cardinal.(R3)

 
 
 
 
184. Acude a nuestra consulta un paciente con clínica de asma bronquial, remitido por
su médico de familia al observar en una radiografía de tórax la presencia de infiltrados
pulmonares bilaterales no cavitados. Aporta una analítica básica donde destaca la
presencia de eosinofília. ¿Cuál sería su sospecha diagnóstica?

1. 1. Poliangeitis microscópica
2. 2. Granulomatosis alérgica de Churg-Strauss
3. 3. Granulomatosis de Wegener
4. 4. Aspergilosis pulmonar invasiva
Gráfico de respuestas
Comentario
Nos plantean un caso tipo que debes tener plasmado en la memoria: Asma + eosinofília +
infiltrados pulmonares bilaterales y no cavitados. Para empezar podemos descartar
razonablemente la aspergilosis invasiva (aparece en inmunodeprimidos, el estado general de
nuestro paciente no se corresponde). En relación a las vasculitis, la granulomatosis de Wegener
suele presentarse con afectación de la vía respiratoria superior, dando lugar a epistaxis y no a
clínica de asma bronquial. Además, aunque también veríamos infiltrados pulmonares bilaterales,
estos suelen ser cavitados. Por último, la poliangeitis microscópica produce afectación pulmonar en
forma de capilaritis siendo más frecuente la hemoptisis y no la clínica de asma con eosinofília
periférica.(R2)

185. Un niño de 6 meses acude con dificultad respiratoria, irritabilidad y rechazo de las
tomas. Durante los días previos padeció un cuadro catarral. En la exploración se objetiva
taquipnea, aleteo nasal y sibilancias espiratorias. El agente causal más implicado en este
cuadro es:

1. 1. Metapneumovirus.
2. 2. Virus parainfluenzae.
3. 3. Virus respiratorio sincitial.
4. 4. Mycoplasma.
Gráfico de respuestas
Comentario

Describen una bronquiolitis aguda. Esta enfermedad es de diagnóstico clínico, y se define como el
primer episodio de sibilancias en niños menores de 2 años de edad. Su causa más frecuente es el
virus respiratorio sincitial.(R3)

186. Niño de 2 años de edad con cuadro de diarrea sanguinolenta desde hace una
semana. De forma brusca presenta petequias generalizadas y disminución de la diuresis.
Señale la respuesta INCORRECTA respecto a la patología que sospecha:

1. 1. El cuadro ha sido causado por la producción de verotoxinas.


2. 2. Puede existir hipertransaminasemia
3. 3. Presenta una alteración de la coagulación.
4. 4. Es la causa más frecuente de insuficiencia renal aguda en menores de 4 años.
Gráfico de respuestas
Comentario

Debemos sospechar un síndrome hemolítico urémico con el antecedente de gastroenteritis aguda


enteroinvasiva por E. coli O157:H7, germen más frecuentemente implicado, y posteriormente

 
 
 
 
alteraciones hematológicas por TROMBOCITOPENIA leve- moderada, sin alteraciones de la
coagulación, ni CID, que genera disfunción renal causando oligoanuria en más del 50% de los
pacientes. Respuesta incorrecta la 3.(R3)

187. Una niña de un mes y 8 días de vida es remitida a su consulta por presentar estrías
de sangre en las heces desde hace 3 días. Presenta entre 3 a 5 evacuaciones al día,
amarillentas y de consistencia normal. Es regurgitadora habitual y ocasionalmente
presenta algún vómito postprandial. Refieren apetito conservado, que ha presentado
algún episodio de llanto aislado por la noche en la última semana y que ha estado afebril
en todo momento. Está alimentada con lactancia materna exclusiva desde que nació y ha
presentado una buena ganancia ponderal. La exploración física no revela ningún dato de
interés. AP: Recién nacida a término (38+, peso al nacimiento 3,460 gr, Apgar 8/9. Precisó
fototerapia durante 24 horas al tercer día por hiperbilirrubinemia (BRT 18.5 mg/dl).
Pruebas metabólicas y cribado auditivo normal. Ha recibido una dosis de la vacuna de la
hepatitis B. AF: padre 28 años intervenido de apendicitis a los 8 años. Madre 27 años (G1
A0 V1) con antecedente de dermatitis atópica. Respecto a la enfermedad que sospecha,
es CIERTO que:

El Helicobacter pylori es el agente infeccioso implicado con mayor frecuencia, y el


1. 1.
tratamiento antibiótico ha cambiado radicalmente el curso de esta enfermedad.
El hecho de recibir lactancia materna exclusiva no descarta una intolerancia a proteínas de
2. 2. leche de vaca cuyo tratamiento sería sustituir la alimentación por una fórmula sin estas
proteínas, por ejemplo la leche de soya.
La colitis es una manifestación muy rara de la intolerancia/alergia a proteínas de leche de
3. 3.
vaca.
El pronóstico vital y funcional es realmente muy bueno, con una resolución completa de
4. 4.
los síntomas en los próximos años de su vida.
Gráfico de respuestas
Comentario

De todas las formas clínicas de intolerancia/alergia a proteínas de leche de vaca ha aumentado


mucho la incidencia de la colitis cuyo síntoma principal es la presencia de hilos/hebras de sangre
en las heces en un niño por lo demás normal.

La edad de aparición suele ser entre las 2-8 semanas de vida. No siempre estarán alimentados
con fórmula artificial (hasta un 60% reciben lactancia materna exclusiva) pero el diagnóstico se
establece al desparecer la clínica tras exclusión de las proteínas de leche de vaca: ya sea por
sustitución con fórmula hidrolizada o por retirada de lácteos de la dieta materna mientras dure la
lactancia.

No es necesario hacer biopsia salvo en casos refractarios. La fórmula de soya tiene un 30% de
reactividad cruzada con las proteínas de leche de vaca, por lo que la clínica puede persistir. 25%
suele presentar antecedentes familiares de atopia. Las pruebas cutáneas de alergia pueden o no
ser positivas. La gran mayoría serán tolerantes hacia los 2-3 años de vida.(R4)

188. Uno de los siguientes tumores cerebrales primarios tiene mayor incidencia de
hemorragia en su seno:

1. 1. Ependimoma.
2. 2. Astrocitoma pilocítico.
3. 3. Meduloblastoma.

 
 
 
 
4. 4. Meningioma.
Gráfico de respuestas
Comentario
Pregunta de tumores del SNC rentable de cara al MIR por su sencillez en recordar cuales son los
tumores primarios y metastáticos con tendencia al sangrado si te aprendes la regla nemotécnica
COME PURITOS mientras GIME OLGA. Asi pues, los tumores metástasicos del SNC con
tendencia al sangrado son el COriocarcinoma (el que la presenta con mayor frecuencia), el
MElanoma, Pulmón, RIñón (hipernefroma) y Tiroides ( COME PURITos) . Dentro de los primarios
del SNC se encuentran el Glioblastoma multiforme, el MEduloblastoma y el OLigodendroglioma (
GIME OLGA).(R3)

189. Una enferma diagnosticada de LES hace 10 años presenta insuficiencia renal
rápidamente progresiva con hematuria y proteinuria. Se realiza una biopsia renal en la
que se observa aumento de la celularidad mesangial y endotelial en todos los glomérulos
con necrosis celular focal, "asas de alambre" y semilunas en el 50% de los glomérulos.
¿Ante qué entidad nos encontramos?:

1. 1. Glomerulonefritis lúpica mesangial.


2. 2. Glomerulonefritis lúpica focal y segmentaria.
3. 3. Glomerulonefritis lúpica proliferativa difusa.
4. 4. Glomerulonefritis lúpica membranosa.
Gráfico de respuestas
Comentario
Esta pregunta es de dificultad media. Con respecto a la nefropatía lúpica es importante que
recuerdes que la afectación renal acontece en un 50% de los casos de lupus y su presencia
ensombrece el pronóstico. La forma más frecuente en asintomáticos cuya supervivencia es mayor
al 90% a los 5 años es la GN mesangial lúpica. Sin embargo, la forma más grave y la más
frecuente en enfermos SINTOMATICOS es la GN lúpica proliferativa difusa (esto ya nos orientaría
hacia la respuesta correcta). La anatomía patológica de esta forma es la relatada en el enunciado:
proliferación difusa mesangial y endotelial, necrosis fibrinoide, cuerpos hematoxinófilos y asas de
alambre (capilares engrosados) junto a fibrosis, vasculitis y cierto grado de semilunas.(R3)

190. Se incluyen dentro del síndrome alcohólico fetal, todas las siguientes
características, EXCEPTO:

1. 1. Fisura palpebral corta.


2. 2. Retraso mental.
3. 3. Retraso del crecimiento.
4. 4. Macroglosia.
Gráfico de respuestas
Comentario

El síndrome alcohólico fetal se caracteriza por la presencia de alteraciones antropométricas


(retraso del peso, talla y perímetro cefálico), facies peculiar (blefarofimosis, epicanto, micrognatia,
hipoplasia maxilar, labio superior fino), malformaciones cardíacas (defectos del septum), anomalías
de las articulaciones y extremidades (ligera limitación de la movilidad y alteraciones de los surcos y
pliegues palmares) y retraso psicomotor. La macroglosia, en cambio, no forma parte de este
cuadro clínico.(R4)

 
 
 
 
191. Uno de los siguientes NO es un factor predictor de buena respuesta al interferón en
la hepatitis crónica B:

1. 1. Infecciones producidas por la cepa mutante precore.


2. 2. Mujeres.
3. 3. Corta duración de la enfermedad.
4. 4. Niveles bajos de ADN-VHB.
Gráfico de respuestas
Comentario

Los factores predictores de buena respuesta son: el sexo femenino, edad joven, corta historia de
enfermedad, niveles bajos de DNA- VHB, niveles elevados de transaminasas, la escasa fibrosis en
la biopsia hepática y un estado inmunológico adecuado (inmunocompetentes). Sin embargo, la
cepa mutante precore suele ser resistente al tratamiento con interferon. Por ello, el tratamiento que
suele emplearse es la lamivudina. Recuerde que, en caso de no responder a lamivudina, el
siguiente recurso terapéutico sería el adefovir.(R1)

192. En un servicio de Urgencias ingresa un hombre de 65 años que presenta dolor


abdominal cólico, estreñimiento y leve distensión abdominal. El tacto rectal muestra una
gran masa en recto. Indique cuál de las pruebas siguientes es más útil para indicar cirugía
de urgencias:

1. 1. Enema opaco.
2. 2. Radiografía simple de abdomen.
3. 3. Biopsia de la masa.
4. 4. Sigmoidoscopia.
Gráfico de respuestas
Comentario

Pregunta práctica en relación con el tema de la obstrucción intestinal, que es un tema que de cara
al ENARM no le debe plantear muchos problemas y del que debe saber su etiología más frecuente
y su cuadro clínico.

Recuerde que la causa más frecuente de obstrucción a nivel del intestino grueso es el cáncer
colorrectal (tacto rectal sugerente).

En cuanto al diagnóstico es fundamentalmente clínico y la prueba complementaria más útil para


confirmarlo es la radiografía simple de abdomen tomada en decúbito y en bipedestación. En ella se
observa la distensión gaseosa de las asas de intestino por encima de la obstrucción y en la placa
tomada en bipedestación se pueden ver los característicos niveles hidroaéreos dentro de las asas
intestinales distendidas. Este examen además de confirmar el diagnóstico, da información acerca
del nivel o altura de la obstrucción, si es intestino delgado o de colon. No suelen ser necesarias
otras pruebas de imagen para el diagnóstico de obstrucción intestinal.(R2)

193. Varón de 62 años con historia de Síndrome de Sjögren de 15 años de evolución. En


los últimos meses presenta aumento persistente de la parótida derecha y
esplenomegalia. En la RX de tórax se observan infiltrados pulmonares bilaterales. ¿Qué
complicación es necesario descartar?:

1. 1. Amiloidosis.
2. 2. Infección por VIH.

 
 
 
 
3. 3. Obstrucción completa del conducto de Wirsung.
4. 4. Linfoma.
Gráfico de respuestas
Comentario

El caso planteado corresponde claramente a un síndrome de Sjögren. En este contexto, la


presencia de tumefacción parotídea debe ponernos sobre aviso de un posible linfoma, sobre todo
si además tenemos afectación pulmonar, adenopatías o esplenomegalia. Recuerda que, en el
síndrome de Sjögren, otro signo característico que debe hacernos pensar en un linfoma sería el
descenso del título de factor reumatoide.(R4)

194. La causa más frecuente de sangrado uterino anormal y recurrente en una mujer en
edad reproductiva es:

1. 1. Anovulación.
2. 2. Efecto colateral de anticonceptivos.
3. 3. Amenaza de aborto.
4. 4. Traumatismo.
Gráfico de respuestas
Comentario

La causa más frecuente de sangrados uterinos irregulares o anormales es la presencia de ciclos


anovulatorios; esto sucede sobre todo al principio y al final de la vida fértil.(R1)

195. A 10-year-old boy presents to his pediatrician because of a painful and swollen
wound on his hand. The injury occurred 72 hours ago at school, when the patient was
bitten by a classmate. The wound has become swollen and red now. The parents say that
his immunizations are up to date and that his only significant past medical history is that
he had a severe rash when he was 3 years old and was prescribed penicillin for tonsillitis.
The physician explores the wound and finds no signs of osteomyelitis or foreign bodies,
which is subsequently confirmed with X-ray films. Which of the following is the most
appropriate treatment?

1. 1. Clindamycin and ciprofloxacin.


2. 2. Erythromycin and ciprofloxacin.
3. 3. Amoxicillin and clavulanate.
4. 4. Clindamycin and metronidazole.
Gráfico de respuestas
Comentario
Clindamycin and ciprofloxacin. Amoxicillin-clavulanate would be the first-choice treatment.
However, this patient is allergic to penicillin. Therefore, you have to choose an adequate alternative
that can be effective against "human bites". Clindamycin and ciprofloxacin have a very adequate
spectrum and should be prescribed to this patient.(R1)

196. Paciente de 34 años, tercípara, gestante de 39 años que tras 6 horas con trabajo de
parto comienzan a aparecer DIPS tipo II asociados a una ondulatoria baja y taquicardia
fetal. Usted le realiza una microtoma fetal obteniendo resultado de 7.18. Condiciones
obstétricas: dilatación completa, Occípitoiliaca izquierda anterior, II plano. ¿Qué haría a
continuación?:

 
 
 
 
1. 1. Cesárea.
2. 2. Fórceps.
3. 3. Ventosa.
4. 4. Espátulas.
Gráfico de respuestas
Comentario
El resultado del pH es patológico, nos indica la existencia de sufrimiento fetal y por tanto nos marca
nuestra actitud con extracción fetal inmediata. Si volvemos a las condiciones obstétricas (dilatación
completa, II plano), lo más rápido es una cesárea. El parto instrumental posible en un II plano es la
ventosa, pero no es un método rápido y por lo tanto válido en casos de sufrimiento fetal como en
esta pregunta.(R1)

197. Un varón de 16 años tiene cólico abdominal, inapetencia y evacuaciones


reblandecidas, presentes durante los últimos dos días. El examen abdominal origina
hipersensibilidad leve en el cuadrante inferior derecho. La temperatura, que fue de 37.2ºC
ayer, es de 37.8ºC el día de hoy. Los signos radiográficos siguientes son compatibles con
el diagnóstico de apendicitis, EXCEPTO:

Presencia de un fecalito en el cuadrante inferior derecho de la radiografía abdominal


1. 1.
simple.
2. 2. Ausencia de llenado apendicular con el enema de bario.
Engrosamiento de la porción terminal de íleon y un fino hilillo de bario en el enema de
3. 3.
bario.
4. 4. Efecto de masa en el borde inferior del ciego, con el enema de bario.
Gráfico de respuestas
Comentario

Las manifestaciones que afecten al apéndice o al ciego, lógicamente, podrán estar en relación con
una apendicitis aguda. Lo mismo sucede con la respuesta 1, ya que un fecalito podría ser el
causante de este cuadro. Sin embargo, la respuesta 4 no es coherente con este diagnóstico, sino
con una ileítis. Podríamos encontrar una imagen como ésta en la enfermedad de Crohn.

Recuerda que, a pesar de las alteraciones radiológicas mostradas en esta pregunta, la radiografía
abdominal más frecuente en la apendicitis aguda es una placa de abdomen normal.(R3)

198. De los descritos, ¿cuál es el síntoma más constante de la apendicitis aguda?

1. 1. Vómitos.
2. 2. Estreñimiento.
3. 3. Anorexia.
4. 4. Distensión abdominal.
Gráfico de respuestas
Comentario
Pregunta fácil si conoces la clínica de la apendicitis aguda. Un paciente con hambre NO tiene una
apendicitis aguda. La anorexia es una característica constante del cuadro y su ausencia nos debe
hacer dudar de este diagnóstico.(R3)

199. Señale cuál de los siguientes volúmenes pulmonares NO se puede medir con
espirometría y requiere dilución de helio o pletismografía corporal:

 
 
 
 
1. 1. Capacidad vital.
2. 2. Volumen de reserva espiratoria.
3. 3. Capacidad residual funcional.
4. 4. Volumen corriente.
Gráfico de respuestas
Comentario
No se puede medir el volumen no movilizable, es decir, el volumen de reserva. Igualmente
tampoco se podra medir ninguna capacidad que incluya el VR entre sus elementos, como la
capacidad pulmonar total o la capacidad residual funcional.(R3)

200. A 9-year-old girl is brought to consultation for presenting a history of recurrent


respiratory infections and diarrhea. The mother says that the girl's nose does not stop
running and the pediatric otolaryngologist has diagnosed her with nasal polyps. Which
of the following conditions is most likely associated to these manifestations?

1. 1. Atopia
2. 2. Cystic fibrosis
3. 3. Lymphoblastic acute leukemia
4. 4. Adenoid hypertrophy
Gráfico de respuestas
Comentario
Cystic fibrosis. Cystic fibrosis is an inherited disease that produces an abnormal increase in
viscosity and thickness of mucous secretions. These secretions favor the formation of mucus plugs
and therefore frequent respiratory infections. Nasal polyps are rare in children, and their
appearance should make us suspect this disease. Adenoid hypertrophy is a common childhood
problem and also produces abundant nasal secretions but is not associated with nasal polyps.
Diarrhea in cystic fibrosis is due to pancreatic insufficiency.(R2)

201. Un paciente de 58 años fumador y bebedor en exceso desde la adolescencia,


presenta dolor e inflamación en la cara interna del muslo derecho. A la exploración se
aprecia una induración dolorosa a la palpación en el recorrido de la safena interna. El
cuadro comenzó por debajo de la rodilla hace 10 días y ha ido progresando a la vez que
cedía distalmente. El paciente comenta que sufrió un episodio similar hace un mes y que
empieza a estar preocupado. En la exploración cutánea general, usted no observa otras
lesiones destacables. Como médico, usted debe alarmarse ante esta dermatosis porque:

1. 1. Indica la existencia de un trastorno grave de la coagulación.


2. 2. El paciente podría tener asociado un adenocarcinoma pancreático.
3. 3. Este paciente seguramente tiene un síndrome antifosfolípido.
4. 4. El paciente muestra signos típicos de una vasculitis necrotizante.
Gráfico de respuestas
Comentario

El cuadro que nos describen corresponde a una tromboflebitis migratoria, también conocida como
síndrome de Trousseau. Este cuadro es potencialmente paraneoplásico y se relaciona de forma
especial con el cáncer de páncreas (respuesta 2 correcta). Sin embargo, también debes saber que
el síndrome de Trousseau no siempre se vincula a un cáncer. También se ha descrito en el
contexto de la enfermedad de Buerger y en algunas infecciones por Rickettsia.(R2)

 
 
 
 
202. Señale cuál de las siguientes opciones NO se corresponde con el concepto de pulso
paradójico:

1. 1. Es un pulso de volumen mucho menor en la inspiración que en la espiración.


Se cuantifica como el descenso mayor de 10 mmHg de la presión arterial diastólica
2. 2.
durante la inspiración.
En el taponamiento cardíaco aparece como consecuencia del abombamiento del septo
3. 3. interventricular hacia el ventrículo izquierdo debido al aumento de llenado del ventrículo
derecho durante la inspiración.
En las obstrucciones bronquiales graves se produce por distensión de la raíz aórtica debida
4. 4.
a la gran presión negativa intratorácica generada durante la inspiración.
Gráfico de respuestas
Comentario

Uno de los signos exploratorios de mayor importancia en el contexto de la patología pericárdica es


la presencia de pulso paradójico, que se define como la caída de más de 10 mmHg de la presión
SISTÓLICA en relación con la inspiración. Recuerde que no es más que la exacerbación de un
fenómeno fisiológico, ya que en la inspiración se reduce la diferencia de presiones entre los
capilares pulmonares y el ventrículo izquierdo, lo cual produce una caída de la presión arterial muy
ligera. La presencia de pulso paradójico es muy típica del taponamiento (aunque no
patognomónica, ya que puede aparecer en enfermedades respiratorias obstructivas y en la
pericarditis constrictiva, entre otras patologías).(R2)

203. En la utilización de los corticoides en la ruptura prematura de membranas antes de


las 34 semanas de gestación, las siguientes afirmaciones son ciertas menos una. Indique
cuál:

1. 1. Aumenta la mortalidad perinatal por infección.


2. 2. Disminuye la aparición de distrés respiratorio del RN.
3. 3. Disminuye el riesgo de enterocolitis necrotizante.
4. 4. Acelera la maduración pulmonar.
Gráfico de respuestas
Comentario

Respuesta incorrecta la número 1, ya que el uso de corticoides no incrementa la mortalidad


perinatal secundaria a infección. Entre los beneficios del uso de corticoides: acelera la maduración
pulmonar, disminuye el riesgo de enterocolitis necrotizante y el riesgo de hemorragia de la matriz
germinal.(R1)

204. Una mujer de 70 años de edad tras sufrir una caída acude a su consulta por dolor e
intensa inflamación a nivel de la muñeca derecha. Presenta una temperatura de 37,9 ºC.
Al realizar el estudio radiológico se observa una imagen de doble contorno en el carpo.
¿Qué hallazgo confirmaría su sospecha diagnóstica?

1. 1. Calcificaciones en los meniscos de las rodillas y en la sínfisis del pubis.


Identificación de cristales en el líquido sinovial muy pequeños y con birrefringencia nega-
2. 2.
tiva.
Presencia en el líquido sinovial de cristales romboidales de escaso tamaño con
3. 3.
birrefringencia débilmente positiva.
4. 4. Un "cardenal" en la muñeca.
Gráfico de respuestas

 
 
 
 
Comentario

Nos presentan un cuadro de monoartritis aguda en una paciente de 70 años, localizada en el


carpo.

La causa más frecuente de monoartritis, a esta edad, es la artropatía por microcristales,


probablemente de pirofosfato cálcico. Al realizar el estudio radiológico se observa la presencia de
calcificación del cartílago (condrocalcinosis) descrita como imagen de doble contorno, que es
característica de los cristales que tienen calcio (pirofosfato cálcico, oxalato calcio o hidroxiapatita
cálcica). La articulación del carpo es habitual localización de los cristales de pirofosfato cálcico, que
se identifican en el líquido sinovial como de forma romboidal con birrefringencia positiva.

El antecedente traumático no descarta que se trate de una artritis microcristalina. De hecho, los
ataques agudos de artritis por pirofosfato pueden desencadenarse por traumatismos.(R3)

205. ¿Qué diagnostico le sugiere la aparición brusca de una masa renal palpable en un
lactante en un estado de deshidratación grave?

1. 1. Pionefrosis.
2. 2. Tumor de Willms.
3. 3. Hidronefrosis gigante.
4. 4. Trombosis de la vena renal.
Gráfico de respuestas
Comentario

Ante una masa renal palpable en un recién nacido con deshidratación grave, deberiamos de
pensar en una posible trombosis de la vena renal (Respuesta 4). Recuerde que este cuadro se
caracteríza por una masa sólida palpable junto con HTA, hematuria y trombopenia asociada a
deshidratación, policitemia, hijo de madre diabética y sepsis.(R4)

206. El síndrome de hiperventilación produce:

1. 1. Aumento de la PCO2 arterial.


2. 2. Disminución de la PCO2 arterial.
3. 3. Hipocalcemia.
4. 4. Hiperpotasemia.
Gráfico de respuestas
Comentario

Se trata de una pregunta fácil para cuya respuesta deberían bastar los conocimientos básicos de la
licenciatura, sin necesidad de estudio específico del MIR. Cuando se hiperventila, se produce un
aumento del intercambio gaseoso a nivel pulmonar, lo que condiciona una mayor eliminación de
CO2, por lo que la PCO2 arterial disminuye (opción 3 correcta). Dentro de la clínica de la
hiperventilación, el síntoma más frecuente es la disnea, ya que la hiperventilación se asocia a un
aumento del impulso respiratorio, del esfuerzo muscular y del volumen minuto. La hipocapnia da
lugar a alcalosis, la cual produce diversos síntomas neurológicos (mareos, síncopes, convulsiones
y trastornos visuales) que se deben a la vasoconstricción cerebral.(R2)

207. ¿Cuál de los siguientes factores NO es un factor de riesgo para desprendimiento


prematuro de placenta normalmente inserta?

 
 
 
 
1. 1. Hipertensión arterial crónica.
2. 2. Nuliparidad.
3. 3. Tabaquismo.
4. 4. Hipofibrinogenemia congenita.
Gráfico de respuestas
Comentario

Pregunta sumamente importante para el ENARM debe dominarla. Las hemorragias del tercer
trimestre son un capítulo fundamental dentro del bloque de Obstetricia. Esta pregunta podría
considerarse de dificultad media-baja, al ser directa y sobre un tema habitual.

La etiología del abruptio placentae (desprendimiento prematuro de la placenta normalmente


inserta) es poco conocida, pero se ha asociado con los siguientes factores:

•   Multiparidad. Es más frecuente en multíparas que en nulíparas (respuesta 2 correcta).


•   Edad: es más usual en mayores de 35 años.
•   Enfermedad vascular: la preeclampsia predispone claramente a esta complicación, sobre
todo cuando existe enfermedad vasculorrenal subyacente (diabéticas, nefrópatas,
hipertensas).
•   Traumatismos: a veces, se asocia a un traumatismo o al antecedente de amniocentesis.
También se ha relacionado con la rápida reducción del tamaño uterino al romperse la bolsa
amniótica en un polihidramnios.
•   Déficit de ácido fólico.
•   Tabaquismo, alcohol, cocaína.
•   Hipofibrinogenemia congénita.

(R2)

208. La complicación más frecuente de un catarro de vías altas es:

1. 1. Amigdalitis.
2. 2. Bronquiolitis.
3. 3. Otitis media.
4. 4. Sinusitis.
Gráfico de respuestas
Comentario

Esta pregunta sobre los catarros es relativamente sencilla y es algo que debemos saber.

No olvide que la complicación mas frecuente de un catarro de vías altas es la otitis media. Es
frecuente la diseminación retrograda vía tubárica de gérmenes desde el tracto respiratorio superior
al oído.

La mayor incidencia se presenta entre los 6 meses y los 3 años, debido a peor función de la
trompa y a la existencia de un reservorio de bacterias patógenas en la rinofaringe por la hipertrofia
adenoidea.

Recuerde que la causa mas frecuente de resfriado son los rinovirus seguido de los
coronavirus.(R3)

 
 
 
 
209. A previously healthy 12-year-old girl is brought to clinic due to poor school
performance. The girl has frequent headaches and she had a seizure last week, deemed
to be a febrile seizure. Physical examination shows decreased strength of the right side
of the body, with hyperreflexia. MRI of the brain shows a space occupying lesion on the
left frontal lobe. What tumor is most likely responsible for these symptoms?

1. 1. Medulloblastoma
2. 2. Glioblastoma
3. 3. Astrocytoma
4. 4. Craniopharyngioma
Gráfico de respuestas
Comentario
Astrocytoma. The most common malignancies during childhood are leukemias followed by brain
tumors. The most frequent type is benign astrocytoma, as in this case. Glioblastoma and
meningioma occurs at a later age and are not the main suspected diagnosis. Craniopharyngioma's
characteristic symptoms are polyphagia anddiabetes insipidus.(R3)

210. Las convulsiones febriles en la infancia son:

1. 1. Parciales mioclónicas.
2. 2. Parciales motoras.
3. 3. Espasmos infantiles.
4. 4. Generalizadas tónico-clónicas.
Gráfico de respuestas
Comentario

Las convulsiones febriles se pueden producir desde los 6 meses a los 6 años. Se producen con los
cambios bruscos de temperatura tanto el ascenso como el descenso. Son más frecuentemente
producidas por infecciones víricas. Son tonicoclónicas generalizadas, suelen durar menos de 15
minutos con pérdida de consciencia y recuperación total rapida. No se suelen repetir las primeras
24 horas. Esto es la descripción de un CF típica. Respuesta 4 correcta.(R4)

211. ¿Qué profilaxis antipalúdica es la más adecuada para una mujer embarazada que
planea viajar a una zona cloroquín-resistente?:

1. 1. Cloroquina.
2. 2. Atovacuona.
3. 3. Doxiciclina.
4. 4. Halofantrina.
Gráfico de respuestas
Comentario

Quimiprofilaxis de Paludismo

Forma sensibles a cloroquina: 1 semana previa al viaje y 4 semanas tras el regreso, es seguro en
embarazadas.

En esta pregunta te dicen ZONA CLOROQUIN RESISTENTE: entonces la Atovacuona + proguanil


2-3 días antes del viaje y 1 semana tras la vuelta.

 
 
 
 
La excepción sería la infección por los plasmodios vivax y ovale, capaces de quedar "durmientes"
como hipnozoitos (que escapan al efecto de los antipalúdicos) en el hígado para reactivarse
después. Así pues, ante una infección por estos plasmodios, usaremos primaquina para
erradicarlos.(R2)

212. Un paciente de 45 años que presenta un nódulo de 2 cm de diámetro en la cabeza


del páncreas ha sido sometido a una punción-aspiración con aguja fina y el diagnóstico
citológico ha sido de adenocarcinoma de páncreas; el estudio de extensión es negativo.
Una vez planteado el caso en la sesión clínica de su servicio, se decide que el tratamiento
más adecuado para intentar curar al paciente es la operación de Whipple o
duodenopancreatectomía cefálica. Uno de los gestos quirúrgicos que se enumeran a
continuación NO está incluido de forma estándar en esta intervención. Señálelo:

1. 1. Colecistectomía.
2. 2. Gastrectomía.
3. 3. Pancreatectomía cefálica.
4. 4. Vagotomía y piloroplastia.
Gráfico de respuestas
Comentario

Se trata de una pregunta específica sobre la intervención de Whipple o duodenopancreatectomía


cefálica, que es la técnica quirúrgica de elección para el cáncer de cabeza de páncreas, cuando lo
tratamos con intención curativa.

En la técnica de Whipple, se reseca el antro gástrico, todo el marco duodenal y unos centímetros
del yeyuno proximal en bloque, junto con la cabeza del páncreas, que se secciona a nivel del
llamado cuello pancreático sobre la porta y que incluye la vía biliar principal distal, es decir el
colédoco distal. Como gesto añadido, se debe resecar la vesícula biliar, lo cual es mandatorio
siempre que se practica una intervención en la vía biliar principal. Así pues, debemos reconstruir
haciendo tres anastomosis:

1) Entre el conducto de Wirsung y el tubo digestivo.

2) Entre la vía biliar y el tubo digestivo.

3) Entre el estómago y el tubo digestivo.

En general, todas estas anastomosis se realizan sobre la misma asa de yeyuno. No se realiza
vagotomía, pues puede producirse un retraso del vaciamiento gástrico y estómago de retención. Y,
por supuesto, tampoco piloroplastia; ya hemos comentado que se reseca el antro gástrico, píloro
inclusive.(R4)

213. A 60-year-old female comes to the physician complaining of a history of diarrhea and
flatulence as well as weight loss and diffuse abdominal pain in the past 2 years. She links
all these symptoms to her surgery 3 years ago in which they "took part of her stomach
because she vomited blood". Her blood pressure is 140/95 mmHg, her pulse is 62/min and
her temperature is 96ºF (35.5ºC). Physical examination reveals a distended abdomen with
identifiable succussion splash. Laboratory shows: Hb 9 mg/dL, MCV 122 fl, WBC
700/mm3. What kind of surgery did this patient undergo 3 years ago?

 
 
 
 
1. 1. Billroth II.
2. 2. Mason.
3. 3. Roux-en-Y.
4. 4. Griffen.
Gráfico de respuestas
Comentario
Bilroth II. Late complications appear especially after Billroth II type surgery, explained by an afferent
loop left close to the gastric remnant.(R1)

214. En pacientes con lupus eritematoso sistémico aparecen lesiones cutáneas de lupus
eritematoso subagudo en:

1. 1. 10%.
2. 2. 45%.
3. 3. 85%.
4. 4. No aparecen nunca.
Gráfico de respuestas
Comentario
La afectación cutánea en el lupus eritematoso sistémico (LES) la observamos en el 80% de los
casos, pudiendo aparecer en forma de lesiones agudas, subagudas o crónicas. Las lesiones
subagudas se observan en el 10% de los pacientes. Se trata de lesiones eritematosas anulares
confluentes (anular policíclico), en las que en ocasiones predomina la descamación
(psoriaciforme). Aparecen en zonas de exposición solar como cuello, escote, brazos y se asocian a
la aparición de anticuerpos anti- Ro.(R2)

215. Señale cuál de las siguientes respuestas acerca de la pubertad y la adolescencia NO


es correcta:

El primer signo de la pubertad en niñas es la aparición del botón mamario, que puede
1. 1.
iniciarse ya a los 8 años.
El primer signo externo en el hombre es el crecimiento testicular, de comienzo hacia los 8
2. 2.
años, pero con evolución de la pubertad más lenta que en las niñas.
El crecimiento durante esta época es asimétrico, primero las zonas distales y después el
3. 3.
tronco, lo que da un aspecto desgarbado al adolescente.
Aproximadamente la mitad de los hombres prepúberes tiene ginecomastia, bilateral, que
4. 4.
se resuelve espontáneamente en unos 3 años y se debe al aumento de estrógenos.
Gráfico de respuestas
Comentario

De esta pregunta acerca de la pubertad lo más importante que debe recordar es que el primer
signo de pubertad en las niñas es la aparición del botón mamario entre los 8-13 años y en los niños
el crecimiento testicular entre los 9-14 años.(R2)

216. Regarding measles, which of the following options is NOT correct?

Fetuses can acquire passive immunity from their mother through the placenta (provided
1. 1.
the mother has had measles).
2. 2. It can aggravate a pre-existing tuberculosis.
3. 3. Maximal viral dissemination occurs after the onset of skin rash.
4. 4. Measles is caused by an RNA virus, which belongs to the Paramyxoviridae family.

 
 
 
 
Gráfico de respuestas
Comentario

Pregunta de dificultad media que podemos contestar sabiendo que el periodo de máxima
diseminación del virus (y por lo tanto de máxima contagiosidad) se produce entre el 5º día antes de
que aparezca el exantema y el 5º después de la aparición del mismo, siendo, por lo tanto, la opción
3 la incorrecta. Es importante destacar la 3, puesto que el sarampión puede producir un descenso
transitorio en la inmunidad celular que puede exacerbar una tuberculosis. Este virus pertenece a la
familia de los paramyxovirus, en concreto al grupo de los morbillivirus (de ahí que un exantema de
características similares a las del sarampión reciba el nombre de morbiliforme) y los anticuerpos
maternos, al ser de tipo IgG pueden atravesar la placenta y conferir inmunidad al niño en los
primeros meses.(R3)

217. En relación al adenoma hepático, señale la opción que considere CORRECTA:

1. 1. Son más frecuentes en el lóbulo hepático izquierdo.


2. 2. La suspensión de anticonceptivos orales puede conseguir disminuir su tamaño.
3. 3. El paciente típico es un hombre de edad avanzada.
4. 4. Es un tumor poco vascularizado, por lo que no es útil el TC abdominal sino la RMN.
Gráfico de respuestas
Comentario

El adenoma hepatocelular es un tumor que puede malignizar siendo más frecuente en mujeres
jóvenes y en el lóbulo hepático derecho. Son tumores muy vascularizados y dentro de su
tratamiento debe considerarse la suspensión de anticonceptivos orales, a cuyo uso se asocian, ya
que puede disminuir el tamaño de los mismos. Son más frecuentes en el lóbulo hepático
derecho(R2)

218. Ante un neonato a término que tras cesárea presenta precozmente taquipnea y
cianosis que desaparece con pequeñas cantidades de oxigeno, en el que la radiografía
muestra marcas vasculares prominentes y liquido en las cisuras sin broncrograma aéreo,
sospecharemos:

1. 1. Taquipnea transitoria del recién nacido.


2. 2. Síndrome de aspiración meconial.
3. 3. Persistencia del la circulación fetal.
4. 4. Síndrome de Wilson-Mikity.
Gráfico de respuestas
Comentario

Tema que debe dominar al 100%, le cuentan el caso clínico típico de una taquipnea transitoria del
recién nacido, lo más importante es el antecedente de que una cesárea o parto rápido en la que
aún no a tenido tiempo para adaptarse presentando taquipnea y cianosis que mejora con oxígeno.
Revise el siguiente cuadro(R1)

 
 
 
 

219. Sobre las luxaciones de cadera, señale cuál es la INCORRECTA:

1. 1. Las más frecuentes son las posteriores.


2. 2. En la luxación anterior, el miembro queda en rotación externa y alargada.
3. 3. La luxación de cadera no requiere de una reducción urgente.
4. 4. En la luxación posterior puede existir una lesión asociada del nervio ciático.
Gráfico de respuestas
Comentario

Ante una luxación de cadera, es prioritario reducirla cuanto antes. De este modo, disminuye el
riesgo de complicaciones, como necrosis avascular de la cabeza del fémur, y además evitamos
que ésta pierda su "derecho de espacio" en la cavidad cotiloidea, siendo su reducción más difícil
cuanto más tiempo pase desde la luxación.(R3)

220. Acude a atender a un accidentado de tráfico. De entre las siguientes lesiones


atenderá prioritariamente:

1. 1. Conmoción cerebral.
2. 2. Neumotórax abierto.
3. 3. Enfisema subcutáneo cervical.
4. 4. Fractura abierta de fémur y tibia izquierdos.
Gráfico de respuestas
Comentario

 
 
 
 
Esta es una pregunta que no puede fallar. El ABCDE del manejo inicial del politraumatizado es un
tema que debe dominar.

Por dicho motivo la respuesta correcta es la 2, ya qu primero hay que controlar la vía aerea.(R2)

221. Un paciente de 45 años ha sido diagnosticado de varias neumonías de repetición.


No es fumador. Se descarta infección por VIH u otros tipos de inmunodepresión. Según
pasan las semanas, se da cuenta de que estos cuadros coinciden con algunos fines de
semana, que es cuando viaja a su pueblo para ver a su familia, donde tienen un palomar.
Estos cuadros inicialmente ocurrían de forma esporádica, pero cada vez son más
frecuentes y, últimamente, tiene tos persistente y en la radiografía de tórax aparece un
patrón reticular que afecta a ambas bases pulmonares. Teniendo en cuenta el diagnóstico
más probable, señale la afirmación CORRECTA:

Es probable que, en el lavado broncoalveolar, encontremos un aumento de linfocitos T


1. 1.
supresores (CD8).
2. 2. Debería evitar las visitas al palomar, ya que padece una psitacosis.
3. 3. Si se realiza una espirometría, lo más probable sería encontrar una alteración obstructiva.
4. 4. Se trata de una neumonía intersticial descamativa.
Gráfico de respuestas
Comentario

Un paciente joven con neumonías de repetición con patrón reticular y tos cada vez más persistente
que se da cuenta que aparecen cada vez que va a un palomar, tiene una neumonitis por
hipersensibilidad.

Recuerda que es una respuesta inmunológica anómala a la inhalación de material orgánico, y que
siempre existe un precipitante, en este caso las palomas. Es típico de esta enfermedad que en el
LBA haya aumento de linfocitos CD8.(R1)

222.
Un paciente de 64 años fumador y diabético, asintomático desde el punto de vista
cardiológico, se realiza un electrocardiograma de rutina como parte de un estudio
preoperatorio y un Holter que se indica por palpitaciones ocasionales. En el Holter se
detecta lo que aparece en la imagen nº 18, y que cursa de forma asintomática. ¿Cuál cree
que es el diagnóstico más probable del cuadro de este paciente?

1. 1. Isquemia silente.
2. 2. Angina estable.
3. 3. Angina inestable.

 
 
 
 
4. 4. Falsa negatividad del Holter.
Gráfico de respuestas
Comentario

La isquemia silente es la demostración en el Holter de cambios electrocardiográficos sugerentes de


isquemia miocárdica (descensos del segmento ST), pero que no se acompañan de dolor ni otros
síntomas.

Estos episodios de isquemia miocárdica asintomática son frecuentes en los enfermos que tienen
angina de pecho, pero también pueden darse en individuos asintomáticos que nunca han tenido
angina.

La presencia de episodios frecuentes de isquemia silente es un factor de mal pronóstico en los


individuos con angina de pecho. No está claro que el disminuir los episodios de isquemia silente
mediante tratamiento farmacológico mejore el pronóstico. No obstante, para estos enfermos, una
buena elección serían los betabloqueantes.(R1)

223. Varón de 61 años, fumador y bebedor importante, que acude a consulta por otalgia
derecha de 3 meses de evolución. La otoscopia es normal y la rinoscopia y faringoscopia
directas también. De lo siguiente, ¿qué cree que es prioritario en este paciente?

1. 1. Pedir un TAC de mastoides derecha.


Explorar el seno piriforme derecho por laringoscopia indirecta, a falta de otros medios
2. 2.
diagnósticos.
3. 3. Pedir una RMN de la ATM derecha.
4. 4. Hacer una audiometría con timpanometría.
Gráfico de respuestas
Comentario
En todo paciente de las características descritas en el enunciado es mandatorio descartar un tumor
de seno piriforme, ya sea por laringospia indirecta o directa.(R2)

224. A two-year-old child is brought to the pediatrician's office presenting with fever and
malaise. Vital signs are temperature 38 ºC, blood pressure 100/60 mmHg and heart rate
70 bpm. Physical examination reveals redness of his eyes, watery discharge and redness
of the pharyngeal mucose with some whitish lesions on the oral mucosa. An
erythematous rash is present on the back of his ears. What is the most likely diagnosis?

1. 1. Rubeola
2. 2. Chickenpox infection
3. 3. Rubella
4. 4. Scarlet fever
Gráfico de respuestas
Comentario
Rubeola. Measles, also known as rubeola, is a contagious disease caused by paramyxovirus. It
causes a flu-like syndrome with fever, malaise and conjuntivitis. Koplik’s spots are a pathognomonic
sign. The rash is generalized, maculopapular and it starts at the back of the ears spreading to the
head and neck and then most of the body.(R1)

 
 
 
 
225. Un recién nacido presenta dificultad respiratoria y cianosis que empeora al tomar el
alimento y mejora con el llanto. ¿Qué diagnóstico le parece más probable?:

1. 1. Tetralogía de Fallot.
2. 2. Atresia bilateral de coanas.
3. 3. Fístula tráqueo-esofágica.
4. 4. Enfermedad por membranas hialinas.
Gráfico de respuestas
Comentario

Los neonatos respiran como tendríamos que respirar todos los humanos, esto es, incorporando el
aire en reposo a través del valioso filtro nasal. Unicamente inspiran por la boca cuando lloran. Por
este motivo, un niño con atresia bilateral de coanas, mejora cuando está llorando, y se vuelve
incapaz de respirar cuando mama (tiene la boca reservada para la succión). El diagnóstico de la
atresia de coanas se hace a través del paso de una sonda nasogástrica por ambas fosas
nasales.(R2)

226. Referente a la Terapia de Rehidratación Oral (TRO). Señale la afirmación


INCORRECTA:

1. 1. La TRO, no es útil en pacientes con cólera.


2. 2. Puede ser aplicada cualquiera que sea la causa de la diarrea.
Se debe pasar a la Terapia de Rehidratación Intravenosa cuando el pH fecal es menor que
3. 3.
5, Glucocinta +++, Benedict +++, flujo fecal mayor que 10 ml/kg/hora.
La alimentación debe reiniciarse en la fase de mantenimiento en pequeños volúmenes y a
4. 4.
intervalos más cortos.
Gráfico de respuestas
Comentario

No hace falta que termine de leer la primera opción para que se pasa que es la mala. La terápia de
rehidratacion oral tal y como dice la respuesta 2 puede ser usada en cualquier causa de diarrea por
lo tanto también en el cólera. Siempre que se pueda usar la via oral, se debe usar. Se debe pasar
a rehidratación intravenosa cuando las condiciones lo requieren.(R1)

227. ¿Cuál es el síntoma más precoz del cáncer de páncreas?:

1. 1. Dolor epigástrico.
2. 2. Ictericia obstructiva.
3. 3. Pérdida de peso.
4. 4. Vesícula palpable.
Gráfico de respuestas
Comentario
La pérdida de peso no sólo es el síntoma más precoz del cáncer de páncreas si no que también es
el síntoma más frecuente independiente mente de su localización. Esta pérdida de peso es en
general importante con una media de 10 kilos. La pérdida de peso puede producirse inicialmente
como un síntoma aislado frente a un apetito aparentemente normal. Después se asocia
generalmente con anorexia. Una pérdida de peso inexplicable debe ponernos rápidamente en la
búsqueda de una neoplasia oculta, y en adultos y ancianos resulta apropiado hacer un TAC
abdominal con está única indicación.(R3)

 
 
 
 

228. Hombre de 49 años, bebedor de


45 g/día de alcohol y fumador de 20 cig/día, acude a su consulta por astenia. En la
exploración física se observaba un abdomen normal. En los exámenes de laboratorio
destaca: Hb 10.5 g/dl, VCM 65.5 fl, HCM 22.3 pg, leucocitos 9500/mm3 con 4300
neutrófilos, plaquetas 90,000/mm3, glucosa 123 mg/dl, urea 62 mg/dl, creatinina 1.1 mg/dl,
AST 214 U/L, ALT 270 U/L, GGT 540 U/L, bilirrubina total 2.4 mg/dl, proteínas totales 4.1
g/dl, albúmina 2.4 g/dl, fosfatasa alcalina 170 U/L, LDH 120, sodio 135, potasio 4.2.
Serología: antiHBs -, AgHBs -, AntiHBc -, AgHBe -, antiHBe-, anti-VHC +, AgVHD-, IgG
VHA+. En una gastroscopia realizada se observa a nivel antral la imagen presentada.
¿Cuál es el diagnóstico más probable?:

1. 1. Hemorragia por varices esofágicas.


2. 2. Gastropatía hipertensiva.
3. 3. Gastritis por Helicobacter pylori.
4. 4. Neoplasia gástrica.
Gráfico de respuestas
Comentario
Se puede observar que el paciente presenta datos analíticos de hepatopatía crónica de probable
origen alcohólico más posible hepatitis crónica C. La endoscopia muestra una mucosa antral con
múltiples angiodisplasias por las que el paciente presentará seguramente un sangrado crónico que
justifique la anemia microcítica que tiene. Todo ello corresponde a una gastropatía hipertensiva
severa.(R2)

229. En relación al diagnóstico realizado en este paciente, ¿qué actitud sería MENOS
útil?:

1. 1. Octreótido retardado subcutáneo.


2. 2. Fulguración con argón-plasma.
3. 3. Erradicación H. pylori.
4. 4. Prohibir consumo de alcohol.
Gráfico de respuestas
Comentario

 
 
 
 
La erradicación para H. pylori no influirá en la evolución de este paciente ni disminuirá la
hemorragia crónica que tiene. El octreótido en preparados de liberación retardada permitirá una
disminución de la presión y, sobre todo, parece disminuir la formación de neovasos que agraven la
gastropatía. Asimismo la fulguración con argon permitirá quemar en una o varias sesiones
periódicas dichas lesiones para evitar que sangren.(R3)

230. Femenino de 59 años, menopaúsica desde hace 6 años, que acude a la consulta de
ginecología para su revisión anual. La exploración física es rigurosamente normal, pero
se decide realizarla un ultrasonido para seguimiento de unos miomas que han ido
desapareciendo. En el ultrasonido ya no hay rastro de los miomas y el espesor del
endometrio es de 9 mm. ¿Qué haría a continuación?:

1. 1. Mandarla volver a revisión, como venía haciendo.


2. 2. Administración de acetato de medroxiprogesterona en la segunda mitad del ciclo.
3. 3. Histeroscopía con biopsia dirigida.
4. 4. Legrado diagnóstico.
Gráfico de respuestas
Comentario

Pregunta de dificultad moderada sobre la hiperplasia endometrial y el cáncer de endometrio.

La hiperplasia endometrial es una proliferación del endometrio, originada por el estímulo de los
estrógenos no compensada por la adecuada secreción de progesterona.

Se clasifica en simple/ compleja, con/ sin atipia.

El diagnóstico es anatomopatológico mediante histeroscopía con biopsia dirigida (RC- 3). Las
indicaciones para realizar esta prueba es la presencia de metrorragia en mujer postmenopáusica
(es obligatorio descartar cáncer de endometrio) y la presencia de un diámetro del endometrio por
USG transvaginal mayor de 15 mm en premenopáusicas y de 5 mm en postmenopáusicas (en el
caso mide 9 mm).(R3)

231. El cáncer de mama es un tumor dependiente de estrógenos, lo que permite el uso de


múltiples tratamientos hormonales, aparte de los quimioterápicos que también pueden
estar indicados. Señale, entre los siguientes, qué fármaco es un antiestrógeno puro:

1. 1. Raloxifeno.
2. 2. Exemestrano.
3. 3. Fulvestrant.
4. 4. Letrozol.
Gráfico de respuestas
Comentario

El tamoxifeno y raloxifeno tienen acción proestrogénica en endometrio. El exemestrano es un


inhibidor irreversible de la aromatasa. El letrozol es un inhibidor reversible de la aromatasa más
selectivo, por lo que no afecta a hormonas tiroideas, adrenales, etc.(R3)

232. En la amniocentesis precoz para el diagnóstico prenatal, es FALSO:

1. 1. Se realiza por punción transabdominal.

 
 
 
 
2. 2. El momento idóneo es antes de la 18ª semana de gestación.
3. 3. Útil para el estudio del cariotipo fetal.
4. 4. El índice de abortos como complicación de la técnica es del 5%.
Gráfico de respuestas
Comentario

Pregunta sobre los métodos de diagnóstico invasivo prenatal. La amniocentesis precoz es la


técnica más utilizada por su facilidad y bajo índice de complicaciones. La amniocentesis se realiza
antes de las 18 semanas, siendo la 14ª, la semana idónea; se realiza mediante punción
transabdominal extrayendo 100 ml de líquido amniótico.

Sus indicaciones son el estudio del cariotipo fetal, diagnóstico de defectos congénitos enzimáticos
y diagnóstico de infecciones congénitas. Tiene un bajo índice de riesgos y complicaciones, entre
los que figuran un 1% de abortos; por ello, la opción falsa es la 4.(R4)

233. Un paciente masculino del que se desconocen antecedentes es traído a Urgencias


por su familia por fiebre de varios días de evolución con deterioro reciente del nivel de
conciencia y ataxia. En la exploración se objetiva rigidez de nuca, nistagmus y parálisis
facial central. ¿Cuál de los siguientes microorganismos sería el causante más probable
del cuadro del enfermo?

1. 1. Neumococo.
2. 2. Listeria.
3. 3. Enterobacterias.
4. 4. Virus herpes.
Gráfico de respuestas
Comentario

Una pregunta de cierta dificultad, ya que es necesario reconocer unas características determinadas
de la afectación meníngea por uno de los patógenos. El caso no ofrece información, como en otras
ocasiones, de la edad o del LCR. Lo llamativo es la afectación cerebelosa dentro del contexto de
un síndrome meníngeo: ataxia y nistagmo.

De los microorganismos que se ofertan recuerde que la Listeria produce un cuadro de


romboencefalitis. Es típico, aunque no frecuente. Puede producir parálisis asimétricas de pares
craneales, alteración del nivel de conciencia, signos cerebelosos y déficit motor o sensitivo.(R2)

234. La nutrición del cartílago articular en las articulaciones diartrodiales se realiza a


partir de:

1. 1. Vasos linfáticos óseos.


2. 2. Líquido sinovial.
3. 3. Vasos linfáticos de la membrana sinovial.
4. 4. Arteriolas del hueso cortical.
Gráfico de respuestas
Comentario

El cartílago articular es una estructura avascular que carece de vasos tanto sanguíneos como
linfáticos y se nutre por difusión desde el líquido sinovial. En huesos muy jóvenes también puede
realizarse la nutrición desde el hueso subcondral. Recuerde que también el cartílago articular
carece de inervación.(R2)

 
 
 
 
235. En un paciente colecistectomizado hace 8 días, que presenta fiebre de 38º C desde
hace 2 días, ¿cuál sería la causa de fiebre más probable?:

1. 1. Infección urinaria.
2. 2. Flebitis.
3. 3. Infección herida quirúrgica.
4. 4. Absceso intraabdominal.
Gráfico de respuestas
Comentario
Tenemos un caso de fiebre en el 8º día de postoperatorio de una colecistectomía. Se trata de
valorar la causa más frecuente de fiebre en ese tiempo del postoperatorio y debemos recordar que
las causas más frecuentes de fiebre a partir del tercer día de postoperatorio son en primer lugar las
infecciosas: infección de la herida quirúrgica, ITU, absceso intraabdominal, y las fugas y fístulas
anastomóticas. Al tratarse de una colecistectomía no existen anastomosis por lo que descartamos
las fugas y fístulas anastomóticas, centrando nuestras opciones entre la ITU, la infección de la
herida quirúrgica y la presencia de un absceso intraabdominal. Teniendo en cuenta que tanto la
ITU como la infección de la herida suelen ser algo más precoces en su aparición que en el 8º día
nos decidimos por el absceso intraabdominal.(R4)

236. ¿En cuál de las siguientes localizaciones es más frecuente el absceso cerebral a
partir de un foco de otitis crónica?

1. 1. Lóbulo parietal.
2. 2. Lóbulo frontal.
3. 3. Cerebelo.
4. 4. Hipófisis.
Gráfico de respuestas
Comentario

Las paredes del oído medio están en contacto con la fosa craneal posterior y con la fosa media. En
la parte superior, el tegmen tympani y el tegmen mastoideo forman el techo del oído medio y
limitan con la duramadre del lóbulo temporal, donde pueden formarse abscesos, dando clínica de
hipertensión intracraneal, cambio de carácter, torpeza, cefalea, déficit motores, crisis comiciales….
Por detrás, el oído se relaciona con la fosa craneal posterior, donde se encuentra el cerebelo y
donde pueden formarse abscesos, siendo la clínica habitual la alteración del equilibrio y la marcha,
de la coordinación de movimientos e hipotonía.(R3)

237. Ante un paciente con un cuadro consistente en demencia, diarrea, oligoartritis


migratoria y miorritmia oculomasticatoria, ¿cuál de las siguientes exploraciones
realizaría en primer lugar?:

1. 1. Biopsia del nervio sural.


2. 2. Biopsia hepática.
3. 3. Biopsia cutánea.
4. 4. Biopsia del intestino delgado.
Gráfico de respuestas
Comentario
La asociación clínica de diarrea, oligoartritis migratoria y alteraciones del sistema nervioso central
(demencia,miorritmia oculomasticatoria), deben hacernos sospechar como primera posibilidad
diagnóstica la enfermedad de Whipple. Por tanto, estaría indicada la biopsia de intestino delgado,
porque sería

 
 
 
 
diagnóstica: se observarían macrófagos con inclusiones PAS+ en su interior, que corresponden a
las propias bacterias.(R4)

238. ¿Cuál de las siguientes patologías hematológicas se ha asociado a PAN?:

1. 1. Síndrome mielodisplásico.
2. 2. Tricoleucemia.
3. 3. Enfermedad de Hodgkin.
4. 4. Leucemia linfoide aguda.
Gráfico de respuestas
Comentario

Concepto que hasta ahora no se ha preguntado en el ENARM. Se ha demostrado una clara


asociación de la PAN clásica con la infección por virus de la hepatitis B (30%); y, en un menor
caso, asociación con el virus de la hepatitis C. Además, hay casos descritos de asociación con
tricoleucemia. Recuerda que los síndromes mielodisplásicos y linfoproliferativos, en ocasiones
pueden estar asociados con patología reumatológica, como las crioglobulinemias o el síndrome de
Sjogren.(R2)

239. Una mujer de 49 años acude a consulta por presentar rectorragia y heces de aspecto
mucoide. En el examen endoscópico se encuentra una tumoración plana de superficie
vellosa, de 4x3 cm, que se reseca endoscópicamente aparentemente de forma completa.
Histológicamente, se informa de un adenoma velloso, descartándose infiltración
neoplásica del tallo vascular. ¿Cuál es la actitud respecto a esta paciente?

Resección segmentaria de la zona donde implantaba el pólipo, con anastomosis término-


1. 1.
terminal posteriormente.
Realizar los test genéticos oportunos para descartar una poliposis colónica familiar entre
2. 2.
sus familiares.
3. 3. Observación periódica.
4. 4. Tránsito baritado.
Gráfico de respuestas
Comentario

Se trata de un adenoma velloso resecado completamente. El siguiente paso es el seguimiento


endoscópico periódico.(R3)

240. Señale la respuesta FALSA respecto a la Esclerosis Sistémica:

1. 1. Las calcificaciones de partes blandas se localizan en el tejido cutáneo y/o subcutáneo.


Un factor de riesgo para sufrir crisis renal esclerodérmica es la instauración de derrame
2. 2.
pericárdico.
La esclerosis cutánea es una manifestación sin la cual no puede establecerse el
3. 3.
diagnóstico.
4. 4. La miopatía propia de la esclerosis sistémica no produce elevación de enzimas musculares.
Gráfico de respuestas
Comentario
Esta es una pregunta un tanto difícil de la esclerosis sistémica, en el MIR suelen caer las
complicaciones y los anticuerpos. La calcinosis consiste en el depósito de calcio en piel y tejido
celular subcutáneo (hidroxiapatita cálcica). Las alteraciones cardiacas, cuando aparecen, se

 
 
 
 
asocian a mal pronóstico, y las crisis renales aparecen en estos momentos complicando la vida del
enfermo (aunque ha mejorado con los IECAs). La atrofia muscular se produce por desuso debido a
la limitación de la movilidad, pudiendo generar contracturas por flexión; no es pues, una miopatía
inflamatoria como tal, no se eleva la CPK como en las miositis. El hipotiroidismo sucede si la
fibrosis alcanza la glándula tiroidea, aunque puede ser de origen autoinmune por Ac. antitiroideos.
La esclerosis sistémica puede existir sin afectación cutánea, si bien en la mayoría de los enfermos
es una constante, pero no imprescindible para el diagnóstico.(R3)

241. En relación con la edad gestacional (EG), marque la CORRECTA:

1. 1. El ultrasonido prenatal tiene mayor dispersión y confiabilidad a mayor EG.


2. 2. La valoración neuromuscular es suficiente para determinar la EG.
3. 3. Los periodos de adaptación podrían alterar al Balllard y al Dubowitz.
4. 4. Siempre hay concordancia con los datos obstétricos.
Gráfico de respuestas
Comentario

Respuesta sencilla de la edad gestacional. La respuesta correcta es la 3, ya que por obvias


razones los periodos de adaptación pueden alterar la escala de Ballard y de Dubowitz.

El ultrasonido prenatal tiene una mayor confiabilidad a menor EG. La valoración neuromuscular no
es suficiente para determinar la EG, existen otras determinaciones más sensibiles y específicas y
no todos los datos obtenidos serán concordantes entres sí.(R3)

242. ¿Cuál de los siguientes nos ayudaría más a diferenciar una anemia ferropénica de
una anemia de las enfermedades crónicas?:

1. 1. Saturación de transferrina.
2. 2. VCM.
3. 3. CHCM.
4. 4. IDE.
Gráfico de respuestas
Comentario
Es la típica pregunta en donde debéis diferenciar dos síndromes anémicos. La sideremia se
encuentra disminuida en ambas patologías por lo que es poco útil para diferenciarlas (opción 1
falsa). El VCM es un parámetro poco especifico, recordad que hay muchas anemias que pueden
cursar con microcitosis, entre ellas la ferropenia y la anemia de trastornos crónicos (opción 2 falsa).
También descartamos fácilmente la opción 4, la saturación de trasferrina suele estar aumentada en
ambas, luego la duda surge entre las opciones 4 y 5. La CHCM es un índice que expresa la
concentración de Hb de cada hematíe, ambas anemias por ser hiporegenerativas, suelen cursar
con valores bajos (opción 4 falsa). El IDE o ADE o RDW (amplitud de la distribución eritrocitaria)
indica el grado de anisocitosis que hay en un frotis sanguíneo. Se considera normal un ADE del
12% aproximadamente, y debéis recordar que en la anemia ferropénica esta aumentado (opción 5
verdadera), mientras que permanece normal en otras anemias microciticas como la de trastornos
crónicos y algunas talasemias.(R4)

243. Todas las siguientes son características de la lesión típica de morfea en placas
EXCEPTO una, señálela:

1. 1. Placa con coloración central blanca, nacarada o amarillenta, a veces con telangiectasias.

 
 
 
 
2. 2. Ausencia de folículos pilosos y alteración de la secreción.
3. 3. Prurito o sensación de quemazón importante en las zonas de la placa en evolución.
4. 4. El tronco es la zona del cuerpo más afectada por las lesiones.
Gráfico de respuestas
Comentario
Son diversas las clasificaciones que podemos encontrar acerca de la esclerodermia, como la que
diferencia tres tipos de cuadros: esclerodermia circunscrita o morfea (no cursa con afectación de
órganos internos), esclerosis sistémica progresiva (ESP) y cuadros esclerodermiformes
(idiopáticos, genéticos, tóxicos...). Con independencia de la forma clínica, la evolución de los
distintos tipos de morfea es muy similar: lentamente una o varias áreas del cuerpo se vuelven
eritematosas y se van indurando en la zona central, la cual va blanqueando, quedando en los
bordes una coloración violácea (lilac ring). Tras 1ó2 años el eritema desaparece y persiste sólo una
placa dura, nacarada y sin anejos. Durante la evolución descrita los pacientes no suelen referir
sintomatología subjetiva significativa (respuesta 4 correcta). A esta evolución general deberíamos
sumar posteriormente las características peculiares de cada tipo de morfea ( distribución,
afectación de planos profundos..).(R3)

244. El cáncer de cérvix se origina preferentemente en:

1. 1. Ectocérvix.
2. 2. Unión escamo-columnar.
3. 3. Glándulas cervicales.
4. 4. Quistes de Naboth.
Gráfico de respuestas
Comentario

La unión escamo-columnar es el sitio en donde se encuentra el mayor número de cáncer de cérvix.


Respuesta 2 correcta.(R2)

245. A 32-year-old secondigravid woman at 9 week's gestation comes to your office for
her first prenatal visit. She has been feeling some morning sickness, but besides that she
has no other complaints. Her first child was born prematurely at 33-week's gestation, she
denies smoking or drinking and she takes no medications. Physical examination reveals
a 9-week sized non-tender uterus. The patient wants information on vitamin
supplementation during pregnancy. Which of the following represents the correct amount
of vitamin A supplementation this patient should take daily?

1. 1. 100 000 UI
2. 2. 50 000 UI
3. 3. 25 000 UI
4. 4. Vitamin A supplementation during pregnancy is not recommended
Gráfico de respuestas
Comentario
Vitamin A supplementation during pregnancy is not recommended. Pregnant women should take 10
micrograms of vitamin D and 400 micrograms of folic acid per day. However, vitamin A supplements
are not recommended, as too much of this vitamin could harm the baby.(R4)

246. De los siguientes factores, ¿cuál NO se considera de riesgo para el cáncer de


mama?:

 
 
 
 
1. 1. Fallo ovárico precoz.
2. 2. Síndrome de Li-Fraumeni.
3. 3. Raza blanca.
4. 4. Historia familiar de cáncer mamario.
Gráfico de respuestas
Comentario

Debe recitar todos los factores de riesgo más importantes de cáncer de mama para el ENARM.

El síndrome de Li-Fraumeni implica una mutación en el gen que codifica la proteína p53, lo que
predispone en gran medida al padecimiento de tumores muy diversos. Las lesiones proliferativas
con atipias son las que más predisponen al cáncer de mama en la mastopatía fibroquística. Sin
embargo, las proliferativas sin atipias también implican cierto riesgo, aunque no tan elevado. Las
que no predisponen al cáncer de mama son las lesiones no proliferativas.

La solución correcta es la 1. El fallo ovárico precoz no se ha relacionado con el cáncer de mama,


ya que supone un acortamiento de la edad fértil y, por ello, menor exposición a estrógenos por
parte de la glándula.(R1)

247. Nos remiten a la consulta una paciente mujer de 9 años de edad, para realizar estudio
por talla baja. Entre los antecedentes gestacionales de la niña tenemos que el peso al
nacimiento fue de 3,050 Kg, talla al nacimiento 49.5 cm, y antecedentes familiares de
estatura paterna 161 cm, estatura materna de 152 cm y menarquia a los 13 años. En el
momento de la consulta la paciente presenta una estatura de 115 cm. En los distintos
estudios, tras la determinación de los niveles de hormona de crecimiento, se diagnostica
a la paciente de talla baja familiar, planificándose la siguiente revisión a los 6 meses.
Como hemos visto la talla baja puede ser idiopática o aparecer en el contexto de diversos
síndromes genéticos y polimalformativos. Sobre la talla baja familiar, señale cuál es la
respuesta INCORRECTA:

1. 1. La talla baja familiar es la causa más frecuente de talla baja.


2. 2. No existe discordancia entre la edad cronológica y la edad ósea.
3. 3. Las pruebas de laboratorio son normales.
4. 4. La pubertad se alcanza después que la mayoría de los niños.
Gráfico de respuestas
Comentario

Todas las afirmaciones se cumplen en la talla baja familiar, salvo la respuesta número 4: la
pubertad se retrasa en el retraso constitucional del crecimiento y el desarrollo, lo dice el propio
nombre.

En la talla baja familiar el crecimiento se mantiene por debajo del percentil 3 en paralelo a las
líneas de crecimiento normales para la edad, y no hay retraso madurativo, por tanto tampoco hay
discordancia entre las edades ósea y cronológica. Es, simplemente, una cuestión de origen
genético; de padres bajitos, difícilmente podrá obtenerse un niño más alto que la media.(R4)

248. Señale lo FALSO con respecto a las complicaciones de la parotiditis:

1. 1. La complicación más frecuente de la parotiditis es la meningoencefalitis.


En el LCR de pacientes con parotiditis y, meningoencefalitis no se aísla el virus de la
2. 2.
parotiditis.

 
 
 
 
3. 3. La púrpura trombocitopénica es una complicación infrecuente.
Hasta en un 13% de los adultos afectos se encuentran alteraciones en el ST del
4. 4.
electrocardiograma.
Gráfico de respuestas
Comentario

Pregunta de dificultad alta sobre las complicaciones de la parotiditis, ya que no encontramos todas
las opciones en el manual. La opción 1 es correcta y es el dato básico que debemos saber acerca
de las complicaciones de esta enfermedad. No se suele encontrar el virus en el LCR, por lo que la
respuesta falsa es la 2. El siguiente dato que debemos recordar es que la sordera es más
frecuentemente bilateral. El resto de opciones son difíciles y no son rentables en el examen.(R2)

249. Un señor de 60 años, empleado de comercio, acude a Urgencias acompañado de su


familia; éstos cuentan que el día anterior, de forma bastante brusca, comenzó a decir
cosas raras, a no responder a lo que le preguntaban y a mostrarse confuso incluso
respecto de su propio nombre. Efectivamente en la exploración parece no entender lo
que se le pregunta, no recuerda nada de lo que le ha pasado y no sabe ni el día, ni el lugar
en el que está. El diagnóstico sería:

1. 1. Psicosis breve.
2. 2. Delirium.
3. 3. Esquizofrenia.
4. 4. Trastorno de ansiedad.
Gráfico de respuestas
Comentario

El caso que nos describen caracteriza bastante bien un delirium (síndrome confusional agudo).Es
una situación clínica muy usual y preguntada en el MIR.

El síndrome confusional agudo suele aparecer en pacientes mayores, con alguna patología
orgánica subyacente. En este caso, no la mencionan, pero esto no excluye el diagnóstico, ya que
podría no estar diagnosticada. El delirium se manifiesta de forma aguda, con deterioro del nivel de
conciencia, acompañado, a veces, de síntomas psicóticos (alucinaciones, delirios poco
sistematizados). Recuerda que lo más característico es el deterioro del nivel de conciencia y la
desorientación.

Dado que el delirium obedece a causas orgánicas, suele desaparecer con el tratamiento de las
mismas. El factor de riesgo más importante es la edad, aumentando la frecuencia cuando se trata
de pacientes hospitalizados.

Respecto a las otras opciones, pueden descartarse fácilmente:

•   R1: el término psicosis breve hace referencia a un cuadro que aparece después de un
estrés psicosocial reconocible, con una duración mayor de una hora, pero menor de dos
semanas. Dado que falta un factor identificable, no podríamos emitir este diagnóstico.
•   R3: la enfermedad de Alzheimer se manifiesta lentamente, con el deterioro crónico propio
de un cuadro demencial.
•   R4: la esquizofrenia exige para su diagnóstico un mínimo de seis meses. Ni siquiera
llegaría a ser un trastorno esquizofreniforme (duración menor de seis meses, pero mayor
de un mes). Por otra parte, la esquizofrenia difícilmente aparecería a los 60 años.

 
 
 
 
•   R5: una cosa es un trastorno de ansiedad, y otra muy distinta es alcanzar este grado de
desorientación (no sabe el día ni el lugar, e incluso está confuso respecto a su propio
nombre…).

(R2)

250. Una mujer de 61 años de edad padece dolores óseos de años de evolución; para
calmarlos, toma cantidades importantes de fenacetina. En una revisión se encuentra
piuria y elevación de las cifras de creatinina sérica. ¿Qué esperaría encontrar en esta
enferma?:

1. 1. Alcalosis metabólica.
2. 2. Hipopotasemia.
3. 3. Hiponatremia.
4. 4. Necrosis papilar.
Gráfico de respuestas
Comentario
Pregunta de considerable dificultad sobre la nefropatía por analgésicos. Esta enfermedad suele
presentarse con los siguientes datos: insuficiencia renal (IR), anemia desproporcionada para el
grado de IR, piuria con cultivos negativos, y el antecedente de ingesta prolongada y en gran
cantidad de analgésicos. Debes recordar que se asocia característicamente a dos patologías: la
necrosis papilar (respuesta 5 correcta) y los carcinomas de células transicionales (uroteliomas) en
pelvis y uréter. Recuerda que la necrosis papilar se puede diagnosticar por el ?signo del anillo? en
la pielografía iv, que es patognomónico de esta enfermedad. Representa a la papila radiolúcida
anulada, rodeada por el material de contraste radiodenso en el cáliz. Los efectos de los AINEs
sobre el aparato urinario incluyen cinco síndromes: 1) necrosis tubular aguda tóxica; 2) nefritis
intersticial inmunoalérgica aguda; 3) nefritis intersticial crónica con necrosis papilar; 4) síndrome
nefrótico por cambios mínimos; y 5) uroteliomas.(R4)

251. Mujer de 34 años, con disnea de inicio hace 3 años. Acude por empeoramiento súbito
de su disnea basal. La Rx de tórax revela la presencia de un pequeño neumotórax junto
a un patrón reticulonodular difuso y mínimo derrame pleural. Se realiza una
toracocentesis, obteniéndose un líquido de aspecto quiloso. Uno de los diagnósticos
más probables es:

1. 1. Proteinosis alveolar pulmonar.


2. 2. Fibrosis pulmonar idiopática.
3. 3. Linfangioleiomiomatosis.
4. 4. Amiloidosis pulmonar.
Gráfico de respuestas
Comentario
El cuadro clínico descrito es muy sugestivo de una linfangioleiomiomatosis. Esta enfermedad que
afecta típicamente a mujeres en edad fértil cursa clínicamente con disnea, tos y hemoptisis. El
neumotórax puede aparecer en el 50% de los casos y puede ser recurrente y bilateral. El quilotórax
es muy característico de esta enfermedad aunque raro al diagnóstico apareciendo en un tercio de
los pacientes. La ooforectomía y el tratamiento con tamoxifeno pueden mejorar algo la
sintomatología. Cuando el anterior tratamiento fracasa se debe considerar el trasplante de
pulmón.(R3)

 
 
 
 
252. En el histerocele ¿Cuáles son los ligamentos más comprometidos?

1. 1. Cardinales.
2. 2. Redondos.
3. 3. Útero – ovárico.
4. 4. Infundíbulo – pélvico.
Gráfico de respuestas
Comentario

El prolapso uterino es una patología muy frecuente en la mujer y la corrección de las distopías, se
debe buscar reposicionar la vagina a la posición más próxima posible de la original, sea usando los
ligamentos naturales de suspensión vaginal cardinales (respuesta correcta 1 ) y sacrouterinos, o
empleando otros puntos de suspensión (tales como el ligamento sacroespinosos o a la fascia del
músculo íleococcígeo) o, también, buscando la creación de neo-ligamentos por medio de
prótesis.(R1)

253. Un lactante de 3 semanas llega al servicio de urgencia en coma, con hemorragias


retinianas e intensa palidez. Nació en su casa y fue visto por vez primera por un médico,
10 días después, y tratado con amoxicilina para una otitis media. El día anterior al ingreso,
sus padres le llevaron a una excursión de montaña por una carretera abrupta en un
vehículo de 4 ruedas. Ocho horas después comenzó con convulsiones y su estado
empeoró ininterrumpidamente en las siguientes 16 horas. Presenta hemorragia por todos
los sitios de punción venosa. El diagnóstico más probable de este niño es:

1. 1. Escorbuto grave.
2. 2. Enfermedad hemorrágica del recién nacido.
3. 3. Malos tratos (zarandeo del niño).
4. 4. Hipervitaminosis A.
Gráfico de respuestas
Comentario

La enfermedad hemorrágica del recién nacido se puede manifestar en distintos períodos de


tiempo. Especialmente precoz y grave es su presentación en los primeros días de vida, en los hijos
de madre consumidora de antiepilépticos (fenobarbital, fenitoína). Pero también se puede
presentar más tardíamente, como en este enunciado. Se puede prevenir con la administración de
vitamina K i.m.(R2)

254. El test post coital (Test de Sims Hünner) positivo valora. Marque lo FALSO:

1. 1. El factor cérvico vaginal.


2. 2. El factor masculino.
3. 3. Factor tubario – peritoneal.
4. 4. No es útil en infección cervical.
Gráfico de respuestas
Comentario

Pregunta complicada.

Es la prueba que permite la evaluación de la habilidad espermática para penetrar en el moco


cervical pre-ovulatorio. El propósito de esta prueba no es solo determinar el número de
espermatozoides activos en el moco cervical, sino además evaluar la supervivencia y el

 
 
 
 
comportamiento espermático varias horas después del coito. La presencia de un número adecuado
de espermatozoides móviles, 6 a 10 horas después del coito, en el moco, excluye el factor cervical
como posible causa de infertilidad. En casos de infección cervical esta prueba carece de utilidad.

La respuesta que debemos elegir es la 3.(R3)

255. Indique cuál de las siguientes características de la mucoviscidosis es FALSA:

1. 1. La incidencia de ulcus péptico en los niños con mucoviscidosis es elevada.


2. 2. Se asocia a una alteración en el brazo largo del cromosoma 7.
La presencia de Diabetes Mellitus tipo I es mayor en los niños con este cuadro que en el
3. 3.
resto de la población infantil.
El test del sudor se considera positivo si presenta valores superiores a 70 mEq/L de cloro y
4. 4.
sodio.
Gráfico de respuestas
Comentario

La fibrosis quística es poco importante en el ENARM, si bien esta pregunta no tiene mucha
relevancia. Lo que sí debe recordar es que el defecto fundamental consiste en la alteración de la
proteína reguladora de la conductancia de tras membrana codificado por un gen situado en el
brazo largo del cromosoma 7, que se hereda como rasgo autosómico recesivo y que la mutación
más prevalente es la AF508.

El test del sudor es el método diagnóstico principal y que es positivo con valores de cloruro por
encima de 60mEq/l. El íleo meconial se resuelve con enemas de gastrografin, diatrizoato de
meglumina, o N acetilcisteina y si no, con cirugia.

El 10 % tiene diabetes y es debida a la insulinopenia secundaria a fibrosis del páncreas con


destrucción de las células beta (no a la destrucción autoinmune de la diabetes tipo 1). La clínica es
aparece en mayores de 10 años, es muy insidiosa y parecida a la diabetes tipo 2 ya que la
secreción residual de insulina se mantiene mucho tiempo, por lo que no suelen presentar
cetoacidosis ni el cuadro típico de poliuria, polidispsia de la diabetes tipo1.(R3)

256. An 18-month-old child is brought to the ER due to swelling in his right knee after
playing in the park, with no apparent trauma. He is up-to-date on his immunizations. The
mother refers that her uncle had similar problems. Ultrasound examination is consistent
with hemarthrosis, and lab tests show an APTT of 52 seconds (normal 25-35). Which of
the following is the most likely diagnosis?

1. 1. Marfan syndrome.
2. 2. Ehlers Danlos disease.
3. 3. Von Willebrand disease.
4. 4. Hemophilia A.

(R4)

257. ¿Cuál es la localización más frecuente del cáncer gástrico?:

1. 1. Cardias.
2. 2. Cuerpo.
3. 3. Antro.

 
 
 
 
4. 4. Prepilórico.
Gráfico de respuestas
Comentario
Se trata de una pregunta directa. Los tumores gástricos más frecuentes son los adenocarcinomas
gástricos de tipo intestinal. Y son éstos los que tienen su localización más frecuente a nivel distal,
en el antro gástrico. En los últimos años parece haber una tendencia a aumentar los tumores
gástricos con localización en cardias. El adenocarcinoma gástrico de tipo difuso,mucho menos
frecuente, se encuentra formando pliegues gástricos engrosados, por toda la superficie
gástrica.(R3)

258. Hombre de 84 años con antecedentes de tabaquismo y de hepatocarcinoma tratado


mediante alcoholización hace 7 días que consulta en urgencias por fiebre de hasta 39 °C
con escalofríos, dolor pleurítico y síndrome tóxico. En la radiografía de tórax se aprecia
una condensación pulmonar en lóbulo inferior izquierdo con broncograma aéreo
sugestiva de neumonía, por lo que se decide ingreso y tratamiento antibiótico
endovenoso. Inicialmente, el paciente experimenta una franca mejoría, pero al tercer día
del ingreso presenta nuevamente fiebre de 38,3 °C y un fuerte dolor pleurítico izquierdo
por lo que el médico de guardia solicita una nueva radiografía de tórax en la que observa
la condensación parenquimatosa anteriormente descrita y una lesión de márgenes
obtusos de nueva aparición posterior izquierda. ¿Cuál es el diagnóstico y el tratamiento
más adecuado ante la nueva situación del paciente?

Se trata de la progresión normal de la neumonía y lo único que debernos hacer es esperar a


1. 1.
que responda al tratamiento antibiótico.
Podría tratarse de un mesotelioma que pasara desapercibido en la radiografía inicial de
2. 2.
Urgencias.
Dados los antecedentes de tabaquismo del paciente, el infiltrado inicialmente interpretado
3. 3. como una neumonía podría tratarse de una neoplasia de pulmón y esta nueva lesión de una
metástasis pleural que deberíamos puncionar.
Puede tratarse de una neumonía complicada con un derrame, el líquido debería ser
4. 4.
analizado por si es tributario de la colocación de un drenaje pleural.
Gráfico de respuestas
Comentario

Un caso clínico de dificultad media sobre una de las complicaciones de la neumonía: el derrame
paraneumónico.

Se denomina así al que se asocia con infección bacteriana pulmonar no tuberculosa (una
neumonía bacteriana, como en este caso). El 40% de los pacientes con neumonía bacteriana
tienen derrame (su presencia se asocia con una mayor mortalidad).

En nuestro medio, el derrame paraneumónico es la causa más común de exudado. Siempre que
se evalúe un paciente con neumonía debe descartarse su presencia.

La evolución que nos muestran en este caso es bastante típica. La neumonía responde
favorablemente a los antibióticos y, a los pocos días, el paciente vuelve a empeorar y reaparece la
fiebre. En estos casos debe descartarse la presencia de esta complicación. En esta pregunta
debemos sospecharla, puesto que la radiografía de tórax nos habla de una nueva imagen en
región posterior izquierda, compatible con derrame pleural.

 
 
 
 
Recuerda que, para visualizar correctamente un derrame pleural es muy conveniente una
radiografía de tórax en decúbito lateral. Se considera no significativo si, en esta posición, la
cantidad de líquido acumulada es menor de 10 mm, en cuyo caso no se realiza toracocentesis. Si
excede de 10 mm, sí se realiza.(R4)

259. Recién nacido de 12 horas de madre primípara de 18 años de edad, quien a las 32
semanas presentó infección urinaria gestacional desencadenando el parto. Al exámen:
peso al nacer 1,400 gramos y test de Silverman Anderson de 8. BH normal ¿cuál es su
diagnóstico?

1. 1. Taquipnea transitoria.
2. 2. Neumonía connatal.
3. 3. Síndrome de aspiración.
4. 4. Enfermedad de membrana hialina.
Gráfico de respuestas
Comentario

Revisar el siguiente cuadro las veces que sean necesarias:

260. Una de las siguientes afirmaciones en infección urinaria en el niño, es FALSA:

La presencia de fiebre, densidad urinaria < 1.010, con presencia de cilindros leucocitarios
1. 1.
orienta a infección urinaria alta.
En pediatría, la tendencia a la formación de cicatrices renales es mayor en niños menores
2. 2.
de 2 años.

 
 
 
 
La nitrofurantoína está indicada en el tratamiento inicial de pacientes con infecciones
3. 3.
urinarias altas.
4. 4. Ninguna de las anteriores.
Gráfico de respuestas
Comentario

Cuando en un cuadro de clínica miccional aparece fiebre, dolor abdominal y/o lumbar, vómitos y
malestar general, orienta a una IVU alta o pielonefritis. En este caso el tratamiento a tener en
cuenta seria:

- Neonatos (sin sepsis y con niveles de creatinina normal): ampicilina y gentamicina i.v. cinco o
siete días pasando a v.o. con monoterapia según antibiograma hasta completar 10-14 días.

- Lactantes mayores de tres meses o NIÑOS: cefixima v.o. durante 10-14 días siempre y cuando el
niño no presente clínica de sepsis, tenga buena tolerancia oral al antibiótico y buena hidratación, y
la creatinina sea normal. Si no es así, será necesario ingresarlo y administrar tratamiento con
gentamicina i.v. durante cinco o siete días, pasando a v.o. con monoterapia según antibiograma
hasta completar 10-14 días. Si los niveles de creatinina son elevados, no se ha de utilizar de
entrada aminoglucósidos, y se recomienda cefotaxima.

Teniendo en cuenta esto la respuesta falsa es la 3.(R3)

261. En los niños menores de 2 años con tuberculosis pulmonar primaria el hallazgo
radiológico más frecuente es:

1. 1. Adenopatias hiliares o mediastinicas.


2. 2. Derrame pleural.
3. 3. Atelectasia lobar o segmentaria.
4. 4. Infiltrado micronodular en lóbulo superior.
Gráfico de respuestas
Comentario

La primoinfección tuberculosa cursa, en general, de forma asintomática o paucisintomática,


produciendo una neumonitis inespecífica que afecta fundamentalmente a lobulos medios o
inferiores, y que se acompaña de adenopatías hiliares. Es la forma de predominio en la
infancia.(R1)

262. ¿Cuál de las siguientes caracteristicas de una lesión coronaria causante de


cardiopatía isquémica NO se incluye dentro de los criterios de indicación de una
angioplastia coronaria transluminal percutánea?:

1. 1. Enfermedad del tronco de la arteria coronaria izquierda.


2. 2. Lesión proximal a la bifurcación arterial.
3. 3. Lesión no calcificada.
4. 4. Afectación de dos vasos.
Gráfico de respuestas
Comentario
En este tipo de preguntas, en las que nos piden indicación de cirugía de revascularización o de
angiplastia, lo que hay que saberse bien son las indicaciones de cirugía, y que el resto, lo que
queda es para ACTP. Las indicaciones de cirugía de cirugía de revasularización coronaria en la
cardiopatía isquémica que debemos conocer para el examen son las siguientes: - Estenosis >50%

 
 
 
 
del tronco de la coronaria izquierda. - Enfermedad de 3 vasos (más indicada si hay disfunción
ventricular) - Enfermedad de dos vasos si uno de ellos es la descendente anterior proximal. Por
tanto, la respuesta 1 es quirúrgica, con lo que debemos marcarla directamente.(R1)

263. Un paciente de 58 años, con antecedentes de hipertensión arterial y dislipemia,


acude por haber presentado un cuadro transitorio de 5 minutos de duración consistente
en debilidad de miembros derechos y dificultad para el habla. El estudio por TC craneal
no demostró alteraciones, mientras que un estudio ecográfico de los troncos
supraaórticos mostró una estenosis del 85% a nivel del origen de la arteria carótida
interna izquierda, que se confirmó en un estudio angiográfico convencional. Además del
control de los factores de riesgo vascular, ¿qué tratamiento adicional recomendaría con
la intención de disminuir el riesgo de ictus establecido?:

1. 1. Antiagregación con ácido acetilsalicílico.


2. 2. Anticoagulación oral.
3. 3. Endarterectomía carotídea, con antiagregación crónica posterior.
4. 4. Ninguno, a excepción del control de los factores de riesgo vascular.
Gráfico de respuestas
Comentario

Pregunta muy importante sobre manejo de la estenosis carotídea. Este aspecto sobre el
tratamiento ha sido muy preguntado.

La endarterectomía carotídea sería el tratamiento de elección en pacientes con estenosis carotídea


sintomática superior al 70% de la luz. Si la estenosis es menor al 50% el paciente debe ser
antiagregado. Cuando la estenosis se encuentra entre el 50- 69% la decisión terapeútica depende
de los factores de riesgo vascular (podría indicarse en varones, con factores de riesgo y síntomas
hemisféricos recientes, aunque no hay claro consenso).

En estenosis carotídeas asintomáticas, se ha recomendado la antiagregación. Sin embargo,


cuando la estenosis es hemodinámicamente significativa y evolutiva en el tiempo, puede ser
beneficiosa la endarterectomía carotídea, siempre que la morbimortalidad operatoria no supere el
5%.(R3)

264. Mujer de 38 semanas de gestación que acude a urgencias refiriendo sensación de


dinámica uterina. A su llegada, se la toma una tensión de 155/98. La paciente no refiere
cefalea, fotopsias ni otra sintomatología. El laboratorio nos informa de proteinuria
significativa. El registro cardiotocográfico fetal es reactivo, sin desaceleraciones. En la
exploración se evidencia una dilatación de 3 centímetros. ¿Qué conducta adoptaría?:

1. 1. Alta domiciliaria.
Estimulación con oxitocina, junto con administración de antihipertensivos, para intentar un
2. 2.
parto vaginal.
3. 3. Ingreso para control de tensiones.
4. 4. Maduración pulmonar, antihipertensivos y estimulación de parto.
Gráfico de respuestas
Comentario
En este caso clínico nos presentan a una gestante a término, con contracciones y dilatación de
parto. Nos añaden un diagnóstico de preclampsia sin criterios de gravedad ni repercusión fetal. Por

 
 
 
 
tanto lo mas lógico es intentar una parto vaginal, dada la exploración que nos proponen,
controlando las tensiones de la paciente con medicación antihipertensiva.(R2)

265. La mutación del gen JAK-2 constituye uno de los criterios mayores de diagnóstico
en una de estas patologías:

1. 1. Policitemia vera.
2. 2. Leucemia mieloide aguda tipo M4 de la FAB.
3. 3. Síndrome mielodisplásico tipo anemia refractaria.
4. 4. Leucemia mieloide crónica.
Gráfico de respuestas
Comentario

La pregunta no debe ofrecer dudas. La mutación de la tirosín-kinasa JAK2 se ha observado en el


95% de los casos de policitemia vera y 50%-60% de los de trombocitosis esencial y mielofibrosis
primaria (es decir, en la mayoría de los casos de síndromes mieloproliferativos crónicos distintos de
la leucemia mieloide crónica). La mutación incrementa la proliferación de la célula clonal e inhibe
su apoptosis.(R1)

266. ¿En cuál de las siguientes situaciones cree que NO estaría indicado el tratamiento
con análogos de la GnRH o LHRH?:

1. 1. Endometriosis.
2. 2. Mioma uterino.
3. 3. Tumores hormonodependientes.
4. 4. Amenorrea hipofisaria.
Gráfico de respuestas
Comentario

El hipotálamo produce GnRH que estimula en la hipófisis la producción de gonadotropinas (LH y


FSH). La liberación se produce de manera pulsátil, de tal forma que los pulsos lentos
sobreestimulan FSH, y los rápidos LH (como ocurre en el SOP).

La liberación continua de GnRH desensibiliza las células por internalización de sus receptores
inhibiendo la FSH y la LH, provocando un estado de hipoestrogenismo, por eso son útiles en el
tratamiento de la endometriosis, los miomas uterinos, la pubertad precoz, la estimulación ovárica y
en algunos casos de tumores hormonodependientes (mama).(R4)

267. En la patogenia de la cetoacidosis diabética:

1. 1. Es determinante el aumento del umbral renal de glucosa.


2. 2. Existe una reducción en la producción de acetoacetato hepático.
3. 3. Aumenta el aporte de ácidos grasos libres al hígado.
4. 4. Existe una reducción en los niveles de las hormonas de contra regulación.
Gráfico de respuestas
Comentario

Una pregunta sencilla sobre un tema bastante preguntado en el examen MIR. No se puede fallar
preguntas como ésta.

 
 
 
 
En la cetoacidosis diabética, existe un disbalance entre las hormonas insulares y contrainsulares:
déficit de insulina y exceso de glucagón. Debido al déficit insulínico, el organismo no es capaz de
introducir las moléculas de glucosa al interior celular, con la consiguiente hiperglucemia. El
glucagón incrementa la gluconeogénesis hepática, agravando la situación e incrementando todavía
más las cifras de glucemia. Dado que las células no pueden hacer uso de la glucosa circulante, se
produce un incremento de la lipólisis, con el consiguiente aumento de los ácidos grasos libres en el
torrente sanguíneo (respuesta 4 correcta).(R3)

268. Un paciente que padeció un infarto de miocardio de localización anterior hace 5 años
acude a Urgencias por palpitaciones, sin otros síntomas añadidos. El ECG revela una
taquicardia regular a 180 lpm, con un QRS de 150 mseg de duración. Presión arterial
140/70 mmHg. Auscultación pulmonar normal. Entre las siguientes medidas terapéuticas,
la MENOS indicada sería:

1. 1. Procainamida i.v.
2. 2. Verapamil i.v.
3. 3. Cardioversión eléctrica.
4. 4. Masaje del seno carotídeo.
Gráfico de respuestas
Comentario

Un a taquicardia regular de QRS ancho (más de 120ms) mientras no se demuestre lo contrario es


una taquicardia ventricular. Además nos dicen que se trata de un paciente con un IAM previo, una
razón añadida para “pensar mal”. Recuerda que la causa más frecuente de TV es la reentrada a
través de la cicatriz de un IAM previo. En la TV esta CONTRAINDICADO el verapamilo iv, ya que
deprime la contractilidad y puede deprimir seriamente el gasto cardíaco.(R2)

269. ¿Cuál de las siguientes estructuras es más precoz en el desarrollo embrionario?:

1. 1. El blastocele.
2. 2. La gástrula.
3. 3. La mórula.
4. 4. La notocorda.
Gráfico de respuestas
Comentario

Pregunta de embriología de primer año de la carrera. La mórula es una fase del desarrollo
embrionario que incluye la fase de 16, 32 y 64 células. Es una sólida, no como la blástula, el
siguiente paso, que es un balón hueco.(R3)

270. ¿Qué seguimiento se realizará en un paciente con colitis ulcerosa para evitar el
desarrollo de un cáncer de colon?:

1. 1. Enema opaco.
2. 2. Colonoscopia e investigación de la displasia.
3. 3. Determinación de antígeno carcinoembrionario.
4. 4. Investigación de sangre oculta en heces.
Gráfico de respuestas
Comentario
En la colitis ulcerosa hay un riesgo aumentado de adenocarcinoma de colon, mayor que en la
enfermedad de Crohn (en la que hay más incidencia de adenocarcinoma de intestino delgado). Se

 
 
 
 
debe a la formación de verdaderos pólipos inflamatorios, que pueden tener degeneración maligna.
El riesgo es mayor en los pacientes con más de 10 años de evolución de la enfermedad y en
aquellos con afectación más extensa del colon (pancolitis), ya que suelen desarrollar más número
de pólipos. Para el seguimiento de los enfermos con colitis ulcerosa, en la prevención del
adenocarcinoma de colon, se debe emplear la colonoscopia, con toma de biopsias, para valorar la
presencia de displasias.(R2)

271. Señale la INCORRECTA en cuanto a la insuficiencia suprarrenal:

Las causas secundarias son más frecuentes que las primarias en la etiología de la crisis
1. 1.
suprarrenal.
Los síntomas gastrointestinales pueden ser la forma de presentación, llegando a veces a
2. 2.
simular un abdomen agudo.
La hiperpigmentación es llamativa en la insuficiencia suprarrenal primaria, por lo que en
3. 3.
casos con clínica típica y aparición simultánea de vitíligo, se descarta el diagnóstico.
Como en los casos de hiperfunción suprarrenal, pueden aparecer cambios de la
4. 4.
personalidad y alteraciones del comportamiento.
Gráfico de respuestas
Comentario

Pregunta relacionada con las manifestaciones clínicas de la insuficiencia suprarrenal. El síntoma


más frecuente es la astenia, inicialmente en épocas de estrés y, según avanza la enfermedad,
incluso en reposo. En ocasiones, el cuadro clínico se asemeja con un abdomen agudo, existiendo
nauseas, vómitos y dolor abdominal. La causa más frecuente de crisis suprarrenal es la suspensión
brusca de un tratamiento corticoideo prolongado. Se produce por la supresión de la ACTH y,
secundariamente, la incapacidad de la suprarrenal para sintetizar cortisol (insuficiencia suprarrenal
secundaria). Aunque la hiperpigmentación sea un signo característico en las formas primarias, el
hecho de que exista vitílgo no descarta el diagnóstico. De hecho, ambas entidades, por su origen
autoinmune, pueden aparecer juntas en los síndromes pluriglandulares autoinmunes tipo 1 y 2.(R3)

272. ¿Cuál de las siguientes características NO corresponde con la diabetes mellitus tipo
1 (insulinodependiente)?

1. 1. Precisa de tratamiento con insulina desde el inicio para sobrevivir.


Se desencadena por un proceso autoinmune con insulinitis que destruye los islotes
2. 2.
pancreáticos productores de insulina.
Patogénicamente se produce una resistencia a la insulina en los receptores del hígado,
3. 3.
músculo y adipocitos.
Sus principales complicaciones a largo plazo son la patología de pequeño vaso
4. 4.
(microangiopatía), de mediano-grande vaso (macroangiopatía) y la neuropatía.
Gráfico de respuestas
Comentario

Esta pregunta no ofrece ninguna dificultad conociendo la distinta patogenia de la diabetes mellitus
tipo 1 y 2.

Hace referencia a la patogenia de la diabetes mellitus tipo 1. Debes conocer que es una
enfermedad típica de personas jóvenes (en general, menores de 30 años) en donde el tratamiento
con insulina se inicia desde el diagnóstico y es fundamental para la vida. Existe un déficit absoluto
de insulina por la destrucción autoinmune de las células beta pancreáticas. Las complicaciones
crónicas derivan de la afectación microangiopática (nefropatía, retinopatía, neuropatía) y

 
 
 
 
macroangiopática (cardiopatía isquémica...). La resistencia insulínica periférica es característica de
la diabetes mellitus tipo 2.(R3)

273. Entre los niños no inmunizados y los malnutridos, ¿cuál de las siguientes
infecciones puede tener mayor morbilidad y mortalidad?

1. 1. Sarampión.
2. 2. Parotiditis.
3. 3. Roséola.
4. 4. Varicela.
Gráfico de respuestas
Comentario

Dentro de las 4 enfermedades, en pacientes no inmunizados o con malnutrición, que podría afectar
con mayor relevacia a los niños es sarampión, respuesta 1 correcta.(R1)

274. Paciente femenino de 32 años de edad, con antecedentes personales de miomas


uterinos, en tratamiento con anticonceptivos orales hasta hace dos años, que acude a
consulta por retraso menstrual de 7 días, al que se asocian fatiga y somnolencia diurna
y sensación de tensión mamaria. Se realiza determinación de beta-HCG, que confirma la
existencia de embarazo. Una de las siguientes serologías no forma parte del cronograma
de seguimiento al embarazo:

1. 1. Sífilis.
2. 2. VIH.
3. 3. Rubéola.
4. 4. Hepatitis A.
Gráfico de respuestas
Comentario

Pregunta bastante sencilla y básical para el ENARM. Son las hepatitis B y C las que pueden
afectar a la madre cursando de manera asintomática en alguna de sus fases, motivo por el que
merece la pena cribarlas haciendo serologías para tratar de evitar la transmisión vertical. La
hepatitis A cursa de manera aguda, su contagio es fecal-oral y no supone ningún riesgo para el
feto.(R4)

275. Mujer de 37 años sexualmente activa visita a su médico por presentar historia de
dolor desde hace dos días en el área de sus genitales. El examen pélvico muestra úlcera
pequeña, superficial, muy sensible, con bases rojas en las regiones vulvar y vaginal. Un
extendido de Tzanc muestra la presencia de células gigantes multinucleadas. Cuál de los
siguientes agentes infecciosos es el causante más probable de estos hallazgos:

1. 1. Cándida albicans.
2. 2. Chlamydia trachomatis.
3. 3. Virus herpes simple.
4. 4. Treponema pallidum.
Gráfico de respuestas
Comentario

 
 
 
 
Varias enfermedades venéreas, que es lo que parece que tiene esta mujer, pueden producir
úlceras genitales:

-La sífilis primaria (Treponema pallidum) típicamente produce una úlcera dura, limpia e indolora.

-El chancroide (Haemophilus ducreyi) produce una o varias úlceras blandas, de fondo sucio y
dolorosas.

-El herpes genital (virus herpes simplex, generalmente tipo 2) produce úlceras superficiales
dolorosas. El test de Tzanck, que consiste en coger un frotis del fondo de una úlcera y examinarlo
al microscopio, previa tinción, muestra típicamente células multinucleadas, pues el virus herpes
induce la fusión de las células infectadas.

Luego queda claro que la pregunta hace referencia a un herpes genital.(R3)

276. En relación al climaterio. Marque lo FALSO:

1. 1. Son frecuentes los ciclos anovulatorios.


2. 2. El estradiol es el estrógeno principal.
3. 3. La estrona es el estrógeno de mayor producción.
4. 4. Hay aumento de andrógenos.
Gráfico de respuestas
Comentario

Pregunta muy sencilla que se puede responder sin conocimientos previos, ya que en la respuesta
2 mencionan que el estradiol es el estrógeno principal y en la respuesta 3 mencionan que es la
estrona, por lo que la respuesta falsa se encontrará en una de estas. Respuesta falsa 2, durante el
climaterio disminuye la producción de estradiol.(R2)

277. Señale la afirmación CORRECTA respecto al test de Apgar:

1. 1. Se debe practicar únicamente a los RN patológicos o supuestamente patológicos.


2. 2. Entre los parámetros que se valoran se encuentra la frecuencia respiratoria.
3. 3. Se debe realizar al minuto y a los 5 minutos de vida.
4. 4. La palidez cutánea generalizada supone una puntuación de 1 en el test de Apgar.
Gráfico de respuestas
Comentario

El test de Apgar se debe realziar al minuto y 5 minutos de vida. Respuesta 3 correcta. Repase el
test de Apgar.(R3)

 
 
 
 

278. Un enfermo con antecedentes de fractura de fémur reciente, acude por presentar
ceguera súbita de un ojo. El fondo de ojo muestra una retina blanquecina, en cuyo centro
destaca una mancha oval rojo. ¿Qué diagnóstico haría?:

1. 1. Neuritis óptica.
2. 2. Trombosis de la vena central de la retina.
3. 3. Oclusión de la arteria central de la retina.
4. 4. Hemorragia macular.
Gráfico de respuestas
Comentario
Esta pregunta debes poder acertarla con facilidad. La obstrucción de la arteria central de la retina
cursa como ceguera de instauración BRUSCA y el fondo de ojo descrito en la pregunta también es
típico de esta patología: retina blanquecina por ausencia de irrigación que contrasta con la mancha
rojo cereza que corresponde a la mácula irrigada por la coriocapilar.(R3)

279. Paciente primigesta Rh negativa no sensibilizada con esposo Rh positivo, recién


nacido Rh positivo. La conducta a seguir es:

1. 1. Vacunarla a la semana del parto.


2. 2. No requiere vacunación hasta el próximo parto.
3. 3. Vacunarla dentro de las 72 horas posparto.
4. 4. Vacunarlas en cualquier momento.
Gráfico de respuestas
Comentario

La respuesta correcta es la 3. Revise el siguiente cuadro.(R3)

 
 
 
 

280. Niño de 5 años diagnosticado de infección por VIH de transmisión vertical. Presenta
un cuadro de tos de unas semanas de evolución. La auscultación pulmonar y cardíaca es
normal, al igual que el resto de la exploración excepto linfadenopatías generalizadas. TA
90/50 mmHg, FC 98 lpm, FR 40 rpm, Tª 36.3 ºC, Saturación periférica de oxígeno: 90%. Es
FALSO que:

Las infecciones oportunistas en el niño son más frecuentes que en al adulto, siendo de las
1. 1. no oportunistas las más incidentes las infecciones bacterianas, especialmente la neumonía
por Streptococcus pneumoniae.
La neumonía por Pneumocystis jirovecii en niños tiene una incidencia máxima entre los 3
2. 2. y los 6 meses de edad, y una gran mortalidad sobre todo por encima del año de edad,
aunque ha disminuido con las nuevas pautas de tratamiento intensivo.
El cuadro que más probablemente presenta este paciente es causado por la propia
3. 3.
infección del VIH.
El tratamiento de la complicación que más probablemente presenta el paciente será con
4. 4.
corticoides orales.
Gráfico de respuestas
Comentario

De las complicaciones respiratorias del VIH en la infancia hay que saber el diagnóstico diferencial
entre la neumonía por Pneumocystis jirovecii, la neumonía intersticial linfoide y otros cuadros
comunes.

La neumonía por Pneumocystis jirovecii es un cuadro agudo de fiebre, taquipnea, disnea e


hipoxemia severa, con infiltrados intersticiales bilaterales rápidamente progresiva. El diagnóstico se
confirme detectando el germen en una muestra obtenida mediante lavado broncoalveolar.
Tratamiento con Trimetoprim-Sulfametoxazol.

La neumonía intersticial linfoide es un cuadro crónico que ocurre en un 25% de los pacientes.
Cursa con taquipnea y tos insidiosas e hipoxemia leve-moderada con acropaquias en los casos
avanzados. En la radiografía se aprecia un patrón reticulonodular difuso. Se asocia a la infección

 
 
 
 
por el propio VIH y la hiperplasia linfoide que ocasiona en el epitelio bronquiolar. Se trata con
corticoides orales y broncodilatadores.

Son muy frecuentes las infecciones por gérmenes comunes sobre todo el neumococo, pero
también virus respiratorios, Haemophylus, Staphylococcus aureus, Pseudomonas y Legionella. La
tuberculosis con frecuencia tiene extensión extrapulmonar.

En la infección por VIH infantil son más frecuentes las infecciones de repetición por bacterias
comunes que las infecciones oportunistas típicas aunque éstas tienden a ser más graves que en el
adulto.(R1)

281. Una mujer de 38 años consulta por una secreción vaginal anormal con mal olor,
como a “pescado podrido”, que se hace más intenso después del coito. No tiene prurito
genital. En la exploración se observa una abundante secreción blanco-grisácea, que no
se adhiere a las paredes vaginales. Al mezclar una muestra de la secreción con una gota
de hidróxido potásico al 10% se aprecia claramente el mal olor referido. ¿Cuál de los
siguientes es el tratamiento de elección?

1. 1. Amoxicilina con ácido clavulánico por vía oral.


2. 2. Clotrimazol por vía intravaginal.
3. 3. Doxiciclina por vía oral.
4. 4. Clindamicina por vía intravaginal.
Gráfico de respuestas
Comentario

Pregunta fácil acerca de un tema relativamente sencillo y que se encuentra bien resumido en la
siguiente tabla sobre las vulvovaginitis.

En este caso se trata de una infección por Gardnerella, que es la causa más común de
vulvovaginitis (cuidado porque la causa más frecuente de vulvovaginitis SINTOMÁTICA y, por
tanto, de consulta, es la infección por Cándida). En este caso lo identificamos por la ausencia de
prurito, siendo el principal síntoma la presencia de mal olor que se intensifica al añadir KOH.

El tratamiento se basa en el uso de clindamicina o metronidazol intravaginal (pese a que también


se puede dar por vía sistémica) (respuesta 4 correcta).(R4)

Diferencias entre las vulvovaginitis

Clínica: Candida; Trichomonas

Flujo blanquecino; Secreción abundante.

Secreción vaginal espeso, en grumos con burbujas

Vulva-vagina Eritema Eritema, cuello con colpitis fresa

 
 
 
 

pH vaginal <4,5 >4,5

Aminas volátiles con KOH (10%) Negativo Ocasional

Microscopia con suero salino Esporas Polimorfonucleares tricomonas

Tratamiento: Azoles; Metronidazol oral (también local)

282. En un RN con sospecha de ictericia hemolítica se hacen las siguientes pruebas,


EXCEPTO:

1. 1. Frotis en sangre.
2. 2. Bilirrubina directa e indirecta.
3. 3. Pruebas de capacidad de fijación de la bilirrubina.
4. 4. Biometría hemática, hematocrito, reticulocitos.
Gráfico de respuestas
Comentario

En todo RN con ictericia hemolítica debe de realizar:

Frotis de sangre periférica ya que podría orientarte en las causas de la hemólisis, como por
ejemplo podría ver esquistocitos.

Bilirrubian total, directa e indirecta: esto con el fin de determinar si la icterica se debe a una
elevación de la BD o BI, ya que las el diagnóstico podría variar.

BH es muy importante habrá que conocer la severidad de la hemólisis y un recuento elevado de


reticulocitos nos orientarían a un proceso hemolítico.

Por lo tanto la respuesta que debe elegir es la número 3.(R3)

283. Una niña con cuello corto y movilidad cervical restringida por la fusión de varias
vértebras cervicales, probablemente padecerá:

1. 1. Artrosis cervical.
2. 2. Osteomielitis cervical.
3. 3. Síndrome de Klippel-Feil.
4. 4. Hombro de Sprengel.
Gráfico de respuestas
Comentario

El síndrome de Klippel- Feil es una rara afectación de la columna cervical. Afecta por igual a ambos
sexos y se presenta mayoritariamente de forma esporádica, aunque en algunos casos es
hereditario.

El aspecto clínico es el un “hombre sin cuello”. La cabeza parece incluida entre los hombros.
Tienen brevedad y rigidez cervical, implantación baja del cabello, anomalías faciales y tortícolis

 
 
 
 
irreductible. Se asocia a elevación de la escápula y aparición de un hueso omovertebral y pueden
aparecer trastornos neurológicos (sordera, alteraciones sensitivo- motoras de miembros superiores
o afectación de pares craneales).

En la radiografía se aprecia a nivel cervical uno o varios bloques vertebrales, que incluyen dos o
más vértebras y hemivértebras laterales, por lo que se produce una actitud en tortícolis. Estas
alteraciones pueden tener dos consecuencias funcionales: oblicuidad del cuello e inestabilidad
cervical. El número de agujeros de conjunción es correcto y suelen tener defectos de cierre dorsal
del canal raquídeo. Cuando hay menos de tres cuerpos vertebrales fusionados, los síntomas son
más leves que si existe una unión cérvico- occipital o de C2- C3. Generalmente, no precisa
tratamiento, excepto en casos sintomáticos o con inestabilidad, precisando una fusión a ese
nivel.(R3)

284. ¿Cómo clasificaría a una gestación que antes del embarazo tenía tensiones arteriales
normales; que en la primera consulta, realizada en la semana 8 de edad gestacional, se
le detecta una tensión arterial de 140/90 mmHg; y que en la semana 28 tiene una tensión
arterial de 170/110 mmHg, sin edemas, y con una proteinuria en orina de 24 horas de 300
mg?

1. 1. Preeclampsia grave.
2. 2. Hipertensión inducida por el embarazo.
3. 3. Hipertensión crónica.
4. 4. Hipertensión crónica con preeclampsia sobreañadida.
Gráfico de respuestas
Comentario

Está claro que la paciente de la pregunta padece una preeclampsia grave, ya que cumple los
criterios necesarios. No obstante, no puede decirse que padezca una hipertensión arterial (HTA)
crónica. Los criterios actuales de HTA se basan al menos en dos mediciones. En esta paciente
existen graves inconvenientes para emitir este diagnóstico:

•   El propio enunciado nos dice que, antes del embarazo, las tensiones arteriales eran
normales.
•   Sólo disponemos de una medición con una presión arterial elevada, justo en las cifras
límite: 140/90 mmHg en la octava semana.
•   La medición de la semana 28 no puede considerarse válida para hablar de hipertensión
arterial crónica, ya que estarían justificadas por la preeclampsia.

Por lo tanto, en esta paciente, en rigor, sólo podríamos hacer el diagnóstico de hipertensión crónica
con preeclampsia sobreañadida (respuesta 4 correcta).(R4)

285. En un recién nacido pretérmino de 32-36 semanas de edad gestacional, ¿cuál es la


incidencia de enfermedad de membrana hialina?:

1. 1. 15%.
2. 2. 30%.
3. 3. 50%.
4. 4. 100%.
Gráfico de respuestas
Comentario

 
 
 
 
La enefermedad de membrana hialina es la modaidad de distrés respiratorio de causa pulmonar
propia de los prematuros. Es tanto más frecuente cuanto más prematuro sea el pequeño. Así, en
menores de 28 semanas, la inicidecia aproximada es de 80%. No hace falta que lo memorice para
el ENARM, pero la incidencia de hialina en RN de 32-36 semanas en de un 15-20%.(R1)

286. Indique cuál de las siguientes respuestas, en torno a las complicaciones cardíacas
de la HTA, es FALSA:

El corazón, sometido a una sobrecarga de trabajo por el aumento de la tensión arterial, se


1. 1.
hipertrofia.
La angina de pecho es frecuente, debido a la arteriosclerosis acelerada que provoca la
2. 2.
HTA y a la propia hipertrofia del ventrículo izquierdo.
La mayoría de las muertes debidas a HTA son por IAM o insuficiencia cardíaca
3. 3.
congestiva.
Al principio existe disfunción sistólica, que suele ser asintomática y requiere estudios
4. 4.
ecocardiográficos para su diagnóstico.
Gráfico de respuestas
Comentario

La presión arterial elevada supone un aumento muy importante de la tensión de la pared del
ventrículo izquierdo, que se adapta a la misma hipertrofiándose, como mecanismo de
compensación (recuerda la ley de Laplace). En esta hipertrofia, no sólo interviene la propia presión,
sino que también son fundamentales los sistemas neuroendocrinos (eje renina angiotensina…) y
especialmente la angiotensina II, por sus propiedades mitogénicas.

Los pacientes hipertensos frecuentemente acaban padeciendo cardiopatía isquémica, bien por
afectación aterosclerótica precoz por la propia HTA (la HTA es uno de los principales factores de
riesgo de aterosclerosis) y, en ocasiones, por la intensa hipertrofia del VI (angina con coronarias
normales). La causa de esta angina es que hay un exceso de músculo, desproporcionado al flujo
coronario. Las causas más frecuentes de mortalidad en estos pacientes son cardiovasculares (IAM
e ICC), pero no te olvides de los accidentes cerebrovasculares.

Lo que es imprescindible que no se te olvide es que la HTA es una causa frecuente de ICC
DIASTÓLICA, porque un ventrículo hipertrófico se relaja mal. Por supuesto, para el diagnóstico,
recurriremos a la ecocardiografía(R4)

287. La causa más frecuente de hemoptisis en la actualidad es:

1. 1. Bronquitis crónica.
2. 2. Ca. Oat cell.
3. 3. Ca. Bronquioalveolar.
4. 4. Adenocarcinoma.
Gráfico de respuestas
Comentario
Pregunta simple que va directamente al grano y que debes recordar pues es posible que se repita
en el MIR. La hemoptisis es la presencia de sangre en el esputo y suele ser casi siempre un signo
de gran preocupación. Pese a que nos puede anunciar patologías graves como un cáncer, que
siempre hay que descartar, debemos pensar que hay muchas enfermedades que son más
frecuentes que el cáncer y que por ello constituyen la causa MAS FRECUENTE (que como hemos
dicho no es la más grave) de hemoptisis, y entre ellas destaca la bronquitis crónica. Recuerda en el
MIR lo más frecuente no es siempre lo más típico ni lo más grave.(R1)

 
 
 
 
288. Una paciente de 75 años es intervenida tras una fractura de cadera con implantación
de prótesis completa en articulación coxofemoral izquierda. A los 2 meses se ve aquejada
de dolor en región inguinal, glútea y cara lateral del muslo. Ocasionalmente, picos febriles
aislados. El dolor se exacerba con la deambulación y progresivamente presenta
impotencia funcional parcial. Señale la opción correcta:

La infección de una prótesis rara vez cursa de forma indolente con síntomas leves y es
1. 1.
difícil que se retrase en el diagnóstico.
2. 2. En el 80% de los casos se aísla el germen (gram +) en el líquido articular.
La cifra de glucosa es baja y el lactato alto en el líquido sinovial, siendo este dato muy
3. 3.
específico.
Es de considerar la administración intraarticular de antibióticos en altas dosis para evitar
4. 4.
posteriores secuelas.
Gráfico de respuestas
Comentario

Estamos ante un caso de infección protésica. Esta se produce en un 1- 2% de los casos y se


caracteriza por una clínica indolente, con un retraso en el diagnostico de 2 a 8 meses. La mayoría
de las infecciones protésicas están producidas por gérmenes Gram positivos (S. epidermidis, S.
aureus). En caso de realizar una punción de la articulación el líquido será característico de
infección (glucosa baja, > 50.000 células, aumento de proteínas) pero NO es específico de
infección protésica. El tratamiento inicial empírico de la infección aguda puede hacerse mediante la
asociación de vancomicina/linezolid/daptomicina con una cefalosporina de 4º generación
intravenosos.(R2)

289. En el lactante, el movimiento de pinzamiento índice-pulgar se inicia desde los:

1. 1. Cuatro meses.
2. 2. Ocho meses.
3. 3. Doce meses.
4. 4. Dieciséis meses.
Gráfico de respuestas
Comentario

Pregunta memorística. Los movimientos de pinza aparecen a los 8 meses, antes de que se ponga
en pie solo con ayuda de objetos. Respuesta 2 correcta.(R2)

290. Señale cuál de las siguientes afirmaciones es correcta en cuanto a la enfermedad


quística medular:

Cuando se asocia a degeneración de la retina, la herencia es autosómica recesiva, y la


1. 1.
insuficiencia se produce antes de los 20 años de edad.
Hay una dilatación hasta proporciones quísticas de los conductos colectores,
2. 2.
selectivamente en los extremos que llegan a las papilas.
Es raro que se desarrolle insuficiencia renal, y cuando aparece, la mejor opción terapéutica
3. 3.
es el trasplante.
La hipertensión arterial es un acontecimiento terminal, que se debe a la dificultad para la
4. 4.
eliminación hidrosalina.
Gráfico de respuestas
Comentario

 
 
 
 
La enfermedad quística medular apenas se ha preguntado en el MIR. También es conocida como
“nefronoptisis”. Sobre las distintas respuestas:

1.- La nefronoptisis autosómica recesiva es claramente más grave que la dominante. Por este
motivo, la insuficiencia renal aparecería durante los primeros años de la vida, o en cualquier caso
antes de los 20 años, no en la vida adulta.

2.- La dilatación de los conductos se produce a nivel medular y córtico- medular, no sólo en la
región de las papilas, por lo que la respuesta 2 es falsa.

3.- Al tratarse de un trastorno tubular, es infrecuente la presencia de hematuria microscópica. Por


otra parte, tampoco es habitual encontrar cilindros granulosos en el sedimento.

4.- La insuficiencia renal se produce con frecuencia.

5.- El motivo de la hipertensión arterial es la desestructuración del parénquima renal, con la


consiguiente distorsión de la vascularización, impidiéndose el filtrado y estimulándose
secundariamente el sistema renina- angiotensina- aldosterona.

(R1)

290. ¿Cuál de los siguientes NO está incluido dentro de los criterios diagnósticos de
trastorno esquizofrénico?:

1. 1. Síntomas psicóticos característicos durante al menos una semana.


2. 2. No hay alteraciones afectivas.
3. 3. No existe causa orgánica que motive los síntomas.
4. 4. Signos continuos de enfermedad durante al menos 6 meses.
Gráfico de respuestas
Comentario
La definición DSM de esquizofrenia tiene 4 elementos: 1) Síntomas característicos (psicóticos,
catatónicos, desorganizados, negativos). 2) Duración (más de 6 meses desde el inicio hasta la
resolución total). 3) Repercusión (social, laboral, académica, familiar, etc.). 4) Exclusión de otras
causas de psicosis (médicas, farmacológicas, tóxicas, psiquiátricas). Si la enfermedad dura menos
de 6 meses se habla de trastorno esquizofreniforme; el pronóstico es mejor, sobre todo si el inicio
fue brusco, existieron síntomas afectivos o confusionales o se evidenciaron factores
desencadenantes. Si dura menos de 1 mes se habla de psicosis breves, que tienen un pronóstico
mucho mejor y pueden estar relacionadas con factores estresantes (psicosis reactivas breves).(R2)

291. ¿Qué paciente está sometida a un mayor riesgo de cáncer ovárico de estirpe
epitelial?

1. 1. Síndrome de ovario poliquístico.


2. 2. Disgenesia gonadal.
3. 3. Síndrome de Morris.
4. 4. Tratamiento con clomifeno o gonadotropinas.
Gráfico de respuestas
Comentario
 
 
 
 
El 90% de las tumoraciones malignas de ovario son de estirpe epitelial. Los factores de riesgo
tienen como punto en común el traumatismo constante en el recubrimiento epitelial del ovario, con
la posibilidad de que la reparación no sea adecuada y aparezcan células atípicas. Ejemplo claro de
ello: un ovario sometido a sucesivas inducciones de la ovulación, con numerosos folículos
reclutados desarrollándose y rompiéndose en cada ciclo (respuesta 4). Serán factores protectores,
por tanto, el consumo de anticonceptivos hormonales (anovulatorios) y el SOP (dónde con gran
frecuencia se producen ciclos anovulatorios). Aunque el síndrome de Morris predispone a un
cáncer de ovario, no sería de estirpe epitelial, sino germinal (el disgerminoma) y algunas
disgenesias gonadales también (el síndrome de Swyer se asocia con la aparición de
gonadoblastomas).(R4)

292. Tras valorar la imagen adjunta, cuál


de las siguientes afirmaciones es cierta desde el punto de vista clínico y radiológico de
un paciente que acude al Servicio de Urgencias de un hospital:

La imagen corresponde con un infarto de ACM derecha, con transformación hemorrágica.


1. 1.
Clínicamente hemiparesia izquierda.
La imagen corresponde con un infarto completo de arteria carótida derecha con clínica de
2. 2.
hemiparesia de predominio crural.
La imagen corresponde con un infarto de ACM derecha. La clínica compatible sería una
3. 3.
hemiparesia de predominio facio-braquial izquierda.
La imagen corresponde con un infarto de ACM izquierda y el paciente presenta
4. 4.
característicamente un cuadro de afasia global.
Gráfico de respuestas
Comentario

Una de las ventajas de las pruebas de neuroimagen es que el cerebro es (o debería ser)
perfectamente simétrico, por lo que no resulta difícil identificar las áreas afectas si comparamos
con las contralaterales. En este caso, podemos ver un cambio de densidad en las regiones
derechas del parénquima cerebral, extendiéndose por el territorio de la cerebral media.
Clínicamente, la

 
 
 
 
hemiparesia debería ser de predominio facio- braquial contralateral (izquierda). Recuerda que, para
que el predominio sea crural, el daño debería producirse en el territorio de la cerebral anterior.(R3)

293. Valorando la imagen, cual sería


probablemente el diagnóstico más probable.

1. 1. Ictus isquémico por oclusión arterial de arteria carótida derecha.


2. 2. Se puede diagnosticar de una meningoencefalitis, muy probablemente herpética.
Ictus isquémico en territorio ACM derecha con efecto masa sobre ventrículo lateral
3. 3.
ipsilateral.
4. 4. Ictus isquémico con transformación hemorrágica en territorio ACM derecha.
Gráfico de respuestas
Comentario

Aparte de algunos hematíes normales, lo que observamos en este frotis son otros de volumen
mucho más reducido, con una coloración más intensa (ya que albergan una cantidad semejante de
hemoglobina, pero en un volumen menor). Se trata, por tanto, de esferocitos. Como sabes, los
esferocitos pueden verse en la esferocitosis hereditaria, pero también en las anemias
inmunohemolíticas. Por ello, carece de sentido solicitar un test de Ham (test de la hemólisis ácida).
Esta prueba es útil para el diagnóstico de la hemoglobinuria paroxística nocturna, donde también
existe una hemólisis, pero que no es mediada por anticuerpos.(R3)

294. ¿Cuál de los siguientes síntomas menopáusicos no está relacionado con la terapia
de sustitución con estrógenos?

1. 1. Relajación vaginal.
2. 2. Depresión.
3. 3. Vaginitis atrófica.
4. 4. Dispareunia.

 
 
 
 
Gráfico de respuestas
Comentario

Pregunta extraña y complicada.

Las respuestas 2, 3 y 4 son síntomas de la menopausia que pueden mejorar con la sustitución con
estrógenos.

La respuesta 1, relajación vaginal, que se refiere a la relajación del suelo pélvico que sucede en
muchas mujeres postmenopáusicas, no mejora con esta terapia.(R1)

295. A 67-year-old man has a 10-year-history of shortness of breath with exertion and dry
cough. He was diagnosed with chronic bronchitis. He presents today with hemoptisis and
malaise. On further questioning, it is known that he used to work in a quarry. Chest X-ray
reveals multiple and small pulmonary nodules in upper lobes with a larger cavitating
nodule. Which of the following is the most likely diagnosis?

1. 1. Tuberculosis in a patient with silicosis.


2. 2. Epidermopid lung carcinoma.
3. 3. Sarcoidosis.
4. 4. Amyloidosis.

(R1)

296. Una gestante de 34 semanas de edad gestacional ingresa por Urgencias con cefalea,
escotomas visuales centelleantes y dolor epigástrico en barra. Su tensión arterial es de
170/110 mm Hg, tiene edemas y proteinuria 3+. En los exámenes de laboratorio el
hematocrito, las plaquetas y las enzimas hepáticas son normales. ¿Qué actitud
recomendaría?

Tratamiento con sulfato de magnesio y después inducción del parto o cesárea si no reúne
1. 1.
condiciones favorables para la inducción.
2. 2. Inducción del parto.
3. 3. Cesárea inmediata.
Tratamiento con hidralazina, y con glucocorticoides para lograr la maduración pulmonar
4. 4.
fetal y pasadas 24 horas inducir el parto.
Gráfico de respuestas
Comentario

Pregunta de dificultad moderada que hace referencia a un tema muy preguntado y muy clásico en
el ENARM, y que, por tanto, es obligatorio estudiarlo con atención: la preeclampsia.

En este caso clínico se nos hace mención a una serie de datos (cefalea, escotomas, dolor
epigástrico…) que sugieren signos inminentes que esta a punto de convulsionar y preeclampsia
que padece la paciente es grave. Se debe manejar con urgencia esta situación, que
obligatoriamente requiere terminar el embarazo. Hay que advertir la diferencia entre la
preeclampsia grave y el síndrome HELLP, que queda descartado por los datos de laboratorio
mencionados en el enunciado, le dicen que las enzimas hepáticas y plaquetas son normales.

 
 
 
 
La vía preferente para terminar el embarazo en la preeclampia es la vía vaginal, por lo que
descartamos la respuesta 3. La preeclampsia grave no es indicación de cesárea per se, si la
paciente responde al tratamiento anti hipertensivo y profilaxis con sulfato de magnesio. El
tratamiento antihipertensivo solo no reestablecerá la situación basal, además si hemos
mencionado anteriormente que la preeclampsia grave requiere terminar el embarazo, no tiene
sentido la respuesta 4. Respecto a la respuesta 4, siendo la gestación de 34 semanas, ya se
considera que el pulmón del feto ha alcanzado la madurez, por lo que no tiene sentido administrar
ahora corticoides con ese fin.

La respuesta 1 es la correcta. El sulfato de magnesio además de tratar las convulsiones (entonces


estamos ante una eclampsia) las previene, por lo que tiene sentido administrarlo en esta situación.
La inducción del parto será mejor opción que la cesárea, siempre y cuando sea posible
realizarla(R1)

297. El 90% de los aneurismas del ventrículo izquierdo, como complicación de un infarto
de miocardio transmural, son secundarios a una oclusión aguda de la arteria coronaria:

1. 1. Tronco común de la arteria coronaria izquierda.


2. 2. Arteria descendente anterior.
3. 3. Arteria descendente posterior.
4. 4. Arteria coronaria derecha.
Gráfico de respuestas
Comentario
Pregunta difícil. Para responderla, hay que recordar que la mayoría de aneurismas post-infarto se
dan en la cara anterior del ventrículo izquierdo y relacionar esto con la arteria correspondiente, es
decir, la descendente anterior (respuesta correcta: 2)(R2)

298. Para diferenciar una insuficiencia renal prerrenal de una insuficiencia renal
intrínseca, tenemos los siguientes indicadores SALVO:

1. 1. Fracción de excreción de sodio menor de 1.


2. 2. Una relación entre el nitrógeno ureico urinario y el plasmático mayor de 8.
3. 3. Una osmolaridad urinaria menor de 300.
4. 4. Una relación entre nitrógeno ureico plasmático y la creatinina mayor de 20.
Gráfico de respuestas
Comentario

En cuanto a la insuficiencia renal aguda es muy importante dominar bien el diagnostico diferencial
de la prerrenal, parenquimatosa y postrenal. No olvides que la osmolaridad urinaria en el fracaso
prerrenal debe ser mayor de 500, puesto que la que nos aparece menor de 300 nos orientaría
hacia una NTA. Has de conocer bien las diferencias entre ambas entidades, que aparecen
reflejadas en la tabla siguiente.(R3)

Datos analíticos

 
 
 
 

299. Señale la FALSA respecto a la mediastinitis crónica:

1. 1. La mayor parte de los casos se deben a tuberculosis o histoplasmosis.


2. 2. Los pacientes con mediastinitis granulomatosa suelen estar asintomáticos.
Está indicado el tratamiento antituberculoso una vez diagnosticada la fibrosis
3. 3.
mediastínica.
Los síntomas más frecuentes son los secundarios a compresión de estructuras
4. 4.
mediastínicas.
Gráfico de respuestas
Comentario
Pregunta difícil de un tema muy poco importante, que no han preguntado nunca en el MIR. La
opción 4 es claramente falsa porque la TBC no produce fibrosis mediastínica, si no una
mediastinitis granulomatosa. Recuerda que las causas más frecuentes de mediastinitis crónica son
la TBC y la histoplasmosis, y que ambas producen una mediastinitis granulomatosa, cursando
éstas de forma asintomática. Para el diagnóstico se suele realizar la exploración quirúrgica para
diferenciar procesos malignos, y en cuanto al tratamiento de las mediastinitis crónicas ningún
tratamiento, médico o quirúrgico, se ha mostrado eficaz.(R3)

 
 
 
 

300. Señale en qué grupo estaría clasificado un niño diagnosticado de SIDA de 3 años de
edad, con 780 linfocitos CD4 totales y que presenta una neumonía intersticial linfoide:

1. 1. C3.
2. 2. B2.
3. 3. A1.
4. 4. B3.
Gráfico de respuestas
Comentario

Pregunta de dificultad elevada debido a que la clasificación en estadíos del SIDA pediátrico no se
estudia aunque viene en el manual y por lo tanto, al ser una pregunta muy directa no podemos
deducirla.

La respuesta correcta es la 2, puesto que en un niño entre 1 y 5 años con 780 CD4 nos situamos
en un nivel de inmunosupresión moderada y la NIL se encuentra encuadrada dentro de la
categoría clínica tipo B (moderadamente sintomáticos). Al cruzar ambos parámetros, categoría
inmunológica y clínica, obtenemos el estadío B2, que es la respuesta a la pregunta.(R2)

301. El factor desencadenante del mecanismo etiopatogénico de la HTA del embarazo


sería:

1. 1. Alteración de los niveles renina angiotensina.


2. 2. Lesión vascular placentaria.
3. 3. Vasoconstricción.
4. 4. Incremento de la sensibilidad vascular.
Gráfico de respuestas
Comentario

En la preeclampsia, se produce una alteración en la tolerancia inmunológica a la placenta,


apareciendo entonces isquemia placentaria. Esta isquemia produce una liberación de sustancias
vasoconstrictoras como el tromboxano, que a su vez produce una lesión endotelial diseminada
(respuesta 2 correcta). El endotelio glomerular es particularmente sensible a este fenómeno,
apareciendo una lesión muy característica, que es la endoteliosis glomerular.(R2)

302.
Varón de 53 años fumador e hipertenso, que acude a su consulta por presentar desde

 
 
 
 
hace 7 meses episodios de molestias precordiales que define como un peso, irradiado a
hombro izquierdo coincidiendo con la subida de cuestas. Explica que se le calman
después de 2-3 minutos de reposo. El último episodio de dolor fue hace 2 días. Usted
realiza un ECG que muestra la imagen nº 4. Esta alteración ya estaba presente en ECG
previos. De los siguientes enunciados señale la respuesta correcta:

1. 1. El paciente presenta un síndrome coronario agudo sin elevación del segmento ST.
En estos momentos estaría indicado iniciar tratamiento con antiagregantes plaquetarios y
2. 2.
heparina de bajo peso molecular.
El tratamiento con nitratos transdérmicos puede mejorar la calidad de vida de los
3. 3.
pacientes.
El fármaco de primera elección para el tratamiento de la HTA en este paciente son los
4. 4.
IECA.
Gráfico de respuestas
Comentario

El ECG nos muestra un ritmo sinusal a 70 lpm con un bloqueo completo de rama izquierda del haz
de His. El caso clínico nos muestra un varón que presenta la clínica típica de una angina de pecho
estable (respuesta: 1 incorrecta). El tratamiento farmacológico de la angina estable se basa en la
combinación de antiagregantes plaquetarios, estatinas, betabloqueantes e IECA (que aumentan la
supervivencia), así como en el uso de nitratos u otros antianginosos para el control de los síntomas
y, por tanto, para mejorar la calidad de vida del paciente (respuesta: 2 incorrecta, respuesta: 4
correcta). Para el diagnóstico de isquemia, en este caso no sería útil una ergometría de esfuerzo
porque los cambios en el ECG no son valorables al existir un bloqueo de rama izquierda
(respuesta: 3 incorrecta). Sería más útil el uso de un ecocardiograma de estrés. La HTA en un
paciente con cardiopatía isquémica se trata de primera elección tanto con BB como con los IECA
(respuesta: 5 incorrecta).(R3)

303. En una paciente de 46 años, la presencia de una masa anexial sólida e hidrotórax
debe hacer pensar en:

1. 1. Cistoadenoma mucinoso.
2. 2. Ginandroblastoma.
3. 3. Cistadenoma seroso.
4. 4. Fibroma de ovario.
Gráfico de respuestas
Comentario

Esta pregunta hace referencia al síndrome de Meigs que consiste en la asociación de tumos sólido
de ovario, derrame pleural y ascitis. La causa mas frecuente son los tumores de cordones
sexuales- estroma, sobre todo el fibroma. El tumor de Brenner también lo produce con cierta
frecuencia. Recuerde que el síndrome de hiperestimulación hay una extravasación de liquido
asociado a ovarios llenos de folículos. El cistoadenoma mucinoso se asocia a pseudomixoma
peritoneal. El giandroblastoma es un tumor secretor de estrógenos y andrógenos.(R4)

304. ¿Cuál de estas glomerulonefritis NO es hipoclompementémica?

1. 1. Glomerulonefritis postinfecciosa.
2. 2. Nefropatía lúpica.
3. 3. Glomerulonefritis mesangio-capilar.
4. 4. Nefritis del shunt.
Gráfico de respuestas

 
 
 
 
Comentario
Debes recordar la fisiología renal a la hora de estudiar las tubulopatías, pues te ayudará a su
comprensión. Dentro de éstas, el síndrome de Bartter es uno de los más preguntados en el
examen. Se trata de una tubulopatía con herencia autonómica recesiva en el que existe una
alteración del transportador situado en la rama ascendente gruesa del asa de Henle (Na+, K+, Mg,
2CL). Existe una deficiencia en la reabsorción de K+ (hipopotasemia) y Na+. El sodio llega en gran
cantidad al túbulo contorneado distal, intercambiándose con calcio (hipercalciuria) y al túbulo
colector, intercambiándose con H+ y K+ (alcalosis metabólica). Estas alteraciones primarias
suponen una serie de consecuencias: - Disminución de la reabsorción de Na+ # deplección de
volumen # aumento aldosterona. - Hipopotasemia # aumento del SRAA con hipertrofia aparato
yuxtaglomerular. - Angiotensina II, aldosterona # aumento de calicreína # aumento de bradicininas
que contrarrestan el efecto hipertensor de las primeras, por lo que nunca hay HTA, a pesar de la
estimulación con angiotensina exógena. En el síndrome de Liddle existe HTA, con respuesta a
angiotensina exógena (resp 1 falsa). En la acidosis tubular tipo uno lo caracetístico es la presencia
de acidosis metabólica y la hipercalciuria; ninguno de estos datos aparece en el enunciado (resp 3
falsa). La clínica de las respuestas 4 y 5 no se corresponde con la clínica de nuestro paciente (resp
4 y 5 falsas).(R1)

305. ¿Cuál de las siguientes afirmaciones es FALSA respecto a la ictericia fisiológica del
recién nacido?:

1. 1. Es más intensa entre el 3° y 4° día de vida.


2. 2. Dura alrededor de un mes.
3. 3. El estado general es bueno.
4. 4. Las orinas son normales.
Gráfico de respuestas
Comentario

Ictericia fisiológica es un tema que repasamos continuamente, es la ictericia neonatal más


frecuente. Es el resultado de la inmadurez de los distintos pasos del metabolismo de los grupos
hem y de un mayor turnover de éstos. Se caracteriza por ser monosintomática, fugaz (desaparece
antes de una semana), poco intensa, sin afectación del estado general y aparece pasadas las
primeras 24 horas de vida. Esta ictericia no recibe tratamiento médico.(R2)

306. Hombre de 38 años, que presenta dolor súbito epigástrico, de 1 hora de evolución,
con náuseas, sudoración y taquicardia. En la exploración física presenta defensa
abdominal involuntaria, con abdomen «en tabla». La primera prueba que usted debe
emplear en la orientación diagnóstica sería:

1. 1. TC abdominal.
2. 2. ECG.
3. 3. Rx de abdomen.
4. 4. Rx de tórax en bipedestación.
Gráfico de respuestas
Comentario

Debe tener mucho cuidado en esta pregunta. Le exponen un caso de irritación peritoneal súbito. La
sospecha diagnóstica sería perforación, por lo que la prueba a realizar es la Rx de tórax en
bipedestación, ya que observaría aire libre subdiafragmático. Obviamente un TC daría mucha más
información, pero la realización de la Rx de tórax sería más rápida y confirmaría la sospecha.(R4)

 
 
 
 
307. Para la contracepción oral combinada, el estrógeno más empleado es:

1. 1. Benzestrol.
2. 2. Etinilestradiol.
3. 3. Epimestrol.
4. 4. Benzoato de estradiol.
Gráfico de respuestas
Comentario

Casi todos los anticonceptivos orales utilizan etinilestradiol.(R2)

308. Todas las siguientes son posibles manifestaciones de inicio de la fibrosis quística
y, por tanto, indicaciones para la realización del test del sudor, EXCEPTO:

1. 1. Sabor salado al besarle.


2. 2. Pólipos nasales.
3. 3. Esteatorrea.
4. 4. Invaginación intestinal.
Gráfico de respuestas
Comentario

La FQ es la enfermedad genética más letal en raza caucásica. Se debe a mutación del gen CFTR
(proteína reguladora de la conductancia transmembrana), que está en el brazo largo cromosoma 7.
La mutación más prevalente en nuestro medio es la delta- F508. Es una enfermedad con herencia
AR, por lo que es necesario tener mutados los dos alelos para expresar la enfermedad.

La alteración de la regulación de los canales iónicos de las membranas celulares produce


secreciones deshidratadas y espesas, lo que origina la clínica. La afectación pulmonar determina la
mortalidad de esta enfermedad.

Para el diagnóstico es necesario una sospecha y una confirmación. La sospecha es por la clínica,
antecedentes familiares o cribado neonatal positivo (elevación de tripsinógeno inmunoreactivo). La
confirmación es mediante el test del sudor, alteración en la diferencia de potencial nasal y estudio
genético (este último indicado en menores de 3 meses).

En el manual viene una lista con las distintas manifestaciones clínicas que son indicación para
realizar el test del sudor. De las 4 respuestas, todas son correctas menos la invaginación intestinal.
No la confunda con el íleo meconial, que sí sería sugestivo.(R4)

309. Acerca de la neumonía por Legionella pneumophila, indique la respuesta


CORRECTA:

Se adquiere por inhalación de las gotitas de flügge a partir de pacientes que tosen o
1. 1.
estornudan.
2. 2. Tiene una mortalidad global que supera el 70%.
Puede originar brotes epidémicos, pero puede ser también causa de neumonía en casos
3. 3.
esporádicos.
4. 4. Afecta raramente a personas sanas.
Gráfico de respuestas
Comentario

 
 
 
 
Pregunta de dificultad media que profundiza en la neumonía por Legionella pneumophila.

Legionella pneumophila es una bacteria gramnegativa, su hábitat natural son los reservorios de
agua como lagunas y arroyos. Las legionelas pueden sobrevivir en una amplia variedad de
condiciones ambientales.

La incidencia de neumonía depende del grado de contaminación del reservorio de agua, de la


susceptibilidad y las características inmunitarias de las personas expuestas al reservorio y de la
intensidad de la exposición; puede afectar tanto a pacientes inmunocomprometidos como
inmunocompetentes.

La Legionella se introduce en los pulmones mediante aspiración o por inhalación directa a partir de
las fuentes de agua contaminadas (como, por ejemplo, torres de refrigeración o sistemas de
distribución de agua). La mortalidad depende de la enfermedad subyacente del enfermo y su
situación inmunitaria; así en inmunodeprimidos que no reciben tratamiento llega al 80%, pero con
la pauta antibiótica adecuada en pacientes inmunocompetentes oscila entre el 0-11%.
Efectivamente, puede originar brotes epidémicos, pero puede ser también causa de neumonía en
casos esporádicos.(R3)

310. Neonato de 24 horas de vida que comienza a presentar ictericia conjuntival y en cara.
No tiene fiebre, ni taquicardia ni taquipnea, ni otros datos de infección. No encontramos
cefalohematoma, ni hepatoesplenomegalia, ni ningún otro dato a la exploración. Entre
los antecedentes del niño tenemos: edad gestacional de 38 semanas, parto vaginal,
cefálico y eutócico; S. agalactiae negativo y rotura de membranas 4 horas antes del
expulsivo, con líquido claro. Peso de recién nacido: 3,050 g. Está con lactancia materna
exclusiva. Ha recibido la primera dosis de la vacuna de hepatitis B y la profilaxis con
vitamina K y ocular. En los exámenes de laboratorio encontramos Hb 12 g/dl con Hto
40%; la madre es 0 Rh + y el niño A Rh -, el test de Coombs directo es ligeramente positivo.
¿Cuál es su diagnóstico de sospecha?:

1. 1. Incompatibilidad AB0.
2. 2. Incompatibilidad Rh.
3. 3. Ictericia fisiológica del RN.
4. 4. Esferocitosis hereditaria.
Gráfico de respuestas
Comentario

La ictericia que aparece en las primeras 24 horas de vida siempre es patológica. Debe pensar en
cuatro entidades: sepsis, infección connatal, hemorragia interna y hemólisis. En cuanto a las
hemólisis, lo más frecuente es que se den por isoinmunización; dentro de éstas, lo más frecuente
es la incompatibilidad ABO. Para que esta aparezca, se precisa (sin sesibilización previa) que la
madre sea O y el hijo, A o B.(R1)

311. Todos los siguientes, EXCEPTO uno, son hallazgos histológicos típicos de
bronquitis crónica:

1. 1. Hiperplasia de células caliciformes.


2. 2. Tapones mucosos intraluminales.
3. 3. Destrucción de tabiques alveolares.
4. 4. Aumento del índice de Reid.
Gráfico de respuestas

 
 
 
 
Comentario
Pregunta sencilla sobre la anatomía patológica de la bronquitis crónica. Este tema no ha sido muy
preguntado en el MIR, pero siempre que hacen preguntas sobre características del enfisema o
bronquitis tratan de confundirte mezclando las características de uno con el otro, por lo que debes
tener claro lo que pertenece a cada una. Aquí ocurre lo mismo: la opción que no corresponde a la
bronquitis es la destrucción de tabiques alveolares, característica definitoria del enfisema. El resto
de las opciones son correctas. En la bronquitis hay hiperplasia e hipertrofia de las glándulas
mucosas (formando tapones mucosos) y un aumento de las células caliciformes en los bronquios.
El índice de Reid refleja la relación entre el espesor de las glándulas submucosas y la pared
bronquial. Su valor normal es de 0.44 +/- 0.09, y en la bronquitis es de 0.52 +/- 0.08. En las vías
más pequeñas se aprecia un infiltrado de predominio mononuclear, tapones mucosos, metaplasia
de las células caliciformes, hipertrofia del músculo liso y distorsión por fibrosis.(R3)

312. A 5-year-old girl is brought to consultation by her mother who says that her shoes
suddenly does not fit her and are too tight. She also presents asthenia and fatigability.
Physical examination shows generalized edema, predominantly in both lower limbs. Vital
signs are normal. Urinalysis shows severe proteinuria. If a renal byopsy was performed,
which of the following would you expect to find in a small piece of kidney tissue?

1. 1. Deposits of IgA in the space between glomerular capillaries


2. 2. A thickened glomerular basement membrane without a hypercellular glomerulus
3. 3. Sclerosis of segments of some glomerules
4. 4. Normal glomerular structure
Gráfico de respuestas
Comentario
Normal glomerular structure. Edema of this child are due to hypoalbuminemia caused by renal
losses. The minimal changes nephropathy is typical of childhood, and as the name suggests, there
are no glomerular pathological changes in biopsy specimens. Treatment is mainly based on
steroids and evolution is favorable in most cases. The disease manifests as nephrotic proteinuria,
without elevation of serum creatinine.(R4)

313. A full term newborn examination (at birth) shows: pulse 60 bpm, no respiratory effort,
cyanosis, hypotonia, slight grimaces when he is stimulated with nasogastric tube. Which
of the following is his Apgar score?

1. 1. 0.
2. 2. 2.
3. 3. 3.
4. 4. 4.
Gráfico de respuestas
Comentario
El test de Apgar sirve para cuantificar de forma estandarizada el grado de depresión de un recién
nacido. En nuestro enunciado, el niño presenta FC<100 (por ello, le damos 1 punto), no genera
esfuerzo respiratorio (0 puntos por tanto), no presenta movilidad alguna (así, le damos 0 puntos),
hace una mueca a la estimulación (1 punto) y su coloración es plenamente cianótica (0 puntos). La
resultante es 2 puntos totales.(R2)

314. Respecto a la criptorquidia, señale cuál de las siguientes opciones NO es cierta:

 
 
 
 
1. 1. La incidencia es inversamente proporcional a la edad gestacional.
La tasa de esterilidad es similar a la de la población general, cuando se trata de una
2. 2.
criptorquidia unilateral.
En los casos no tratados o tratados más allá de la pubertad, existe riesgo de desarrollar
3. 3.
gonadoblastoma.
4. 4. El tratamiento hormonal no sustituye al quirúrgico.
Gráfico de respuestas
Comentario

La criptorquidia es un transtorno frecuente. Es más frecuente en priematuros por lo que más


frecuente a menor edad gestacional.

La tasa de esterilidad es similar a la de la población general ya que, al igual que pasa con los
riñones, cuando solo hay uno que está afectado no se pierde la función.

El cáncer que se desarrolla en los testículos criptorquídicos no es el gonadoblastoma, sino de


seminoma.(R3)

315. A 72-year-old male comes to the ED with a two-day history of high grade fever and
intense pelvic pain, especially with micturition. Physical examination shows suprapubic
pain and rectal examination reveals an enlarged and tender prostate. His vital signs are
heart rate 98/min, respiratory rate 17/min, blood pressure 125/80 mmHg and temperature
38.5 C (101.3 F). Laboratory workup: Hb 12.9 g/dL RWC 13.500 /mm3 Platelets 450.000
/mm3 Urinary dipstick is positive for nitrates. Which of the following is the most likely
etiologic pathogen?

1. 1. S. aureus
2. 2. S. epidermidis
3. 3. E. coli
4. 4. P. mirabilis
Gráfico de respuestas
Comentario
E. coli. The most frequent infectious agent of both UTI and acute prostatitis is E. coli.(R3)

316. Neonato de 24 horas de vida que presenta esplenomegalia de 7 cm con discreta


hepatomegalia. En la inspección, encontramos grandes ampollas fláccidas en manos y
pies, que se descaman dejando áreas denudadas, y rinorrea con obstrucción nasal que
le impide la toma adecuada de alimento. En la palpación se descubren linfadenopatías
diseminadas y llanto con la palpación de los codos y rodillas. En la radiografía de las
extremidades, encontramos zonas de refuerzo en las epífisis que forman dichas
articulaciones. ¿Qué tipo de patología sospecha en este niño?:

1. 1. Sepsis precoz.
2. 2. Sífilis congénita tardía.
3. 3. Sífilis congénita precoz.
4. 4. Infección connatal por VIH.
Gráfico de respuestas
Comentario

 
 
 
 
Existe una sencilla regla mnemotécnica para recordar el cuadro clínico producido por la sífilis
precoz. Esta regla no es perfecta, como verá, pero puede ahorrarnos pasar un mal rato… Sólo
tiene que recordar la palabra PRECOZ:

- P: Pénfigo sifilítico (lesiones ampollosas).

- R: Rinitis.

- E: “Epatoesplenomegalia”. Ya lo sé, pero de lo contrario no funciona…

- C: Condilomas planos (no confundir con “condiloma acuminado” o verruga genital).

- O: Osteítis.

- Z: “Zangre”: Leucocitosis, anemia, trombopenia.


(R3)

317. En un viaje
lejos de nuestro país, para atender personas sin recursos, acude a nuestro consultorio
un varón de 28 años de edad refiriendo tumoración en región externa en pierna derecha
que ha tenido “desde que recuerda”. En un viaje a España, le realizamos estudios
mediante TAC que usted puede contemplar. La extirpación del tumor permite mejorar la
sintomatología. El estudio de Anatomía Patológica nos confirma que estábamos ante el
tumor óseo benigno primario más frecuente, que es un:

1. 1. Tumor de células gigantes.


2. 2. Osteocondroma.
3. 3. Quiste óseo aneurismático.
4. 4. Defecto fibroso cortical.
Gráfico de respuestas
Comentario
El tumor óseo benigno primario más frecuente es el osteocondroma. Suele localizarse en las
metáfisis distal del fémur y proximal de la tibia, pero cualquier metáfisis puede desarrollar un

 
 
 
 
osteocondroma. Puede crecer durante el crecimiento del individuo, estabilizando después su
tamaño. Es un tumor benigno, pero si provoca compresión y dolor de las estructuras vecinas puede
ser subsidiario de extirpación quirúrgica. Existe una osteocondromatosis múltiple hereditaria (OMH)
conocida también como aclasia diafisaria, que presenta un riesgo de malignización de los
osteocondromas hacia condrosarcomas. Por este motivo, la extirpación profiláctica de los
osteocondromas estaría justificada en este cuadro.(R2)

318. A diferencia del tumor óseo benigno primario más frecuente que ha respondido en
la pregunta anterior, cuál es el tumor óseo maligno primario más frecuente y cuál es su
localización:

1. 1. Sarcoma Ewing – diáfisis.


2. 2. Osteosarcoma – diáfisis.
3. 3. Mieloma – diáfisis.
4. 4. Condrosarcoma – metáfisis.
Gráfico de respuestas
Comentario
El tumor óseo maligno primario (que se origina en el hueso) es el mieloma. Su localización
característica es la diáfisis de huesos largos. No hay que confundir el tumor maligno más frecuente
“en el hueso” (la metástasis), con el tumor maligno más frecuente “del hueso” (mieloma). El
mieloma nace de la medular del hueso. El resto de asociaciones son correctas, pero ninguna de
ellas son más frecuentes que el mieloma.(R3)

319. Respecto a la artritis reumatoide. ¿Cuál de las siguientes afirmaciones es FALSA?

La determinación de los anticuerpos anti-péptido cíclico citrulinado (anti-CCP) tiene valor


1. 1.
en el diagnóstico de la artritis reumatoide de inicio.
2. 2. La mayoría de las veces, la enfermedad intersticial pulmonar es asintomática.
Un rasgo característico de la afectación articular periférica es la presencia de un patrón de
3. 3. afectación radial (participación de diversas articulaciones de un mismo dedo, con
preservación de las de los otros dedos de manos o pies).
La rigidez matutina de larga duración es el síntoma de comienzo más frecuente en la
4. 4.
artritis reumatoide, precediendo a la artritis en semanas o incluso meses.
Gráfico de respuestas
Comentario

Una pregunta de dificultad media-alta. Vamos a comentar especialmente la respuesta 1, ya que


hace referencia a un concepto que nunca antes había aparecido en el examen ENARM.

Los anticuerpos antipéptido citrulinado cíclico (anti-CCP) corresponden a un grupo de


autoanticuerpos con alta especificidad para el diagnóstico de la artritis reumatoide. Son el resultado
de una respuesta autoinmune específica generada contra péptidos citrulinados que están
presentes en la membrana sinovial.

Como dice la respuesta 1, tienen interés para el diagnóstico de inicio, ya que esta enfermedad es
importante diagnosticarla cuanto antes para instaurar tratamientos modificadores agresivos desde
el principio, lo que mejorará la evolución de la misma.

 
 
 
 
La opción falsa sería la 3. El patrón que describe es claramente incorrecto, ya que una de las
características fundamentales de la artritis reumatoide es la simetría. Por consiguiente, sería
impropio de esta enfermedad encontrar afectación de varias articulaciones de un mismo dedo,
respetándose los otros y los de la otra mano o pie.

El resto de las opciones son correctas y hacen referencia a aspectos más conocidos y ya
preguntados sobre esta enfermedad. Respecto a la opción 2, merece la pena recordar que, en la
artritis reumatoide, la enfermedad intersticial pulmonar produce una afectación preferente de los
lóbulos inferiores.(R3)

320. Mujer de 38 años, tercípara, gestante en su 35 semanas. En una revisión rutinaria se


le detectan edemas en extremidades inferiores, con TA de 145/90 mmHg que se repite en
sucesivas tomas. Exámenes de laboratorio: destaca creatinina de 1.2 mg/dl, con
hemograma normal. EGO: proteinuria+++. Se decide ingresar a la paciente, en reposo
absoluto y con dieta rica en proteínas, además se le pone sedación con diazepam. Tras
una semana con cifras tensionales bien controladas comienza con cefalea frontal, visión
borrosa y la TA asciende a 160/110. ¿Cuál sería la actitud más adecuada ante esta
situación?:

Dieta rica en proteínas, reposo absoluto, sedación con diazepam, control de la TA con
1. 1.
furosemida e inducción del parto.
Dieta rica en proteínas, reposo absoluto, sedación con sulfato de magnesio, control de la
2. 2.
TA con diazóxido e inducción del parto.
Dieta rica en proteínas, reposo absoluto, sedación con sulfato de magnesio, control de la
3. 3.
TA con hidralacina y cesárea inmediata.
Dieta rica en proteínas, reposo absoluto, sedación con sulfato de magnesio, control de la
4. 4.
TA con hidralacina e inducción del parto.
Gráfico de respuestas
Comentario

Ante una paciente que presenta un cuadro de hipertensión grave persistente como es este caso,
igual o superior a 34 semanas, debe finalizarse la gestación, y la vía de elección es la vaginal. Se
ha demostrado ser más útil el sulfato de magnesio que el diazepam en la prevención y tratamiento
de las convulsiones. Los diuréticos están contraindicados en el embarazo porque disminuyen el
volumen plasmático y el flujo útero-placentario, por tanto, no se debe emplear furosemida. El
diazóxido aunque es un potente hipotensor, no se recomienda durante el embarazo por sus efectos
secundarios en la madre y efectos teratógenos. La hidralacina tiene un efecto rápido y por tanto es
el indicado en las crisis hipertensivas. La opción correcta es la nº4.(R4)

321. La respuesta inicial, como mecanismo compensador de la asfixia, da una


vasocontrición del lecho vascular pulmonar, intestinal, renal, muscular y cutáneo con
redistribución del flujo sanguíneo hacia:

1. 1. El corazón.
2. 2. El hígado.
3. 3. El cerebro.
4. 4. A y C.
Gráfico de respuestas
Comentario

 
 
 
 
Los órganos que se deben preservar es el corazón y el cerebro, por lo que el mecanismo
compensador no afectará a estos órganos respuesta 4 correcta.(R4)

322. A 66 year-old man comes to the physician after been diagnosed on an emergency
department of left vocal cord palsy when he was on vacation. He reports a 1 month-history
of progressively worsening dysphonia. Which of the following is the most efficient
management?

1. 1. Request a chest X-ray and refer the patient to the ENT (ear, nose and throat) specialist.
2. 2. Refer the patient to the ENT specialist without any other study.
3. 3. Request a viral serology and follow-up after 6 months if positive.
4. 4. Start vitamin B1-B6-B12 therapy and follow-up after 3 months.
Gráfico de respuestas
Comentario

Aunque el especialista encargado de valorar la disfonía es el otorrinolaringólogo, como este


paciente ya está diagnosticado de parálisis recurrencial, en un paciente de edad mediana y
fumador lo primero que hay que hacer es descartar un cáncer de pulmón, cosa que a veces se
detecta con una Rx simple de tórax. Recuerde que el nervio recurrente, encargado de la inervación
motora de las cuerdas vocales, en el lado izquierdo del cuerpo baja hasta el mediastino y recurre
en el cayado de la aorta; por lo que patologías pulmonares, cardíacas o mediastínicas pueden
comprimirlo. En caso de verse un imagen sugestiva de tumor pulmonar, este paciente además de
ser enviado al ORL debería ser enviado de forma preferente a un servicio de Neumología para
estudio.(R1)

323. Femenino de 21 años,


G2P2. Ciclos menstruales normales. Acude a urgencias refieriendo metrorragia escasa y
oscura de tres días de evolución junto a molestias hipogástricas escasas. Exploración
ginecológica: genitales externos y vagina normales, cérvix cerrado, con sangrado escaso
menor que regla, no doloroso a la movilización, utero y anexos no se palpan patológicos.
Test de embarazo positivo. En el USG transvaginal se observa un útero como el que se
muestra en la imagen. Anexos sin hallazgos patológicos. No líquido libre. ¿Qué haría a
continuación?

 
 
 
 
1. 1. Determinación de progesterona.
2. 2. Legrado bajo anestesia general.
3. 3. Realizar laparoscopía diagnóstica.
4. 4. Determinación de HCG sérica cada 48 horas.
Gráfico de respuestas
Comentario

El USG nos muestra un útero vacío, donde no encontramos vesícula embrionaria, y sin embargo
está decidualizado. Esto podría corresponder, por tanto, a un embarazo ectópico, cuyo diagnóstico
de elección sería con laparoscopía. Sin embargo, también nos dicen que los anexos no muestran
hallazgos patológicos, con lo que también cabe la posibilidad de que la vesícula embrionaria esté
en el útero, pero todavía no se vea, por lo que lo más correcto es hacer lo que dice la respuesta 4.
Si se trata de una gestación normal, la curva de ascenso de la hCG será diferente a la del
embarazo ectópico, lo que podría ser de utilidad para salir de dudas, en vez de proceder
directamente a una prueba invasiva.(R4)

324. ¿Qué técnica nos daría el diagnóstico de certeza en la paciente del caso clínico
anterior?

1. 1. Determinación de HCG sérica.


2. 2. Ecografía transvaginal.
3. 3. Laparoscopía.
4. 4. Resonancia magnética.
Gráfico de respuestas
Comentario

Tal como se explicaba en el comentario anterior, el diagnóstico y tratamiento de un embarazo


ectópico se alcanzaría mediante laparoscopía.(R3)

325. Con respecto a la patología que presente la paciente del caso clínico anterior ¿Cuál
NO se considera factor de riesgo para embarazo ectópico?

1. 1. Primiparidad.
2. 2. Malformaciones uterinas.
3. 3. Endometriosis.
4. 4. Enfermedad inflamatoria pélvica.
Gráfico de respuestas
Comentario

La primiparidad, hasta el momento, no se ha relacionado con una mayor incidencia de gestación


ectópica. Tal como explicamos en el Manual, algunos de los más importantes serían los siguientes:

- Antecedentes de gestación ectópica.

- Cirugía tubárica previa.

- Enfermedad inflamatoria pélvica.

- DIU. Es controvertido. Disminuye radicalmente los embarazos intrauterinos y levemente los


extrauterinos, por lo que los extrauterinos aumentan relativamente.

 
 
 
 
- Endometriosis.

- Ligadura tubárica.

- Infertilidad (posible obstrucción tubárica, etc.)

- Técnicas de reproducción asistida (inducción de la ovulación, etc.).

(R1)

326. Respecto a los parámetros diagnósticos del síndrome de ovarios poliquísticos, es


CIERTO que:

1. 1. Hasta el 70% de las mujeres serán obesas.


2. 2. Entre el 5-10 % de las pacientes tienen niveles normales de testosterona total.
3. 3. Hasta un 80% de las pacientes tienen elevada la LH.
4. 4. Por USG no es necesario que los dos ovarios cumplan criterios de ovario poliquístico.
Gráfico de respuestas
Comentario

En el SOP hasta el 50% de las mujeres serán obesas. Los andrógenos son responsables del
cuadro clínico, pero entre el 20-30 % de las pacientes tienen niveles normales de testosterona
total, no obstante la androstendiona está elevada en el 40% de las pacientes. Hasta un 50% de las
pacientes tienen elevada la LH, siendo mayor que la FSH en un 95% de las veces. Por USG no es
necesario que los dos ovarios cumplan criterios de ovario poliquístico (presencia de 12 o más
folículos con diámetros de 2- 9 mm y/o un volumen ovárico mayor de 10 ml).(R4)

327. ¿Qué antibiótico emplearía para tratar una infección producida por Escherichia coli
productor de betalactamasa de espectro extendido - BLEE-?

1. 1. Amoxicilina.
2. 2. Amoxicilina-ácido clavulánico.
3. 3. Piperacilina-tazobactam.
4. 4. Meropenem.
Gráfico de respuestas
Comentario

En la última década se ha descrito, cada vez con mayor frecuencia, la infección por enterobacterias
Gram negativas productoras de betalactamasa de espectro extendido (BLEE). También es posible
encontrar en la literatura su denominación como betalactamasa de espectro ampliado (BLEA). Las
dos especies en las que con más frecuencia se han descrito estas betalactamasas son Escherichia
coli y Klebsiella pneumoniae. Como indica su nombre, son enzimas capaces de escindir el anillo
batalactámico pero, en contra de lo que ocurre con las betalactamasas habituales, su acción no es
contrarrestada por los inhibidores de betalactamasas (como el ácido clavulánico, el sulbactam o el
tazobactam). A efectos prácticos, el único grupo de antibióticos betalactámicos activos frente a este
tipo de bacterias son los carbapenemes: imipenem, meropenem, ertapenem, doripenem. No se
puede emplear para su tratamiento las penicilinas, las penicilinas asociadas a inhibidor de
betalactamasas, los monobactámicos como el aztreonam ni ninguna de las cefalosporinas.(R4)

 
 
 
 

328. Mujer de 30
años diagnosticada de LES, en tratamiento con AINE, por artritis de rodilla. Acude al
servicio de Urgencias por hematemesis. PA 120/40 FC 60. Hb 14,5. Se realiza una
endoscopia urgente observándose (ver imagen). ¿Qué binomio diagnóstico-terapéutico
considera correcto?

1. 1. Úlcera duodenal Forrest Ib Esclerosis endoscópica + hemoclip + perfusión i.v. de IBP.


2. 2. Hemorragia por varices Esclerosis + somatostatina i.v.
3. 3. Úlcera duodenal Forrest III IBP vía oral y alta hospitalaria.
4. 4. Úlcera duodenal Forrest IIa Esclerosis endoscópica e IBP vía oral.
Gráfico de respuestas
Comentario

Dependiendo del aspecto endoscópico de la úlcera, el riesgo de resangrado es variable. La


clasificación de Forrest permite predecirlo con bastante precisión.(R3)

CLASIFICACIÓN DE
FORREST

Clasificación Hallazgo Resangrado


endoscópico

 
 
 
 

Hemorragia activa Ia Hemorragia en 55%


chorro

Ib Hemorragia en 50%
babeo

Hemorragia reciente IIa Vaso visible 43%

IIb Coágulo adherido 22%

IIc Mancha plana 7%

Sin signos sangrado III Base de fibrina 2%

En esta pregunta, se trata claramente de un Forrest III. El fondo de fibrina se ve amarillento, como
en este caso. Dado que el riesgo de resangrado es muy bajo, la actitud correcta sería el
tratamiento domiciliario con inhibidores de la bomba de protones.

329. What is the recommended treatment for a patient with ovarian cancer?

1. 1. Radiotherapy.
2. 2. Cytoreductive surgery and chemotherapy.
3. 3. Surgery without chemotherapy.
4. 4. Immunosuppressive therapy.
Gráfico de respuestas
Comentario

El tratamiento del cáncer de ovario es sencillo pero es importante a la hora de estudiar este tema.
Simplemente recuerda que la actitud inicial es siempre quirúrgica, que nos sirve también para
estadificar y se sigue normalmente de quimioterapia adyuvante.

La cirugía citorreductora consiste en eliminar la mayor cantidad de neoplasia y el protocolo para el


cancer de ovario es: histerectomía total con anexectomía bilateral, omnetectomía, apendicetomía
(especialmente en tumores mucinosos), linfadenectomía iliaca, pélvica y paraortica, lavado y
biopsias del peritoneo.

 
 
 
 
La radioterapia se puede utilizar en disgerminoma, que es radiosensible.

La cirugía sin tratamiento quimioterápico sólo se utiliza en los estadio Ia y Ib bien diferenciados.

La inmunosupresión y la hormonoterapia no se utilizan.(R2)

330. Todas las afirmaciones sobre el sarampión son ciertas, EXCEPTO:

1. 1. La contagiosidad es máxima durante la erupción, que dura 7 a 10 días.


2. 2. Las complicaciones son: neumonía, otitis media y encefalitis.
3. 3. La inmunización activa y pasiva sirven para prevenir la enfermedad.
La frecuencia de encefalitis consecutiva a la inmunización es 1000 veces menor que la
4. 4.
secundaria a la enfermedad espontánea.
Gráfico de respuestas
Comentario

Pregunta de dificultad media acerca del sarampión.

El dato que nos pide la pregunta puede que sea difícil de recordar, pero debemos saber que el
período de máxima contagiosidad del sarampión es 5 diás antes de la erupción y hasta 5 días
después de que aparezca (podemos también recordar por analogía que el de la Rubéola es 7 días
antes a 7 días después) y por lo tanto, la opción a elegir es la 1. El resto de las opciones son
ciertas y conviene recordarlas por si aparecieran en el examen.(R1)

331. Un paciente de 60 años es diagnosticado de carcinoma epidermoide del lóbulo


inferior del pulmón derecho (T2 N1 M0). Tiene un FEV1 del 80% (superior a 2,5 l). ¿Cuál
sería la mejor opción terapéutica?

1. 1. Quimioterapia neoadyuvante y cirugía posterior.


2. 2. Radioterapia con intención curativa.
3. 3. Cirugía y radioterapia posterior.
4. 4. Cirugía sólo.
Gráfico de respuestas
Comentario

El cáncer de pulmón es un tema muy preguntado en el MIR, por lo que deberías conocerlo bien.

El caso que nos describen corresponde a un paciente con un carcinoma epidermoide en estadio II.
El tratamiento clásico de este estadio era la cirugía aislada. Sin embargo, se ha demostrado
recientemente que la quimioterapia asociada a la cirugía supone un aumento de la supervivencia
en estos enfermos. Por este motivo, aunque en la plantilla de respuestas inicial el Ministerio aceptó
la 4 como válida, después del período de reclamaciones la cambió por la 1. Fíjate que se trata de
una pregunta de reserva (257), de modo que éstas también pueden ser impugnadas.(R1)

332. El síntoma principal de la artrosis de rodilla es:

1. 1. Inestabilidad articular.
2. 2. Dolor.
3. 3. Tumefacción.
4. 4. Crujidos articulares.

 
 
 
 
Gráfico de respuestas
Comentario
Es una pregunta difícil por tener que jerarquizar los síntomas de la gonartrosis. Sin embargo,
pregúntale a tu abuela cuál es su síntoma principal, su mayor queja: el dolor, y con ello habrás
acertado esta pregunta. Si tenemos en cuenta que el problema radica en un desgaste cartilaginoso
y que se producen fisuras que terminan por agredir incluso al hueso, dejando de tener efecto
amortiguador de carga y roces, es comprensible. También surge de la distensión ligamentosa,
compresión de nervios, contractura muscular,? Es cierto que existe tumefacción de partes blandas,
breve rigidez, crujidos articulares en fases avanzadas (como si de una bisagra mal engranada se
tratara), que las deformidades articulares tardías provocan inestabilidad y dificultan la
deambulación, especialmente al bajar escaleras que es donde se sobrecarga más la articulación?
pero el dolor mecánico domina el cuadro desde el principio: empeora con el movimiento y mejora
con el reposo. La analgesia es tan importante como mantener la funcionalidad articular para
mejorar la calidad de vida del paciente.(R2)

333. A 70-year-old diabetic patient with a 50-pack-year history of smoking is admitted to


a coronary care unit for acute anterior wall myocardial infarction. A few hours later he
starts complaining of moderate dyspnea and signs of hypoperfusion are observed. Which
of the following options is not indicated in this case?

1. 1. Beta-blockers in order to reduce ischemia and decrease the risk of cardiac rupture.
2. 2. Diuretics and vasodilators since they are first choice drugs.
3. 3. Echocardiogram to rule out mechanical complications.
If the previous therapeutic measures are ineffective, an intraaortic balloon pump must be
4. 4.
placed.
Gráfico de respuestas
Comentario

Las complicaciones del infarto es un tema fundamental, y debe manejar con soltura el diagnóstico
diferencial entre las distintas entidades.

Nos describen un caso clínico de insuficiencia cardiaca con posible edema agudo de pulmón por
fallo de ventrículo y nos piden que digamos cual es la opción terapéutica menos indicada. Debe
recordar que en los betabloqueantes están CONTRAINDICADOS en el edema agudo de pulmón
ya que son fármacos INO y CRONO- tropos negativos, y lo único que harían en fase aguda es
empeorar la situación hemodinámica del paciente.

Tenga cuidado con la respuesta 2. El paciente está entrando en edema agudo de pulmón (observe
que ya ha desarrollado disnea de moderada intensidad), por lo que deberíamos emplear diuréticos
y vasodilatadores (venosos) para evitarlo. De acuerdo que el paciente está hipotenso, pero lo más
prioritario es evitar que muera asfixiado. Si fuera necesario, se podría considerar el empleo de
fármacos vasopresores si la situación lo requiriera.(R1)

334. Uno de los siguientes tipos de apendicitis aguda NO requiere intervención quirúrgica
urgente, señálelo:

1. 1. Apendicitis gangrenosa.
2. 2. Plastrón apendicular.
3. 3. Mucocele apendicular.
4. 4. Apendicitis catarral.

 
 
 
 
Gráfico de respuestas
Comentario

De cara al ENARM, debe quedar claro que el tratamiento de la apendicitis aguda es


QUIRURGICO, excepto en el caso de que el diagnóstico se haga en la fase de plastrón
apendicular (masa palpable y más de una semana de evolución) en cuyo caso se instaurará
tratamiento antibiótico intravenoso con cirugía diferida.(R2)

335. A patient with abdominal pain located in the epigastrium and right hypochondrium
along with jaundice and loss of about 10 kg of body weight in the last 15 months
undergoes an abdominal ultrasound that shows a mass in the pancreas head.
Unfortunately, the radiologist can not determine if that mass is pancreatitis or pancreatic
cancer. Given this result, which of the following techniques is the most appropriate in
order to yield a definitive diagnosis?

1. 1. Arteriography.
2. 2. ERCP (Endoscopic retrograde cholangiopancreatography).
3. 3. MRI.
4. 4. Studies of pancreatic function.
Gráfico de respuestas
Comentario

Esta es una pregunta de dificil respuesta pues entre las respuestas figuran técnicas de imagen que
se consideran como las de más utilidad en el estudio de las masas pancreáticas y de ellas sin
dudas la más ampliamente utilizada es el TAC.

Sin embargo partimos ya del diagnóstico de masa pancreática y nos están pidiendo el diagnóstico
diferencial entre una pancreatitis crónica y un cáncer de cabeza de páncreas. Así pues este
diagnóstico diferencial sólo lo podremos conseguir fielmente mediante la obtención de una biopsia
o citología que nos permita el diagnóstico de cáncer y entre las pruebas enumeradas la única que
nos lo permite es la CPRE y por otra parte la mejor prueba de imagen en el diagnóstico de una
pancreatitis crónica es la CPRE.

En contrapartida el TAC nos permite hacer el estudio de extensión del cáncer mientras que esto no
es posible con la CPRE. En la práctica clínica si un USG demuestra litiasis biliar sin evidencia de
una masa pancreática la prueba siguiente a realizar en el estudio es una CPRE. Por otra parte si
una USG no demuestra la existencia de una litiasis biliar la presencia de coledocolitiasis es poco
probable, y la existencia de un cáncer de cabeza de páncreas o de la ampolla de Vater o bien una
pancreatitis crónica son las causas más probables de obstrucción y generalmente el TAC es la
siguiente prueba a realizar. Además hay que tener en cuenta que el TAC puede perder masas
pancreáticas menores de 2 cm, y en estos casos los hallazgos del TAC pueden limitarse a la
presencia de dilatación del conducto biliar o pancreático. Estos hallazgos son altamente
sospechosos de neoplasia y requieren de una posterior evaluación mediante CPRE.(R2)

 
 
 
 

336. Paciente
femenino primigesta de 32 años, embarazada de 38 semanas, con las siguientes
condiciones obstétricas intraparto: feto cefálica, occipitoiliaca izquierda anterior, II plano,
10 cm de dilatación, bolsa rota y líquido claro. El registro cardiotocográfico es el que se
muestra en la imagen. ¿Qué haría a continuación?:

1. 1. Prueba de Pose.
2. 2. Cesárea.
3. 3. Microtoma fetal.
4. 4. Ventosa.
Gráfico de respuestas
Comentario
Efectivamente el RCTE muestra Dips tipo II y muy baja variabilidad, siendo por lo tanto sospechoso
de sufrimiento fetal. La prueba diagnóstica de sufrimiento fetal intraparto por excelencia es el pH
fetal, y deberá ser en función de su resultado nuestra actitud posterior.(R3)

337. Señale la respuesta FALSA en relación con la anomalía de Ebstein:

1. 1. La valva septal de la válvula tricúspide está desplazada hacia el ventrículo.


2. 2. La aurícula derecha es pequeña.
3. 3. Los trastornos del ritmo son frecuentes.
4. 4. Asocia comunicación interauricular.
Gráfico de respuestas
Comentario
Las cardiopatías congénitas constituyen uno de los temas más preguntados en el MIR dentro de la
cardiología, por lo que es importante que conozcas las características de las cardiopatías
principales. La anomalía de Ebstein no es muy preguntada pero aprovecha la ocasión para
recordar que consiste en un desplazamiento hacia abajo de la válvula tricúspide en el VD debido a
una inserción anómala de las valvas, por lo que la VD será más pequeño de lo normal, mientras
que la AD será mayor de lo normal. Puede asociar disfunción del VD y es importante que

 
 
 
 
recuerdes su posible asociación al síndrome de WPW. En cuanto al pronóstico de estos pacientes,
la mayoría alcanza los 30 años de vida.(R2)

338. Sobre la púrpura de Schönlein-Henoch, señale la afirmación INCORRECTA:

La lesión glomerular característica consiste en una intensa proliferación extracapilar,


1. 1.
formando imágenes en semiluna.
2. 2. Las exploraciones complementarias mostrarán leucocitosis y elevación de la IgA.
La afectación renal produce hematuria y proteinuria, siendo muy rara la aparición de
3. 3.
insuficiencia renal.
4. 4. Más del 50% de los pacientes padecen síntomas articulares, fundamentalmente artralgias.
Gráfico de respuestas
Comentario

La opción 1 es falsa ya que la proliferación característica del Schönlein- Henoch es a nivel


mesangial (difusa o focal) con una GN proliferativa segmentaria. La proliferación extracapilar
determina una lesión glomerular muchísimo más grave, en la glomerulonefritis rápidamente
progresiva.(R1)

339. Niño de 12 meses de edad que presenta irritabilidad y pérdida de apetito de varios
meses de evolución. En la exploración se observan las encías hinchadas y azuladas,
petequias en la piel y que llora intensamente al movilizarle las extremidades. En la
radiografía de rodillas se observa un adelgazamiento de la cortical, una línea metafisaria
gruesa y por debajo de ella una zona de rarefacción. La calcemia y la fosforemia son
normales. El niño está alimentado con leche materna exclusivamente y no recibe
suplementos vitamínicos. ¿Qué diagnóstico le sugiere?:

1. 1. Sífilis congénita.
2. 2. Raquitismo carencial.
3. 3. Hipovitaminosis C.
4. 4. Deficiencia de tiamina.
Gráfico de respuestas
Comentario

Esta pregunta nos presenta una caso de escorbuto infantil, consecuencia de un déficit de vitamina
C (ácido ascórbico). Se puede producir cuando se alimenta a los lactantes con leche materna
exclusivamente durante el primer año de vida. Se manifiesta por irritabilidad, hipersensibilidad ósea
con tumefacción y pseudoparálisis de las piernas, hemorragias subperiosticas, hiperqueratosis de
los folículos pilosos y una serie de trastornos mentales.

El beri beri infantil (déficit de tiamina) aparece en los niños malnutridos, los que reciben
hiperalimentación sin suplementos o leche hervida. Predominan los síntomas cardiológicos agudos
y tienen un llanto afónico característico.

El déficit de niacina (pelagra) se caracteriza por debilidad, dermatitis, diarrea y demencia.

De los déficit de micronutrientes el más importante es el de vitamina D que ocasiona raquitismo, de


el que además de los signos clínicos típicos (fontanelas agrandadas, rosario condrocostal
craneotabes...) debe recordar que el nivel de calcio suele ser normal pero el de fósforo siempre
está descendido y la fosfatasa alcalina elevada.

 
 
 
 
Los síntomas mas importantes de la sífilis congénita son: pénfigo, rinorrea, exantema, condilomas,
afectación ósea y renal(R3)

340. Un paciente de 65 años diagnosticado desde hace 30 años de reflujo gastroesofágico


con esofagitis, y de esófago de Barrett desde hace 8 años, seguido con revisiones
anuales en la consulta. En la última biopsia realizada se ha observado displasia severa.
Señale cuál es la actitud más correcta en este caso:

1. 1. Endoscopia y nueva biopsia dentro de 6 meses.


2. 2. Esofagectomía distal programada.
3. 3. Iniciar tratamiento con antihistamínicos anti-H2, y nueva revisión en 6 meses.
4. 4. Iniciar tratamiento con omeprazol, y nueva revisión en 6 meses.
Gráfico de respuestas
Comentario
La presencia en un esófago de Barret de displasia severa debe considerarse como que ya existe
un adenocarcinoma o que el riesgo de desarrollarlo es muy elevado y en forma inminente. Por todo
ello, en esa situación, lo que se recomienda es la esofaguectomía de la zona afecta. Ultimamente
se ha desarrollado procedimientos endoscópicos como la abrasión, pero su eficacia en estudios
amplios todavía no ha sido suficientemente contrastada.(R2)

341. La aparición de un osteosarcoma en una persona de 65 años de edad nos haría


pensar en la existencia previa de uno de los siguientes antecedentes:

1. 1. Mucopolisacaridosis.
2. 2. Enfermedad de Paget.
3. 3. Displasia fibrosa.
4. 4. Corticoterapia.
Gráfico de respuestas
Comentario

Pregunta de dificultad media-baja que trata acerca de las complicaciones de la enfermedad de


Paget.

La enfermedad de Paget es, tras la osteoporosis, la osteopatía más frecuente en los países de
nuestro entorno. Con más frecuencia en varones, aparece fundamentalmente en torno a los 65
años con fuerte tendencia a la agregación familiar.

Entre las complicaciones de la enfermedad de Paget podemos destacar la elevación del gasto
cardíaco debido a la proliferación de vasos sanguíneos en el hueso; fracturas patológicas,
síndromes neurológicos compresivos, trastornos bucodentales; también como complicación de la
enfermedad de Paget destacaremos cálculos urinarios (por hipercalciuria), y en relación con la
pregunta que nos compete, el sarcoma.

Este osteosarcoma aparece en el 1% de los pacientes con enfermedad de Paget, siendo la


complicación más grave. El aumento del dolor y de la tumefacción, junto con un aumento
exagerado de los niveles de fosfatasa alcalina, nos debe hacer sospechar su presencia. Así, el
osteosarcoma en términos generales tiene una frecuencia bimodal: la 2ª década y la 7ª década en
personas con antecedentes de radioterapia o enfermedad de Paget. Se suele localizar en fémur,
húmero, cráneo, huesos de la cara y pelvis.

 
 
 
 
En cuanto al resto de las opciones:

•   No está demostrado que un traumatismo óseo suponga un factor de riesgo para la


aparición de un osteosarcoma. En algunos casos, sí podemos decir que traumatismos
previos pueden relacionarse con la aparición de una lesión pseudotumoral llamada quiste
óseo aneurismático (proceso reactivo no neoplásico) que está compuesto por múltiples
cavidades rellenas de líquido hemático.
•   La mucopolisacaridosis constituye un grupo de enfermedades genéticas en las que, debido
a un defecto enzimático, se acumula material de distinto tipo (dermatán sulfato, queratán
sulfato…). En general, ocasionan alteraciones oculares (opacidad corneal), hematológicas
(linfocitos granulados), alteraciones neurológicas… Pero no produce la aparición de un
osteosarcoma entre sus manifestaciones clínicas.
•   La displasia fibrosa no supone un factor de riesgo para la aparición de un osteosarcoma,
puesto que la displasia fibrosa no es otra cosa que una alteración en el proceso de
osificación que ocurre en el desarrollo de uno o varios efectos esqueléticos. Como
curiosidad, podemos decir que puede asociarse a manchas cutáneas hiperpigmentadas y
pubertad precoz en niñas, formando parte del síndrome McCune-Albright.
•   La corticoterapia no es factor de riesgo para la aparición de un osteosarcoma. Lo que te
interesa recordar de cara al MIR es que la corticoterapia prolongada puede causar necrosis
avascular ósea (típicamente cabeza de fémur o cabeza humeral).

(R2)

342. ¿Cuál de los siguientes supuestos NO es cierto?:

1. 1. La glucosa pasa de la madre al feto a través de la placenta.


2. 2. La hiperglucemia fetal estimula la secreción de insulina.
3. 3. Los cuerpos cetónicos no atraviesan la placenta.
4. 4. Los aminoácidos atraviesan la placenta.
Gráfico de respuestas
Comentario

Pregunta complicada sobre la transferencia placentaria. La glucosa pasa de la madre al feto por
difusión facilitada, habiendo mayor concentración en la madre. La hiperglucemia fetal estimula la
producción de insulina fetal (evento que se ve en los fetos de gestantes diabéticas, de ahí que
exista mayor macrosomía fetal). Los aminoácidos atraviesan la barrera placentaria por transporte
activo. El glucagón no atraviesa la placenta, al igual que la insulina. Por lo tanto, la opción 3 es la
incorrecta, ya que los cuerpos cetónicos sí atraviesan la placenta.(R3)

343. ¿Cuál de las siguientes afirmaciones respecto al tratamiento intensivo con insulina
para la diabetes (uso de una bomba de insulina externa o tres o más inyecciones diarias
de insulina en función de controles frecuentes de glucemia) es correcta?:

1. 1. Todos los pacientes con diabetes mellitus deben recibir este tratamiento.
Se ha demostrado de forma definitiva que, comparado con el tratamiento estándar, este
2. 2. tratamiento intensivo reduce la probabilidad de retinopatía en los pacientes con diabetes
mellitus insulinodependiente.
3. 3. Con un control cuidadoso, se evita un aumento en el número de episodios hipoglucémicos.
El tratamiento intensivo con insulina no reduce la concentración de hemoglobina
4. 4.
glucosilada.
Gráfico de respuestas

 
 
 
 
Comentario

El tratamiento intensivo tiene el problema principal de producir un mayor número de hipoglucemias.


Por este motivo, no debe utilizarse en los pacientes más vulnerables frente a este problema, ni en
aquellos que tendrán mayores dificultades para adherirse al tratamiento (niños y ancianos).

Aunque las cifras de glucemia no llegan a ser iguales que las de un paciente no diabético, se ha
visto que disminuyen significativamente las complicaciones microvasculares (nefropatía,
retinopatía, neuropatía), por lo que la respuesta correcta es la 2.(R2)

344. Hombre de 65 años que presenta de forma súbita fiebre, disnea, edemas en
miembros inferiores y sensación de mareo llegando a tener episodios de síncope. En la
exploración física destaca la existencia de un soplo mitral, crepitantes bilaterales y
lesiones nodulares dolorosas en pulpejos de los dedos. En el ECG su detecta en bloqueo
A-V de grado III. ¿Cuál le parece la actitud más correcta?:

Realizar ecocardiograma urgente y valorar cirugía urgente pues se trata de una


1. 1.
endocarditis y posible absceso miocárdico.
Extraer hemocultivos, realizar fondo de ojo e instaurar tratamiento empírico con
2. 2.
Cloxacilina.
3. 3. Realizar Rx de tórax, biopsia de las lesiones nodulares y tratamiento con O2 y diurético.
Extraer hemocultivos e instaurar tratamiento diurético, antibioterapia con vancomicina y
4. 4.
colocar un marcapasos.
Gráfico de respuestas
Comentario

En esta pregunta tenemos una serie de claves para llegar al diagnóstico y otras para adivinar la
actitud terapéutica más correcta, que es lo que nos preguntan.

Comencemos por lo primero. Se trata de un cuadro súbito de insuficiencia cardiaca congestiva


(edemas en miembros inferiores, crepitantes bilaterales = edema agudo de pulmón) provocada por
un bloqueo AV de tercer grado en un paciente en el que no nos comentan antecedentes previos de
interés. Podemos saber que la causa de todo ello es una endocarditis porque nos dicen que tiene
fiebre y nos hablan de unos nódulos en el pulpejo de los dedos que son lesiones características de
esta enfermedad (nódulos de Osler). Por lo que ya tenemos el diagnóstico.

Pasemos a deducir el tratamiento. Por lo que vemos es un paciente que está muy grave e
inestable ya que se encuentra en insuficiencia cardiaca congestiva, por un bloqueo AV de tercer
grado, todo ello secundario a la endocarditis aguda que padece. En la endocarditis, cuando se
produce un bloqueo de tercer grado hay que pensar que sea por un absceso miocárdico, lo cual se
diagnostica con ecocardiografía (urgente, dada la inestabilidad del paciente) y se trata mediante
cirugía urgente. Por lo tanto la respuesta correcta es la 1.(R1)

345. Una de las siguientes respuestas es falsa respecto a la neuronitis vestibular:

1. 1. Se caracteriza por una crisis de vértigo.


2. 2. En la electronistagmografía hay hipo o arreflexia vestibular.
3. 3. Se afecta la primera neurona del nervio vestibular.
4. 4. Se acompaña de hipocusia y acúfenos.
Gráfico de respuestas
Comentario

 
 
 
 
La neuronitis vestibular es un proceso que afecta sólo al nervio vestibular del VIII par craneal, por
lo que presenta vértigo y no síntomas auditivos. La etiología se piensa que puede ser vírica o
infecciosa y afecta a la primera neurona del nervio vestibular que disminuye o cesa en su función
provocando hipo o arreflexia vestibular. Clínicamente se presenta como una crisis intensa de
aparición brusca que irá mejorando en el curso de días o semanas.(R4)

346. ¿Cuál de los siguientes factores puede producir una disminución del riesgo de
padecer un cáncer de mama?

1. 1. Administración repetida de estrógenos.


2. 2. Menopausia tardía.
3. 3. Hiperplasias ductales o lobulillares.
4. 4. Multiparidad.
Gráfico de respuestas
Comentario

Pregunta relativamente fácil sobre los factores de riesgo para el cáncer de mama.

Si recordamos el carácter hormonodependiente del cáncer de mama, podemos descartar


directamente las opciones 1 y 2, y directamente llegaremos a la respuesta correcta (opción 4) ya
que la multiparidad es un factor protector por los cambios producidos en la diferenciación del tejido
mamario.

Las lesiones proliferativas en la mama aumentan el riesgo a padecer el cáncer de mama, por lo
que la opción 3 es falsa.(R4)

347. La disección aguda de la aorta torácica tipo B de Stanford se caracteriza por los
siguientes hallazgos anatómicos:

Disección que afecta la raíz de la aorta y la válvula aórtica, pero preservando el resto de la
1. 1.
aorta ascendente.
2. 2. La disección solamente afecta el cayado o arco aórtico.
3. 3. Disección de la aorta descendente distal a la arteria subclavia izquierda.
4. 4. Disección de toda la aorta torácica.
Gráfico de respuestas
Comentario

La disección de aorta ocurre cuando la sangre penetra en la pared de la aorta a través de una
solución de continuidad en la íntima y se propaga longitudinalmente, separando la íntima de la
media, formando un canal lleno de sangre dentro de la pared aórtica. La disección aórtica se
asocia en un 70% a HTA que es, con mucho, el agente etiológico más destacado. La máxima
incidencia se genera en la sexta y séptima década de la vida y es dos veces más frecuente en los
hombres que en las mujeres.

El síntoma más usual y precoz es el dolor torácico, intenso y brusco que, a menudo, se describe
como muy desgarrador. Otra característica del dolor es su tendencia a migrar a diferentes zonas
según la disección se extiende.

Recuerde que para el diagnóstico, la técnica de elección es el ecocardiograma transesofágico o el


TC si no se dispone del primero.

 
 
 
 
El pronóstico de la disección aórtica es malo y, sin el tratamiento adecuado, resulta casi siempre
fatal. El tratamiento es distinto según la localización de la disección. Así hay dos clasificaciones
que diferencian los tipos de disección según el lugar de la aorta que se afecte:

La disección de la aorta ascendente es la más frecuente y la de peor pronóstico. Para el


tratamiento es importante que recuerde lo siguiente:

•   La disección que afecta a la aorta ascendente siempre requiere una intervención


quirúrgica.
•   La disección que NO afecta a la aorta ascendente (es decir, que afecta solamente a la
aorta descendente) NO requiere cirugía, mientras que el paciente esté estable y no haya
complicaciones. Lo que se usa es tratamiento médico con fármacos hipotensores como el
nitroprusiato y los betabloqueantes intravenosos (recuerde que la hidralacina y el diazóxido
están contraindicados).

(R3)

Clasificación de la disección de aorta

CLASIFICACIÓN DE DE
BAKEY

I Desgarro en Ao ascendente y se extiende a


descendente

II Desgarro en Ao ascendente (confinado a ésta)

III Desgarro en Ao descendente

CLASIFICACIÓN DE
STANDORD

A (proximales) Afectan a Ao ascendente

 
 
 
 

B (distales) No afectan a Ao ascendente

Regla mnemotécnica

348. En un varón de 20 años con un ataque agudo de angioedema hereditario, ¿qué


administraría?:

1. 1. Corticoides.
2. 2. Antihistamínicos.
3. 3. C1 inhibidor.
4. 4. Danazol.
Gráfico de respuestas
Comentario
El angioedema hereditario o edema angioneurótico familiar de Quincke se caracteriza por la
aparición de brotes de edema del tejido celular subcutáneo sobretodo en cara, manos y pies. No se
acompaña de habones ni prurito. En algunos casos se produce afectación de la mucosa intestinal y
respiratoria lo cual puede manifestarse en forma de vómitos, diarrea o broncoespasmos. Se debe a
un déficit real o funcional del C1 inhibidor y se hereda de forma autosómico dominante. El
tratamiento de las crisis agudas se hace con concentrados de C1 inhibidor o con plasma fresco
congelado y la profilaxis con danazol o estanozolol. El ácido tranexámico, un fármaco
antifibrinolítico, se ha utilizado para el tratamiento de la forma aguda y crónica de la enfermedad
con menos efectos secundarios.(R3)

349. Señale la afirmación FALSA, de entre las siguientes, respecto a la isoinmunización


Rh:

1. 1. La enfermedad hemolítica grave suele afectar al segundo hijo.


2. 2. Suele cursar con ictericia, que aparece en las primeras 24 horas de vida.
Debe realizarse profilaxis con gammaglobulina específica anti-D antes de que pasen 72
3. 3.
horas del parto, cuando el test de Coombs directo es positivo.
4. 4. El 90% de los casos de incompatibilidad se deben al antígeno D.

 
 
 
 
Gráfico de respuestas
Comentario

La prevención de la isoinmunización anti-D se hará sólo si la madre no está sensibilizada, esto es,
si su test de Coombs indirecto es NEGATIVO. Respuesta 3 falsa.(R3)

350. Paciente varón de 20 años de edad que acude por ojo rojo doloroso. El problema
comenzó hace 5 días cuando tras bañarse en un lago con las lentillas puestas. Fue
valorado inicialmente por el médico de familia que le mando colirio de tobramicina. En
vista de la mala respuesta se pensó que pudiera tratarse de una queratitis herpética y ha
estado en tratamiento con aciclovir, sin que tampoco se objetivara mejoría. En esta
situación, señale el diagnóstico más probable:

1. 1. Queratitis bacteriana, resistente a tobramicina.


2. 2. Queratitis vírica resistente a aciclovir.
3. 3. Queratitis por toxoplasma.
4. 4. Queratitis por amebas.
Gráfico de respuestas
Comentario
El hecho de que el paciente sea usuario de lentes de contacto y haya nadado en agua dulce no
clorada, debe hacerte sospechar que haya podido sufrir una queratitis por amebas. Recuerda que
estas úlceras tienen muy mal pronóstico. A pesar de los tratamientos médicos, suele ser necesario
finalmente someter al paciente a una queratoplastia.(R4)

351. De entre los siguientes efectos secundarios a una insuficiencia renal crónica, señale
cuál tiende a persistir a pesar del tratamiento del cuadro:

1. 1. Acidosis metabólica.
2. 2. Hipocalcemia.
3. 3. Asterixis.
4. 4. Prurito.
Gráfico de respuestas
Comentario
La insuficiencia renal crónica la intentamos manejar con tratamiento conservador y cuando no es
posible con sustitutivo. El objetivo de esta pregunta es repasar el prurito (recuerda que ya fue
preguntado una vez en el examen MIR). El prurito es frecuente en la IRC, se debe a una elevación
de la PTH y a calcificaciones subcutáneas, recuerda que es una indicación relativa de instauración
de diálisis aunque no suele mejorar con la misma. Se suele realizar tratamiento sintomático con
antihistamínicos, cremas hidratantes, sedantes. En las formas graves pueden beneficiarse de la
paratiroidectomía. No olvides que la pericarditis urémica y la acidosis metabólica severa son
indicaciones absolutas de diálisis.(R4)

352. En relación con el derrame pleural de etiología tuberculosa, señale la respuesta


INCORRECTA:

1. 1. En niños y adultos jóvenes puede ser la manifestación de una primoinfección tuberculosa.


2. 2. Analíticamente cursa en forma de exudado de predominio mononuclear.
Se caracteriza por la presencia de cifras elevadas de ADA, interferón-gamma y amilasa
3. 3.
pleural.

 
 
 
 
4. 4. Suele constituir una forma de enfermedad poco contagiosa.
Gráfico de respuestas
Comentario

En efecto, la pleuritis tuberculosa puede constituir la forma de presentación de la primoinfección en


niños y adultos jóvenes. Suele ser unilateral, de comienzo brusco y habitualmente cursa con un
exudado de predominio linfocitario que, característicamente, presenta pobreza de células
mesoteliales, elevación de las cifras de interferón- gamma y de la isoenzima 2 de la
adenosindeaminasa (ADA, y cifras bajas de amilasa (respuesta 3 incorrecta). La presencia de
bacilos tuberculosos en el líquido pleural es poco frecuente, por lo que una baciloscopia negativa
no excluye el diagnóstico, que habitualmente debe realizarse mediante biopsia pleural (ya sea
ciega o dirigida mediante pleuroscopia). La prueba de la tuberculina puede ser negativa hasta en la
tercera parte de los casos. Si no se asocia a neumonía, la enfermedad es poco contagiosa, ya que
no existe contacto del bacilo con el exterior.(R3)

353. La tinción de calcoflúor sirve para visualizar:

1. 1. Estructuras fúngicas.
2. 2. Esporas de anaerobios.
3. 3. Micobacterias.
4. 4. Parásitos intestinales.
Gráfico de respuestas
Comentario

Pregunta muy difícil sobre un tema árido y muy poco rentable, cual es la microbiología. La tinción
del calcoflúor es una tinción específica para hongos. Este tipo de información sobre hongos no
tiene gran relevancia de cara al ENARM, es mucho más importante que domine los aspectos
clínicos básicos sobre hongos "rentables" como Candida, Aspergillus, Mucor, Criptococo o la
Malassezia furfur.(R1)

354. ¿En qué situación estaría contraindicada la fibrinólisis endovenosa para tratar un
infarto cerebral?

1. 1. Edad superior a 70 años.


2. 2. Historia de tratamiento hipotensor.
3. 3. Mejoría espontánea del déficit neurológico.
4. 4. TC cerebral normal.
Gráfico de respuestas
Comentario

Una pregunta que deberías acertar por sentido común. Evidentemente, si se produce una mejoría
espontánea del déficit neurológico, ya no sería necesario someter al paciente a los riesgos que
supone el tratamiento trombolítico. En la tabla de la pregunta 062, MIR 08-09, exponemos los
criterios de exclusión para el uso de este tipo de tratamiento, pero insistimos en que la lógica
muchas veces permite acertar preguntas aparentemente muy difíciles.(R3)

355. Los indicadores absolutos de la ovulación son:

1. 1. La temperatura corporal basal


2. 2. La excreción del pregnandiol
3. 3. El embarazo o la obtención del óvulo

 
 
 
 
4. 4. El seguimiento ovulatorio ecográfico
Gráfico de respuestas
Comentario

El único indicador absoluto de ovulación, es demostrar la rpesencia de un óvulo fuera del ovario,
que queda evidente si ocurre un embarazo o si se obtiene un óvulo de manera artificial.(R3)

356. Hombre de 67 años a quien se estudia por dolor abdominal y astenia leves. Se realizó
dentro del estudio un ultrasonido abdominal que demostró un nódulo de 3 cm en el
segmento VI hepático. Como consecuencia de ello, se solicitó un TC abdominal con
contraste que demostró la existencia de dicho nódulo de 3.2 cm hipovascular que realza
en fase portal. ¿Cuál es el diagnóstico más probable?:

1. 1. Adenoma hepático.
2. 2. Hepatocarcinoma.
3. 3. Angioma hepático.
4. 4. Metástasis de cáncer de colon.
Gráfico de respuestas
Comentario
Se trata de un nódulo hepático hipovascular y por tanto la lesión más probable es la metástasis de
cáncer de colon que tiene ese comportamiento. Tanto el adenoma hepático, el hepatocarcinoma, el
angioma hepático y la hiperplasia nodular focal tienen un comportamiento típicamente
hipervascular y realzan en la fase arterial.(R4)

357. Gestante de 36 años,


primigesta, de 33 semanas, en la que en el USG se diagnostica un CIR. Usted realiza un
Doppler en la arteria umbilical y observa los hallazgos que se muestran en la imagen.
Señale la CORRECTA:

1. 1. Indicaría nuevo control con Doppler en una semana.


2. 2. Finalizaría la gestación a las 37 semanas.
3. 3. Finalizaría la gestación.
4. 4. Realizaría una prueba de Pose.
Gráfico de respuestas

 
 
 
 
Comentario
Como se ve en la imagen, el flujo en la arteria umbilical es claramente patológico, con un flujo
reverso en la diástole. Es un signo ominoso que se asocia a muerte fetal en el 50% de los casos,
por lo que no esperaría otra semana ni hasta la semana 37, sino que finalizaría ya la gestación,
incluso a pesar de tratarse de una gestación de 33 semanas.

358. Masculino de 25 años de edad, sometido a resección intestinal por enfermedad de


Crohn, que comienza con fiebre inmediatamente después de la cirugía. La causa MENOS
probable de la fiebre es:

1. 1. Manipulación intraoperatoria de material purulento.


2. 2. Flebitis.
3. 3. Reacción transfusional.
4. 4. Fiebre medicamentosa.
Gráfico de respuestas
Comentario
Se trata de determinar la causa MENOS probable de fiebre en un postoperatorio de una cirugía
digestiva que aparece en el primer día de postoperatorio. Debemos recordar que entre las causa
más frecuentes de fiebre en el postoperatorio inmediato (horas después de la intervención) se
encuentran la existencia de una infección previa a la intervención, la manipulación de material
purulento durante la misma o la reacción a fármacos o hemoderivados utilizados durante la misma.
Sin embargo la infección de los catéteres venosos utilizados para el acceso vascular al paciente
durante la intervencióno la de los vasos en los que estos asientan no se produce hasta pasadas al
menos 24 - 72 horas de la misma.(R2)

359. ¿En qué casos haríamos un RCTG estresante (POSE)?

1. 1. Frecuencia cardiaca fetal de 115 lpm.


2. 2. No existen aceleraciones en la frecuencia cardiaca fetal.
3. 3. Si hay escasa variabilidad y deceleraciones.
4. 4. Frecuencia cardiaca basal de 170 lpm.
Gráfico de respuestas
Comentario
Se trata de una pregunta sencilla sobre el registro cardiotocográfico. Los parámetros de normalidad
son:
- Frecuencia cardíaca fetal entre 120- 160 latidos por minuto.
- Variabilidad de 10- 25 lat/min.
- Ascensos o aceleraciones transitorias por encima de 15- 20 lat/min.
- Ausencia de deceleraciones.
Cuando no existen aceleraciones de la frecuencia cardíaca fetal, cabe hablar de un registro
cardiotocográfico negativo. La causa más frecuente de esta situación es el sueño fetal. No
obstante, ante esta circunstancia estaría indicada la prueba de Pose (monitorización fetal
estresante), como siguiente paso.(R2)

360. Un paciente de 58 años, hipertenso, consulta por hiperglucemia detectada


casualmente en una analítica; el paciente tiene un aspecto facial tosco y aumento de
partes acras. El refiere crecimiento de pies y manos, pero dice que toda su familia es
parecida. Ante la alta sospecha clínica de acromegalia, ¿qué determinación analítica
elegiría como screening diagnóstico?

 
 
 
 
1. 1. Dos determinaciones basales de GH.
2. 2. Test de GHRH.
3. 3. Determinación de insulin growing factor tipo I (IGF-I).
4. 4. Sobrecarga oral de glucosa y determinación de IGF-I.
Gráfico de respuestas
Comentario

En los pacientes acromegálicos, los niveles de IGF- I se encuentran elevados respecto a los de los
de la población sana de la misma edad y sexo. La secreción pulsátil de la hormona de crecimiento
provoca que las determinaciones aisladas de GH no sean útiles como método de detección
sistemática de exceso de secreción de GH. Por lo tanto, la determinación más útil en el caso de
sospecha de acromegalia es la medición de niveles de IGF- I. El diagnóstico de confirmación se
realiza demostrando la ausencia de supresión de la GH trascurridas 1 o 2 horas de una sobrecarga
oral de glucosa (75 o 100 gramos). Se define como supresión unos niveles de GH mcg/L
determinados por RIA o una GH< 1mcg/dL cuando el método de determinación es el IRMA.

En esta pregunta, la respuesta correcta sería la 3. La respuesta 4 es incorrecta, porque lo que se


mide después de la sobrecarga de glucosa es la GH, no la IGF- I.(R3)

361. A 45-year-old male comes to your office with an intense pain with defecation followed
by small amounts of bright red blood in her stools. He has suffered from constipation for
many years, but currently it has worsened notably. His vital signs are within normality
and laboratory workup reveals no anemia. Physical examination gives the diagnosis.
Treatment is initiated with a topic nifedipine ointment. Warm sit baths, high-fiber diet and
stool softeners are recommended. After six months, symptoms have not disappeared.
Which is the gold standart treatment for anal fissures?

1. 1. Lateral sphincterotomy
2. 2. Endoscopic anal dilatation
3. 3. Shpincterectomy
4. 4. Fistulotomy
Gráfico de respuestas
Comentario
Lateral sphincterotomy(R1)

362. Un paciente que presenta vómitos biliosos de repetición no relacionados con dolor
abdominal, es diagnosticado de obstrucción parcial duodenal secundaria a páncreas
anular. En este caso, ¿cuál es el tratamiento más correcto?:

1. 1. Resección parcial del páncreas anular.


2. 2. Duodenoyeyunostomía.
3. 3. Intervención de Whipple (duodenopancreatectomía cefálica).
4. 4. Esofagoyeyunostomía.
Gráfico de respuestas
Comentario

Pregunta difícil y poco importante para el ENARM, ya que es una malformación poco frecuente que
hasta ahora no ha sido preguntada. El páncreas anular es un aro o collar anormal de tejido
pancreático que rodea al duodeno, causando una obstrucción intestinal a este nivel. Los síntomas
del páncreas anular son náuseas, vómitos, sensación de plenitud después de comer y problemas

 
 
 
 
de alimentación en los recién nacidos. El tratamiento usual para este trastorno es la cirugía de
derivación del segmento obstruido en el duodeno mediante una duodenoyeyunostomia.(R2)

363. NO entra en la definición de síndrome de Conn:

1. 1. Tamaño pequeño.
2. 2. Más frecuente en mujeres de mediana edad.
3. 3. Frecuencia similar en ambos lados.
4. 4. Aldosterona plasmática y actividad de renina elevadas.
Gráfico de respuestas
Comentario
Esta pregunta hace referencia a una de las posibles causas de hiperaldosteronismo primario. La
causa más frecuente de esta patología es la existencia de un adenoma unilateral secretor de
aldosterona (síndrome de Conn). Estos adenomas suelen tener un pequeño tamaño,
presentándose con la misma frecuencia en ambas suprarrenales, siendo mñas habitual en mujeres
de mediana edad. Dado que es causa de hiperaldosteronismo primario, encontraremos en plasma
cifras de aldosterona elevadas, con una actividad de renina plasmática suprimida. El tratamiento de
elección del síndrome de Conn consiste en la realización de una suprarrenalectomia unilateral.(R4)

364. Con respecto a la endometriosis señale la respuesta INCORRECTA.

1. 1. La severidad del dolor pélvico se correlaciona con la cantidad de focos endometriosicos.


2. 2. El ultrasonido no es muy útil para detectar la mayoría de lesiones.
De acuerdo a la teoría de Sampson, la regurgitación de células endometriales por las
3. 3.
trompas es la causa de la enfermedad.
4. 4. La endometriosis es causa de infertilidad.
Gráfico de respuestas
Comentario

La severidad del dolor pélvico no se correlaciona con la cantidad de implantes endometriósicos,


por lo que la opción correcta es la 1. Generalmente el ultrasonido es una prueba muy útil para
detectar la mayoría de las lesiones. Existen varias teorías que explican la génesis de esta
enfermedad, y una de ellas es la regurgitación de células endometriales por las trompas. Y entre
los síntomas de endometriosis se encuentra la infertilidad.(R1)

365. En una laparotomía por un tumor ovárico, encontramos afectación de ambos


ovarios. La biopsia intraoperatoria informa de la existencia de numerosas células en
anillo de sello, con lo que se confirma su naturaleza metastásica. ¿Dónde buscaría usted
el tumor primario?:

1. 1. En el riñón
2. 2. En el pulmón.
3. 3. En el tubo digestivo.
4. 4. En las glándulas salivares.
Gráfico de respuestas
Comentario

Lo que nos describe este caso clínico es un tumor de Krukenberg. Acerca del mismo, debe
recordar las siguientes características:

 
 
 
 
- Origen más frecuente: estómago, aunque puede proceder de otras regiones del tubo
digestivo.

- Histología: células en anillo de sello.

- Llegada al ovario por vía hematógena, es decir, se trata de una metástasis.

(R3)

366. RN de madre 0- y padre A+ con embarazo no controlado con respecto a los cuadros
clínicos que puede presentar el neonato, señale la opción FALSA.

Si el Coombs indirecto para la incompatibilidad Rh es negativo, se debe realizar profilaxis


1. 1. con gammaglobulina hacia la semana 28 de gestación y en las primeras 72 horas postparto,
si el RN es Rh+.
2. 2. Puede ser necesaria la transfusión intrauterina de hematíes si la hemolisis es intensa.
3. 3. En la incompatibilidad de grupo puede haber afectación del primer hijo.
Si el Coombs directo es positivo frente al Ag Rh, es imprescindible administrar
4. 4.
gammaglobulina a la madre.
Gráfico de respuestas
Comentario

La profilaxis de la isoimmunización por Rh se administra hacia las 28 semanas de gestación (la


primera dosis), si la madre es Rh negativa y se sospecha que el feto pueda ser Rh positivo siempre
y cuando la madre no haya producido ya anticuerpos contra el Rh +, puesto que si ya los ha
producido la profilaxos es inútil.

Si al nacer se confirma el Rh positivo del RN, se debe administrar una segunda dosis a la madre
antes de las 72 horas para evitar isoimmunización en embarazo posteriores.(R4)

 
 
 
 

367. Niño de 25 días que presenta


vómitos desde hace 3 días que han ido en aumento hasta vomitar todas las tomas. No
presenta fiebre ni otra sintomatología y tiene avidez por las tomas. En urgencias la
exploración es normal. Se decide mantener en observación donde se observan vómitos
proyectivos después de la toma, por lo que se decide realizar radiografía ante la
imposibilidad de hacer ultrasonido, observando la imagen que se adjunta. ¿Cuál sería la
asociación más correcta?

1. 1. Imagen de doble burbuja: atresia duodenal.


2. 2. Ausencia de aire en esófago: atresia esofágica.
3. 3. Radiografía normal: gastroenteritis aguda.
4. 4. Distensión gástrica: estenosis hipertrófica de píloro.
Gráfico de respuestas
Comentario

Ante un cuadro de vómitos en un neonato alrededor de las 3 semanas de vida, característicamente


proyectivos y que aumentan progresivamente, aunque el niño mantiene la avidez por las tomas,
debemos descartar la estenosis hipertrófica de píloro.

Ante la sospecha, se debe realizar un ultrasonido abdominal que valore el tamaño y grosor del
píloro. En el caso de no tener disponible el ultrasonido, se puede realizar una radiografía de
abdomen, donde observaremos una distensión gástrica con algo de aire distal, pero escaso, típico
de la estenosis de píloro.

Debemos distinguir esta imagen de la imagen en doble burbuja de la atresia duodenal y de la


imagen en burbuja única en el caso de la atresia de píloro, cuadros que darían sintomatología más
precoz y brusca.(R4)

 
 
 
 
368. Se realiza exámenes de sangre, además de una gasometría para valorar el equilibrio
hidroelectrolítico. ¿Cuál de las siguientes situaciones es más característica de este
cuadro?

1. 1. Alcalosis hipoclorémica e hipokaliémica.


2. 2. Acidosis hipoclorémica e hipokaliémica.
3. 3. Acidosis hiperclorémica e hipokaliémica.
4. 4. Alcalosis hipoclorémica e hiperkaliémica.
Gráfico de respuestas
Comentario
Los vómitos van a producir una pérdida de hidrogeniones y de cloro que van a desencadenar una
alcalosis hipoclorémica, típica de los cuadros con vómitos. El potasio, lo encontraremos bajo
debido a que el potasio se va a intercambiar con los hidrogeniones del interior de las células para
tratar de compensar la alcalosis, por lo que se va a observar una bajada de los niveles de potasio
asociada.(R1)

369. Señale la respuesta CORRECTA respecto de la inclusión de ritonavir en las pautas


de tratamiento frente a VIH.

Actualmente se recomienda como uno de los fármacos de primera elección para el


1. 1. tratamiento de esta infección junto con dos inhibidores de la transcriptasa inversa análogos
de los nucleósidos, siempre que se administre a altas dosis.
Se debe emplear a dosis bajas en combinación con cualquier otro inhibidor de la proteasa,
2. 2.
ya que se pretende que actúe como inhibidor enzimático.
Se trata de un potente inductor enzimático con el que se pretende evitar efectos adversos
3. 3.
de los otros fármacos antirretrovirales que esté recibiendo el paciente.
Sólo está indicado como tratamiento coadyuvante en pacientes que estén recibiendo
4. 4.
tratamiento con un inhibidor de la integrasa como el raltegravir.
Gráfico de respuestas
Comentario
Ritonavir es un inhibidor de la proteasa que actualmente se emplea como inhibidor enzimático.
Este fármaco inhibe de manera muy potente al citocromo p450, que es el enzima hepático a través
del cual se metabolizan los inhibidores de la proteasa. Actualmente se recomienda que cualquier
paciente con infecciónpor VIH que esté siendo tratado con un inhibidor de la proteasa, reciba
también dosis bajas de ritonavir, de modo que al disminuir la destrucción del inhibidor de la
proteasa con el que se combina, se “potencia” su acción terapéutica. Esta estrategia se conoce
como “tratamiento potenciado” de los inhibidores de la proteasa.(R2)

370. A 51-year-old man presents to the emergency department complaining of a 3-week


history of productive cough, fever, and night sweats. His past medical history is
significant for long-standing asthma and psoriasis. He claims that his sputum is
yellowish. His medications include chronic corticosteroid therapy, methotrexate and
inhaled beta-adrenergics. His temperature is 39.0 ºC, pulse is 90/min, respirations are
15/min, and blood pressure is 125/75 mmHg. Scattered rales in his right lower chest are
revealed on auscultation. Chest X-ray shows a right lower lobe cavitary lesion with diffuse
infiltrate. Sputums cultures yield partially acid-fast, gram-positive branching rods. Which
of the following is the most appropriate treatment?

1. 1. Amphotericin B.
2. 2. Penicillin G.

 
 
 
 
3. 3. Trimethoprim-sulfamethoxazole.
4. 4. Linezolid and a third-generation cephalosporin.
Gráfico de respuestas
Comentario
Trimethoprim-sulfamethoxazole. The treatment of choice for nocardiosis is cotrimoxazole (a mixture
of the drugs sulphamethoxazole and trimethoprim). You have to suspect nocardiosis in this patient,
because of the chronic use of corticoteroids and the data given in the rest of the case description.
Nocardia spp is a gram-positive, partially acid-fast, filamentous aerobe. Differential diagnosis
include tuberculosis or Actinomyces infection, since the most common symptoms include weight
loss, fever and night sweats. When disseminated, the setting of simultaneous pumonary and
cerebral involvement is very typical (lung and brain abscesses). It usually occurs in
immunocompromised patients.(R3)

371. Una de las siguientes aseveraciones sobre el ganglio centinela en el cáncer de mama
es CORRECTA:

1. 1. El ganglio centinela es el primer ganglio que recibe el drenaje linfático del tumor primario.
2. 2. El ganglio centinela es el primer ganglio metastatizado.
La existencia de una adenopatía axilar palpable en una paciente con un tumor de 3 cm de
3. 3.
diámetro no se considera una contraindicación a la técnica de detección.
Si durante la identificación del ganglio centinela solo capta la cadena mamaria interna se
4. 4.
considerará como prueba fallida.
Gráfico de respuestas
Comentario

Una pregunta conceptual, muy sencilla. Se conoce como ganglio centinela al primer ganglio de una
cadena linfática en el que drena un territorio determinado. De este modo, las células procedentes
de una lesión cancerosa pasarían inicialmente a su través, antes de alcanzar otros ganglios. De
ahí el interés de localizarlo, para realizar una biopsia de forma selectiva y, en función del resultado,
proceder a la linfadenectomía completa del resto de la zona.(R1)

372. Secundigesta de
28 años, con un parto eutócico previo, gestante de 39 semanas. El embarazo ha cursado
con normalidad. Durante el periodo de dilatación se obtiene el registro que se muestra
en la imagen. Condiciones obstétricas: cefálica, líquido amniótico claro, 5 cm, II plano de
Hodge. ¿Cuál será la conducta a seguir?:

1. 1. Cesárea.

 
 
 
 
2. 2. Realizar microtoma fetal.
3. 3. Administrar betamiméticos.
4. 4. Retirar goteo oxitocina.
Gráfico de respuestas
Comentario

Una pregunta bastante difícil, puesto que dependemos mucho de nuestra capacidad para
interpretar la imagen.

Para empezar, debe darse cuenta de que se trata de un registro cardiotocográfico fetal. Lo
fundamental es que sepa distinguir entre las DIPS tipo I y tipo II. En las tipo I, la bradicardia fetal
ocurre exactamente en el mismo instante que la contracción uterina, sin ningún decalaje temporal
entre ellas. Por el contrario, en el tipo II se produce primero la contracción, y varios segundos
después el descenso de la frecuencia cardíaca.

En este caso, observe que no existe una coincidencia temporal perfecta. Esto se aprecia muy
claramente en el centro de la imagen, un poco a la derecha, donde existe un claro decalaje entre la
contracción uterina (línea negra) y el descenso de la frecuencia cardíaca (línea roja). Estamos, por
tanto, ante un registro con DIPS tipo II, que suelen relacionarse con mayor o menor grado de
sufrimiento fetal. Esto nos obliga a medir el pH mediante una microtoma (respuesta 2 correcta), lo
que nos permitirá definir la actitud siguiente. Recuerde que los valores normales del pH fetal están
entre 7.25 y 7.45. Cuando se encuentran por debajo de 7.20, debemos poner fin a la gestación
inmediatamente, y entre 7.20- 7.25 podríamos adoptar medidas conservadoras (oxígeno, decúbito
lateral izquierdo, etc.) y repetir la microtoma en quince minutos.(R2)

373. Un paciente inmunodeficiente que presenta en la radiografía de tórax/TC una


neumonía con el signo del halo menisco o contorno semilunar, sugiere infección por:

1. 1. Staphylococcus aureus.
2. 2. Streptococcus pneumoniae.
3. 3. Pseudomonas aeruginosa.
4. 4. Aspergillus fumigatus.
Gráfico de respuestas
Comentario

Una pregunta fácil, porque nos dan un dato muy característico: el “signo del halo menisco” o “halo
en semiluna”. Este signo, en el parénquima pulmonar de un paciente inmunodeprimido, sugiere la
colonización por Aspergillus de una caverna preexistente, también llamado aspergiloma (respuesta
5 correcta).

Se trata de formas no invasivas pulmonares. La caverna preexistente puede ser una caverna
tuberculosa, un quiste pulmonar o bronquiectasias. Al ser formas benignas, el tratamiento sería la
observación, y cirugía sólo en caso de hemoptisis grave.

Las respuestas 1, 2 y 4 podrían dar neumonías cavitadas, pero el enunciado habla de un signo
radiológico característico, no de cavitación.(R4)

374. A 31-year-old male patient presents with orogenital ulcerations along with erythema
nodosum and arthritis affecting his knees and ankles. Which of the following additional
findings is the LEAST likely?

 
 
 
 
1. 1. Spondylitis.
2. 2. Lymphocytic chronic meningitis.
3. 3. Deep venous thrombosis.
4. 4. Posterior uveitis.
Gráfico de respuestas
Comentario

En un paciente joven con aftas orales y genitales de repetición debe sospechar de entrada una
enfermedad de Behçet. En este paciente se cumplen criterios para esta enfermedad, ya que
presenta el criterio imprescindible, que son la úlceras orales y además otros dos criterios de entre
los cuatro que se requieren (aftas genitales, lesiones cutáneas tipo eritema nodoso o
pseudofoliculitis, test de patergia positivo y alteraciones oculares como uveítis anterior, posterior o
vasculitis retiniana). Además de esta clínica, los pacientes con esta enfermedad pueden presentar
artritis de grandes articulaciones no erosiva, episodios de trombosis venosa superficial o profunda,
lesiones del tracto digestivo y afectación neurológica en forma de meningitis linfocitaria,
hipertensión intracraneal, demencia, síntomas piramidales...etc.(R1)

375. ¿Cuál es el índice más empleado para establecer la gravedad de la EPOC?.

1. 1. Grado de disnea.
2. 2. Índice de masa corporal.
3. 3. Volumen espiratorio forzado en el primer segundo (FEV1).
4. 4. Distancia recorrida en la prueba de la marcha de 6 minutos.
Gráfico de respuestas
Comentario
Aunque todos los mencionados pueden ser utilizados, el más ampliamente usado, debido a su
facilidad de medición y a su reproducibilidad, es el FEV1.(R3)

376. ¿Cuál de las siguientes NO es una prueba que mida la insuficiencia hepatocelular?

1. 1. Medida de la albúmina.
2. 2. Cuantificación de la amoniemia.
3. 3. Aclaramiento de ácidos biliares.
4. 4. Medida de las transaminasas.
Gráfico de respuestas
Comentario

La medida de las transaminasas no sigue una correlación con la función desarrollada por los
hepatocitos. De hecho la clasificación de Child- Pugh no lo utiliza. La elevación de estas enzimas
traduce la lisis de los hepatocitos, como podría ocurrir en una hepatitis aguda, por ejemplo.

El hepatocito se encarga de la síntesis de proteínas y por ello se determinan los niveles de


albúmina (proteína más abundante en el plasma). También se ocupa del aclaramiento de sales
biliares hacia la bilis, y de la síntesis de factores de coagulación (midiéndose el tiempo de
protombina como expresión de la síntesis de los mismos). La cuantificación de la amoniemia es la
expresión de la capacidad de depuración de productos nitrogenados absorbidos con la dieta. Por
tanto, también refleja su buen o mal funcionamiento.(R4)

377. Varón de 48 años con un infarto inferior de 24 horas de evolución, que presenta
disnea progresiva, ortopnea de dos almohadas y oliguria desde las últimas 6 horas,

 
 
 
 
evidenciando una TA de 100/70 mmHg, crepitantes en tercio inferior de ambas bases y
un soplo sistólico en ápex irradiado a axila, con tercer ruido y cuarto ruido. Presenta unos
gases basales con pO2 48 mmHg, pCO2 23 mmHg, HCO3 de 13 mmHg y pH 7,4. En la
radiografía de tórax presenta líneas B de Kerley y un infiltrado en lóbulo superior derecho.
¿Cuál es su diagnóstico?:

1. 1. Shock séptico por neumonía del lóbulo superior derecho.


2. 2. Tromboembolismo pulmonar.
3. 3. Insuficiencia mitral aguda por necrosis del músculo papilar.
4. 4. Comunicación interventricular.
Gráfico de respuestas
Comentario

La rotura del músculo papilar es una complicación grave de los IAM, porque provoca insuficiencia
mitral aguda, generalmente severa. Al presentarse bruscamente, no da tiempo a dilatar la aurícula
con lo que se elevan rápidamente las presiones de llenado (presión capilar pulmonar, o presión
telediastólica del ventrículo izquierdo) y se produce edema agudo de pulmón. En ocasiones, el
edema pulmonar se localiza en un solo lóbulo (a menudo el superior derecho), porque el jet de
insuficiencia mitral se dirige hacia las venas pulmonares de ese lóbulo y eleva la presión más
intensamente en esa zona. El tratamiento consiste en el empleo de vasodilatadores para tratar de
disminuir las resistencias sistémicas y, por tanto, disminuir el grado de regurgitación. El empleo de
balón de contrapulsación aórtico es útil, ya que también disminuye las resistencias sistémicas. Los
vasoconstrictores arteriales como la noradrenalina estarían contraindicados, ya que aumentarían el
grado de regurgitación, al aumentar la postcarga. El tratamiento definitivo es la cirugía correctora
mitral que, si se puede diferir 10 o 12 días, presenta mejores resultados. No obstante, muchos
pacientes requieren cirugía precoz.(R3)

378. Señale cuál de los siguientes enunciados sobre el test de Apgar es correcto:

Se realiza en los minutos 1 y 5 y si es menor de 9 se realiza cada 5 minutos hasta que sea 9
1. 1.
o 10.
2. 2. Si encontramos un Apgar menor de 3 a los 20 minutos predice mayor morbimortalidad.
3. 3. Si la frecuencia cardiaca es menor de 140 suma un punto
4. 4. La posición del neonato en semiflexión suma un punto.
Gráfico de respuestas
Comentario

El test de Apgar sirve para evaluar el grado de depresión respiratoria y hemodinámica en el


neonato, se realiza a los 1 y 5 minutos del nacimiento y se repite cada 5 minutos si la puntuación a
los 5 minutos es menor de 7.

No tiene valor pronóstico pero a los 20 minutos un Apgar menor de 3 predice una mayor
morbimortalidad.

La frecuencia cardiaca se evalúa sumando 1 punto si es menor de 100 y 2 puntos si es mayor de


100, pero la frecuencia respiratoria no se evalúa, sino el esfuerzo respiratorio (0 si ausente, 1 si
lento e irregular 2 si llanto y bueno).

Respecto al tono muscular: una leve flexión de extremidades sumará un punto mientras que la
hipotonía sumará 0 puntos y los movimientos activos sumarán 2 puntos.(R2)

 
 
 
 
379. Mujer de 30 años, fumadora de 1 paquete/día, alérgica a AINEs, que, como
antecedentes patológicos, refiere una apendicectomía hace 10 años, asma y poliposis
rinosinusal. En la actualidad sigue tratamiento con anticonceptivos orales y
antihistamínicos. Acude a su médico de cabecera pues desde hace 3 semanas presenta
lesiones nodulares, dolorosas a la presión, de coloración eritematoviolácea y localizadas
en la cara anterior de las piernas. No presenta afectación del estado general. Señale la
FALSA en relación con su patología:

1. 1. Debemos iniciar tratamiento con reposo, yoduro potásico y diclofenaco sódico.


2. 2. La histología revelará una paniculitis septal sin vasculitis.
Es conveniente la realización de una radiografía de tórax independientemente de que tenga
3. 3.
o no sintomatología respiratoria.
4. 4. El Mantoux y el ASLO son pruebas que pueden orientarnos sobre la etiología del cuadro.
Gráfico de respuestas
Comentario

Esta pregunta es bastante compleja, y tiene varios aspectos interesantes. Como antecedentes, la
paciente tiene asma, poliposis rinosinusal y alergia a los AINE (estos datos conforman la tríada
ASA). El cuadro que presenta en la actualidad, desde hace 3 semanas, es concordante con un
eritema nodoso, que es una paniculitis septal sin vasculitis; esta enfermedad puede asociarse a
otras, como una infección estreptocócica, una tuberculosis o una infección por Mycoplasma, por lo
que pueden resultr útiles el Mantoux, los ASLO y la radiografía de tóraz. Además de las lesiones
nodulares, es frecuente que haya fiebre y mal estado general. El tratamiento estándar se basa en
reposo, yoduro potásico o AINE (pero en este caso, la paciente es alérgica a AINE, por lo que no
puede utilizarse el diclofenaco sódico, opción 1 incorrecta).(R1)

380. A 35-year-old man is brought to the ER by the police, due to agitated behavior in the
streets. His past medical history is irrelevant. He was found at the top of a tree, shouting
at everyone that they must convert to "Jamism" or otherwise they would die. When asked
about this statement, he refers that God has sent him to create a new religion, that will
help everyone become better people. Which of the following best defines his thought
content?

1. 1. Hallucination
2. 2. Grandiose delusion
3. 3. Illusion
4. 4. Perseveration
Gráfico de respuestas
Comentario
Grandiose delusion. This patient is suffering from delusions, fixed, false beliefs that can be
classified into grandiose, paranoid or somatic. This patient's belief that he has God-given missions
would be considered a classic example of grandiose delusion, which are common during manic
episodes.(R2)

381. La amigdalectomía está indicada en una de las siguientes patologías:

1. 1. Amigdalitis aguda no complicada.


2. 2. Angina de Vincent.
3. 3. Mononucleosis infecciosa.
4. 4. Abscesos periamigdalinos de repetición.

 
 
 
 
Gráfico de respuestas
Comentario

Un primer absceso periamigdalar no constituye una indicación de amigdalectomía, pero sí los


abscesos de repetición (respuesta 5 correcta).

La angina agranulocítica de Schultze se produce en el contexto de la neutropenia, a veces inducida


por fármacos como antitiroideos. Es más frecuente en pacientes de edad avanzada. Se caracteriza
por síntomas sistémicos de gravedad, ulceraciones y necrosis de las amígdalas, así como dolor
severo. El tratamiento se realiza con penicilina más transfusión sanguínea.

La amigdalitis bacteriana aguda no complicada se trata con penicilina. Por supuesto, un episodio
aislado de amigdalitis aguda sin complicaciones no requiere tratamiento quirúrgico.

La angina de Vincent, producida por una asociación fusoespirilar, es de buen pronóstico y se trata
con penicilina (S. pyogenes).

La mononucleosis infecciosa, producida por el virus de Epstein- Barr, tiene tratamiento sintomático,
y en algunas ocasiones de afectación faríngea muy severa, se pueden asociar antibióticos
anaerobicidas.(R4)

382. A los diez días de iniciar por primera vez la toma de un anovulatorio de baja dosis,
una paciente de 24 años comienza con metrorragia escasa (spotting). Se produce:

1. 1. En muchos casos; no es preocupante.


2. 2. Por falta de progestágenos.
3. 3. Por exceso de progestágenos.
4. 4. Por fallo en la administración.
Gráfico de respuestas
Comentario

Los anticonceptivos orales más recientes contienen baja dosis de estrógenos para disminuir la
incidencia de efectos adversos. Aunque esta dosis sea suficiente para mantener el efecto
anovulatorio y anticonceptivo, en ocasiones no es suficiente para inducir un adecuado crecimiento
del endometrio el cual se descama parcialmente por disrupción, apareciendo sangrado
intermenstrual escaso (spotting). En otras ocasiones, la dosis de estrógenos no es capaz de
estimular el endometrio quedando éste atrófico y en amenorrea. Ambos efectos adversos son
relativamente frecuentes y no son preocupantes. Ambos se tratan aumentando la dosis de
estrógenos que contiene el preparado.(R1)

383. Uno de los siguientes NO es criterio de exclusión del programa de mantenimiento


con antagonistas opiáceos:

1. 1. Embarazadas.
2. 2. Consumo esporádico de heroína, cannabis y LSD.
3. 3. Esquizofrenia en tratamiento con clorpromacina.
4. 4. Lactantes.
Gráfico de respuestas
Comentario
Es una pregunta sencilla y de sentido común sobre los programas de rehabilitación del consumo
de drogas. El

 
 
 
 
tratamiento con antagonistas opiáceos se hace con naltrexona, no está exento de riesgos y nos
debemos asegurar que el paciente ya no consume está desintoxicado por se considera un
programa de " alta exigencia", y recuerda que la eficacia es escasa. La opción 2 no es criterio de
exclusión, más bien es su indicación principal. Todas las demás opciones sí se consideran criterios
de exclusión. Manual CTO 4ª Edición, Psiquiatría, Tema 5, Pág. 38(R2)

384. Un varón de 48 años y con antecedentes de hepatitis C (Ag HVC+) es enviado al


Servicio de Nefrología por: edemas maleolares, proteinuria 3.9 gr/día, creatinina 1.4
mg/dl, 6-8 hematíes/campo. ¿Qué proceso glomerular se encontrará en la biopsia renal?:

1. 1. Mínimos cambios.
2. 2. Glomerulonefritis focal y esclerosante.
3. 3. GN membranoproliferativa.
4. 4. Crioglobulinemia.
Gráfico de respuestas
Comentario
Caso clínico de dificultad moderada en el que nos piden hacer el diagnóstico de un paciente con
antecedentes de hepatitis C, proteinuria importante y leve hematuria e insuficiencia renal. Si has
fallado esta pregunta, a partir de ahora no olvides que para el MIR, la GN asociada al VHC es la
GNMP (RESPUESTA 3 CORRECTA), que cursa con síndrome nefótico en el 66% de los casos,
pudiéndose añadir microhematuria y fallo renal. Las etiologías que debes recordar de la GNMP (y
que tienes en el Manual) son: las enfermedades autoinmunes (LES, sarcoidosis, etc.), las
infecciones (VHC, VHB, endocarditis, parásitos, VIH, etc.), tumores (mieloma, leucemia linfática
crónica, linfoma no hodgkin, etc.)(R3)

385. Los siguientes fármacos producen reacciones adversas en el ojo, si bien uno de
ellos produce de forma típica neuritis óptica. Señálelo:

1. 1. Antidepresivos tricíclicos.
2. 2. Corticoides.
3. 3. Cloroquina.
4. 4. Etambutol.
Gráfico de respuestas
Comentario
Muchos fármacos pueden producir efectos secundarios a nivel ocular. La tioridacina, y en general
los antipsicóticos, pueden producir por un efecto acumulativo cuadros similares a la retinosis
pigmetaria. Los antidpresivos tricíclicos por su efecto anticolinérgico, pueden desencadenar en
pacientes hipermétropes un episodio de glaucoma agudo de ángulo estrecho. Los corticoides
pueden subir la presión intraocular y producir catarata. La cloroquina da lugar en ocasiones a la
típica maculopatía en ojo de buey. El etambutol puede producir cuadros de neuropatía óptica,
habitualmente retrobulbar, y por ello la respuesta correcta es la última.(R4)

386. Paciente masculino de 64 años de edad con antecedentes de HTA y cardiopatía


isquémica acude a urgencias por aparición de dolor torácico opresivo mientras veía la
televisión. En la consulta de clasificación (Triaje) de urgencias se detectan cifras de
presión arterial 155/95 mmHg y una saturación capilar de O2 de 95%, ¿cuál es la conducta
más CORRECTA entre las que a continuación se citan?

 
 
 
 
1. 1. Indicarle que vaya a la sala de espera. Le avisarán para la realización de pruebas.
2. 2. Este paciente debe ser atendido en el box de paros.
3. 3. Desde la consulta de Triaje se le remitirá a la unidad coronaria.
4. 4. Haremos un ECG en menos de 10 minutos.
Gráfico de respuestas
Comentario

Pregunta evidente que no se puede fallar. La correcta es claramente la 4. Todo paciente que entre
por la puerta de urgencias con dolor torácico tiene que tener un ECG en menos de 10 minutos.(R4)

387. Producto de un accidente masivo de carretera se presentan las siguientes victimas;


cual de ellos NO tiene indicación de lavado peritoneal diagnóstico:

1. 1. Mujer de 18 años, con traumatismo abdominal penetrante.


Paciente masculino de 37 años, con Glasgow de 9 puntos, Hemodinámicamente inestable,
2. 2.
sin evidencia de lesión hemorrágica externa.
Masculino de 45 años, hemodinámicamente inestable, con traumatismo torácico cerrado y
3. 3.
fractura de tibia y peroné derecho.
4. 4. Todos requieren lavado peritoneal diagnóstico
Gráfico de respuestas
Comentario

Los pacientes de las opciones 2 y 3 son pacientes inestables, por lo que estaría indicado realizar
un LP, mientras que en el caso 1 nos hablan de un traumatismo penetrante, por lo que la
indicación es de laparotomía. Repase el siguiente algoritmo.(R1)

388. Masculino de 79 años se detecta un carcinoma prostático con afectación únicamente


de un lóbulo y sin diseminación a distancia. El paciente presenta una hepatopatía
alcohólica además de un EPOC severa. Señale la respuesta INCORRECTA:

Por ser un estadio localizado, la prostatectomía radical en este paciente es la opción de


1. 1.
elección.
2. 2. La radioterapia podría ser una opción de tratamiento
3. 3. Por sus condicionantes médicos, puede ser preferible la orquiectomía.
4. 4. Es posible que su hepato y neumopatía condicionen su pronóstico vital más que su tumor.
Gráfico de respuestas
Comentario

Pregunta que no es fácil... Nos presentan el caso de un varón de 79 años con cáncer de próstata
aparentemente localizado (T2A). Nos preguntan sobres nuestra actitud. La duda surge si con 79

 
 
 
 
años tiene sentido plantear un tratamiento con intención radical (ya que aparentemente el tumor
parece organoconfinado) o se podría tomar una actitud expectante debido a lento crecimiento
tumoral. La opción de la cirugía (1) no parece aceptable, lo primero por la edad del paciente y
después por el riesgo quirúrgico asociado a su comorbilidad. Es una buena pregunta para
plantearse muchas cosas en relación al cáncer de próstata.(R1)

389. En el estudio gasométrico de la fibrosis pulmonar la alteración más habitual es:

1. 1. Descenso del PaO2 en reposo.


2. 2. Descenso del PaO2 tras esfuerzo.
3. 3. Aumento del PaO2.
4. 4. Descenso del pH.
Gráfico de respuestas
Comentario
En los pacientes con fibrosis pulmonar idiopática, la hipoxemia se debe a la disminución de la
capacidad de difusión por engrosamiento de la membrana alveolocapilar. En condiciones
fisiológicas, el oxígeno sólo requiere aproximadamente el 30% del tiempo de paso de los hematíes
por la unidad alveolo capilar para producirse el intercambio. Esto explica que los pacientes con
fibrosis tengan hipoxemia leve en reposo que se hace grave con el esfuerzo. Es característica la
normo o hipocapnia, que produce alcalosis respiratoria (aumento del pH) y una acidosis metabólica
compensadora (disminución del bicarbonato).(R2)

390. A 9-year-old patient is brought to the doctor's office by her worried father because
she is having trouble completing her homework assignments and she keeps losing
things. She does not seem to listen when she is being talked to. She is very impatient and
interrupts conversations or other people's activities. What is the most likely diagnosis?

1. 1. Borderline personality disorder.


2. 2. Paranoid schizophrenia.
3. 3. Asperger's syndrome.
4. 4. Attention deficit hyperactivity disorder.
Gráfico de respuestas
Comentario
Attention deficit hyperactivity disorder. Attention deficit hyperactivity disorder is a
neurodevelopmental psychiatric disorder in which there are significant problems with executive
functions (e.g., attentional control and inhibitory control) that cause attention deficits, hyperactivity
or impulsiveness. Schizophrenia is a disorder with impaired perception of reality and children
affected by Asperger's syndrome show difficulties in social interaction.(R4)

391. Todos los siguientes cuadros se incluyen clínicamente dentro de los estados
paranoides (o trastornos delirantes), EXCEPTO:

1. 1. Celotipia.
2. 2. Psicosis hipocondríaca monosintomática.
3. 3. Delirio querulante.
4. 4. Síndrome de Briquet.
Gráfico de respuestas
Comentario

 
 
 
 
Cinco son los tipos de delirios que incluye la DSM en su descripción de los trastornos delirantes
crónicos:

1) Delirios de persecución y perjuicio: los más frecuentes.

2) Delirios de celos: a veces llamado síndrome de Otelo; asociados en ocasiones al alcoholismo


(pero no de forma tan frecuente como antes se pensaba).

3) Delirios de grandeza: como Don Quijote; el paciente defiende tener unos derechos sobre títulos,
tierras o patentes; pone frecuentes denuncias y se muestra muy querulante.

4) Delirios de enfermedad: llamados a veces psicosis hipocondríacas monosintomáticas; destacan


los delirios de parasitosis o síndrome de Ekbom.

5) Delirios de enamoramiento: llamado en ocasiones síndrome de Clerambault.

El síndrome de Briquet es el epónimo para el trastorno por somatización, una variante de los
trastornos somatomorfos, en la que los pacientes presentan múltiples somatizaciones de curso
crónico.(R4)

392. ¿Cuál de los siguientes NO es criterio de Stegen y Toledo para el diagnostico de tuberculosis
infantil?

1. 1. Cuadro clínico.
2. 2. Test de ADA positivo.
3. 3. Radiografía sugestiva.
4. 4. Antecedente epidemiológico de contacto.
Gráfico de respuestas
Comentario
(R2)

 
 
 
 

 
 
 
 

393. Un paciente de 52 años


acude al Servicio de Urgencias aquejado de dolor en flanco derecho, náuseas, vómitos y
sensación febril de aparición brusca 4 horas antes. Tras una exploración anodina salvo
por el dolor se realiza una Rx simple de abdomen. Ante dicha imagen es falso que:

El tratamiento de esta patología es sintomático y no requerirá posteriores intervenciones,


1. 1.
siempre que no haya complicaciones.
El tratamiento de esta patología es sintomático pero posteriormente requerirá un
2. 2.
tratamiento definitivo, siempre que no haya complicaciones.
3. 3. Entre sus posibles complicaciones se encuentra el FRA.
4. 4. El paciente no precisa tratamiento urgente en el momento actual.
Gráfico de respuestas
Comentario

Una pregunta difícil, pues dependemos mucho de nuestra habilidad para juzgar correctamente la
radiografía de abdomen. Un dolor en hipocondrio derecho con náuseas y vómitos podría
corresponder a un cólico nefrítico, a un cólico biliar, etc. Sin embargo, en este caso vemos una
imagen radioopaca (densidad calcio) en hemiabdomen derecho, aproximadamente a la altura de
las siluetas renales. Teniendo esto en cuenta, posiblemente se trate de un cólico nefrítico por una
litiasis renal.

La respuesta incorrecta sería la 1. Es verdad que el tratamiento del cólico nefrítico es sintomático,
pero ante una imagen como la que nos muestran, es bastante probable que requiera algún tipo de
intervención posterior para eliminar un cálculo de este tamaño.(R1)

 
 
 
 
394. Tras la vuelta de la sala de rayos el paciente se encuentra sudoroso y comienza con
un cuadro de hipotensión acompañado de fiebre de 39 ºC. En la analítica se observan
2.300 leucos y una disminución en la cifra de plaquetas. En este momento:

1. 1. Tras estabilizar al paciente debería realizarse una derivación con catéter.


2. 2. Tras estabilizar al paciente debería colocarse una nefrostomía percutánea.
3. 3. Tras estabilizar al paciente debería programarse una sesión de litotricia.
Tras estabilizar al paciente se debería realizar un tratamiento antibiótico de 21 días de
4. 4.
duración.
Gráfico de respuestas
Comentario

En la pregunta anterior, el paciente tenía simplemente “sensación febril”. Sin embargo, ahora se
convierte en un hecho objetivo: fiebre de 39ºC. Aparte, nos hablan de hipotensión, leucopenia y
trombopenia. Nos encontramos, por tanto, ante el comienzo de una sepsis de origen urinario. Dado
que el cólico nefrítico no tendría por qué producir fiebre por sí solo, debemos sospechar una
infección urinaria asociada. El material purulento se encontrará proximalmente a la obstrucción, lo
que nos obliga a intentar drenarlo instrumentalmente (respuesta correcta 1).(R1)

395. Niña de 18 meses remitida al hospital por haberle detectado una masa abdominal
con motivo de un examen rutinario de salud. A la exploración se palpa una masa dura a
nivel de fosa renal derecha. Mínimo exoftalmos derecho con hematoma lineal en el
párpado superior derecho. Resto de la exploración no significativa. Entre los exámenes
complementarios destaca la existencia de niveles elevados en sangre de ferritina y de
catecolaminas en orina. ¿Qué tipo de entidad clínica sospecha?:

1. 1. Neuroblastoma.
2. 2. Tumor de Wilms.
3. 3. Hipernefroma.
4. 4. Teratoma.
Gráfico de respuestas
Comentario

Es importante hacer el diagnostico diferencial entre el neuroblastoma y el nefroblastoma (tumor de


Wilms). El neuroblastoma es el tumor sólido extracraneal más frecuente. La clinica más frecuente
es la de una masa abdominal que pasa la linea media y se acompaña de sintomas
paraneoplasicos como el hematoma lineal en párpado, la diarrrea secretora intensa, el síndrome
de opsiclonos mioclonos la HTA, el síndrome de Horner y otros.(R1)

396. A 50-year-old man presents to the ER complaining of cough and purulent sputum for
10 days. His past medical history is irrelevant. He smokes a pack of cigarettes per day
and drinks 3 cans of beer and half a bottle of whisky every day. Currently he takes no
medication. On physical examination, blood pressure is 120/78 mmHg, pulse is 70/min
and temperature is 38.5ºC. Chest x-ray confirms the diagnosis of pneumonia and
antibiotic treatment is started. Five days later, the symptoms have not subsided. New
chest x-ray shows pleural effusion so thoracentesis is performed, showing the presence
of purulent fluid. Which of the following is the best next step in management?

1. 1. Continue with the same antibiotic treatment


2. 2. Discontinue current treatment and initiate treatment with oseltamivir
3. 3. Place a chest tube

 
 
 
 
4. 4. Change antibiotic treatment
Gráfico de respuestas
Comentario
Place a chest tube. Indications for chest tube placement: - Empyema - Pleural fluid pH < 7.2 -
Pleural fluid glucose <60 mg / dl - LDH > 1000 - Loculated effusion - Positive culture(R3)

397. Primigesta de 40 años. Por ecografía, a la semana 19 de amenorrea, se diagnostica


la existencia de un CIR tipo I, sin apreciarse alteraciones morfológicas. Estará indicado
practicar:

1. 1. Determinar alfa-fetoproteína en líquido amniótico.


2. 2. Realizar biopsia corial.
3. 3. Funiculocentesis.
4. 4. Reposo y beta-miméticos.
Gráfico de respuestas
Comentario

Pregunta fácil sobre los métodos de diagnóstico prenatal. Cuando se diagnostica un CIR simétrico,
debemos intentar encontrar la causa, ya sea una cromosomopatía, enfermedad metabólica,
infección congénita, etc... Para ello, si estamos en la semana 19ª, no podremos realizar biopsia
corial (se realiza en la 8ª semana). Tampoco es válida la alfa-fetoproteína en líquido amniótico, ya
que su máxima fidelidad la tenemos a la 14ª-15ª semana. La opción 4 carece de sentido en este
contexto. Debemos realizar una funiculocentesis para poder obtener un diagnóstico prenatal, ya
que estamos en la semana 19ª (opción 3 correcta).(R3)

398. Joven de 26 años nos consulta porque desde el verano tiene máculas blanquecinas
en la espalda de diferentes tamaños pero menores de 2cm. Estas manchas descaman al
ser raspadas con la uña. La visión directa con KOH es positiva. ¿Cuál de los siguientes
tratamientos NO sería efectivo?:

1. 1. Sulfuro de selenio tópico.


2. 2. Miconazol tópico.
3. 3. Ciclopiroxolamina tópica.
4. 4. Griseofulvina oral.
Gráfico de respuestas
Comentario

Se trata de una pregunta fácil perteneciente al tema de infecciones cutáneas en dermatología. El


cuadro que se trata, es una Pitiriasis versicolor, producida por una levadura denominada
Pityrosporum ovale, que en su forma patógena se denomina Malassezia furfur. Da lugar a estas
lesiones maculosas hiper o hipocrómicas de predominio en tronco, y que descaman al rascado
(signo de la uñada). Son de fácil diagnóstico, pues poseen fluorescencia amarillo- naranja con la
luz de Wood, y en el examen en fresco, tras aplicarles una solución de KOH, se aprecian
filamentos y elementos redondos que dan lugar a la típica imagen en spaghetti y albondigas. El
tratamiento es tópico, con sulfuro de selenio, derivados imidazólicos, ciclopiroxolmina, y en los
casos extensos, oral con imidazoles.(R4)

399. What should be done if a 15-year-old boy presents mild gynecomastia and
tenderness at palpation?

 
 
 
 
1. 1. Reassurance
2. 2. Perform a CT scan of the brain
3. 3. Complete hormonal analysis
4. 4. Karyotyping
Gráfico de respuestas
Comentario
Reassurance. Gynecomastia is a male breast enlargement. It may be physiologic in certain stages
of life, like the neonatal period and puberty. In other cases it is due to hormonal disturbances and
even tumors. In this case, the boy is in his teenage years so the most likely cause is pubertal
gynecomastia. This condition is usually self-limited, and resolves before adulthood. Nevertheless,
breast enlargement and discomfort may cause or worsen psychological trauma. The best medical
attitude is to reassure the boy and the family.(R1)

400. ¿Cuál de las siguientes hernias acompaña al cordón espermático en el interior del
cremáster?:

1. 1. Hernia inguinal directa.


2. 2. Hernia inguinal indirecta.
3. 3. Hernia de Littre.
4. 4. Hernia obturatriz.
Gráfico de respuestas
Comentario

Es habitual encontrar preguntas sobre hernias en el ENARM. Es importante que conozca todos los
tipos posibles y además añada algunos datos epidemiológicos que seguro serán de mucha utilidad.
No olvide que la hernia inguinal indirecta protruye por el orifico profundo del conducto inguinal. De
esta forma se coloca dentro de del conducto acompañando al cordón espermático. Por esta razón
es habitual que este tipo de hernias llegue al escroto.(R2)

401. Un varón alcohólico de 42 años, tras un episodio de vómitos, comienza a presentar


dolor torácico retroesternal de gran intensidad, irradiado a la región cervical. En la
exploración física muestra tendencia al sueño, TA 90/45 mmHg, temperatura 39º C.
Auscultación pulmonar: murmullo conservado. Auscultación cardíaca rítmica. Señale lo
correcto:

1. 1. Probablemente se trata de una neumonía aspirativa.


Las esofagitis agudas, como es este caso, suelen cursar con fiebre alta y postración del
2. 2.
enfermo.
3. 3. Probablemente se trata de una mediastinitis aguda.
El paciente ha de ser reevaluado tras unas horas, cuando se pase el efecto de la ingesta
4. 4.
alcohólica, que es la que justifica la somnolencia y la fiebre por situación de estrés.
Gráfico de respuestas
Comentario

Pregunta de dificultad moderada y poco importante para el MIR. Dada la clínica de este paciente,
habría que sospechar una rotura esofágica espontánea tras el vómito (síndrome de Boerhaave),
que es una de las causas de mediastinitis aguda. Esta complicación se inicia de forma brusca tras
un vómito (por la rotura del esófago) o posquirúrgica, y cursa con fiebre, taquicardia, hipotensión,
taquipnea, enfisema subcutáneo y signo de Hamman (crujido sincrónico con el latido cardíaco en
decúbito lateral izquierdo, que en este caso no existe).

 
 
 
 
El tratamiento consiste en el drenaje quirúrgico inmediato, antibioterapia y reposo alimentario, por
lo que quedan descartadas la opción 4 y la 5. La 2 es falsa puesto que una esofagitis aguda no
tiene porque producir fiebre ni tendencia al sueño, y la 1 se descarta porque la clínica no es la de
una neumonía.(R3)

402. Una paciente de 32 años con un cuadro clínico de sangrado menstrual irregular,
dismenorrea, dispareunia y una esterilidad de tres años de evolución, presenta un
estudio hormonal normal y un USG transvaginal que informa un útero normal y múltiples
formaciones quísticas ováricas bilaterales de 4 cm que son signos ecográficos de
sospecha y elevación de CA-125 ¿Cuál sería la orientación diagnóstica?

1. 1. Cáncer de ovario.
2. 2. Hemorragia uterina disfuncional.
3. 3. Síndrome de ovarios poliquísticos.
4. 4. Endometriosis.
Gráfico de respuestas
Comentario

Si alguien se dejo engañar por la elevación del CA-125 es momento que le quede claro que no sólo
se eleva en el cáncer de ovario, puede estar incrementado en la endometriosis.

Estamos ante una paciente en edad fértil, que muestra una clínica compatible con endometriosis:
en primer lugar dolor pélvico de intensidad muy variable. Es característica la aparición de
dismenorrea progresiva, que no cede con la toma de anticonceptivos orales. También puede
manifestarse como dispareunia (dolor con las relaciones sexuales).

Otros síntomas significativos son las alteraciones menstruales y la infertilidad (la endometriosis
supone una causa habitual de esterilidad). La localización más frecuente es ovárica a modo de
quistes que se llenan de sangre (quistes de chocolate). En relación con el diagnóstico, el USG
doppler nos permite evaluar las características de la pelvis, pero actualmente el diagnóstico de
certeza nos lo da la laparoscopía.(R4)

403. The classic triad of Reiter's syndrome includes:

1. 1. Arthritis, prostatitis, conjunctivitis.


2. 2. Conjunctivitis, urethritis, arthritis.
3. 3. Urethritis, arthralgia, uveitis.
4. 4. Conjunctivitis, sexual contact and articular clicking.
Gráfico de respuestas
Comentario

En sentido estricto denominamos Síndrome de Reiter a la asociación de uretritis, artritis y


conjuntivitis. RecuerdE que aunque la uretritis desencadenante, habitualmente producida por
clamidia suele haber desaparecido cuando se presentan las manifestaciones articulares y
extrarticulares, en esta fase podemos encontrar una uretritis esteril. A pesar de ello suelen utilizar
el término de Síndrome de Reiter como sinónimo de artritis reactiva, por lo que no debes ser muy
exquisito con el léxico en este aspecto.(R2)

404. Which is the most common route of dissemination of ovarian cancer?

1. 1. Direct extension or local invasion.

 
 
 
 
2. 2. Transcoelomic spread.
3. 3. Lymphatic spread.
4. 4. Along the genital tract.
Gráfico de respuestas
Comentario

Esta pregunta de cáncer de ovario importante y no se puede fallar. La vía de diseminación mas
frecuente es la implantación directa de células tumorales en el peritoneo. Se implantan fácilmente
por el origen común del ovario y esta serosa. Recuerde que la extensión peritoneal difusa es la
forma más frecuente de presentación del cáncer de ovario. La extensión linfática se produce por
tres vías principales: paraaórticos, pélvicos e iliacos. La vía hematógena es menos frecuente.(R2)

405. La mayoría de las reacciones adversas de la sulfasalazina se deben a:

1. 1. El ácido 5-aminosalicílico.
2. 2. El ácido 4-aminosalicílico.
3. 3. La sulfapiridina.
4. 4. La olsalazina.
Gráfico de respuestas
Comentario
La sulfasalazina pertenece al grupo de los salicilatos, que se emplean en los brotes leves-
moderados de la colitis ulcerosa y de la enfermedad de Crohn. Aunque se usan principalmente en
el tratamiento de mantenimiento de estas enfermedades. Las reacciones adversas provocadas por
la sulfasalazina se deben fundamentalmente a la molécula derivada de las sulfamidas, la
sulfapiridina, que puede desencadenar reacciones alérgicas importantes.(R3)

406. An 80-year-old male patient refers fatigue and weakness that have increased for the
past few months. Complete blood count shows leukocytes 23,000, hemoglobin 7 g/L,
medium corpuscular volume 102 fL, platelets 50,000. Bone marrow biopsy shows
hypercellularity and a bone marrow blast count of 15%. Which of the following options
would be the most appropriate treatment?

1. 1. Regular blood transfusions.


2. 2. Hematopoietic Growth Factors.
3. 3. High-dose cytarabine therapy.
4. 4. Azacitidine.
Gráfico de respuestas
Comentario

Aunque todas las respuestas ofrecen opciones válidas de tratamiento, con asociación ocasional
entre ellas, es evidente que la edad del paciente aconseja terapias poco agresivas y las
transfusiones periódicas constituyen la forma más adecuada de paliar el fracaso medular, teniendo
además en cuenta el mal pronóstico a corto plazo.(R1)

407. El principio de Fick se usa para:

 
 
 
 
1. 1. Determinar presiones.
2. 2. Determinar el gasto cardíaco.
3. 3. Determinar gradientes.
4. 4. Determinar el Qp/Qs.
Gráfico de respuestas
Comentario

El principio de Fick es una técnica ideada a finales del siglo XIX para la medición del gasto
cardiaco, mediante la medición del consumo de O2 por minuto (VO2), el contenido en O2 de la
sangre en la arteria pulmonar (Cv) y el contenido en O2 de la sangre en una arteria periférica (Ca).

Por supuesto, no tendrías que preocuparte en caso de haber fallado una pregunta como ésta.(R2)

408. En el ganglio linfático existen linfocitos B, T, NK y células plasmáticas. Señale en


qué área ganglionar se disponen éstas últimas:

1. 1. Foliculos primarios.
2. 2. Corteza.
3. 3. Paracorteza.
4. 4. Cordones medulares.
Gráfico de respuestas
Comentario
Pregunta muy sencilla sobre la localización de cada célula en el sistema inmune. Estudia bien este
tema pues es una pregunta MIR probable. Los órganos linfoides secundarios son aquellos en los
que los antígenos entran en contacto con los antígenos, siendo éstos el bazo (antígenos
sanguíneos), ganglio linfático (antígeno en la linfa) y tejido linfoide asociado a mucosas (antígenos
externos). En el ganglio linfáTico predominan los linfocitos T, pero si desglosamos por partes: en la
corteza son B, en la parcorTeza son T y en los cordones Medulares son células plasMáticas.
Recuerda que la célula mayoritaria en el bazo son los linfocitos B (recuerda en el Bazo, los
linfocitos B). Es importante tener claro además que los órganos linfoides primarios son aquellos
donde maduran las células inmunes, siendo éstos la médula ósea (linfocitos B durante toda la vida
y los T sólo en adultos) y el timo (linfocitos T durante los primeros años de vida).(R4)

409. La radioterapia con quimioterapia simultánea o concurrente NO es el tratamiento de


elección en estados localmente avanzados de:

1. 1. Carcinoma epidermoide de esófago.


2. 2. Adenocarcinoma de próstata.
3. 3. Carcinoma no microcítico de pulmón.
4. 4. Carcinoma epidermoide de cérvix.
Gráfico de respuestas
Comentario

Pregunta relativamente sencilla. Debes saber que, en el cáncer de próstata, la quimioterapia tiene
un papel muy limitado, siendo más útil la hormonoterapia, aprovechando la dependencia de
andrógenos de este tumor (opción 3 incorrecta). En cambio, la radioterapia sí que puede resultar
de utilidad en tumores limitados a la glándula prostática, si son de un tamaño relativamente
pequeño. En el resto de las opciones, en mayor o menor medida pueden aplicarse QT + RT.(R2)

410. Señale la afirmación ERRÓNEA sobre el oncogén C-erb B2 (Her-2-neu).

 
 
 
 
1. 1. También se llama receptor del factor de crecimiento epidérmico humano 2.
2. 2. Las pacientes con cánceres que lo sobreexpresan pueden ser tratadas con trastuzumab.
3. 3. Los cánceres de mama en los que se sobreexpresa tienen mejor pronóstico.
4. 4. Puede encontrarse sobreexpresión en ganglios axilares metastásicos.
Gráfico de respuestas
Comentario

Esta pregunta puede complicarnoss si no dominamos este concepto.

Los cánceres de mama que sobreexpresan el Her-2-neu (C-erb B2) generalmente son más
agresivos y suelen tener receptores hormonales negativos. Por tanto NO tienen mejor pronóstico
como se afirma en la opción 3. Otra cosa es que tienen un tratamiento específico con Trastuzumab
(Herceptin®) y es lo que nos hace pensar, erróneamente, que tienen mejor pronóstico.(R3)

411. Paciente
de 36 años, con dos embarazos y partos normales, sin antecedentes médicos de interés,
acude a la consulta tras notarse tumoración dolorosa en la mama derecha. A la
exploración se palpan mamas nodulares, con sensibilidad aumentada (fase
premenstrual), identificándose nódulo de 3 cm, doloroso a la palpación, liso, móvil, no
adherido; no se palpan adenopatías. Se pide una mamografía, cuyo resultado se muestra
en la imagen. Ante el diagnóstico de sospecha, que actitud seria la más adecuada:

1. 1. Realizar unl ultrasonido de mama y PAAF si precisa.


La imagen es altamente sugestiva de cáncer de mama, por lo que nos planteamos
2. 2.
tumorectomía y biopsia intraoperatoria.
Se trata con toda seguridad de una mastopatía fibroquística, por lo que le prescribiremos
3. 3.
progesterona percutánea.
4. 4. Por el tamaño, se trata de un quiste simple de mama.
Gráfico de respuestas
Comentario

Es necesaria el ultrasonido mamario para completar el estudio de la lesión que en la mamografía


no tiene características de malignidad. El USG nos informará sobre el tamaño, bordes y

 
 
 
 
sonoluscencia de la lesión, para diferenciar entre lesiones sólidas y quísticas. La punción
aspiración con aguja fina ofrece garantía diagnóstica (citologia) en las mastopatias fibroquisticas de
predominio nodular o quistico; pero si existe alguna duda sobre la naturaleza benigna del proceso
se deberá realizar una biopsia.(R1)

412. Después de realizar una biopsia bajo control ultrasonográfico en la paciente del caso
clínico anterior, sabemos que presenta un riesgo relativo ligeramente aumentado (entre
1.5 y 5 veces) de padecer cáncer de mama. ¿Qué histología se ha encontrado más
probablemente?:

1. 1. Hiperplasia leve sin atipias.


2. 2. Adenosis esclerosante.
3. 3. Quiste simple.
4. 4. Metaplasia apocrina.
Gráfico de respuestas
Comentario

La mayoría de los cambios que se producen en la displasia no se relacionan con un aumento en la


incidencia del càncer de mama. El riesgo esta ligeramente aumentado (1.5- 5) en la hiperplasia
moderada o intensa sin atipia, adenosis esclerosante, fibroadenoma con patron morfológico
complejo y en el papiloma solitario o múltiple. El riesgo moderadamente aumentado (4- 5 veces)
está en la hiperplasia ductal atípica y en la lobulillar atípica.(R2)

413. Respecto al cáncer de vulva, señale la afirmación FALSA:

1. 1. La edad de aparición más frecuente es 65-75 años.


2. 2. El síntoma principal es el prurito vulvar.
3. 3. La localización más frecuente son los labios menores.
4. 4. La vía de extensión más importante es la linfática.
Gráfico de respuestas
Comentario

El cáncer de vulva es una enferedad relativamente rara, en la mayoría de los casos es un


carcinoma epideromide, también denominado escamoso. Suele aparecer en mujeres mayores de
65 años y los factores de riesgo son tabaco, los inmunosurpesores, el cáncer de cérvix, el VPH y la
distrofia vulvar con atipias. El sintoma pricipal es el pruirot y suelen localizarse en los labios
mayores.(R3)

414. ¿Cuál de las siguientes afirmaciones es FALSA con respecto a la neumonía


varicelosa?:

1. 1. Es una complicación que ocurre más frecuentemente en niños.


2. 2. Suele presentarse a los 3-5 días de comenzar con las lesiones cutáneas.
3. 3. Es la complicación más grave de la varicela.
4. 4. En la radiografía suele manifestarse con infiltrados nodulares y neumonitis intersticial.
Gráfico de respuestas
Comentario
Las complicaciones típicas de la varicela han sido preguntas de MIR en varias ocasiones.
Efectivamente la más grave, es la neumonía varicelosa, que presenta todas las características que
se enuncian en las opciones, salvo la de la opción 1: esta complicación es mucho más frecuente y
típica en adultos (20%). Recuerda que la complicación más frecuente globalmente es la

 
 
 
 
sobreinfección bacteriana de las lesiones. Y la extracutánea más frecuente es el compromiso del
SNC (este sí es más frecuente en niños), en forma de ataxia cerebelosa y/o meningitis aséptica,
que suele presentarse a los 21 días desde la aparición del exantema.(R1)

415. Señale cuál de los siguientes gérmenes NO produce típicamente infección


congénita:

1. 1. Virus hepatitis B.
2. 2. Mycoplasma.
3. 3. Virus herpes simple.
4. 4. Treponema pallidum.
Gráfico de respuestas
Comentario

Entre las infecciones congénitas, recuerde el acrónimo TORCH (Toxoplasma, Rubéola, CMV,
Herpes y Otros - lúes, HIV, Parvovirus B19, Enterovirus, hepatitis B y C- ). El Mycpolasma no da
problemas perinatales.(R2)

416. La hemorragia uterina disfuncional puede ser controlada administrando la hormona


faltante que generalmente es:

1. 1. Testosterona.
2. 2. Progesterona.
3. 3. Tiroides.
4. 4. Ninguno.
Gráfico de respuestas
Comentario

La HUD o hemorragia uterina disfuncional puede tratarse administrando progesterona que es muy
útil en caso de endometrios hiperplásicos proliferativos.(R2)

417. Un varón de 62 con antecedentes de tuberculosis presenta elevación de la presión


venosa yugular, hepatomegalia y ascitis. En la radiografía de tórax llama la atención la
presencia de una calcificación pericárdica. ¿Qué modificaciones de las ondas de presión
venosa central esperaría encontrar?:

1. 1. Morfología en raíz cuadrada con seno "x" profundo y seno "y" obliterado.
2. 2. Morfología en dip plateau con seno "x" obliterado y seno "y" profundo.
3. 3. Morfología en W con senos "x" e "y" profundos.
4. 4. Pulso venoso central aplanado con senos "x" e "y" obliterados.
Gráfico de respuestas
Comentario
Pregunta de dificultad moderada en forma de caso clínico sobre la fisiopatología de las
enfermedades del pericardio. La pregunta nos describe la clínica y la radiografía de una pericarditis
constrictiva. Es decir, predominan los datos de congestión sistémica como la ingurgitación yugular,
hepatomegalia, edemas de MMII, esplenomegalia..., esto se debe a que se comporta como una
insuficiencia cardíaca derecha . Existen calcificaciones del pericardio hasta en un 50% de las
ocasiones. En el cateterismo, las ondas "x" e "y" de la presión de la AD y AI son prominentes. La
descendente "y" es la onda más prominente, y está interrumpida por una rápida elevación de la
presión. En el ventrículo, la presión diastólica adquiere una morfología en raíz cuadrada o "dip-

 
 
 
 
plateau", pues la presión aumenta rápidamente cuando se llega al límite de la distensibilidad del
pericardio. Es característico el signo de Kussmaul, que es el aumento de la PVC con la inspiración.
La opción 2 describe el taponamiento cardiaco.(R3)

418. Masculino de 5 años de edad, que acude por presentar cuadro febril y odinofagia
con exudado blanquecino en ambas amígdalas que su pediatra trató con penicilina oral.
Dos días después, presenta petequias y púrpura palpables, principalmente en miembros
inferiores, así como artralgias en ambos tobillos que le impiden la marcha y dolor
abdominal de tipo cólico. Los estudios complementarios revelan 13 g/dl de Hb; leucocitos
10,500/mm3 con fórmula normal; plaquetas 485,000/mm3; tiempo de protrombina de 95%
y tiempo parcial de tromboplastina de 27 segundos (control 25 segundos). La causa más
probable de estos hallazgos es:

1. 1. Reacción a la penicilina.
2. 2. Púrpura de Schönlein-Henoch.
3. 3. Enfermedad de Kawasaki.
4. 4. Púrpura trombocitopénica idiopática.
Gráfico de respuestas
Comentario

Estamos ante púrpura anafilactoide o púrpura de Schonlein-Henoch. Esta enfermedad afecta sobre
todo a niños que presentan púrpura no trombopénica palpable a nivel de los miembros inferiores,
artromialgias y dolor abdominal de tipo cólico. La afectación renal no suele ser grave (GN
mesangial IgA).(R2)

419. La aparición de una metahemoglobinemia en el lactante menor puede estar en


relación con la introducción precoz en la alimentación de espinacas. ¿Cuál sería el
tratamiento indicado en este cuadro?:

1. 1. Exanguinotrasfusión parcial.
2. 2. Ventilación mecánica con una FiO2 mayor del 50%.
3. 3. Fenobarbital.
4. 4. Oxígenoterapia y azul de metileno.
Gráfico de respuestas
Comentario

La metahemoglobinemia en el lactante se debe a la intoxicación por nitratos, generalmente por la


introducción precoz en la alimentación complementaria de verduras como la col, espinacas o
remolacha. Ocasiona distrés respiratorio con cianosis color chocolate. La pulsioximetría y la
gasometría no la detectan de modo que requiere determinar el porcentaje de metahemoglobina. El
tratamiento de elección es la administración de azul de metileno y oxígeno. Respuesta 4
correcta.(R4)

420. Señale cuál de los siguientes virus es sensible a ribavirina:

1. 1. Hantavirus.
2. 2. Virus de la fiebre amarilla.
3. 3. Ebola.
4. 4. Virus de Ebstein-Barr.
Gráfico de respuestas

 
 
 
 
Comentario
Esta es una pregunta difícil sobre una indicación de la ribavirina poco habitual en nuestro medio.
Como recordaréis la ribavirina es un análogo de nucleósido que podemos usar en el tratamiento de
virosis como la hepatitis C (asociada a interferón) y el virus respiratorio sincicial (en pacientes
graves o de riesgo). Pero también se usa en el tratamiento de algunas fiebres hemorrágicas como
la producida por el Hantavirus o la fiebre de Lassa entre otras. Hay otros dos aspectos
preguntables sobre este antiviral: que produce hemólisis en un 10- 15 % de los pacientes (lo cual
siempre se monitoriza durante el tratamiento) y que es MUY teratogénico (categoría X).(R1)

421. El síndrome de los huesos hambrientos es:

1. 1. Las imágenes óseas erosivas en sacabocados características de la artritis reumatoide.


2. 2. Una complicación de la cirugía paratiroidea.
3. 3. La hipocalcemia del hijo con madre afecta de hiperparatiroidismo.
4. 4. La tríada clásica de hipoparatiroidismo.
Gráfico de respuestas
Comentario
Los trastornos del metabolismo del calcio constituyen un tema bastante preguntado en el MIR. El
hiperparatiroidismo primario es la causa más frecuente de hipercalcemia. El adenoma paratiroideo
segrega PTH de forma autónoma que actúa sobre el riñón aumentando la reabsorción de Ca y la
eliminación de P y sobre el hueso aumentando la resorción. Si el hiperparatiroidismo es de larga
evolución puede llegar a producirse osteítis fibrosa quística, secundaria al estimulo de los
osteoclastos por el exceso de PTH. El hueso está tan ávido del Ca que al instaurarse el tto
quirúrgico y eliminar el o los adenomas, se produce una hipocalcemia grave postoperatoria, que es
el denominado síndrome de los huesos hambrientos. Esta complicación quirúrgica debe detectarse
y tratarse precozmente.(R2)

422. Mujer de 40 años que tuvo infecciones amigdalares bacterianas repetidas en la


infancia, diagnosticada hace 10 años de estenosis mitral reumática. Actualmente acude
a urgencias por presentar palpitaciones irregulares acompañada de disnea importante
con expectoración rosada, edema importante de extremidades inferiores e ingurgitación
yugular. El factor desencadenante más probable de este empeoramiento será:

1. 1. Endocarditis infecciosa.
2. 2. Fibrilación auricular.
3. 3. Anemia intensa.
4. 4. Tromboembolismo pulmonar.
Gráfico de respuestas
Comentario

Se trata de un paciente con estenosis mitral, probablemente significativa y de larga evolución, que
ha caído en fibrilación auricular. Esto es prácticamente la norma en las estenosis mitrales
importantes, ya que la aurícula se va agrandando progresivamente y está sometida a mucha
presión, lo que la hace eléctricamente inestable. Por otro lado, en los pacientes con estenosis
mitral, el mantener la contracción auricular es de extraordinaria importancia, ya que necesita que
alguien “empuje” la sangre a través de la válvula estenótica, que supone una dificultad para el
llenado ventricular izquierdo. Por ello, suelen tolerar muy mal la FA, porque supone la pérdida de
esta contribución de la aurícula al llenado.(R2)

 
 
 
 
423. En relación a la madurez del pulmón señale lo CORRECTO:

1. 1. La relación lecitina/esfingomielina superior a 2 indica madurez pulmonar.


2. 2. La relación lecitina/esfingomielina superior a 1.5 indica madurez pulmonar.
3. 3. Los corticoides maternos no modifican la maduración pulmonar.
La betametasona (12 mg/día) IM 48-72 horas antes del parto no acelera la síntesis de
4. 4.
surfactante en fetos con menos de 34 semanas.
Gráfico de respuestas
Comentario

La relación lecitina / esfingomielina, que indica maduración pulmonar es de 2. El uso de corticoides


externos o los maternos ayudan a la maduración pulmonar fetal.(R1)

424. El síndrome que asocia poliposis adenomatosa y tumores del SNC es:

1. 1. Síndrome de Gardner.
2. 2. Síndrome de Peutz-Jeghers.
3. 3. Síndrome de Turcot.
4. 4. Poliposis juvenil.
Gráfico de respuestas
Comentario

Dentro de los citados, los únicos que cursan como poliposis adenomatosas son el síndrome de
Gardner y el de Turcot.

El síndrome de Gardner cursa con polipos adenomatosos como la poliposis colonica familiar pero
también con manifestaciones extraintestinales como osteomas en craneo, mandíbula y huesos
largos, tumores desmoides, anormalidades dentales, quistes epidermoides y sebáceos, lipomas,
fibromas, tumores de tiroides, glándulas suprarrenales arbol biliar e hígado pero no cursa con
tumores del SNC típico del síndrome de Turcot.(R3)

425. Female carriers of which of the following genes have a 50% lifetime risk of developing
breast cancer?

1. 1. Her 2-neu.
2. 2. c-erb B2.
3. 3. BRCA 1.
4. 4. RET.
Gráfico de respuestas
Comentario

Pregunta básica de un tema muy importante. El BRCA1 y el BRCA2 son los genes implicados en el
más de la mitad de los cánceres de mama hereditarios. No olvides el papel de Her 2 neu (c-erb B2)
en el cáncer de mama: los tumores metastásicos que expresan este gen pueden ser tratados con
Trastuzumab, aunque en principio son de peor pronóstico que los que no lo expresan.(R3)

426. Sobre la isquemia intestinal, señale lo INCORRECTO:

En la insuficiencia arterial crónica el dolor aparece 15 a 30 minutos después de las


1. 1.
comidas.
2. 2. La colitis isquémica no produce estenosis.

 
 
 
 
3. 3. La colitis isquémica es casi siempre no oclusiva, por lo que la angiografía no es útil.
La causa principal de la pérdida de peso en la insuficiencia arterial crónica es la
4. 4.
disminución de la ingesta por el miedo al dolor.
Gráfico de respuestas
Comentario

La isquemia mesentérica crónica es un proceso vascular que cursa con dolor anginoso
postpandrial como consecuencia de una obstrucción por placas de arteriosclerosis sobre todo en
arteria mesentérica superior. Ello hace que el paciente tenga miedo a comer y por eso pierde peso.
La prueba diagnóstica de elección es la arteriografía.

En cambio la colitis isquémica se produce por déficit agudo del riego a nivel de mesentérica inferior
pero no oclusivo, produciendo isquemia de mucosa cólica. Cursa en forma de dolor abdominal y
rectorragia. En la radiografía simple de abdomen se ven las características huellas digitales que
son la expresión de hematomas submucosos en la colonoscopia. Suele respetar el recto ya que
esta zona recibe aporte del plexo hemorroidal, siendo la zona más afectada el ángulo esplénico
donde la vascularización es más pobre por recibir a porte de ramas distales de mesentérica
superior e inferior. La arteriografía, al deberse a un trastorno de la microvascularización, no es útil
para el diagnóstico.(R2)

427. Indique cuál de los siguientes le parece que se ajusta más a la posibilidad que existe
de evolucionar a la cronicidad a partir de una hepatitis aguda por virus B en el adulto
joven:

1. 1. 1-2%.
2. 2. 20-30%.
3. 3. 75-80%.
4. 4. 95-98%.
Gráfico de respuestas
Comentario
Los principales factores que determinan la incidencia de cronicidad son la edad de infección y la
ausencia de síntomas. La tasa de cronicidad en pacientes adultos con hepatitis aguda clínica es
del 1- 2% (opción 1 correcta) mientras que las formas subclínicas cronifican en torno al 10%. El
grupo de mayor riesgo para la cronificación es el de los recién nacidos que cronifican hasta el
90%.(R1)

428. ¿Cuál es el síntoma clínico predominante en las fibrosis pulmonares?

1. 1. Expectoración.
2. 2. Hemoptisis.
3. 3. Disnea de esfuerzo.
4. 4. Dolor costal.
Gráfico de respuestas
Comentario

Pregunta de dificultad moderada sobre la fibrosis pulmonar.

En la fibrosis pulmonar se produce una alteración en el intersticio pulmonar que da lugar a una
alteración en el intercambio gaseoso por trastornos en la difusión, y esto se traduce en disnea.

 
 
 
 
En condiciones normales, el intercambio gaseoso tiene lugar en el tercio inicial del recorrido del
capilar por el alveolo, de forma que en los restantes 2/3 no hay difusión de gases porque ya no hay
gradiente de presión. Esto sería una especie de reserva.

Las enfermedades en las que hay alteración de la difusión no suelen producir disnea en reposo
porque se utiliza este espacio de “reserva”, pero durante el ejercicio aumenta la velocidad de la
sangre a su paso por el pulmón, de forma que esa “reserva” se vuelve insuficiente y aparece
disnea durante el ejercicio.(R3)

429. La espondilolisis representa una lesión lítica vertebral que se localiza a nivel de:

1. 1. Láminas.
2. 2. Cuerpo vertebral.
3. 3. Porción interarticular.
4. 4. Apófisis espinosas.
Gráfico de respuestas
Comentario
Pregunta de desgloses que debes recordar. La espondilolisis se define como la fractura,
generalmente por fatiga, de la pars interarticularis (porción vertebral que se encuentra entre las
apófisis articulares superior e inferior), y se aprecia en las radiografías oblícuas como "borramiento
del cuello del perrito de LaChapelle".(R3)

430. Una niña de 6 años es rescatada por los bomberos de un incendio. Inicialmente se
encuentra inconsciente pero responde con el oxígeno y al estímulo; llega a urgencias
alerta. La gasometría muestra PO2 normal, la radiografía de tórax es normal. En relación
a la lesión por inhalación:

1. 1. Debe excluirse dado que tiene una radiografía de tórax normal.


2. 2. Debe ser considerado a pesar de la oxigenación normal.
3. 3. Requiere evaluar los hallazgos de quemaduras facial y de las fibrillas nasales.
4. 4. El paciente puede irse de alta.
Gráfico de respuestas
Comentario

Una de las principales causas de morbi-mortalidad en un incendio es la inhalación de humos que


siempre debe ser considerada (opción 2 correcta), aunque la oxigenación sea normal.(R2)

431. La presencia de hepatopatía asociada a síndrome de Sjögren sugiere:

1. 1. Lupus eritematoso diseminado.


2. 2. Cirrosis biliar primaria.
3. 3. Cirrosis hepática criptogenética.
4. 4. Hepatitis tóxica.
Gráfico de respuestas
Comentario

Es una pregunta no demasiado importante, pero le ayudará a englobar tus conociemientos de


digestivo y reuma. El Sjögren al igual que el síndrome de CREST (esclerodermia en su forma
limitada) se relacionan con la Cirrosis Biliar Primaria, aunque raramente se da esta relación, y
desde luego el Sjögren ha sido menos preguntado en el ENARM. Normalmente el Sjögren

 
 
 
 
primario, pese a tener mayor afectación extraglandular, no afecta al hígado, y tampoco cuando se
asocia a LES. No hay razones para hepatitis tóxica ni cirrosis hepática de otro origen. La CBP es la
única relación típica, con elevación de enzimas de colestasis (poco de transaminasas) aunque sea
bastante rara en la clínica. Así que memoriza: "síndrome seco y CREST pueden asociarse a
CBP".(R2)

432. Niño de 11 meses con un cuadro de diarrea disentérica, que presenta anemia
hemolítica, trombocitopenia e insuficiencia renal aguda, el diagnóstico más probable es:

1. 1. Síndrome nefrótico.
2. 2. Síndrome hemolítico urémico.
3. 3. Púrpura trombótica trombocitopénica..
4. 4. Púrpura de Schönlein Henoch.
Gráfico de respuestas
Comentario

La respuesta correcta en este caso es el síndrome hemolítico urémico o SHU, que forma parte de
las anemias hemolíticas angiopáticas y que suele ser desencadenado por una infección
gastrointestinal por E. COli OH157.

Recuerde: anemia hemolítica microangiopática, trombocitopenia, insuficiencia renal y fiebre.(R2)

433. Ante el hallazgo sobre la cicatriz umbilical de un tejido rosado, con abundante
número de vasos, húmedo y con secreción mucosa o mucopurulenta, usted debe pensar
en:

1. 1. Pólipo umbilical.
2. 2. Granuloma umbilical.
3. 3. Infección de la herida umbilical.
4. 4. Persistencia del conducto ónfalo-mesentérico.
Gráfico de respuestas
Comentario

Tema muy importantee en el bloque de neonatología, en el que debes saber hacer diagnósticos
diferenciales.

Las masa umbilicales se pueden clasificar según su tamaño en dos grupos: las masas grandes
(hernia, onfalocele y gastrosquisis) y las masas pequeñas. Las masas pequeñas son de color rojo.
Estas masas pequeñas o masas rojas son dos: granuloma umbilical (blando, de aspecto vascular y
color rojo pálido) y pólipo (consistencia dura, color rojo brillante).(R2)

434. Señalar lo FALSO en relación a la taquipnea transitoria del RN o síndrome de distrés


respiratorio tipo II (SDR-II):

1. 1. Auscultación sin estertores.


2. 2. Taquipnea de inicio a las 12-24 horas de vida.
3. 3. Se cree que es debida a un retraso de la eliminación del líquido pulmonar fetal.
4. 4. Mejora con administración de oxígeno.
Gráfico de respuestas
Comentario

 
 
 
 
La taquipnea transitoria es la modalidad de distrés respiratorio típica del RN a término o casi a
término. Para su génesis, se requiere además un desencadenante obstétrico (parto por cesárea o
vaginal rápido), que hace que el tórax del RN no sufra compresión en el canal del parto, con lo que
el líquido pulmonar no sería desalojado. Precisamente, ese líquido pulmonar en la radiología te
debe dar la clave diagnóstica, junto con la evolución favorable. El comienzo de los síntomas se
produce hacia las 5- 6 horas de vida.(R2)

435. En un paciente en el que sospecha reflujo vesicoureteral, ¿qué prueba diagnóstica


indicaría?

1. 1. Ultrasonido.
2. 2. TAC.
3. 3. Cistouretrografía miccional.
4. 4. Cistoscopia.
Gráfico de respuestas
Comentario

La cistouretrografía miccional seriada (CUMS) sirve para descartar la presencia de reflujo


vesicoureteral. Está indicada en todos los niños y niñas menores de 5 años con infección del tracto
urinario, los afectos de una IVU febril (independientemente del sexo), en todo varón con una IVU y
en las niñas en edad escolar que hayan presentado dos o más episodios.(R3)

436. Uno de los siguientes hallazgos de laboratorio va en contra del diagnóstico de


anemia de las enfermedades crónicas:

1. 1. VCM: 83
2. 2. Transferrina:285 microg/dL.
3. 3. Sideremia: 30 microg/dL.
4. 4. Reticulocitos: 25%.
Gráfico de respuestas
Comentario
Esta es la típica pregunta donde nos exigen conocer a fondo los datos analíticos de la anemia de
las enfermedades crónicas, pero con un poco de picardía se resuelve bien fácil. Debéis manejar un
concepto muy importante, en estas anemias las citoquinas liberadas alteran la eritopoyesis en la
M.O. por lo tanto existe una inadecuada respuesta de la serie roja frente al cuadro anémico, y los
reticulocitos no aumentan en sangre periférica (opción 5 falsa). Recordad que el porcentaje de
reticulocitos normales es del 1- 2 % (40.000- 90.000).(R4)

437. Embarazada en el primer trimestre de gestación, con antecedentes de hijos


fallecidos por una inmunodeficiencia y cuyo defecto molecular en la familia ya es
conocido. ¿Qué prueba diagnóstica es la más idónea para el diagnóstico prenatal de la
inmunodeficiencia?

1. 1. Estudio molecular en los padres.


2. 2. Biopsia de corion en la 9.ª semana.
3. 3. Determinación de alfa-fetoproteína en la madre.
4. 4. Estudio de subpoblaciones de linfocitos en la madre.
Gráfico de respuestas
Comentario

 
 
 
 
Concepto poco preguntado en el ENARM. La biopsia corial se basa en la obtención de
vellosidades coriales por el cuello uterino o vía transabdominal. Obtiene directamente tejidos
fetales placentarios y los resultados genéticos pueden obtenerse en 48-72 horas. Presenta mayor
número de pérdidas fetales. Es el método que permite el diagnóstico más precoz de
cromosomopatías como la trisomía del 21 y, consecuentemente, de elección si queremos un
diagnóstico citogenético prenatal antes de las 12 semanas de gestación. Se realiza a partir de las
ocho semanas de gestación.(R2)

438. Hombre de 68 años con los siguientes antecedentes personales: exfumador,


hipertenso, diabético de larga evolución y dislipidémico. Intervenido quirúrgicamente
hace 4 años por isquemia crónica de extremidad inferior derecha practicándose by pass
aorto-bifemoral y reintervenido por embolia practicándose embolectomía, es traído a
urgencias por sus familiares presentando intensa palidez cutáneo-mucosa, obnubilación
y distensión abdominal. La familia refiere una evacuación abundante de heces negras,
pastosas y malolientes hace 24 horas. ¿Qué patología habrá que descartar en primer
lugar?:

1. 1. Hemorragia digestiva alta por fístula aorto-entérica.


2. 2. Úlcera péptica.
3. 3. Diverticulosis.
4. 4. Isquemia mesentérica.
Gráfico de respuestas
Comentario

El paciente presenta factores de riesgo cardiovascular y antecedentes de cirugía aórtica con


colocación de una prótesis. Dados esos antecedentes, debemos sospecha ante una hemorragia
digestiva con repercusión hemodinámica, la existencia de una fístula aorto- entérica. Respuesta 1
correcta.(R1)

439. Indique el estadio radiológico que presenta una paciente de 35 años, con
sarcoidosis, en cuya radiografía de tórax se aprecia un patrón difuso reticulonodular:

1. 1. Estadio O.
2. 2. Estadio I.
3. 3. Estadio III.
4. 4. Estadio IV.
Gráfico de respuestas
Comentario
Existen cuatro estadíos radiológicos en la sarcoidosis. El estadío I se caracteriza por adenopatías
hiliares bilaterales sin afectación parenquimatosa; el estadío II muestra adenopatías hiliares
bilaterales y afectación parenquimatosa difusa; en el estadío III se observa afectación
parenquimatosa difusa sin adenopatías hiliares. Cuando aparece patrón de fibrosis pulmonar en la
radiografía de tórax hablamos de estadío IV radiológico.(R3)

 
 
 
 

 
 
 
 

 
 
 
 

 
 
 
 

 
 
 
 

 
 
 
 

 
 
 
 

 
 
 
 

 
 
 
 

 
 
 
 

 
 
 
 

 
 
 
 

 
 
 
 

 
 

Das könnte Ihnen auch gefallen